Esercizi per casa (risolti)ippolitv/pdf/fns1_2020_solutions.pdfEsercizi per casa (risolti) Valerio...

23
Esercizi per casa (risolti) Valerio Ippolito * 13 maggio 2020 1 Lezione 1 (13 marzo 2020) Esercizio 1 Dilatazione dei tempi Qual è la velocità con la quale viaggia un orologio se il suo rate è pari alla metà del rate di un orologio a riposo? Soluzione dell’esercizio 1 Per rate intendiamo che lunghezza di un intervallo di tempo: in altri termini, stiamo guardando un orologio in movimento e lo vediamo scorrere due volte più lentamente di quanto vede chi lo indossa. Poiché la dilatazione dei tempi dipende da γ , questo significa semplicemente che Δt = 2Δt 0 = γ Δt 0 -→ γ =2, e dalla definizione di γ segue naturalmente che γ = 1 p 1 - β 2 =2 1 2 = p 1 - β 2 β 2 = v 2 c 2 = 3 4 v = 3 4 c =2.6 × 10 8 m/s. Esercizio 2 Contrazione delle lunghezze Un osservatore misura la lunghezza di un’asta quando questa è a riposo, ottenendo L =1m, e quando è in moto, ottenendo L 0 =0.5m. A che velocità viaggia l’asta quando è in moto? * [email protected] 1

Transcript of Esercizi per casa (risolti)ippolitv/pdf/fns1_2020_solutions.pdfEsercizi per casa (risolti) Valerio...

Page 1: Esercizi per casa (risolti)ippolitv/pdf/fns1_2020_solutions.pdfEsercizi per casa (risolti) Valerio Ippolito 13 maggio 2020 1 Lezione 1 (13 marzo 2020) Esercizio1 Dilatazionedeitempi

Esercizi per casa (risolti)

Valerio Ippolitolowast

13 maggio 2020

1 Lezione 1 (13 marzo 2020)

Esercizio 1 Dilatazione dei tempiQual egrave la velocitagrave con la quale viaggia un orologio se il suo rate egrave pari alla metagravedel rate di un orologio a riposo

Soluzione dellrsquoesercizio 1Per rate intendiamo che lunghezza di un intervallo di tempo in altri termini

stiamo guardando un orologio in movimento e lo vediamo scorrere due volte piugravelentamente di quanto vede chi lo indossa Poicheacute la dilatazione dei tempi dipendeda γ questo significa semplicemente che

∆t = 2∆tprime = γ∆tprime minusrarr γ = 2

e dalla definizione di γ segue naturalmente che

γ =1radic

1minus β2= 2

1

2=radic

1minus β2

β2 =v2

c2=

3

4

v =

radic3

4c = 26times 108 ms

Esercizio 2 Contrazione delle lunghezzeUn osservatore misura la lunghezza di unrsquoasta quando questa egrave a riposo

ottenendo L = 1 m e quando egrave in moto ottenendo Lprime = 05 m A che velocitagraveviaggia lrsquoasta quando egrave in moto

lowastvalerioippolitoroma1infnit

1

Soluzione dellrsquoesercizio 2La lunghezza a riposo egrave legata alla lunghezza misurata quando lrsquoasta egrave in

movimento dalla relazione Lprime = Lγ per cui γ = 2 La velocitagrave dellrsquoasta egravedunque

γ =1radic

1minus β2= 2

1

2=radic

1minus β2

β2 =v2

c2=

3

4

v =

radic3

4c = 26times 108 ms

Esercizio 3 Limite di piccole velocitagraveUn orologio atomico egrave posto su un Boeing 747 Lrsquoorologio misura lrsquointervallo

di tempo che separa due eventi ottenendo ∆t = 1 h quando si muove con velocitagravev = 1000 kmh rispetto ad un osservatore a terra Qual egrave lrsquointervallo di tempomisurato da un orologio identico ma a riposo rispetto allrsquoosservatore

Soluzione dellrsquoesercizio 3La velocitagrave egrave v = 1000 kmh c per cui possiamo lavorare nel limite

di piccole velocitagrave e semplificare un porsquo i conti Gli stessi eventi misuratidallrsquoosservatore saranno separati da un intervallo di tempo

∆tprime = γ∆t =1radic

1minus β2∆t

asymp(

1 +1

2β2

)∆t

equiv ∆t+ δt

dove δt asymp 15 ns

Esercizio 4 Contrazione delle lunghezzeUnrsquoasta di lunghezza L0 si muove con velocitagrave v lungo la direzione orizzon-

tale Nel sistema di riferimento dellrsquoasta questa forma un angolo θ rispettoallrsquoasse xprime Determinare la lunghezza dellrsquoasta misurata da un osservatore inquiete e lrsquoangolo che lrsquoasta forma con lrsquoasse x

Soluzione dellrsquoesercizio 4Assumiamo che i due sistemi di riferimento abbiano gli assi coincidenti e che

x (e xprime) siano orientati lungo la direzione del moto relativo dellrsquoasta rispetto allaboratorio Il punto in questo esercizio egrave che solo le lunghezze lungo la direzionedel moto x subiscono lrsquoeffetto della contrazione relativistica Dobbiamo dunque

2

scomporre lrsquoasta in lunghezza longitudinale e trasversale che nel sistema diriferimento in cui lrsquoasta egrave ferma valgono

L = L0 cos θ Lperp = L0 sin θ

applicare opportunamente la contrazione di Lorentz nel passaggio al sistema diriferimento del laboratorio (apice prime)

Lprime =1

γL0 cos θ Lprimeperp = L0 sin θ

ottenendo quindi la lunghezza dellrsquoasta misurata nel laboratorio

Lprime =radicLprime

2 + Lprimeperp2 = L0

radicsin2 θ +

1

γ2cos2 θ

= L0

radicsin2 θ + (1minus β2) cos2 θ = L0

radic1minus v2

c2cos2 θ

che egrave ovviamente minore di L0 e lrsquoangolo fra lrsquoasta e lrsquoasse xprime (lrsquoasse orizzontalenel sistema del laboratorio)

θprime = arctan

(LprimeperpLprime

)= arctan

(γy

x

)= arctan (γ tan θ)

Esercizio 5 Trasformazione delle velocitagraveUn osservatore in quiete sulla Terra vede due astronavi avvicinarsi lrsquouna

allrsquoaltra lungo la stessa direzione alla stessa velocitagrave La loro velocitagrave relativaegrave 07c Determinare la velocitagrave delle due astronavi misurata dallrsquoosservatore aTerra

Soluzione dellrsquoesercizio 5Il testo intende che ciascuna delle due astronavi vede lrsquoaltra viaggiare a una

velocitagrave di 07c In altri termini

v(B)A = v

(A)B = 07c equiv a

dove il pedice indica lrsquoastronave di cui misuriamo la velocitagrave e lrsquoapice indica ilsistema di riferimento in cui la velocitagrave egrave misurata Chiamiamo questa quantitagravea per semplificare i conti

Lrsquoincognita del problema egrave la velocitagrave di ciascuna delle due astronavi che egraveuguale in modulo per ipotesi nel sistema di riferimento del laboratorio ndash ovvero

v(lab)A = v

(lab)B equiv x

Si noti che ovviamentev(lab)A = minusv(lab)

B

3

in quanto le due astronavi si avvicinano lrsquouna allrsquoaltraEsprimiamo quindi la velocitagrave di B nel sistema di riferimento di A che egrave

nota in funzione della velocitagrave di B nel sistema di riferimento del laboratorioche egrave ignota tenendo opportunamente conto dei segni

v(A)B =

v(lab)B minus v(lab)

A

1minus v(lab)A

c

v(lab)B

c

=2x

1 + x2

c

= a

da cui

x =2plusmn

radic4minus 4a

2

c2

2ac=c

a

(1plusmn

radic1minus

(ac

)2)asymp 040c

dove abbiamo scartato la soluzione con x gt c Viste dal sistema di riferimentodel laboratorio le astronavi viaggiano quindi a +040c e minus040c

Esercizio 6 Leggi di trasformazioneIn relativitagrave speciale come si trasforma il volume E la densitagrave

Soluzione dellrsquoesercizio 6Il volume di un corpo in moto egrave il prodotto di tre componenti due trasverse e

una parallela al boost di Lorentz Perciograve il volume misurato a terra egrave V = V primeγLe densitagrave invece aumentano di un fattore γ

Esercizio 7 Intervallo invarianteSi considerino anzitutto due punti A e B nello spazio euclideo (rappresentato

per semplicitagrave in due dimensioni) Quale dei due cammini in figura egrave piugrave breve

x

y

A

B

x

y

A

B

Si consideri ora lo spazio di Minkowski (pseudo-euclideo) con lrsquousuale defi-nizione di distanza ds2 = c2dt2 minus dx2 minus dy2 minus dz2 Quale dei due tragitti egrave piugravebreve Si assuma che tutti i segmenti di curva che congiungono A e B siano ditipo tempo

x

ct

A

B

x

ct

A

B

4

Soluzione dellrsquoesercizio 7Nello spazio euclideo la linea retta tra due punti egrave quella che minimizza

la distanza dx Nello spazio di Minkowski a causa del segno negativo allecomponenti spaziali di ds2 la distanza ds egrave invece massima per il cammino inlinea retta

2 Lezione 2 (20 marzo 2020)

Esercizio 8 QuadrivelocitagraveQual egrave la quadrivelocitagrave di una particella che si muove di moto rettilineo uni-forme lungo lrsquoasse x con velocitagrave v = 3

5cSoluzione dellrsquoesercizio 8

La quadrivelocitagrave di una particella egrave rappresentata dal quadrivettore con-travariante

ηmicro equiv dxmicro

dτ=

(c

dt

dx

dy

dz

)

dove τ egrave il tempo proprio della particella ovvero il tempo misurato nel suo siste-ma di riferimento Il problema ci fornisce la velocitagrave nel sistema di riferimentodel laboratorio perciograve ci conviene esprimere il tempo proprio in funzione deltempo misurato nel laboratorio t = γτ

ηmicro =

(γc γ

dx

dt γ

dy

dt γ

dz

dt

)= γ(c v 0 0) =

1radic1minus

(35

)2(c

3

5c

)=c

4(5 3 0 0)

La norma della quadrivelocitagrave vale

ηmicroηmicro = gmicroνηmicroην =

c2

42(52 minus 32 minus 0minus 0) = c2

che egrave manifestamente invariante

Esercizio 9 Decadimento e dilatazione dei tempiMetagrave dei muoni di un fascio composto da muoni di energia fissata sopravvive

dopo aver viaggiato l = 600 m nel sistema di riferimento del laboratorio Qualegrave la velocitagrave dei muoni

Soluzione dellrsquoesercizio 9Da

N

N0= exp

(minus tτ

)= exp

(minus vtvτ

)= exp

(minus l

βcγτ0

)=

1

2

segue che

minus log1

2=

l

βγcτ0

βγ =βradic

1minus β2= minus l

log 12cτ0

equiv λ

5

ed elevando al quadrato

β =

radicλ2

1 + λ2asymp 080

Esercizio 10 Calcolo tensorialeUn tensore le cui componenti contravarianti si indicano come amicroν egrave la ge-

neralizzazione a due indici di un quadrivettore Mentre un quadrivettore ha 4componenti un tensore a due indici ne ha 4times 4 = 16 Se nel passaggio da unsistema di riferimento O in quiete a uno Oprime in moto lungo lrsquoasse x con velocitagraveV = βc le coordinate di un quadrivettore trasformano secondo la legge

xmicroprime = Λmicroνxν

dove le Λmicroν sono le componenti della matrice

Λ =

γ minusβγ 0 0minusβγ γ 0 0

0 0 1 00 0 0 1

analogamente un tensore trasforma secondo

amicroν prime = ΛmicroρΛνσaρσ

Analogamente a quanto accade per i quadrivettori le coordinate contravariantidi un tensore sono legate a quelle covarianti dalla relazione

amicroν = gmicroρgνσaρσ

Un tensore si dice simmetrico se le sue componenti sono uguali sotto scambiodegli indici (amicroν = aνmicro) e antisimmetrico se sono uguali ma di segno opposto(amicroν = minusaνmicro)

1 Quanti elementi indipendenti ci sono in un tensore simmetrico (siconsiderano dipendenti ad esempio a12 e a21 = a12)

2 E in un tensore antisimmetrico3 Simmetria e antisimmetria sono caratteristiche che si mantengono sotto

trasformazione di Lorentz4 Se un tensore amicroν egrave simmetrico lo egrave anche la sua versione covariante amicroν

E se egrave antisimmetrico5 Se amicroν egrave un tensore simmetrico e bmicroν un tensore antisimmetrico quanto

vale amicroνbmicroν6 Scomporre un tensore generico amicroν nella somma di due tensori uno

simmetrico e lrsquoaltro antisimmetricoSoluzione dellrsquoesercizio 10

6

1 Un tensore simmetrico ha 10 componenti indipendenti (a11 a12 a13 a14 a22 a23 a24 a33 a34 a44)

2 Un tensore antisimmetrico ha 6 componenti indipendenti (a12 a13 a14 a23 a24 a34)

3 Se un tensore egrave simmetrico significa che

amicroν = aνmicro

La versione trasformata del termine a sinistra egrave

amicroν prime = ΛmicroρΛνσaρσ

mentre quella del termine di destra egrave

aνmicroprime = ΛνρΛmicroσaρσ

ma applicando la definizione di tensore simmetrico nel riferimento di par-tenza (coordinate senza apici) e applicando la proprietagrave commutativa aitermini della matrice di Lorentz si ha

aνmicroprime = ΛνρΛmicroσaρσ = ΛνρΛmicroσa

σρ = ΛmicroσΛνρaσρ = ΛmicroρΛνσa

ρσ = amicroν prime

dove abbiamo ndash nellrsquoultimo passaggio ndash usato il fatto che sia ρ che σ sonoindici muti Ne concludiamo che un tensore simmetrico rimane tale dopotrasformazione di Lorentz

Se il tensore egrave antisimmetrico analogamente significa che

amicroν = minusaνmicro

La versione trasformata del termine a sinistra egrave sempre

amicroν prime = ΛmicroρΛνσaρσ

Applicando ancora una volta la definizione di tensore antisimmetrico nelriferimento di partenza e la proprietagrave commutativa si ha

aνmicroprime = ΛνρΛmicroσaρσ = ΛνρΛmicroσ (minusaσρ) = minusΛmicroσΛνρa

σρ = minusΛmicroρΛνσaρσ = minusamicroν prime

cioegrave anche lrsquoantisimmetria egrave mantenuta dopo trasformazione di Lorentz

4 Per il tensore simmetrico

amicroν = gmicroρgνσaρσ = gmicroρgνσa

σρ = gνσgmicroρaσρ = amicroν

Nel caso antisimmetrico

amicroν = gmicroρgνσaρσ = gmicroρgνσ (minusaσρ) = minusgνσgmicroρaσρ = minusamicroν

7

5 Vale

amicroνbmicroν = aνmicrob

νmicro = amicroν (minusbmicroν) = minus (amicroνbmicroν)rarr amicroνbmicroν = 0

dove al primo passaggio abbiamo semplicemente rinominato gli indici mu-ti e al secondo abbiamo usato le proprietagrave di simmetriaantisimmetriadei tensori di partenza Ne segue dunque che il prodotto fra un tensoresimmetrico e uno antisimmetrico egrave zero

Suggerimento almeno una volta nella vita conviene esplicitare la som-ma implicita nella notazione di Einstein per comprendere davvero questoformalismo cosigrave compatto

6 Basta costruire due somme che per costruzione allo scambio di indicisono lrsquouna simmetrica

amicroν equiv 1

2(tmicroν + tνmicro)

e lrsquoaltra antisimmetrica cioegrave

amicroν equiv 1

2(tmicroν minus tνmicro)

Egrave evidente che questi due tensori sono simmetriciantisimmetrici sottoscambio degli indici

Esercizio 11 Relazione fra forza e accelerazioneIn relativitagrave speciale forza e accelerazione sono in generale proporzionali fra

loro Usare la definizione di forza F = dpdt

Suggerimento scomporre lrsquoaccelerazione nella somma di un termine paral-lelo e un termine ortogonale alla direzione del moto (cioegrave alla velocitagrave)

Soluzione dellrsquoesercizio 11Scomponiamo lrsquoaccelerazione a lungo le direzioni ortogonale e parallela al

moto della particella (cioegrave a v)

a equiv aperp + a

Usando il fatto che

dt=

dv

dv

dt=

d

[(1minus v2

c2

)minus 12

]dt

dv

dt= minus1

2

(minus2v

c2

)(1minus v2

c2

)minus 32 dv

dt=

v

c2γ3 dv

dt

e che dalla definizione di prodotto scalare fra vettori

vdv

dt= v middot a

8

otteniamo che

F =dpdt

=d(mγv)

dt= mγ

dvdt

+mdγ

dtv

= mγa +mγ3 (v middot a) vc2

= mγ(aperp + a) +mγ3(v middot aperp + v middot a)v

= mγaperp +mγ3

(1

γ2+v2

c2

)a

= mγaperp +mγ3

(1minus v2

c2+v2

c2

)a

= mγaperp +mγ3a

Si noti come in relativitagrave speciale la forza non egrave in generale proporzionaleallrsquoaccelerazione

Esercizio 12 Classificazione dei quadrivettoriIl quadrimpulso egrave un quadrivettore di tipo spazio tempo o luce

Soluzione dellrsquoesercizio 12Poicheacute P 2 = E2

c2 minus p2 = m2 ge 0 saragrave di tipo tempo per particelle massive (e

di tipo luce per particelle senza massa)

Esercizio 13 Energia cineticaQuanto lavoro bisogna compiere per aumentare la velocitagrave di un elettrone

(m = 511 keVc2) dalla posizione di riposo a1 050c2 0990c3 09990c

Soluzione dellrsquoesercizio 13A questi elettroni dovremo dare una certa energia cinetica T in modo da

far passare lrsquoenergia totale da quella a riposo (γ = 0) cioegrave

Ei = mc2

aEf = T +mc2

Dalla relazioneEf = mγc2

segue

T = Ef minusmc2 = m(γ minus 1)c2 = m

1radic1minus v2

c2

minus 1

c2

per cui nei tre casi indicati servono rispettivamente 79 keV 31 MeV e 109 MeV

9

Esercizio 14 Energia di sogliaSupponiamo di far scontrare un fascio di protoni con un protone a riposo

Qual egrave lrsquoenergia minima che devono avere i protoni del fascio percheacute la reazione

p+ prarr p+ p+ p+ p

sia permessa (La massa del protone egrave pari a quella dellrsquoantiprotone p e vale938 MeVc2)

Esercizio 15 Diffusione elasticaChiamiamo elastico un urto (ldquoscatteringrdquo) in cui le particelle dello stato ini-

ziale e dello stato finale sono le stesse Si consideri un urto elastico fra unaparticella di massa nulla e una particella di massa m (bersaglio) che si trovaa riposo nel sistema di riferimento del laboratorio qual egrave la massima energiatrasferita dalla particella incidente al bersaglio Suggerimento si lavori nelsistema di riferimento del laboratorio e si espliciti il prodotto scalare fra gliimpulsi spaziali della particella di massa nulla prima e dopo lrsquourto in funzionedellrsquoangolo sempre nel sistema di riferimento del laboratorio fra la direzioneiniziale e finale della particella incidente

Se la particella incidente egrave un fotone e il bersaglio egrave un elettrone atomico ariposo di quanto varia la lunghezza drsquoonda del fotone fra prima e dopo lrsquourto

Soluzione dellrsquoesercizio 15Per scattering elastico intendiamo un processo in cui le particelle dello stato

iniziale sono le stesse di quelle dello stato finaleDenotiamo con k e P i quadrimpulsi della particella incidente e del bersa-

glio prima dellrsquourto e indichiamo con lrsquoapice le stesse quantitagrave dopo lrsquourto ilproblema ci dice che

k = (Ek)

kprime = (Eprimekrsquo)P = (mc0)

Partiamo dalla conservazione del quadrimpulso durante lrsquourto isoliamo la quan-titagrave che non misuriamo direttamente ndash cioegrave il quadrimpulso del bersaglio dopolrsquourto P prime ndash ed eleviamo al quadrato

k + P = kprime + P prime

P prime = k + P minus kprimem2c2 = 0 +m2c2 + 0 + 2Emminus 2(EEprime minus k middot krsquo)minus 2mEprime

e se indichiamo con θprime lrsquoangolo ndash nel riferimento del laboratorio ndash fra la direzioneiniziale e finale della particella incidente e usiamo il fatto che |k|c = E e |krsquo|c =

10

Eprime

0 = 2mc2(E minus Eprime)minus 2(EEprime minus EEprime cos θprime)

mc2(Eprime minus E) = minusEEprime(1minus cos θprime)

Eprime(mc2 + E(1minus cos θprime)) = mc2E

Eprime =E

1 + Emc2 (1minus cos θprime)

Il bersaglio rinculeragrave di una energia E minus Eprime massima per θ = π Il valoremassimo di questrsquoenergia di rinculo

E minus E

1 + 2 Emc2

= E2Emc2

1 + 2Emc2

prende il nome ndash nel caso dello scattering Compton in cui la particella incidenteegrave un fotone e il bersaglio egrave un elettrone atomico ndash di picco Compton

Cosa cambia fra un fotone di energia E ed uno di energia Eprime Dalla mecca-nica quantistica

E = hν =hc

λ

cioegrave cambia la lunghezza drsquoonda del fotone

Eprime =hc

λprime=

hcλ

1 +hcλ

mc2 (1minus cos θprime)

1

λprime=

1 +hcλ

mc2 (1minus cos θprime)

λprime = λ

(1 +

hc

λmc2(1minus cos θprime)

)

λprime = λ+h

mc(1minus cos θprime) equiv λ+ λc(1minus cos θprime)

dove abbiamo definito la lunghezza drsquoonda Compton dellrsquoelettrone λc che rap-presenta la scala di lunghezza sotto la quale gli effetti della meccanica quantisticarelativistica divengono importanti

3 Lezione 3 (27 marzo 2020)

Esercizio 16 Trasformazione delle velocitagraveUn oggetto si muove di moto rettilineo uniforme con velocitagrave costante αc versoun secondo oggetto immobile A che velocitagrave dobbiamo muoverci noi lungo lastessa direzione per vedere entrambi gli oggetti muoversi con velocitagrave uguali eopposte

11

Soluzione dellrsquoesercizio 16Se indichiamo con lrsquoapice la velocitagrave nel nostro sistema di riferimento che

egrave in moto rispetto al sistema di riferimento del secondo oggetto la legge ditrasformazione delle velocitagrave ci dice che

vprime1 =v1 minus V

1minus v1cVc

=αcminus V1minus αVc

vprime2 =v2 minus V

1minus v2cVc

= minusV

Stiamo cercando V la nostra velocitagrave nel sistema di riferimento del secondooggetto tale che le velocitagrave dei due oggetti nel nostro sistema di riferimentosiano uguali e opposte

vprime1 = minusvprime2 = V

perciograve la richiesta egrave

αcminus V1minus αVc

= V

αcminus V minus V + αV 2

c= 0

(αc)V 2 + (minus2)V + (α)c = 0

V =1minusradic

1minus α2

αc

dove abbiamo scelto la soluzione dellrsquoequazione di secondo grado con V le c

Esercizio 17 Conseguenze della relativitagraveUna navicella spaziale in moto rettilineo uniforme con velocitagrave 05c in al-

lontanamento dalla Terra egrave in orbita verso Plutone che si trova a 75times 109 kmdi distanza dalla Terra Non appena raggiunto il pianeta la comandante inviaun segnale radio alla base a Houston per chiedere lrsquoautorizzazione allrsquoatter-raggio Quanto tempo impiega la richiesta a raggiungere la base secondo lacomandante E secondo i suoi colleghi a Houston

Soluzione dellrsquoesercizio 17Lrsquoonda radio viaggia a velocitagrave c secondo ogni sistema di riferimento Per

lrsquoosservatore lrsquoonda radio percorreragrave il tragitto Plutone-Terra in un tempo

tTerra =L

c=

75times 109 km

3times 108 msasymp 25 000 s

mentre per la comandante saragrave passato un tempo inferiore che dipende dal fat-tore γ dellrsquoastronave nel sistema di riferimento della Terra secondo la relazione

tcomandante =tTerra

γasymp 25 000 stimes

radic1minus 052 asymp 21 650 s

12

Esercizio 18 Energia cineticaHa piugrave energia un protone che viaggia a 09999999896c o un Boeing 747 al

decollo

Esercizio 19 Dilatazione dei tempiUna scienziata misura che un fascio di particelle selezionate con impulso di

10 GeVc si degrada dellrsquo84 dopo aver percorso 1 m Se la massa di questeparticelle egrave 498 MeVc2 qual egrave la loro vita media

Soluzione dellrsquoesercizio 19La distanza misurata dalla scienziata egrave chiaramente riferita al suo sistema

di riferimento quello del laboratorio ed egrave legata alla vita media delle particelledel fascio τ dalla relazione

Ldecay = 1 m = βγcτ =pc

E

E

mc2cτ =

p

mccτ

e dalla legge del decadimento se indichiamo con N0 il numero di particelleinizialmente presenti nel fascio e con N il numero di particelle misurato si haN

N0= 1minus084 = exp

(minus L

Ldecay

)= exp

(minus L

pmccτ

)rarr τ =

mcL

pc log(N0

N

) asymp 9times 10minus11 s

Esercizio 20 Dilatazione dei tempiVi trovate a dover studiare un fascio di particelle di cui conoscete lrsquoenergia

ndash 2 GeV ndash ma non la massa a quanto ne sapete potrebbero essere composti daelettroni (di massa 511 keVc2) o protoni (938 MeVc2) Avete a disposizionedue rivelatori identici in grado di registrare con precisione il tempo in cui unaparticella li attraversa Come potete utilizzarli per determinare se il vostrofascio contiene elettroni o protoni

Soluzione dellrsquoesercizio 20Disponendo i due rivelatori lungo la direzione del fascio a distanza ∆L

lrsquouno dallrsquoaltro si puograve misurare il tempo impiegato dalle particelle per passaredallrsquouno allrsquoaltro e quindi la loro massa infatti se ∆t egrave la distanza temporalefra i segnali dei due rivelatori

∆L = v∆t = βc∆t =pc2

E∆t

Questo principio egrave usato nei cosiddetti rivelatori di time of flight che sono usatiper discriminare diversi tipi di particelle una particella di tipo 1 e una di tipo2 infatti percorreranno la distanza ∆L in tempi diversi legati alle rispettivemasse dalla relazione

∆L1 = ∆L2

p1c2

E1∆t1 =

radicE2 minusm2

1c2c

E∆t1 =

radicE2 minusm2

2c2c

E∆t2

13

Esercizio 21 Energia di sogliaDue fisici delle particelle vogliono produrre il bosone Z una particella di

carica neutra e di massamZ = 91 GeVc2 e discutono come fare Alice proponedi far scontrare fasci di elettroni e positroni di energia identica che viaggianodunque con impulso spaziale uguale in modulo e direzione ma di verso oppostoproducendo Z tramite il processo

e+ + eminus rarr Z

mentre Bob preferisce scontrare un fascio di protoni su un bersaglio fisso diidrogeno tramite il processo

p+ prarr Z + p+ p

Chi dei due avragrave bisogno di fasci di particelle di energia piugrave alta La massa delprotone egrave di 9383 MeVc2 quella dellrsquoelettrone di 511 keVc2

Esercizio 22 Leggi di conservazioneUn fotone (particella di massa nulla) puograve decadere in un elettrone e in un

positrone (entrambi di massa 511 keVc2) tramite il processo

γ rarr e+ + eminus

Esercizio 23 Decadimento βminus

Quali sono lrsquoenergia minima e massima dellrsquoelettrone nel decadimento

nrarr p+ eminus + νe

se il neutrone decade da fermo La massa del neutrone egrave di 9396 MeVc2 quelladel protone di 9383 MeVc2 e quella dellrsquoelettrone di 511 keVc2 si assuma chelrsquoantineutrino elettronico νe abbia massa nulla

Esercizio 24 Leggi di conservazioneIl decadimento

prarr n+ e+ + νe

egrave permesso

4 Lezione 4 (3 aprile 2020)

Esercizio 25Usando il fatto che hc = 1973 MeVfm si dimostri che in un sistema di unitagrave dimisura in cui h = c = 1 vale

14

1 1 GeVminus2 = 0389 mb

2 1 m = 5068times 1015 GeVminus1

3 1 s = 15times 1024 GeVminus1

Ricordiamo che 1 b = 1times 10minus28 m2 e che

[hc] = [Jsms] = [E][L]

Soluzione dellrsquoesercizio 25Lrsquoidea egrave di capire per quale potenza di hc e c va moltiplicato il termine a

sinistra di ciascuna equazione per ottenere il termine di destraPer cui

bull [1 GeVminus2][hc]α = [E]minus2[E]α[L]α = [0389 mb] = [L]2 da cui segue α = 2

e 1 GeVminus2(hc)2 = 1973 MeVfm1 GeV =

(01973times 10minus15 GeVm

1 GeV

)2

= 0389 mb

bull [1 m][hc]α = [L][E]α[L]α = [5068times 1015 GeVminus1] = [E]minus1 da cui segueα = minus1 e 1 m(hc)minus1 = 1 m

1973 MeVfm = 1 m01973times 10minus15 GeVm

= 5068times 1015 GeVminus1

bull [1 s][hc]α[c]β = [T ][E]α[L]α[L]β [T ]minusβ = [T ][E]α[L]α+β [T ]minusβ = [15times 1024 GeVminus1] =[E]minus1 da cui segue α = minus1 β = 1 e 1 s(hc)minus1c = 1 s

1973 MeVfm299 792 458 ms =299 792 458 m

01973times 10minus15 GeVm= 15times 1024 GeVminus1

Esercizio 26 Massa invarianteTre protoni (mp = 938 MeVc2) hanno impulsi uguali in modulo (p =

3 GeVc) e che formano angoli di 120 lrsquouno con lrsquoaltro Qual egrave la massainvariante del sistema

Esercizio 27 Energia di sogliaSi consideri il processo

γ + prarr p+ π0

dove il fotone ha massa nulla il protone ha massa di 938 MeVc2 e il π0 hamassa di 135 MeVc2

1 Se il protone egrave a riposo qual egrave lrsquoenergia minima che deve avere il fotoneincidente percheacute la reazione abbia luogo

2 La stessa reazione puograve avvenire nellrsquouniverso in cui un protone dei raggicosmici di alta energia puograve collidere con uno dei fotoni della radiazionecosmica di fondo di energia dellrsquoordine di 1 meV Qual egrave in questo casolrsquoenergia minima che deve avere il protone percheacute la reazione abbia luogo

Esercizio 28 Conseguenze della relativitagraveUn misterioso supereroe pattuglia a velocitagrave molto elevata la periferia roma-

na Allrsquoincrocio con via di Tor Bella Monaca incontra un semaforo e ndash vedendolo

15

verde ndash attraversa senza rallentare Una pattuglia della polizia municipale loferma e lo multa per esser passato col rosso Assumendo sia il supereroe che ivigili siano nel giusto a che velocitagrave viaggiava il supereroe

Soluzione dellrsquoesercizio 28Per la polizia municipale il semaforo emette fotoni di energia

E0 = hν0 =hc

λrossoasymp hc

630 nm

mentre il supereoe vede fotoni di energia

E = hν =hc

λverdeasymp hc

490 nm

e dalle trasformazioni di Lorentz indicando con γ e β le variabili calcolateusando la velocitagrave del supereroe misurata dalla municipale possiamo scrivere

E = γ(E0minusβp0) = γ(E0minusβE0) =1minus βradic1minus β2

E0 =1minus βradic

(1 + β)(1minus β)E0 =

radic1minus βradic1 + β

E0

dove abbiamo usato il fatto che i fotoni hanno massa nulla Perciograve

E

E0=

hcλverdehc

λrosso

=λrosso

λverde=

radic1minus βradic1 + β

rarr v = βc = 025c

Esercizio 29 Conseguenze della relativitagraveI neutrini sono particelle di massa molto piccola e al momento ignota Uno

dei modi con cui egrave stato possibile dedurre un limite superiore al suo valore egravestata lrsquoosservazione nel 1987 di neutrini prodotti dalla supernova 1987A1 chesi trova a 168000 anni luce dalla Terra Sono stati osservati due segnali dineutrini che possono essere schematizzati come segue si egrave osservato prima unneutrino di 35 MeV di energia seguito a 9 s di distanza da un secondo segnaledi 13 MeV Si assuma che questo ritardo sia dovuto al fatto che la massa delneutrino non egrave nulla e si calcoli questrsquoultima

Esercizio 30 DecadimentoAnimali e piante assumono dallrsquoatmosfera diversi composti contenenti car-

bonio Il carbonio presente nellrsquoatmosfera egrave predominantemente 126 C ma sono

presenti piccole concentrazioni del suo isotopo 146 C (un atomo ogni 1012) che

decade con emissione di elettroni attraverso il processo2

146 C rarr14

7 N + eminus + νe

con un tempo di dimezzamento di 5700 anni1httpsenwikipediaorgwikiSN_1987ANeutrino_emissions2Di altro non si tratta che del decadimento βminus nrarr p+ e+ νe

16

1 Qual egrave la concentrazione di 146 C dopo 11400 anni

2 Animali e piante assumono in vita proporzioni fisse di 146 C e 12

6 C mentrealla loro morte la quantitagrave di 14

6 C inizia a diminuire Avete a portata dimano un relitto di legno per cui misurate una emissione di elettroni daldecadimento di 14

6 C pari al 61 di quella di un pezzo di legno vivodella stessa massa quanti anni ha il manufatto

Esercizio 31 Energia cinetica e trasformazioni di LorentzDue particelle identiche di massa m ed energia cinetica T collidono frontal-

mente Qual egrave la loro energia cinetica relativa (ossia lrsquoenergia cinetica di unaparticella misurata nel sistema di riferimento dellrsquoaltra particella)

Esercizio 32 Energia nel centro di massaLa reazione

πminusprarr K0Λ0

avviene con unrsquoenergia nel centro di massa diradics = 3 GeV La massa del πminus egrave

di 1396 MeVc2 la massa del protone egrave di 938 MeVc2 la massa del Kminus egrave di498 MeVc2 e quella della Λ0 di 11 GeVc2

1 Calcolare lrsquoimpulso di πminus e Λ0 nel sistema di riferimento del centro dimassa

2 Se il protone egrave a riposo il K puograve essere emesso allrsquoindietro nel sistemadi riferimento del laboratorio

Esercizio 33 DecadimentoIl mesone φ0 egrave una particella neutra3 di circa 1 GeVc2 di massa che puograve

decadere in una coppia di particelle

φ0 rarr K+ +Kminus

di massa identicamK = 494 MeVc2 Si assuma di produrre φ0 di impulso notoegrave possibile che uno dei due K sia prodotto a riposo nel sistema di riferimentodel laboratorio

Esercizio 34 DecadimentoUn fascio di anti-neutrini muonici νmicro si puograve generare selezionando pioni

o kaoni π+ e K+ e facendoli passare in un lungo tubo in cui egrave stato fattoil vuoto4 in modo che dopo un certo tragitto L una buona parte di loro saragrave

3Il collisore DAFNE ai Laboratori Nazionali di Frascati produce specificatamente parti-celle di questo tipo tramite il processo e+ + eminus rarr φ0 httpswwwyoutubecomwatchv=L5yB9gDGKms

4Una tecnica di questo tipo egrave stata usata per inviare ai Laboratori Nazionali del Gran Sassodei fasci di neutrini prodotti al CERN di Ginevra httpsvideoscernchrecord985892

17

decaduta tramite i processi

π+ rarr microminus + νmicro

K+ rarr microminus + νmicro

Se lrsquoimpulso di pioni e kaoni egrave di 200 GeVc e la loro vita media di 26 ns e12 ns rispettivamente

1 Quanto a lungo viaggiano nel laboratorio i due tipi di particelle2 Se L = 1000 m quale saragrave la frazione di pioni e kaoni che saragrave decaduta

alla fine del tubo3 Qual egrave lrsquoenergia massima dei neutrini che egrave possibile misurare nel sistema

di riferimento del laboratorio nei due casi

5 Lezione 5 (16 aprile 2020)

Esercizio 35 Energia cineticaTra le eccellenze con sede in territorio elvetico primeggiano indiscutibilmente lacioccolata al latte e il Large Hadron Collider (LHC) Questrsquoultimo egrave un collisoredi particelle situato al CERN di Ginevra che fa scontrare due fasci identici diprotoni di impulso uguale in modulo e direzione ma verso opposto I fasci sonocomposti da circa 2800 gruppi (pacchetti) di 1011 particelle ciascuno Sapendoche lrsquoenergia nel centro di massa della collisione fra due protoni egrave

radics = 13 TeV

quanta cioccolata al latte dovete mangiare per assumere un numero di caloriepari allrsquoenergia cinetica di ciascun fascio di protoni di LHC

Soluzione dellrsquoesercizio 35Poicheacute si tratta di un collider e le energie in gioco sono molto maggiori della

massa del protone lrsquoenergia di ciascun protone egrave diradics2 = 65 TeV e quella

totale di un fascio egrave

2800times 1011 times 65 middot 1012 times 16 middot 10minus19J asymp 300 MJ

Secondo Google 100 g di cioccolata al latte apportano circa 500 cal = 500 times4184 J asymp 2 MJ Lrsquoenergia di un fascio di LHC corrisponde quindi a quella dicirca 15 kg di cioccolata

Esercizio 36 Scattering RutherfordUn fascio di particelle α di 100 MeV di energia e 032 nA di corrente5 collide

contro un bersaglio fisso di alluminio spesso 1 cm Una sperimentatrice prendeun rivelatore di 1 cm times 1 cm di superficie e lo posiziona ad un angolo di 30

5Per una spiegazione breve su come (e percheacute) si misura la corrente di un fascio di particel-le vedi httpswwwlhc-closerestaking_a_closer_look_at_lhc0beam_current Unatrattazione piugrave completa egrave data ad esempio da httpscdscernchrecord1213275filesp141pdf

18

rispetto al fascio di particelle a 1 m di distanza dal bersaglio Quante particelleα incideranno sul rivelatore ogni secondo

Soluzione dellrsquoesercizio 36Lrsquoalluminio ha una densitagrave di 27 gcm3 numero atomico 13 e massa atomica

27 uPoicheacute le particelle α sono nuclei di elio hanno carica 2e e la corrente di

032 nA corrisponde a un miliardo di particelle incidenti al secondo

dNidt

=032 nCs

2times 16times 10minus19 C= 1times 109 sminus1

Il rivelatore vede un angolo solido di

∆Ω equiv superficieraggio

2

=1 cm2

(1 m)2= 1times 10minus4 sr

Si tratta di uno scattering alla Rutherford per cui la sezione drsquourto per unitagravedi angolo solido rilevata ad un certo angolo θ vale

dΩ=

(zαzAle

2

4πε04E

1

sin2(θ2)

)2

pari a

dΩasymp(

2times 13times 4times etimes 16times 10minus19 C

4π times 89times 10minus12 Fmtimes 4times 100times 106 eV

1

sin2(π180 times 302)

)2

asymp 2times 10minus30 m2sr = 20 mbsr

(1)

e il numero di particelle visto dal rivelatore vale se indichiamo con nAl = ρAlNAAAl

la densitagrave numero di atomi di alluminio e con d lo spessore del rivelatore

dNrivelate

dt= ∆Ω

dΩnAld

dNi

dt

asymp 1times 10minus4 srtimes 2times 10minus30 m2srtimes 1times 104 cm2m2 times 27 gcm3 6times 1023 molminus1

27 gmol

= 120 Hz

Esercizio 37 Sezione drsquourtoUn bersaglio di idrogeno liquido di densitagrave ρ = 0071 gcm3 e volume V =

125 cm3 egrave bombardato da un fascio monoenergetico di pioni negativi con unflusso φ = 2times 107 mminus2sminus1 e si osserva la reazione

πminus + prarr π0 + n

rivelando i fotoni del decadimento π0 rarr γγ che avviene nel 988 dei casi Sela sezione drsquourto di quellrsquointerazione egrave σ = 40 mb quanti fotoni sono emessi ognisecondo

19

Esercizio 38 Sezione drsquourtoLa sezione drsquourto dellrsquoeffetto fotoelettrico per raggi X di 10 keV in carbonio

egrave 40 b per atomo Data una lastra di carbonio di 4 mm di spessore si calcoli1 il numero di bersagli per unitagrave di volume2 il coefficiente di assorbimento per effetto fotoelettrico dei raggi X di tale

energia3 la probabilitagrave che un raggio X incidente sulla lastra produca un elettrone

per effetto fotoelettrico

Esercizio 39 Sezioni drsquourtoUn bersaglio drsquooro di densitagrave superficiale ρS = 097 mgcm2 e superficie

SB = 1 cm2 viene colpito da un fascio di particelle α la cui sezione trasversaegrave contenuta completamente nellrsquoarea del bersaglio Sul bersaglio impattano37times 104 αs La sezione drsquourto di diffusione elastica ad un certo angolo θ valedσ

ddΩ = 1 bsr Calcolare1 la densitagrave di atomi del bersaglio per unitagrave di superficie2 il numero di particelle α rivelate in unrsquoora da un rivelatore di superficie

SR = 2 cm2 posto allrsquoangolo θ e a distanza DR = 01 m dal bersaglio3 Se il fascio di particelle viene sostituito da una sorgente radioattiva di

particelle α con distribuzione isotropa su tutto lrsquoangolo solido che vieneposta lungo la stessa linea del fascio a distanza DB = 20 cm dal bersaglioquanto tempo egrave necessario per rivelare con lo stesso rivelatore lo stessonumero di particelle calcolato sopra a paritagrave di sezione drsquourto

Esercizio 40 CinematicaUn fascio di positroni di 100 MeV di energia annichila su una targhetta fissa

di elettroni producendo due fotoni tramite il processo

e+ + eminus rarr γ + γ

Se uno dei due fotoni egrave emesso a 30 nel sistema del centro di massa1 quanto vale lrsquoenergia dei due fotoni nel sistema del centro di massa2 che energia e direzione avranno nel sistema del laboratorio

Esercizio 41 Sezione drsquourtoGli antineutrini νe prodotti da un reattore nucleare con potenza P = 16 GW

attraversano un bersaglio da 2000 l drsquoacqua posto a 50 m di distanza dal reat-tore

1 Supponendo che per ogni fissione sia prodotta unrsquoenergia termica di200 MeV e vengano emessi 6 antineutrini calcolare il numero medio direazioni

νe + prarr e+ + n

prodotte al giorno se la sezione drsquourto egrave σ = 1times 10minus43 cm2

20

2 Qual egrave la minima energia del neutrino per cui la reazione egrave permessa

6 Lezione 7 (8 maggio 2020)

Esercizio 42Un protone interagisce con un bersaglio producendo un pione di energia E =300 MeV Con un tracciatore posto a distanza d = 6 cm dal bersaglio egrave possi-bile rivelare la traiettoria del pione e risalire al punto di produzione del pionesul bersaglio Determinare lrsquoerrore sulla misura della posizione di tale puntocausato dalla presenza di un piano di alluminio di spessore L = 05 mm postoimmediatamente davanti al tracciatore (quindi a distanza d dal bersaglio) nellrsquoi-potesi che gli angoli delle tracce rispetto alla normale alle superfici del bersaglioe del piano di alluminio siano piccoli [mπ = 1396 MeV Al (Z = 13 A = 27ρ = 27 gcm3)]

Soluzione dellrsquoesercizio 42La deviazione standard dellrsquoangolo di diffusione coulombiana multipla vale

radic〈θ2〉 = 21 MeV

z

βc|p|

radicx

X0= 21 MeV

z

βc|p|

radicLρ

X0

dove p =radicE2 minusm2

π = 266 MeV β = pE = 0885 z = 1 Per lrsquoalluminioabbiamo

X0 = 24 gcm2 rarr X0ρ = 89 cm

per cui radic〈θ2〉 asymp 00067

e lo spostamento medio corrispondente rispetto alla posizione nominale egrave

δs = tan(00067)times 6 cm = 04 mm

Esercizio 43Un fascio misto di elettroni e antiprotoni passa attraverso una regione con

B = 2 T e dopo aver curvato per R = 3 m raggiunge una regione senza campomagnetico

1 Qual egrave lrsquoimpulso delle particelle selezionate2 Come discriminereste fra i due tipi di particelle3 Se usaste due scintillatori posti a 15 m di distanza quale risoluzione

temporale sarebbe necessaria per discriminare i due tipi di particelle4 Se i due scintillatori fossero spessi 2 cm e avessero una lunghezza di ra-

diazione X0 = 40 cm quanta energia perderebbero elettroni e protoninellrsquoattraversarli Si assuma una perdita di energia per ionizzazione di2 MeVcm e 25 MeVcm per protoni ed elettroni rispettivamente

21

5 Che indice di rifrazione dovrebbe avere un rivelatore a luce Cherenkovper discriminare elettroni e protoni

Soluzione dellrsquoesercizio 43Dalla definizione di forza

F =dpdt

= qvtimesB

|F| = mv2

R

per cui se il campo magnetico egrave ortogonale alla traiettoria

pc = qcBR

che poicheacute c = 03 mns possiamo esprimere in maniera piugrave conveniente come

pc[GeV] = 03B[T]R[m]

Ne segue che nel nostro esercizio (e tornando a c = 1)

p = 18 GeV

Trascurando le perdite di energia e ricordando che me mp i tempi divolo di elettroni e protoni valgono rispettivamente

te =L

βecasymp L

casymp 50 ns

etp =

L

βpc=

Lpradic

p2+m2p

casymp 56 ns

per cui serve una risoluzione dellrsquoordine del ns per discriminare elettroni e muonidal tempo di volo

Se gli scintillatori sono spessi 2 cm i protoni perderanno in ciascuno di essi2 cm times 2 MeVcm = 4 MeV per ionizzazione Gli elettroni perderanno invecenel primo rivelatore 5 MeV per ionizzazione e (me asymp 0)

18 GeV times(

1minus exp

(minus 2 cm

40 cm

))asymp 88 MeV

cioegrave dopo il primo rivelatore avranno 18 GeVminus93 MeV di energia e nel secondone perderanno altri

5 MeV + (18 GeV minus 93 MeV)

(1minus exp

(minus 2 cm

40 cm

))asymp 88 MeV

Lrsquoangolo di emissione di luce Cherenkov egrave dato da

cos θc =1

βnle 1

22

e poicheacute βe asymp 1 βp asymp 0887 per avere luce Cherenkov solo al passaggio deglielettroni (e non dei protoni) egrave necessario che

1 lt n lt 113

Esercizio 44Come misurereste la massa di una particella carica E quella di una parti-

cella neutra

Esercizio 45Un muone di energia E = 400 GeV penetra verticalmente nel mare Attra-

verso quale processo puograve essere rivelato A quale profonditagrave arriva prima didecadere

Esercizio 46Avete a disposizione dei tubi fotomoltiplicatori sensibili a lunghezze drsquoonda

fra 300 nm e 500 nm e volete rivelare la luce Cherenkov prodotta dal passaggio diun elettrone di 1 MeV di energia in un metro drsquoacqua Quanti fotoni vi aspettateche vengano prodotti Confrontate con il numero di elettroni di ionizzazioneche produrrebbe una particella α di 5 keV nello stesso rivelatore

Esercizio 47Determinare quali sono i processi piugrave probabili (cioegrave quelli di sezione drsquourto

piugrave alta) nellrsquointerazione fra1 fotoni di 1 MeV e atomi di alluminio2 fotoni di 100 keV e H23 fotoni di 100 keV e atomi di ferro4 fotoni di 10 MeV e atomi di carbonio5 fotoni di 10 MeV e atomi di piombo

Esercizio 48Volete misurare lrsquoimpulso di una particella carica che attraversa un campo

magnetico B ortogonale alla sua traiettoria Avete a disposizione tre rivelatori diposizione della stessa risoluzione spaziale δx come li disponete Che risoluzionein impulso vi aspettate di ottenere

23

  • Lezione 1 (13 marzo 2020)
  • Lezione 2 (20 marzo 2020)
  • Lezione 3 (27 marzo 2020)
  • Lezione 4 (3 aprile 2020)
  • Lezione 5 (16 aprile 2020)
  • Lezione 7 (8 maggio 2020)
Page 2: Esercizi per casa (risolti)ippolitv/pdf/fns1_2020_solutions.pdfEsercizi per casa (risolti) Valerio Ippolito 13 maggio 2020 1 Lezione 1 (13 marzo 2020) Esercizio1 Dilatazionedeitempi

Soluzione dellrsquoesercizio 2La lunghezza a riposo egrave legata alla lunghezza misurata quando lrsquoasta egrave in

movimento dalla relazione Lprime = Lγ per cui γ = 2 La velocitagrave dellrsquoasta egravedunque

γ =1radic

1minus β2= 2

1

2=radic

1minus β2

β2 =v2

c2=

3

4

v =

radic3

4c = 26times 108 ms

Esercizio 3 Limite di piccole velocitagraveUn orologio atomico egrave posto su un Boeing 747 Lrsquoorologio misura lrsquointervallo

di tempo che separa due eventi ottenendo ∆t = 1 h quando si muove con velocitagravev = 1000 kmh rispetto ad un osservatore a terra Qual egrave lrsquointervallo di tempomisurato da un orologio identico ma a riposo rispetto allrsquoosservatore

Soluzione dellrsquoesercizio 3La velocitagrave egrave v = 1000 kmh c per cui possiamo lavorare nel limite

di piccole velocitagrave e semplificare un porsquo i conti Gli stessi eventi misuratidallrsquoosservatore saranno separati da un intervallo di tempo

∆tprime = γ∆t =1radic

1minus β2∆t

asymp(

1 +1

2β2

)∆t

equiv ∆t+ δt

dove δt asymp 15 ns

Esercizio 4 Contrazione delle lunghezzeUnrsquoasta di lunghezza L0 si muove con velocitagrave v lungo la direzione orizzon-

tale Nel sistema di riferimento dellrsquoasta questa forma un angolo θ rispettoallrsquoasse xprime Determinare la lunghezza dellrsquoasta misurata da un osservatore inquiete e lrsquoangolo che lrsquoasta forma con lrsquoasse x

Soluzione dellrsquoesercizio 4Assumiamo che i due sistemi di riferimento abbiano gli assi coincidenti e che

x (e xprime) siano orientati lungo la direzione del moto relativo dellrsquoasta rispetto allaboratorio Il punto in questo esercizio egrave che solo le lunghezze lungo la direzionedel moto x subiscono lrsquoeffetto della contrazione relativistica Dobbiamo dunque

2

scomporre lrsquoasta in lunghezza longitudinale e trasversale che nel sistema diriferimento in cui lrsquoasta egrave ferma valgono

L = L0 cos θ Lperp = L0 sin θ

applicare opportunamente la contrazione di Lorentz nel passaggio al sistema diriferimento del laboratorio (apice prime)

Lprime =1

γL0 cos θ Lprimeperp = L0 sin θ

ottenendo quindi la lunghezza dellrsquoasta misurata nel laboratorio

Lprime =radicLprime

2 + Lprimeperp2 = L0

radicsin2 θ +

1

γ2cos2 θ

= L0

radicsin2 θ + (1minus β2) cos2 θ = L0

radic1minus v2

c2cos2 θ

che egrave ovviamente minore di L0 e lrsquoangolo fra lrsquoasta e lrsquoasse xprime (lrsquoasse orizzontalenel sistema del laboratorio)

θprime = arctan

(LprimeperpLprime

)= arctan

(γy

x

)= arctan (γ tan θ)

Esercizio 5 Trasformazione delle velocitagraveUn osservatore in quiete sulla Terra vede due astronavi avvicinarsi lrsquouna

allrsquoaltra lungo la stessa direzione alla stessa velocitagrave La loro velocitagrave relativaegrave 07c Determinare la velocitagrave delle due astronavi misurata dallrsquoosservatore aTerra

Soluzione dellrsquoesercizio 5Il testo intende che ciascuna delle due astronavi vede lrsquoaltra viaggiare a una

velocitagrave di 07c In altri termini

v(B)A = v

(A)B = 07c equiv a

dove il pedice indica lrsquoastronave di cui misuriamo la velocitagrave e lrsquoapice indica ilsistema di riferimento in cui la velocitagrave egrave misurata Chiamiamo questa quantitagravea per semplificare i conti

Lrsquoincognita del problema egrave la velocitagrave di ciascuna delle due astronavi che egraveuguale in modulo per ipotesi nel sistema di riferimento del laboratorio ndash ovvero

v(lab)A = v

(lab)B equiv x

Si noti che ovviamentev(lab)A = minusv(lab)

B

3

in quanto le due astronavi si avvicinano lrsquouna allrsquoaltraEsprimiamo quindi la velocitagrave di B nel sistema di riferimento di A che egrave

nota in funzione della velocitagrave di B nel sistema di riferimento del laboratorioche egrave ignota tenendo opportunamente conto dei segni

v(A)B =

v(lab)B minus v(lab)

A

1minus v(lab)A

c

v(lab)B

c

=2x

1 + x2

c

= a

da cui

x =2plusmn

radic4minus 4a

2

c2

2ac=c

a

(1plusmn

radic1minus

(ac

)2)asymp 040c

dove abbiamo scartato la soluzione con x gt c Viste dal sistema di riferimentodel laboratorio le astronavi viaggiano quindi a +040c e minus040c

Esercizio 6 Leggi di trasformazioneIn relativitagrave speciale come si trasforma il volume E la densitagrave

Soluzione dellrsquoesercizio 6Il volume di un corpo in moto egrave il prodotto di tre componenti due trasverse e

una parallela al boost di Lorentz Perciograve il volume misurato a terra egrave V = V primeγLe densitagrave invece aumentano di un fattore γ

Esercizio 7 Intervallo invarianteSi considerino anzitutto due punti A e B nello spazio euclideo (rappresentato

per semplicitagrave in due dimensioni) Quale dei due cammini in figura egrave piugrave breve

x

y

A

B

x

y

A

B

Si consideri ora lo spazio di Minkowski (pseudo-euclideo) con lrsquousuale defi-nizione di distanza ds2 = c2dt2 minus dx2 minus dy2 minus dz2 Quale dei due tragitti egrave piugravebreve Si assuma che tutti i segmenti di curva che congiungono A e B siano ditipo tempo

x

ct

A

B

x

ct

A

B

4

Soluzione dellrsquoesercizio 7Nello spazio euclideo la linea retta tra due punti egrave quella che minimizza

la distanza dx Nello spazio di Minkowski a causa del segno negativo allecomponenti spaziali di ds2 la distanza ds egrave invece massima per il cammino inlinea retta

2 Lezione 2 (20 marzo 2020)

Esercizio 8 QuadrivelocitagraveQual egrave la quadrivelocitagrave di una particella che si muove di moto rettilineo uni-forme lungo lrsquoasse x con velocitagrave v = 3

5cSoluzione dellrsquoesercizio 8

La quadrivelocitagrave di una particella egrave rappresentata dal quadrivettore con-travariante

ηmicro equiv dxmicro

dτ=

(c

dt

dx

dy

dz

)

dove τ egrave il tempo proprio della particella ovvero il tempo misurato nel suo siste-ma di riferimento Il problema ci fornisce la velocitagrave nel sistema di riferimentodel laboratorio perciograve ci conviene esprimere il tempo proprio in funzione deltempo misurato nel laboratorio t = γτ

ηmicro =

(γc γ

dx

dt γ

dy

dt γ

dz

dt

)= γ(c v 0 0) =

1radic1minus

(35

)2(c

3

5c

)=c

4(5 3 0 0)

La norma della quadrivelocitagrave vale

ηmicroηmicro = gmicroνηmicroην =

c2

42(52 minus 32 minus 0minus 0) = c2

che egrave manifestamente invariante

Esercizio 9 Decadimento e dilatazione dei tempiMetagrave dei muoni di un fascio composto da muoni di energia fissata sopravvive

dopo aver viaggiato l = 600 m nel sistema di riferimento del laboratorio Qualegrave la velocitagrave dei muoni

Soluzione dellrsquoesercizio 9Da

N

N0= exp

(minus tτ

)= exp

(minus vtvτ

)= exp

(minus l

βcγτ0

)=

1

2

segue che

minus log1

2=

l

βγcτ0

βγ =βradic

1minus β2= minus l

log 12cτ0

equiv λ

5

ed elevando al quadrato

β =

radicλ2

1 + λ2asymp 080

Esercizio 10 Calcolo tensorialeUn tensore le cui componenti contravarianti si indicano come amicroν egrave la ge-

neralizzazione a due indici di un quadrivettore Mentre un quadrivettore ha 4componenti un tensore a due indici ne ha 4times 4 = 16 Se nel passaggio da unsistema di riferimento O in quiete a uno Oprime in moto lungo lrsquoasse x con velocitagraveV = βc le coordinate di un quadrivettore trasformano secondo la legge

xmicroprime = Λmicroνxν

dove le Λmicroν sono le componenti della matrice

Λ =

γ minusβγ 0 0minusβγ γ 0 0

0 0 1 00 0 0 1

analogamente un tensore trasforma secondo

amicroν prime = ΛmicroρΛνσaρσ

Analogamente a quanto accade per i quadrivettori le coordinate contravariantidi un tensore sono legate a quelle covarianti dalla relazione

amicroν = gmicroρgνσaρσ

Un tensore si dice simmetrico se le sue componenti sono uguali sotto scambiodegli indici (amicroν = aνmicro) e antisimmetrico se sono uguali ma di segno opposto(amicroν = minusaνmicro)

1 Quanti elementi indipendenti ci sono in un tensore simmetrico (siconsiderano dipendenti ad esempio a12 e a21 = a12)

2 E in un tensore antisimmetrico3 Simmetria e antisimmetria sono caratteristiche che si mantengono sotto

trasformazione di Lorentz4 Se un tensore amicroν egrave simmetrico lo egrave anche la sua versione covariante amicroν

E se egrave antisimmetrico5 Se amicroν egrave un tensore simmetrico e bmicroν un tensore antisimmetrico quanto

vale amicroνbmicroν6 Scomporre un tensore generico amicroν nella somma di due tensori uno

simmetrico e lrsquoaltro antisimmetricoSoluzione dellrsquoesercizio 10

6

1 Un tensore simmetrico ha 10 componenti indipendenti (a11 a12 a13 a14 a22 a23 a24 a33 a34 a44)

2 Un tensore antisimmetrico ha 6 componenti indipendenti (a12 a13 a14 a23 a24 a34)

3 Se un tensore egrave simmetrico significa che

amicroν = aνmicro

La versione trasformata del termine a sinistra egrave

amicroν prime = ΛmicroρΛνσaρσ

mentre quella del termine di destra egrave

aνmicroprime = ΛνρΛmicroσaρσ

ma applicando la definizione di tensore simmetrico nel riferimento di par-tenza (coordinate senza apici) e applicando la proprietagrave commutativa aitermini della matrice di Lorentz si ha

aνmicroprime = ΛνρΛmicroσaρσ = ΛνρΛmicroσa

σρ = ΛmicroσΛνρaσρ = ΛmicroρΛνσa

ρσ = amicroν prime

dove abbiamo ndash nellrsquoultimo passaggio ndash usato il fatto che sia ρ che σ sonoindici muti Ne concludiamo che un tensore simmetrico rimane tale dopotrasformazione di Lorentz

Se il tensore egrave antisimmetrico analogamente significa che

amicroν = minusaνmicro

La versione trasformata del termine a sinistra egrave sempre

amicroν prime = ΛmicroρΛνσaρσ

Applicando ancora una volta la definizione di tensore antisimmetrico nelriferimento di partenza e la proprietagrave commutativa si ha

aνmicroprime = ΛνρΛmicroσaρσ = ΛνρΛmicroσ (minusaσρ) = minusΛmicroσΛνρa

σρ = minusΛmicroρΛνσaρσ = minusamicroν prime

cioegrave anche lrsquoantisimmetria egrave mantenuta dopo trasformazione di Lorentz

4 Per il tensore simmetrico

amicroν = gmicroρgνσaρσ = gmicroρgνσa

σρ = gνσgmicroρaσρ = amicroν

Nel caso antisimmetrico

amicroν = gmicroρgνσaρσ = gmicroρgνσ (minusaσρ) = minusgνσgmicroρaσρ = minusamicroν

7

5 Vale

amicroνbmicroν = aνmicrob

νmicro = amicroν (minusbmicroν) = minus (amicroνbmicroν)rarr amicroνbmicroν = 0

dove al primo passaggio abbiamo semplicemente rinominato gli indici mu-ti e al secondo abbiamo usato le proprietagrave di simmetriaantisimmetriadei tensori di partenza Ne segue dunque che il prodotto fra un tensoresimmetrico e uno antisimmetrico egrave zero

Suggerimento almeno una volta nella vita conviene esplicitare la som-ma implicita nella notazione di Einstein per comprendere davvero questoformalismo cosigrave compatto

6 Basta costruire due somme che per costruzione allo scambio di indicisono lrsquouna simmetrica

amicroν equiv 1

2(tmicroν + tνmicro)

e lrsquoaltra antisimmetrica cioegrave

amicroν equiv 1

2(tmicroν minus tνmicro)

Egrave evidente che questi due tensori sono simmetriciantisimmetrici sottoscambio degli indici

Esercizio 11 Relazione fra forza e accelerazioneIn relativitagrave speciale forza e accelerazione sono in generale proporzionali fra

loro Usare la definizione di forza F = dpdt

Suggerimento scomporre lrsquoaccelerazione nella somma di un termine paral-lelo e un termine ortogonale alla direzione del moto (cioegrave alla velocitagrave)

Soluzione dellrsquoesercizio 11Scomponiamo lrsquoaccelerazione a lungo le direzioni ortogonale e parallela al

moto della particella (cioegrave a v)

a equiv aperp + a

Usando il fatto che

dt=

dv

dv

dt=

d

[(1minus v2

c2

)minus 12

]dt

dv

dt= minus1

2

(minus2v

c2

)(1minus v2

c2

)minus 32 dv

dt=

v

c2γ3 dv

dt

e che dalla definizione di prodotto scalare fra vettori

vdv

dt= v middot a

8

otteniamo che

F =dpdt

=d(mγv)

dt= mγ

dvdt

+mdγ

dtv

= mγa +mγ3 (v middot a) vc2

= mγ(aperp + a) +mγ3(v middot aperp + v middot a)v

= mγaperp +mγ3

(1

γ2+v2

c2

)a

= mγaperp +mγ3

(1minus v2

c2+v2

c2

)a

= mγaperp +mγ3a

Si noti come in relativitagrave speciale la forza non egrave in generale proporzionaleallrsquoaccelerazione

Esercizio 12 Classificazione dei quadrivettoriIl quadrimpulso egrave un quadrivettore di tipo spazio tempo o luce

Soluzione dellrsquoesercizio 12Poicheacute P 2 = E2

c2 minus p2 = m2 ge 0 saragrave di tipo tempo per particelle massive (e

di tipo luce per particelle senza massa)

Esercizio 13 Energia cineticaQuanto lavoro bisogna compiere per aumentare la velocitagrave di un elettrone

(m = 511 keVc2) dalla posizione di riposo a1 050c2 0990c3 09990c

Soluzione dellrsquoesercizio 13A questi elettroni dovremo dare una certa energia cinetica T in modo da

far passare lrsquoenergia totale da quella a riposo (γ = 0) cioegrave

Ei = mc2

aEf = T +mc2

Dalla relazioneEf = mγc2

segue

T = Ef minusmc2 = m(γ minus 1)c2 = m

1radic1minus v2

c2

minus 1

c2

per cui nei tre casi indicati servono rispettivamente 79 keV 31 MeV e 109 MeV

9

Esercizio 14 Energia di sogliaSupponiamo di far scontrare un fascio di protoni con un protone a riposo

Qual egrave lrsquoenergia minima che devono avere i protoni del fascio percheacute la reazione

p+ prarr p+ p+ p+ p

sia permessa (La massa del protone egrave pari a quella dellrsquoantiprotone p e vale938 MeVc2)

Esercizio 15 Diffusione elasticaChiamiamo elastico un urto (ldquoscatteringrdquo) in cui le particelle dello stato ini-

ziale e dello stato finale sono le stesse Si consideri un urto elastico fra unaparticella di massa nulla e una particella di massa m (bersaglio) che si trovaa riposo nel sistema di riferimento del laboratorio qual egrave la massima energiatrasferita dalla particella incidente al bersaglio Suggerimento si lavori nelsistema di riferimento del laboratorio e si espliciti il prodotto scalare fra gliimpulsi spaziali della particella di massa nulla prima e dopo lrsquourto in funzionedellrsquoangolo sempre nel sistema di riferimento del laboratorio fra la direzioneiniziale e finale della particella incidente

Se la particella incidente egrave un fotone e il bersaglio egrave un elettrone atomico ariposo di quanto varia la lunghezza drsquoonda del fotone fra prima e dopo lrsquourto

Soluzione dellrsquoesercizio 15Per scattering elastico intendiamo un processo in cui le particelle dello stato

iniziale sono le stesse di quelle dello stato finaleDenotiamo con k e P i quadrimpulsi della particella incidente e del bersa-

glio prima dellrsquourto e indichiamo con lrsquoapice le stesse quantitagrave dopo lrsquourto ilproblema ci dice che

k = (Ek)

kprime = (Eprimekrsquo)P = (mc0)

Partiamo dalla conservazione del quadrimpulso durante lrsquourto isoliamo la quan-titagrave che non misuriamo direttamente ndash cioegrave il quadrimpulso del bersaglio dopolrsquourto P prime ndash ed eleviamo al quadrato

k + P = kprime + P prime

P prime = k + P minus kprimem2c2 = 0 +m2c2 + 0 + 2Emminus 2(EEprime minus k middot krsquo)minus 2mEprime

e se indichiamo con θprime lrsquoangolo ndash nel riferimento del laboratorio ndash fra la direzioneiniziale e finale della particella incidente e usiamo il fatto che |k|c = E e |krsquo|c =

10

Eprime

0 = 2mc2(E minus Eprime)minus 2(EEprime minus EEprime cos θprime)

mc2(Eprime minus E) = minusEEprime(1minus cos θprime)

Eprime(mc2 + E(1minus cos θprime)) = mc2E

Eprime =E

1 + Emc2 (1minus cos θprime)

Il bersaglio rinculeragrave di una energia E minus Eprime massima per θ = π Il valoremassimo di questrsquoenergia di rinculo

E minus E

1 + 2 Emc2

= E2Emc2

1 + 2Emc2

prende il nome ndash nel caso dello scattering Compton in cui la particella incidenteegrave un fotone e il bersaglio egrave un elettrone atomico ndash di picco Compton

Cosa cambia fra un fotone di energia E ed uno di energia Eprime Dalla mecca-nica quantistica

E = hν =hc

λ

cioegrave cambia la lunghezza drsquoonda del fotone

Eprime =hc

λprime=

hcλ

1 +hcλ

mc2 (1minus cos θprime)

1

λprime=

1 +hcλ

mc2 (1minus cos θprime)

λprime = λ

(1 +

hc

λmc2(1minus cos θprime)

)

λprime = λ+h

mc(1minus cos θprime) equiv λ+ λc(1minus cos θprime)

dove abbiamo definito la lunghezza drsquoonda Compton dellrsquoelettrone λc che rap-presenta la scala di lunghezza sotto la quale gli effetti della meccanica quantisticarelativistica divengono importanti

3 Lezione 3 (27 marzo 2020)

Esercizio 16 Trasformazione delle velocitagraveUn oggetto si muove di moto rettilineo uniforme con velocitagrave costante αc versoun secondo oggetto immobile A che velocitagrave dobbiamo muoverci noi lungo lastessa direzione per vedere entrambi gli oggetti muoversi con velocitagrave uguali eopposte

11

Soluzione dellrsquoesercizio 16Se indichiamo con lrsquoapice la velocitagrave nel nostro sistema di riferimento che

egrave in moto rispetto al sistema di riferimento del secondo oggetto la legge ditrasformazione delle velocitagrave ci dice che

vprime1 =v1 minus V

1minus v1cVc

=αcminus V1minus αVc

vprime2 =v2 minus V

1minus v2cVc

= minusV

Stiamo cercando V la nostra velocitagrave nel sistema di riferimento del secondooggetto tale che le velocitagrave dei due oggetti nel nostro sistema di riferimentosiano uguali e opposte

vprime1 = minusvprime2 = V

perciograve la richiesta egrave

αcminus V1minus αVc

= V

αcminus V minus V + αV 2

c= 0

(αc)V 2 + (minus2)V + (α)c = 0

V =1minusradic

1minus α2

αc

dove abbiamo scelto la soluzione dellrsquoequazione di secondo grado con V le c

Esercizio 17 Conseguenze della relativitagraveUna navicella spaziale in moto rettilineo uniforme con velocitagrave 05c in al-

lontanamento dalla Terra egrave in orbita verso Plutone che si trova a 75times 109 kmdi distanza dalla Terra Non appena raggiunto il pianeta la comandante inviaun segnale radio alla base a Houston per chiedere lrsquoautorizzazione allrsquoatter-raggio Quanto tempo impiega la richiesta a raggiungere la base secondo lacomandante E secondo i suoi colleghi a Houston

Soluzione dellrsquoesercizio 17Lrsquoonda radio viaggia a velocitagrave c secondo ogni sistema di riferimento Per

lrsquoosservatore lrsquoonda radio percorreragrave il tragitto Plutone-Terra in un tempo

tTerra =L

c=

75times 109 km

3times 108 msasymp 25 000 s

mentre per la comandante saragrave passato un tempo inferiore che dipende dal fat-tore γ dellrsquoastronave nel sistema di riferimento della Terra secondo la relazione

tcomandante =tTerra

γasymp 25 000 stimes

radic1minus 052 asymp 21 650 s

12

Esercizio 18 Energia cineticaHa piugrave energia un protone che viaggia a 09999999896c o un Boeing 747 al

decollo

Esercizio 19 Dilatazione dei tempiUna scienziata misura che un fascio di particelle selezionate con impulso di

10 GeVc si degrada dellrsquo84 dopo aver percorso 1 m Se la massa di questeparticelle egrave 498 MeVc2 qual egrave la loro vita media

Soluzione dellrsquoesercizio 19La distanza misurata dalla scienziata egrave chiaramente riferita al suo sistema

di riferimento quello del laboratorio ed egrave legata alla vita media delle particelledel fascio τ dalla relazione

Ldecay = 1 m = βγcτ =pc

E

E

mc2cτ =

p

mccτ

e dalla legge del decadimento se indichiamo con N0 il numero di particelleinizialmente presenti nel fascio e con N il numero di particelle misurato si haN

N0= 1minus084 = exp

(minus L

Ldecay

)= exp

(minus L

pmccτ

)rarr τ =

mcL

pc log(N0

N

) asymp 9times 10minus11 s

Esercizio 20 Dilatazione dei tempiVi trovate a dover studiare un fascio di particelle di cui conoscete lrsquoenergia

ndash 2 GeV ndash ma non la massa a quanto ne sapete potrebbero essere composti daelettroni (di massa 511 keVc2) o protoni (938 MeVc2) Avete a disposizionedue rivelatori identici in grado di registrare con precisione il tempo in cui unaparticella li attraversa Come potete utilizzarli per determinare se il vostrofascio contiene elettroni o protoni

Soluzione dellrsquoesercizio 20Disponendo i due rivelatori lungo la direzione del fascio a distanza ∆L

lrsquouno dallrsquoaltro si puograve misurare il tempo impiegato dalle particelle per passaredallrsquouno allrsquoaltro e quindi la loro massa infatti se ∆t egrave la distanza temporalefra i segnali dei due rivelatori

∆L = v∆t = βc∆t =pc2

E∆t

Questo principio egrave usato nei cosiddetti rivelatori di time of flight che sono usatiper discriminare diversi tipi di particelle una particella di tipo 1 e una di tipo2 infatti percorreranno la distanza ∆L in tempi diversi legati alle rispettivemasse dalla relazione

∆L1 = ∆L2

p1c2

E1∆t1 =

radicE2 minusm2

1c2c

E∆t1 =

radicE2 minusm2

2c2c

E∆t2

13

Esercizio 21 Energia di sogliaDue fisici delle particelle vogliono produrre il bosone Z una particella di

carica neutra e di massamZ = 91 GeVc2 e discutono come fare Alice proponedi far scontrare fasci di elettroni e positroni di energia identica che viaggianodunque con impulso spaziale uguale in modulo e direzione ma di verso oppostoproducendo Z tramite il processo

e+ + eminus rarr Z

mentre Bob preferisce scontrare un fascio di protoni su un bersaglio fisso diidrogeno tramite il processo

p+ prarr Z + p+ p

Chi dei due avragrave bisogno di fasci di particelle di energia piugrave alta La massa delprotone egrave di 9383 MeVc2 quella dellrsquoelettrone di 511 keVc2

Esercizio 22 Leggi di conservazioneUn fotone (particella di massa nulla) puograve decadere in un elettrone e in un

positrone (entrambi di massa 511 keVc2) tramite il processo

γ rarr e+ + eminus

Esercizio 23 Decadimento βminus

Quali sono lrsquoenergia minima e massima dellrsquoelettrone nel decadimento

nrarr p+ eminus + νe

se il neutrone decade da fermo La massa del neutrone egrave di 9396 MeVc2 quelladel protone di 9383 MeVc2 e quella dellrsquoelettrone di 511 keVc2 si assuma chelrsquoantineutrino elettronico νe abbia massa nulla

Esercizio 24 Leggi di conservazioneIl decadimento

prarr n+ e+ + νe

egrave permesso

4 Lezione 4 (3 aprile 2020)

Esercizio 25Usando il fatto che hc = 1973 MeVfm si dimostri che in un sistema di unitagrave dimisura in cui h = c = 1 vale

14

1 1 GeVminus2 = 0389 mb

2 1 m = 5068times 1015 GeVminus1

3 1 s = 15times 1024 GeVminus1

Ricordiamo che 1 b = 1times 10minus28 m2 e che

[hc] = [Jsms] = [E][L]

Soluzione dellrsquoesercizio 25Lrsquoidea egrave di capire per quale potenza di hc e c va moltiplicato il termine a

sinistra di ciascuna equazione per ottenere il termine di destraPer cui

bull [1 GeVminus2][hc]α = [E]minus2[E]α[L]α = [0389 mb] = [L]2 da cui segue α = 2

e 1 GeVminus2(hc)2 = 1973 MeVfm1 GeV =

(01973times 10minus15 GeVm

1 GeV

)2

= 0389 mb

bull [1 m][hc]α = [L][E]α[L]α = [5068times 1015 GeVminus1] = [E]minus1 da cui segueα = minus1 e 1 m(hc)minus1 = 1 m

1973 MeVfm = 1 m01973times 10minus15 GeVm

= 5068times 1015 GeVminus1

bull [1 s][hc]α[c]β = [T ][E]α[L]α[L]β [T ]minusβ = [T ][E]α[L]α+β [T ]minusβ = [15times 1024 GeVminus1] =[E]minus1 da cui segue α = minus1 β = 1 e 1 s(hc)minus1c = 1 s

1973 MeVfm299 792 458 ms =299 792 458 m

01973times 10minus15 GeVm= 15times 1024 GeVminus1

Esercizio 26 Massa invarianteTre protoni (mp = 938 MeVc2) hanno impulsi uguali in modulo (p =

3 GeVc) e che formano angoli di 120 lrsquouno con lrsquoaltro Qual egrave la massainvariante del sistema

Esercizio 27 Energia di sogliaSi consideri il processo

γ + prarr p+ π0

dove il fotone ha massa nulla il protone ha massa di 938 MeVc2 e il π0 hamassa di 135 MeVc2

1 Se il protone egrave a riposo qual egrave lrsquoenergia minima che deve avere il fotoneincidente percheacute la reazione abbia luogo

2 La stessa reazione puograve avvenire nellrsquouniverso in cui un protone dei raggicosmici di alta energia puograve collidere con uno dei fotoni della radiazionecosmica di fondo di energia dellrsquoordine di 1 meV Qual egrave in questo casolrsquoenergia minima che deve avere il protone percheacute la reazione abbia luogo

Esercizio 28 Conseguenze della relativitagraveUn misterioso supereroe pattuglia a velocitagrave molto elevata la periferia roma-

na Allrsquoincrocio con via di Tor Bella Monaca incontra un semaforo e ndash vedendolo

15

verde ndash attraversa senza rallentare Una pattuglia della polizia municipale loferma e lo multa per esser passato col rosso Assumendo sia il supereroe che ivigili siano nel giusto a che velocitagrave viaggiava il supereroe

Soluzione dellrsquoesercizio 28Per la polizia municipale il semaforo emette fotoni di energia

E0 = hν0 =hc

λrossoasymp hc

630 nm

mentre il supereoe vede fotoni di energia

E = hν =hc

λverdeasymp hc

490 nm

e dalle trasformazioni di Lorentz indicando con γ e β le variabili calcolateusando la velocitagrave del supereroe misurata dalla municipale possiamo scrivere

E = γ(E0minusβp0) = γ(E0minusβE0) =1minus βradic1minus β2

E0 =1minus βradic

(1 + β)(1minus β)E0 =

radic1minus βradic1 + β

E0

dove abbiamo usato il fatto che i fotoni hanno massa nulla Perciograve

E

E0=

hcλverdehc

λrosso

=λrosso

λverde=

radic1minus βradic1 + β

rarr v = βc = 025c

Esercizio 29 Conseguenze della relativitagraveI neutrini sono particelle di massa molto piccola e al momento ignota Uno

dei modi con cui egrave stato possibile dedurre un limite superiore al suo valore egravestata lrsquoosservazione nel 1987 di neutrini prodotti dalla supernova 1987A1 chesi trova a 168000 anni luce dalla Terra Sono stati osservati due segnali dineutrini che possono essere schematizzati come segue si egrave osservato prima unneutrino di 35 MeV di energia seguito a 9 s di distanza da un secondo segnaledi 13 MeV Si assuma che questo ritardo sia dovuto al fatto che la massa delneutrino non egrave nulla e si calcoli questrsquoultima

Esercizio 30 DecadimentoAnimali e piante assumono dallrsquoatmosfera diversi composti contenenti car-

bonio Il carbonio presente nellrsquoatmosfera egrave predominantemente 126 C ma sono

presenti piccole concentrazioni del suo isotopo 146 C (un atomo ogni 1012) che

decade con emissione di elettroni attraverso il processo2

146 C rarr14

7 N + eminus + νe

con un tempo di dimezzamento di 5700 anni1httpsenwikipediaorgwikiSN_1987ANeutrino_emissions2Di altro non si tratta che del decadimento βminus nrarr p+ e+ νe

16

1 Qual egrave la concentrazione di 146 C dopo 11400 anni

2 Animali e piante assumono in vita proporzioni fisse di 146 C e 12

6 C mentrealla loro morte la quantitagrave di 14

6 C inizia a diminuire Avete a portata dimano un relitto di legno per cui misurate una emissione di elettroni daldecadimento di 14

6 C pari al 61 di quella di un pezzo di legno vivodella stessa massa quanti anni ha il manufatto

Esercizio 31 Energia cinetica e trasformazioni di LorentzDue particelle identiche di massa m ed energia cinetica T collidono frontal-

mente Qual egrave la loro energia cinetica relativa (ossia lrsquoenergia cinetica di unaparticella misurata nel sistema di riferimento dellrsquoaltra particella)

Esercizio 32 Energia nel centro di massaLa reazione

πminusprarr K0Λ0

avviene con unrsquoenergia nel centro di massa diradics = 3 GeV La massa del πminus egrave

di 1396 MeVc2 la massa del protone egrave di 938 MeVc2 la massa del Kminus egrave di498 MeVc2 e quella della Λ0 di 11 GeVc2

1 Calcolare lrsquoimpulso di πminus e Λ0 nel sistema di riferimento del centro dimassa

2 Se il protone egrave a riposo il K puograve essere emesso allrsquoindietro nel sistemadi riferimento del laboratorio

Esercizio 33 DecadimentoIl mesone φ0 egrave una particella neutra3 di circa 1 GeVc2 di massa che puograve

decadere in una coppia di particelle

φ0 rarr K+ +Kminus

di massa identicamK = 494 MeVc2 Si assuma di produrre φ0 di impulso notoegrave possibile che uno dei due K sia prodotto a riposo nel sistema di riferimentodel laboratorio

Esercizio 34 DecadimentoUn fascio di anti-neutrini muonici νmicro si puograve generare selezionando pioni

o kaoni π+ e K+ e facendoli passare in un lungo tubo in cui egrave stato fattoil vuoto4 in modo che dopo un certo tragitto L una buona parte di loro saragrave

3Il collisore DAFNE ai Laboratori Nazionali di Frascati produce specificatamente parti-celle di questo tipo tramite il processo e+ + eminus rarr φ0 httpswwwyoutubecomwatchv=L5yB9gDGKms

4Una tecnica di questo tipo egrave stata usata per inviare ai Laboratori Nazionali del Gran Sassodei fasci di neutrini prodotti al CERN di Ginevra httpsvideoscernchrecord985892

17

decaduta tramite i processi

π+ rarr microminus + νmicro

K+ rarr microminus + νmicro

Se lrsquoimpulso di pioni e kaoni egrave di 200 GeVc e la loro vita media di 26 ns e12 ns rispettivamente

1 Quanto a lungo viaggiano nel laboratorio i due tipi di particelle2 Se L = 1000 m quale saragrave la frazione di pioni e kaoni che saragrave decaduta

alla fine del tubo3 Qual egrave lrsquoenergia massima dei neutrini che egrave possibile misurare nel sistema

di riferimento del laboratorio nei due casi

5 Lezione 5 (16 aprile 2020)

Esercizio 35 Energia cineticaTra le eccellenze con sede in territorio elvetico primeggiano indiscutibilmente lacioccolata al latte e il Large Hadron Collider (LHC) Questrsquoultimo egrave un collisoredi particelle situato al CERN di Ginevra che fa scontrare due fasci identici diprotoni di impulso uguale in modulo e direzione ma verso opposto I fasci sonocomposti da circa 2800 gruppi (pacchetti) di 1011 particelle ciascuno Sapendoche lrsquoenergia nel centro di massa della collisione fra due protoni egrave

radics = 13 TeV

quanta cioccolata al latte dovete mangiare per assumere un numero di caloriepari allrsquoenergia cinetica di ciascun fascio di protoni di LHC

Soluzione dellrsquoesercizio 35Poicheacute si tratta di un collider e le energie in gioco sono molto maggiori della

massa del protone lrsquoenergia di ciascun protone egrave diradics2 = 65 TeV e quella

totale di un fascio egrave

2800times 1011 times 65 middot 1012 times 16 middot 10minus19J asymp 300 MJ

Secondo Google 100 g di cioccolata al latte apportano circa 500 cal = 500 times4184 J asymp 2 MJ Lrsquoenergia di un fascio di LHC corrisponde quindi a quella dicirca 15 kg di cioccolata

Esercizio 36 Scattering RutherfordUn fascio di particelle α di 100 MeV di energia e 032 nA di corrente5 collide

contro un bersaglio fisso di alluminio spesso 1 cm Una sperimentatrice prendeun rivelatore di 1 cm times 1 cm di superficie e lo posiziona ad un angolo di 30

5Per una spiegazione breve su come (e percheacute) si misura la corrente di un fascio di particel-le vedi httpswwwlhc-closerestaking_a_closer_look_at_lhc0beam_current Unatrattazione piugrave completa egrave data ad esempio da httpscdscernchrecord1213275filesp141pdf

18

rispetto al fascio di particelle a 1 m di distanza dal bersaglio Quante particelleα incideranno sul rivelatore ogni secondo

Soluzione dellrsquoesercizio 36Lrsquoalluminio ha una densitagrave di 27 gcm3 numero atomico 13 e massa atomica

27 uPoicheacute le particelle α sono nuclei di elio hanno carica 2e e la corrente di

032 nA corrisponde a un miliardo di particelle incidenti al secondo

dNidt

=032 nCs

2times 16times 10minus19 C= 1times 109 sminus1

Il rivelatore vede un angolo solido di

∆Ω equiv superficieraggio

2

=1 cm2

(1 m)2= 1times 10minus4 sr

Si tratta di uno scattering alla Rutherford per cui la sezione drsquourto per unitagravedi angolo solido rilevata ad un certo angolo θ vale

dΩ=

(zαzAle

2

4πε04E

1

sin2(θ2)

)2

pari a

dΩasymp(

2times 13times 4times etimes 16times 10minus19 C

4π times 89times 10minus12 Fmtimes 4times 100times 106 eV

1

sin2(π180 times 302)

)2

asymp 2times 10minus30 m2sr = 20 mbsr

(1)

e il numero di particelle visto dal rivelatore vale se indichiamo con nAl = ρAlNAAAl

la densitagrave numero di atomi di alluminio e con d lo spessore del rivelatore

dNrivelate

dt= ∆Ω

dΩnAld

dNi

dt

asymp 1times 10minus4 srtimes 2times 10minus30 m2srtimes 1times 104 cm2m2 times 27 gcm3 6times 1023 molminus1

27 gmol

= 120 Hz

Esercizio 37 Sezione drsquourtoUn bersaglio di idrogeno liquido di densitagrave ρ = 0071 gcm3 e volume V =

125 cm3 egrave bombardato da un fascio monoenergetico di pioni negativi con unflusso φ = 2times 107 mminus2sminus1 e si osserva la reazione

πminus + prarr π0 + n

rivelando i fotoni del decadimento π0 rarr γγ che avviene nel 988 dei casi Sela sezione drsquourto di quellrsquointerazione egrave σ = 40 mb quanti fotoni sono emessi ognisecondo

19

Esercizio 38 Sezione drsquourtoLa sezione drsquourto dellrsquoeffetto fotoelettrico per raggi X di 10 keV in carbonio

egrave 40 b per atomo Data una lastra di carbonio di 4 mm di spessore si calcoli1 il numero di bersagli per unitagrave di volume2 il coefficiente di assorbimento per effetto fotoelettrico dei raggi X di tale

energia3 la probabilitagrave che un raggio X incidente sulla lastra produca un elettrone

per effetto fotoelettrico

Esercizio 39 Sezioni drsquourtoUn bersaglio drsquooro di densitagrave superficiale ρS = 097 mgcm2 e superficie

SB = 1 cm2 viene colpito da un fascio di particelle α la cui sezione trasversaegrave contenuta completamente nellrsquoarea del bersaglio Sul bersaglio impattano37times 104 αs La sezione drsquourto di diffusione elastica ad un certo angolo θ valedσ

ddΩ = 1 bsr Calcolare1 la densitagrave di atomi del bersaglio per unitagrave di superficie2 il numero di particelle α rivelate in unrsquoora da un rivelatore di superficie

SR = 2 cm2 posto allrsquoangolo θ e a distanza DR = 01 m dal bersaglio3 Se il fascio di particelle viene sostituito da una sorgente radioattiva di

particelle α con distribuzione isotropa su tutto lrsquoangolo solido che vieneposta lungo la stessa linea del fascio a distanza DB = 20 cm dal bersaglioquanto tempo egrave necessario per rivelare con lo stesso rivelatore lo stessonumero di particelle calcolato sopra a paritagrave di sezione drsquourto

Esercizio 40 CinematicaUn fascio di positroni di 100 MeV di energia annichila su una targhetta fissa

di elettroni producendo due fotoni tramite il processo

e+ + eminus rarr γ + γ

Se uno dei due fotoni egrave emesso a 30 nel sistema del centro di massa1 quanto vale lrsquoenergia dei due fotoni nel sistema del centro di massa2 che energia e direzione avranno nel sistema del laboratorio

Esercizio 41 Sezione drsquourtoGli antineutrini νe prodotti da un reattore nucleare con potenza P = 16 GW

attraversano un bersaglio da 2000 l drsquoacqua posto a 50 m di distanza dal reat-tore

1 Supponendo che per ogni fissione sia prodotta unrsquoenergia termica di200 MeV e vengano emessi 6 antineutrini calcolare il numero medio direazioni

νe + prarr e+ + n

prodotte al giorno se la sezione drsquourto egrave σ = 1times 10minus43 cm2

20

2 Qual egrave la minima energia del neutrino per cui la reazione egrave permessa

6 Lezione 7 (8 maggio 2020)

Esercizio 42Un protone interagisce con un bersaglio producendo un pione di energia E =300 MeV Con un tracciatore posto a distanza d = 6 cm dal bersaglio egrave possi-bile rivelare la traiettoria del pione e risalire al punto di produzione del pionesul bersaglio Determinare lrsquoerrore sulla misura della posizione di tale puntocausato dalla presenza di un piano di alluminio di spessore L = 05 mm postoimmediatamente davanti al tracciatore (quindi a distanza d dal bersaglio) nellrsquoi-potesi che gli angoli delle tracce rispetto alla normale alle superfici del bersaglioe del piano di alluminio siano piccoli [mπ = 1396 MeV Al (Z = 13 A = 27ρ = 27 gcm3)]

Soluzione dellrsquoesercizio 42La deviazione standard dellrsquoangolo di diffusione coulombiana multipla vale

radic〈θ2〉 = 21 MeV

z

βc|p|

radicx

X0= 21 MeV

z

βc|p|

radicLρ

X0

dove p =radicE2 minusm2

π = 266 MeV β = pE = 0885 z = 1 Per lrsquoalluminioabbiamo

X0 = 24 gcm2 rarr X0ρ = 89 cm

per cui radic〈θ2〉 asymp 00067

e lo spostamento medio corrispondente rispetto alla posizione nominale egrave

δs = tan(00067)times 6 cm = 04 mm

Esercizio 43Un fascio misto di elettroni e antiprotoni passa attraverso una regione con

B = 2 T e dopo aver curvato per R = 3 m raggiunge una regione senza campomagnetico

1 Qual egrave lrsquoimpulso delle particelle selezionate2 Come discriminereste fra i due tipi di particelle3 Se usaste due scintillatori posti a 15 m di distanza quale risoluzione

temporale sarebbe necessaria per discriminare i due tipi di particelle4 Se i due scintillatori fossero spessi 2 cm e avessero una lunghezza di ra-

diazione X0 = 40 cm quanta energia perderebbero elettroni e protoninellrsquoattraversarli Si assuma una perdita di energia per ionizzazione di2 MeVcm e 25 MeVcm per protoni ed elettroni rispettivamente

21

5 Che indice di rifrazione dovrebbe avere un rivelatore a luce Cherenkovper discriminare elettroni e protoni

Soluzione dellrsquoesercizio 43Dalla definizione di forza

F =dpdt

= qvtimesB

|F| = mv2

R

per cui se il campo magnetico egrave ortogonale alla traiettoria

pc = qcBR

che poicheacute c = 03 mns possiamo esprimere in maniera piugrave conveniente come

pc[GeV] = 03B[T]R[m]

Ne segue che nel nostro esercizio (e tornando a c = 1)

p = 18 GeV

Trascurando le perdite di energia e ricordando che me mp i tempi divolo di elettroni e protoni valgono rispettivamente

te =L

βecasymp L

casymp 50 ns

etp =

L

βpc=

Lpradic

p2+m2p

casymp 56 ns

per cui serve una risoluzione dellrsquoordine del ns per discriminare elettroni e muonidal tempo di volo

Se gli scintillatori sono spessi 2 cm i protoni perderanno in ciascuno di essi2 cm times 2 MeVcm = 4 MeV per ionizzazione Gli elettroni perderanno invecenel primo rivelatore 5 MeV per ionizzazione e (me asymp 0)

18 GeV times(

1minus exp

(minus 2 cm

40 cm

))asymp 88 MeV

cioegrave dopo il primo rivelatore avranno 18 GeVminus93 MeV di energia e nel secondone perderanno altri

5 MeV + (18 GeV minus 93 MeV)

(1minus exp

(minus 2 cm

40 cm

))asymp 88 MeV

Lrsquoangolo di emissione di luce Cherenkov egrave dato da

cos θc =1

βnle 1

22

e poicheacute βe asymp 1 βp asymp 0887 per avere luce Cherenkov solo al passaggio deglielettroni (e non dei protoni) egrave necessario che

1 lt n lt 113

Esercizio 44Come misurereste la massa di una particella carica E quella di una parti-

cella neutra

Esercizio 45Un muone di energia E = 400 GeV penetra verticalmente nel mare Attra-

verso quale processo puograve essere rivelato A quale profonditagrave arriva prima didecadere

Esercizio 46Avete a disposizione dei tubi fotomoltiplicatori sensibili a lunghezze drsquoonda

fra 300 nm e 500 nm e volete rivelare la luce Cherenkov prodotta dal passaggio diun elettrone di 1 MeV di energia in un metro drsquoacqua Quanti fotoni vi aspettateche vengano prodotti Confrontate con il numero di elettroni di ionizzazioneche produrrebbe una particella α di 5 keV nello stesso rivelatore

Esercizio 47Determinare quali sono i processi piugrave probabili (cioegrave quelli di sezione drsquourto

piugrave alta) nellrsquointerazione fra1 fotoni di 1 MeV e atomi di alluminio2 fotoni di 100 keV e H23 fotoni di 100 keV e atomi di ferro4 fotoni di 10 MeV e atomi di carbonio5 fotoni di 10 MeV e atomi di piombo

Esercizio 48Volete misurare lrsquoimpulso di una particella carica che attraversa un campo

magnetico B ortogonale alla sua traiettoria Avete a disposizione tre rivelatori diposizione della stessa risoluzione spaziale δx come li disponete Che risoluzionein impulso vi aspettate di ottenere

23

  • Lezione 1 (13 marzo 2020)
  • Lezione 2 (20 marzo 2020)
  • Lezione 3 (27 marzo 2020)
  • Lezione 4 (3 aprile 2020)
  • Lezione 5 (16 aprile 2020)
  • Lezione 7 (8 maggio 2020)
Page 3: Esercizi per casa (risolti)ippolitv/pdf/fns1_2020_solutions.pdfEsercizi per casa (risolti) Valerio Ippolito 13 maggio 2020 1 Lezione 1 (13 marzo 2020) Esercizio1 Dilatazionedeitempi

scomporre lrsquoasta in lunghezza longitudinale e trasversale che nel sistema diriferimento in cui lrsquoasta egrave ferma valgono

L = L0 cos θ Lperp = L0 sin θ

applicare opportunamente la contrazione di Lorentz nel passaggio al sistema diriferimento del laboratorio (apice prime)

Lprime =1

γL0 cos θ Lprimeperp = L0 sin θ

ottenendo quindi la lunghezza dellrsquoasta misurata nel laboratorio

Lprime =radicLprime

2 + Lprimeperp2 = L0

radicsin2 θ +

1

γ2cos2 θ

= L0

radicsin2 θ + (1minus β2) cos2 θ = L0

radic1minus v2

c2cos2 θ

che egrave ovviamente minore di L0 e lrsquoangolo fra lrsquoasta e lrsquoasse xprime (lrsquoasse orizzontalenel sistema del laboratorio)

θprime = arctan

(LprimeperpLprime

)= arctan

(γy

x

)= arctan (γ tan θ)

Esercizio 5 Trasformazione delle velocitagraveUn osservatore in quiete sulla Terra vede due astronavi avvicinarsi lrsquouna

allrsquoaltra lungo la stessa direzione alla stessa velocitagrave La loro velocitagrave relativaegrave 07c Determinare la velocitagrave delle due astronavi misurata dallrsquoosservatore aTerra

Soluzione dellrsquoesercizio 5Il testo intende che ciascuna delle due astronavi vede lrsquoaltra viaggiare a una

velocitagrave di 07c In altri termini

v(B)A = v

(A)B = 07c equiv a

dove il pedice indica lrsquoastronave di cui misuriamo la velocitagrave e lrsquoapice indica ilsistema di riferimento in cui la velocitagrave egrave misurata Chiamiamo questa quantitagravea per semplificare i conti

Lrsquoincognita del problema egrave la velocitagrave di ciascuna delle due astronavi che egraveuguale in modulo per ipotesi nel sistema di riferimento del laboratorio ndash ovvero

v(lab)A = v

(lab)B equiv x

Si noti che ovviamentev(lab)A = minusv(lab)

B

3

in quanto le due astronavi si avvicinano lrsquouna allrsquoaltraEsprimiamo quindi la velocitagrave di B nel sistema di riferimento di A che egrave

nota in funzione della velocitagrave di B nel sistema di riferimento del laboratorioche egrave ignota tenendo opportunamente conto dei segni

v(A)B =

v(lab)B minus v(lab)

A

1minus v(lab)A

c

v(lab)B

c

=2x

1 + x2

c

= a

da cui

x =2plusmn

radic4minus 4a

2

c2

2ac=c

a

(1plusmn

radic1minus

(ac

)2)asymp 040c

dove abbiamo scartato la soluzione con x gt c Viste dal sistema di riferimentodel laboratorio le astronavi viaggiano quindi a +040c e minus040c

Esercizio 6 Leggi di trasformazioneIn relativitagrave speciale come si trasforma il volume E la densitagrave

Soluzione dellrsquoesercizio 6Il volume di un corpo in moto egrave il prodotto di tre componenti due trasverse e

una parallela al boost di Lorentz Perciograve il volume misurato a terra egrave V = V primeγLe densitagrave invece aumentano di un fattore γ

Esercizio 7 Intervallo invarianteSi considerino anzitutto due punti A e B nello spazio euclideo (rappresentato

per semplicitagrave in due dimensioni) Quale dei due cammini in figura egrave piugrave breve

x

y

A

B

x

y

A

B

Si consideri ora lo spazio di Minkowski (pseudo-euclideo) con lrsquousuale defi-nizione di distanza ds2 = c2dt2 minus dx2 minus dy2 minus dz2 Quale dei due tragitti egrave piugravebreve Si assuma che tutti i segmenti di curva che congiungono A e B siano ditipo tempo

x

ct

A

B

x

ct

A

B

4

Soluzione dellrsquoesercizio 7Nello spazio euclideo la linea retta tra due punti egrave quella che minimizza

la distanza dx Nello spazio di Minkowski a causa del segno negativo allecomponenti spaziali di ds2 la distanza ds egrave invece massima per il cammino inlinea retta

2 Lezione 2 (20 marzo 2020)

Esercizio 8 QuadrivelocitagraveQual egrave la quadrivelocitagrave di una particella che si muove di moto rettilineo uni-forme lungo lrsquoasse x con velocitagrave v = 3

5cSoluzione dellrsquoesercizio 8

La quadrivelocitagrave di una particella egrave rappresentata dal quadrivettore con-travariante

ηmicro equiv dxmicro

dτ=

(c

dt

dx

dy

dz

)

dove τ egrave il tempo proprio della particella ovvero il tempo misurato nel suo siste-ma di riferimento Il problema ci fornisce la velocitagrave nel sistema di riferimentodel laboratorio perciograve ci conviene esprimere il tempo proprio in funzione deltempo misurato nel laboratorio t = γτ

ηmicro =

(γc γ

dx

dt γ

dy

dt γ

dz

dt

)= γ(c v 0 0) =

1radic1minus

(35

)2(c

3

5c

)=c

4(5 3 0 0)

La norma della quadrivelocitagrave vale

ηmicroηmicro = gmicroνηmicroην =

c2

42(52 minus 32 minus 0minus 0) = c2

che egrave manifestamente invariante

Esercizio 9 Decadimento e dilatazione dei tempiMetagrave dei muoni di un fascio composto da muoni di energia fissata sopravvive

dopo aver viaggiato l = 600 m nel sistema di riferimento del laboratorio Qualegrave la velocitagrave dei muoni

Soluzione dellrsquoesercizio 9Da

N

N0= exp

(minus tτ

)= exp

(minus vtvτ

)= exp

(minus l

βcγτ0

)=

1

2

segue che

minus log1

2=

l

βγcτ0

βγ =βradic

1minus β2= minus l

log 12cτ0

equiv λ

5

ed elevando al quadrato

β =

radicλ2

1 + λ2asymp 080

Esercizio 10 Calcolo tensorialeUn tensore le cui componenti contravarianti si indicano come amicroν egrave la ge-

neralizzazione a due indici di un quadrivettore Mentre un quadrivettore ha 4componenti un tensore a due indici ne ha 4times 4 = 16 Se nel passaggio da unsistema di riferimento O in quiete a uno Oprime in moto lungo lrsquoasse x con velocitagraveV = βc le coordinate di un quadrivettore trasformano secondo la legge

xmicroprime = Λmicroνxν

dove le Λmicroν sono le componenti della matrice

Λ =

γ minusβγ 0 0minusβγ γ 0 0

0 0 1 00 0 0 1

analogamente un tensore trasforma secondo

amicroν prime = ΛmicroρΛνσaρσ

Analogamente a quanto accade per i quadrivettori le coordinate contravariantidi un tensore sono legate a quelle covarianti dalla relazione

amicroν = gmicroρgνσaρσ

Un tensore si dice simmetrico se le sue componenti sono uguali sotto scambiodegli indici (amicroν = aνmicro) e antisimmetrico se sono uguali ma di segno opposto(amicroν = minusaνmicro)

1 Quanti elementi indipendenti ci sono in un tensore simmetrico (siconsiderano dipendenti ad esempio a12 e a21 = a12)

2 E in un tensore antisimmetrico3 Simmetria e antisimmetria sono caratteristiche che si mantengono sotto

trasformazione di Lorentz4 Se un tensore amicroν egrave simmetrico lo egrave anche la sua versione covariante amicroν

E se egrave antisimmetrico5 Se amicroν egrave un tensore simmetrico e bmicroν un tensore antisimmetrico quanto

vale amicroνbmicroν6 Scomporre un tensore generico amicroν nella somma di due tensori uno

simmetrico e lrsquoaltro antisimmetricoSoluzione dellrsquoesercizio 10

6

1 Un tensore simmetrico ha 10 componenti indipendenti (a11 a12 a13 a14 a22 a23 a24 a33 a34 a44)

2 Un tensore antisimmetrico ha 6 componenti indipendenti (a12 a13 a14 a23 a24 a34)

3 Se un tensore egrave simmetrico significa che

amicroν = aνmicro

La versione trasformata del termine a sinistra egrave

amicroν prime = ΛmicroρΛνσaρσ

mentre quella del termine di destra egrave

aνmicroprime = ΛνρΛmicroσaρσ

ma applicando la definizione di tensore simmetrico nel riferimento di par-tenza (coordinate senza apici) e applicando la proprietagrave commutativa aitermini della matrice di Lorentz si ha

aνmicroprime = ΛνρΛmicroσaρσ = ΛνρΛmicroσa

σρ = ΛmicroσΛνρaσρ = ΛmicroρΛνσa

ρσ = amicroν prime

dove abbiamo ndash nellrsquoultimo passaggio ndash usato il fatto che sia ρ che σ sonoindici muti Ne concludiamo che un tensore simmetrico rimane tale dopotrasformazione di Lorentz

Se il tensore egrave antisimmetrico analogamente significa che

amicroν = minusaνmicro

La versione trasformata del termine a sinistra egrave sempre

amicroν prime = ΛmicroρΛνσaρσ

Applicando ancora una volta la definizione di tensore antisimmetrico nelriferimento di partenza e la proprietagrave commutativa si ha

aνmicroprime = ΛνρΛmicroσaρσ = ΛνρΛmicroσ (minusaσρ) = minusΛmicroσΛνρa

σρ = minusΛmicroρΛνσaρσ = minusamicroν prime

cioegrave anche lrsquoantisimmetria egrave mantenuta dopo trasformazione di Lorentz

4 Per il tensore simmetrico

amicroν = gmicroρgνσaρσ = gmicroρgνσa

σρ = gνσgmicroρaσρ = amicroν

Nel caso antisimmetrico

amicroν = gmicroρgνσaρσ = gmicroρgνσ (minusaσρ) = minusgνσgmicroρaσρ = minusamicroν

7

5 Vale

amicroνbmicroν = aνmicrob

νmicro = amicroν (minusbmicroν) = minus (amicroνbmicroν)rarr amicroνbmicroν = 0

dove al primo passaggio abbiamo semplicemente rinominato gli indici mu-ti e al secondo abbiamo usato le proprietagrave di simmetriaantisimmetriadei tensori di partenza Ne segue dunque che il prodotto fra un tensoresimmetrico e uno antisimmetrico egrave zero

Suggerimento almeno una volta nella vita conviene esplicitare la som-ma implicita nella notazione di Einstein per comprendere davvero questoformalismo cosigrave compatto

6 Basta costruire due somme che per costruzione allo scambio di indicisono lrsquouna simmetrica

amicroν equiv 1

2(tmicroν + tνmicro)

e lrsquoaltra antisimmetrica cioegrave

amicroν equiv 1

2(tmicroν minus tνmicro)

Egrave evidente che questi due tensori sono simmetriciantisimmetrici sottoscambio degli indici

Esercizio 11 Relazione fra forza e accelerazioneIn relativitagrave speciale forza e accelerazione sono in generale proporzionali fra

loro Usare la definizione di forza F = dpdt

Suggerimento scomporre lrsquoaccelerazione nella somma di un termine paral-lelo e un termine ortogonale alla direzione del moto (cioegrave alla velocitagrave)

Soluzione dellrsquoesercizio 11Scomponiamo lrsquoaccelerazione a lungo le direzioni ortogonale e parallela al

moto della particella (cioegrave a v)

a equiv aperp + a

Usando il fatto che

dt=

dv

dv

dt=

d

[(1minus v2

c2

)minus 12

]dt

dv

dt= minus1

2

(minus2v

c2

)(1minus v2

c2

)minus 32 dv

dt=

v

c2γ3 dv

dt

e che dalla definizione di prodotto scalare fra vettori

vdv

dt= v middot a

8

otteniamo che

F =dpdt

=d(mγv)

dt= mγ

dvdt

+mdγ

dtv

= mγa +mγ3 (v middot a) vc2

= mγ(aperp + a) +mγ3(v middot aperp + v middot a)v

= mγaperp +mγ3

(1

γ2+v2

c2

)a

= mγaperp +mγ3

(1minus v2

c2+v2

c2

)a

= mγaperp +mγ3a

Si noti come in relativitagrave speciale la forza non egrave in generale proporzionaleallrsquoaccelerazione

Esercizio 12 Classificazione dei quadrivettoriIl quadrimpulso egrave un quadrivettore di tipo spazio tempo o luce

Soluzione dellrsquoesercizio 12Poicheacute P 2 = E2

c2 minus p2 = m2 ge 0 saragrave di tipo tempo per particelle massive (e

di tipo luce per particelle senza massa)

Esercizio 13 Energia cineticaQuanto lavoro bisogna compiere per aumentare la velocitagrave di un elettrone

(m = 511 keVc2) dalla posizione di riposo a1 050c2 0990c3 09990c

Soluzione dellrsquoesercizio 13A questi elettroni dovremo dare una certa energia cinetica T in modo da

far passare lrsquoenergia totale da quella a riposo (γ = 0) cioegrave

Ei = mc2

aEf = T +mc2

Dalla relazioneEf = mγc2

segue

T = Ef minusmc2 = m(γ minus 1)c2 = m

1radic1minus v2

c2

minus 1

c2

per cui nei tre casi indicati servono rispettivamente 79 keV 31 MeV e 109 MeV

9

Esercizio 14 Energia di sogliaSupponiamo di far scontrare un fascio di protoni con un protone a riposo

Qual egrave lrsquoenergia minima che devono avere i protoni del fascio percheacute la reazione

p+ prarr p+ p+ p+ p

sia permessa (La massa del protone egrave pari a quella dellrsquoantiprotone p e vale938 MeVc2)

Esercizio 15 Diffusione elasticaChiamiamo elastico un urto (ldquoscatteringrdquo) in cui le particelle dello stato ini-

ziale e dello stato finale sono le stesse Si consideri un urto elastico fra unaparticella di massa nulla e una particella di massa m (bersaglio) che si trovaa riposo nel sistema di riferimento del laboratorio qual egrave la massima energiatrasferita dalla particella incidente al bersaglio Suggerimento si lavori nelsistema di riferimento del laboratorio e si espliciti il prodotto scalare fra gliimpulsi spaziali della particella di massa nulla prima e dopo lrsquourto in funzionedellrsquoangolo sempre nel sistema di riferimento del laboratorio fra la direzioneiniziale e finale della particella incidente

Se la particella incidente egrave un fotone e il bersaglio egrave un elettrone atomico ariposo di quanto varia la lunghezza drsquoonda del fotone fra prima e dopo lrsquourto

Soluzione dellrsquoesercizio 15Per scattering elastico intendiamo un processo in cui le particelle dello stato

iniziale sono le stesse di quelle dello stato finaleDenotiamo con k e P i quadrimpulsi della particella incidente e del bersa-

glio prima dellrsquourto e indichiamo con lrsquoapice le stesse quantitagrave dopo lrsquourto ilproblema ci dice che

k = (Ek)

kprime = (Eprimekrsquo)P = (mc0)

Partiamo dalla conservazione del quadrimpulso durante lrsquourto isoliamo la quan-titagrave che non misuriamo direttamente ndash cioegrave il quadrimpulso del bersaglio dopolrsquourto P prime ndash ed eleviamo al quadrato

k + P = kprime + P prime

P prime = k + P minus kprimem2c2 = 0 +m2c2 + 0 + 2Emminus 2(EEprime minus k middot krsquo)minus 2mEprime

e se indichiamo con θprime lrsquoangolo ndash nel riferimento del laboratorio ndash fra la direzioneiniziale e finale della particella incidente e usiamo il fatto che |k|c = E e |krsquo|c =

10

Eprime

0 = 2mc2(E minus Eprime)minus 2(EEprime minus EEprime cos θprime)

mc2(Eprime minus E) = minusEEprime(1minus cos θprime)

Eprime(mc2 + E(1minus cos θprime)) = mc2E

Eprime =E

1 + Emc2 (1minus cos θprime)

Il bersaglio rinculeragrave di una energia E minus Eprime massima per θ = π Il valoremassimo di questrsquoenergia di rinculo

E minus E

1 + 2 Emc2

= E2Emc2

1 + 2Emc2

prende il nome ndash nel caso dello scattering Compton in cui la particella incidenteegrave un fotone e il bersaglio egrave un elettrone atomico ndash di picco Compton

Cosa cambia fra un fotone di energia E ed uno di energia Eprime Dalla mecca-nica quantistica

E = hν =hc

λ

cioegrave cambia la lunghezza drsquoonda del fotone

Eprime =hc

λprime=

hcλ

1 +hcλ

mc2 (1minus cos θprime)

1

λprime=

1 +hcλ

mc2 (1minus cos θprime)

λprime = λ

(1 +

hc

λmc2(1minus cos θprime)

)

λprime = λ+h

mc(1minus cos θprime) equiv λ+ λc(1minus cos θprime)

dove abbiamo definito la lunghezza drsquoonda Compton dellrsquoelettrone λc che rap-presenta la scala di lunghezza sotto la quale gli effetti della meccanica quantisticarelativistica divengono importanti

3 Lezione 3 (27 marzo 2020)

Esercizio 16 Trasformazione delle velocitagraveUn oggetto si muove di moto rettilineo uniforme con velocitagrave costante αc versoun secondo oggetto immobile A che velocitagrave dobbiamo muoverci noi lungo lastessa direzione per vedere entrambi gli oggetti muoversi con velocitagrave uguali eopposte

11

Soluzione dellrsquoesercizio 16Se indichiamo con lrsquoapice la velocitagrave nel nostro sistema di riferimento che

egrave in moto rispetto al sistema di riferimento del secondo oggetto la legge ditrasformazione delle velocitagrave ci dice che

vprime1 =v1 minus V

1minus v1cVc

=αcminus V1minus αVc

vprime2 =v2 minus V

1minus v2cVc

= minusV

Stiamo cercando V la nostra velocitagrave nel sistema di riferimento del secondooggetto tale che le velocitagrave dei due oggetti nel nostro sistema di riferimentosiano uguali e opposte

vprime1 = minusvprime2 = V

perciograve la richiesta egrave

αcminus V1minus αVc

= V

αcminus V minus V + αV 2

c= 0

(αc)V 2 + (minus2)V + (α)c = 0

V =1minusradic

1minus α2

αc

dove abbiamo scelto la soluzione dellrsquoequazione di secondo grado con V le c

Esercizio 17 Conseguenze della relativitagraveUna navicella spaziale in moto rettilineo uniforme con velocitagrave 05c in al-

lontanamento dalla Terra egrave in orbita verso Plutone che si trova a 75times 109 kmdi distanza dalla Terra Non appena raggiunto il pianeta la comandante inviaun segnale radio alla base a Houston per chiedere lrsquoautorizzazione allrsquoatter-raggio Quanto tempo impiega la richiesta a raggiungere la base secondo lacomandante E secondo i suoi colleghi a Houston

Soluzione dellrsquoesercizio 17Lrsquoonda radio viaggia a velocitagrave c secondo ogni sistema di riferimento Per

lrsquoosservatore lrsquoonda radio percorreragrave il tragitto Plutone-Terra in un tempo

tTerra =L

c=

75times 109 km

3times 108 msasymp 25 000 s

mentre per la comandante saragrave passato un tempo inferiore che dipende dal fat-tore γ dellrsquoastronave nel sistema di riferimento della Terra secondo la relazione

tcomandante =tTerra

γasymp 25 000 stimes

radic1minus 052 asymp 21 650 s

12

Esercizio 18 Energia cineticaHa piugrave energia un protone che viaggia a 09999999896c o un Boeing 747 al

decollo

Esercizio 19 Dilatazione dei tempiUna scienziata misura che un fascio di particelle selezionate con impulso di

10 GeVc si degrada dellrsquo84 dopo aver percorso 1 m Se la massa di questeparticelle egrave 498 MeVc2 qual egrave la loro vita media

Soluzione dellrsquoesercizio 19La distanza misurata dalla scienziata egrave chiaramente riferita al suo sistema

di riferimento quello del laboratorio ed egrave legata alla vita media delle particelledel fascio τ dalla relazione

Ldecay = 1 m = βγcτ =pc

E

E

mc2cτ =

p

mccτ

e dalla legge del decadimento se indichiamo con N0 il numero di particelleinizialmente presenti nel fascio e con N il numero di particelle misurato si haN

N0= 1minus084 = exp

(minus L

Ldecay

)= exp

(minus L

pmccτ

)rarr τ =

mcL

pc log(N0

N

) asymp 9times 10minus11 s

Esercizio 20 Dilatazione dei tempiVi trovate a dover studiare un fascio di particelle di cui conoscete lrsquoenergia

ndash 2 GeV ndash ma non la massa a quanto ne sapete potrebbero essere composti daelettroni (di massa 511 keVc2) o protoni (938 MeVc2) Avete a disposizionedue rivelatori identici in grado di registrare con precisione il tempo in cui unaparticella li attraversa Come potete utilizzarli per determinare se il vostrofascio contiene elettroni o protoni

Soluzione dellrsquoesercizio 20Disponendo i due rivelatori lungo la direzione del fascio a distanza ∆L

lrsquouno dallrsquoaltro si puograve misurare il tempo impiegato dalle particelle per passaredallrsquouno allrsquoaltro e quindi la loro massa infatti se ∆t egrave la distanza temporalefra i segnali dei due rivelatori

∆L = v∆t = βc∆t =pc2

E∆t

Questo principio egrave usato nei cosiddetti rivelatori di time of flight che sono usatiper discriminare diversi tipi di particelle una particella di tipo 1 e una di tipo2 infatti percorreranno la distanza ∆L in tempi diversi legati alle rispettivemasse dalla relazione

∆L1 = ∆L2

p1c2

E1∆t1 =

radicE2 minusm2

1c2c

E∆t1 =

radicE2 minusm2

2c2c

E∆t2

13

Esercizio 21 Energia di sogliaDue fisici delle particelle vogliono produrre il bosone Z una particella di

carica neutra e di massamZ = 91 GeVc2 e discutono come fare Alice proponedi far scontrare fasci di elettroni e positroni di energia identica che viaggianodunque con impulso spaziale uguale in modulo e direzione ma di verso oppostoproducendo Z tramite il processo

e+ + eminus rarr Z

mentre Bob preferisce scontrare un fascio di protoni su un bersaglio fisso diidrogeno tramite il processo

p+ prarr Z + p+ p

Chi dei due avragrave bisogno di fasci di particelle di energia piugrave alta La massa delprotone egrave di 9383 MeVc2 quella dellrsquoelettrone di 511 keVc2

Esercizio 22 Leggi di conservazioneUn fotone (particella di massa nulla) puograve decadere in un elettrone e in un

positrone (entrambi di massa 511 keVc2) tramite il processo

γ rarr e+ + eminus

Esercizio 23 Decadimento βminus

Quali sono lrsquoenergia minima e massima dellrsquoelettrone nel decadimento

nrarr p+ eminus + νe

se il neutrone decade da fermo La massa del neutrone egrave di 9396 MeVc2 quelladel protone di 9383 MeVc2 e quella dellrsquoelettrone di 511 keVc2 si assuma chelrsquoantineutrino elettronico νe abbia massa nulla

Esercizio 24 Leggi di conservazioneIl decadimento

prarr n+ e+ + νe

egrave permesso

4 Lezione 4 (3 aprile 2020)

Esercizio 25Usando il fatto che hc = 1973 MeVfm si dimostri che in un sistema di unitagrave dimisura in cui h = c = 1 vale

14

1 1 GeVminus2 = 0389 mb

2 1 m = 5068times 1015 GeVminus1

3 1 s = 15times 1024 GeVminus1

Ricordiamo che 1 b = 1times 10minus28 m2 e che

[hc] = [Jsms] = [E][L]

Soluzione dellrsquoesercizio 25Lrsquoidea egrave di capire per quale potenza di hc e c va moltiplicato il termine a

sinistra di ciascuna equazione per ottenere il termine di destraPer cui

bull [1 GeVminus2][hc]α = [E]minus2[E]α[L]α = [0389 mb] = [L]2 da cui segue α = 2

e 1 GeVminus2(hc)2 = 1973 MeVfm1 GeV =

(01973times 10minus15 GeVm

1 GeV

)2

= 0389 mb

bull [1 m][hc]α = [L][E]α[L]α = [5068times 1015 GeVminus1] = [E]minus1 da cui segueα = minus1 e 1 m(hc)minus1 = 1 m

1973 MeVfm = 1 m01973times 10minus15 GeVm

= 5068times 1015 GeVminus1

bull [1 s][hc]α[c]β = [T ][E]α[L]α[L]β [T ]minusβ = [T ][E]α[L]α+β [T ]minusβ = [15times 1024 GeVminus1] =[E]minus1 da cui segue α = minus1 β = 1 e 1 s(hc)minus1c = 1 s

1973 MeVfm299 792 458 ms =299 792 458 m

01973times 10minus15 GeVm= 15times 1024 GeVminus1

Esercizio 26 Massa invarianteTre protoni (mp = 938 MeVc2) hanno impulsi uguali in modulo (p =

3 GeVc) e che formano angoli di 120 lrsquouno con lrsquoaltro Qual egrave la massainvariante del sistema

Esercizio 27 Energia di sogliaSi consideri il processo

γ + prarr p+ π0

dove il fotone ha massa nulla il protone ha massa di 938 MeVc2 e il π0 hamassa di 135 MeVc2

1 Se il protone egrave a riposo qual egrave lrsquoenergia minima che deve avere il fotoneincidente percheacute la reazione abbia luogo

2 La stessa reazione puograve avvenire nellrsquouniverso in cui un protone dei raggicosmici di alta energia puograve collidere con uno dei fotoni della radiazionecosmica di fondo di energia dellrsquoordine di 1 meV Qual egrave in questo casolrsquoenergia minima che deve avere il protone percheacute la reazione abbia luogo

Esercizio 28 Conseguenze della relativitagraveUn misterioso supereroe pattuglia a velocitagrave molto elevata la periferia roma-

na Allrsquoincrocio con via di Tor Bella Monaca incontra un semaforo e ndash vedendolo

15

verde ndash attraversa senza rallentare Una pattuglia della polizia municipale loferma e lo multa per esser passato col rosso Assumendo sia il supereroe che ivigili siano nel giusto a che velocitagrave viaggiava il supereroe

Soluzione dellrsquoesercizio 28Per la polizia municipale il semaforo emette fotoni di energia

E0 = hν0 =hc

λrossoasymp hc

630 nm

mentre il supereoe vede fotoni di energia

E = hν =hc

λverdeasymp hc

490 nm

e dalle trasformazioni di Lorentz indicando con γ e β le variabili calcolateusando la velocitagrave del supereroe misurata dalla municipale possiamo scrivere

E = γ(E0minusβp0) = γ(E0minusβE0) =1minus βradic1minus β2

E0 =1minus βradic

(1 + β)(1minus β)E0 =

radic1minus βradic1 + β

E0

dove abbiamo usato il fatto che i fotoni hanno massa nulla Perciograve

E

E0=

hcλverdehc

λrosso

=λrosso

λverde=

radic1minus βradic1 + β

rarr v = βc = 025c

Esercizio 29 Conseguenze della relativitagraveI neutrini sono particelle di massa molto piccola e al momento ignota Uno

dei modi con cui egrave stato possibile dedurre un limite superiore al suo valore egravestata lrsquoosservazione nel 1987 di neutrini prodotti dalla supernova 1987A1 chesi trova a 168000 anni luce dalla Terra Sono stati osservati due segnali dineutrini che possono essere schematizzati come segue si egrave osservato prima unneutrino di 35 MeV di energia seguito a 9 s di distanza da un secondo segnaledi 13 MeV Si assuma che questo ritardo sia dovuto al fatto che la massa delneutrino non egrave nulla e si calcoli questrsquoultima

Esercizio 30 DecadimentoAnimali e piante assumono dallrsquoatmosfera diversi composti contenenti car-

bonio Il carbonio presente nellrsquoatmosfera egrave predominantemente 126 C ma sono

presenti piccole concentrazioni del suo isotopo 146 C (un atomo ogni 1012) che

decade con emissione di elettroni attraverso il processo2

146 C rarr14

7 N + eminus + νe

con un tempo di dimezzamento di 5700 anni1httpsenwikipediaorgwikiSN_1987ANeutrino_emissions2Di altro non si tratta che del decadimento βminus nrarr p+ e+ νe

16

1 Qual egrave la concentrazione di 146 C dopo 11400 anni

2 Animali e piante assumono in vita proporzioni fisse di 146 C e 12

6 C mentrealla loro morte la quantitagrave di 14

6 C inizia a diminuire Avete a portata dimano un relitto di legno per cui misurate una emissione di elettroni daldecadimento di 14

6 C pari al 61 di quella di un pezzo di legno vivodella stessa massa quanti anni ha il manufatto

Esercizio 31 Energia cinetica e trasformazioni di LorentzDue particelle identiche di massa m ed energia cinetica T collidono frontal-

mente Qual egrave la loro energia cinetica relativa (ossia lrsquoenergia cinetica di unaparticella misurata nel sistema di riferimento dellrsquoaltra particella)

Esercizio 32 Energia nel centro di massaLa reazione

πminusprarr K0Λ0

avviene con unrsquoenergia nel centro di massa diradics = 3 GeV La massa del πminus egrave

di 1396 MeVc2 la massa del protone egrave di 938 MeVc2 la massa del Kminus egrave di498 MeVc2 e quella della Λ0 di 11 GeVc2

1 Calcolare lrsquoimpulso di πminus e Λ0 nel sistema di riferimento del centro dimassa

2 Se il protone egrave a riposo il K puograve essere emesso allrsquoindietro nel sistemadi riferimento del laboratorio

Esercizio 33 DecadimentoIl mesone φ0 egrave una particella neutra3 di circa 1 GeVc2 di massa che puograve

decadere in una coppia di particelle

φ0 rarr K+ +Kminus

di massa identicamK = 494 MeVc2 Si assuma di produrre φ0 di impulso notoegrave possibile che uno dei due K sia prodotto a riposo nel sistema di riferimentodel laboratorio

Esercizio 34 DecadimentoUn fascio di anti-neutrini muonici νmicro si puograve generare selezionando pioni

o kaoni π+ e K+ e facendoli passare in un lungo tubo in cui egrave stato fattoil vuoto4 in modo che dopo un certo tragitto L una buona parte di loro saragrave

3Il collisore DAFNE ai Laboratori Nazionali di Frascati produce specificatamente parti-celle di questo tipo tramite il processo e+ + eminus rarr φ0 httpswwwyoutubecomwatchv=L5yB9gDGKms

4Una tecnica di questo tipo egrave stata usata per inviare ai Laboratori Nazionali del Gran Sassodei fasci di neutrini prodotti al CERN di Ginevra httpsvideoscernchrecord985892

17

decaduta tramite i processi

π+ rarr microminus + νmicro

K+ rarr microminus + νmicro

Se lrsquoimpulso di pioni e kaoni egrave di 200 GeVc e la loro vita media di 26 ns e12 ns rispettivamente

1 Quanto a lungo viaggiano nel laboratorio i due tipi di particelle2 Se L = 1000 m quale saragrave la frazione di pioni e kaoni che saragrave decaduta

alla fine del tubo3 Qual egrave lrsquoenergia massima dei neutrini che egrave possibile misurare nel sistema

di riferimento del laboratorio nei due casi

5 Lezione 5 (16 aprile 2020)

Esercizio 35 Energia cineticaTra le eccellenze con sede in territorio elvetico primeggiano indiscutibilmente lacioccolata al latte e il Large Hadron Collider (LHC) Questrsquoultimo egrave un collisoredi particelle situato al CERN di Ginevra che fa scontrare due fasci identici diprotoni di impulso uguale in modulo e direzione ma verso opposto I fasci sonocomposti da circa 2800 gruppi (pacchetti) di 1011 particelle ciascuno Sapendoche lrsquoenergia nel centro di massa della collisione fra due protoni egrave

radics = 13 TeV

quanta cioccolata al latte dovete mangiare per assumere un numero di caloriepari allrsquoenergia cinetica di ciascun fascio di protoni di LHC

Soluzione dellrsquoesercizio 35Poicheacute si tratta di un collider e le energie in gioco sono molto maggiori della

massa del protone lrsquoenergia di ciascun protone egrave diradics2 = 65 TeV e quella

totale di un fascio egrave

2800times 1011 times 65 middot 1012 times 16 middot 10minus19J asymp 300 MJ

Secondo Google 100 g di cioccolata al latte apportano circa 500 cal = 500 times4184 J asymp 2 MJ Lrsquoenergia di un fascio di LHC corrisponde quindi a quella dicirca 15 kg di cioccolata

Esercizio 36 Scattering RutherfordUn fascio di particelle α di 100 MeV di energia e 032 nA di corrente5 collide

contro un bersaglio fisso di alluminio spesso 1 cm Una sperimentatrice prendeun rivelatore di 1 cm times 1 cm di superficie e lo posiziona ad un angolo di 30

5Per una spiegazione breve su come (e percheacute) si misura la corrente di un fascio di particel-le vedi httpswwwlhc-closerestaking_a_closer_look_at_lhc0beam_current Unatrattazione piugrave completa egrave data ad esempio da httpscdscernchrecord1213275filesp141pdf

18

rispetto al fascio di particelle a 1 m di distanza dal bersaglio Quante particelleα incideranno sul rivelatore ogni secondo

Soluzione dellrsquoesercizio 36Lrsquoalluminio ha una densitagrave di 27 gcm3 numero atomico 13 e massa atomica

27 uPoicheacute le particelle α sono nuclei di elio hanno carica 2e e la corrente di

032 nA corrisponde a un miliardo di particelle incidenti al secondo

dNidt

=032 nCs

2times 16times 10minus19 C= 1times 109 sminus1

Il rivelatore vede un angolo solido di

∆Ω equiv superficieraggio

2

=1 cm2

(1 m)2= 1times 10minus4 sr

Si tratta di uno scattering alla Rutherford per cui la sezione drsquourto per unitagravedi angolo solido rilevata ad un certo angolo θ vale

dΩ=

(zαzAle

2

4πε04E

1

sin2(θ2)

)2

pari a

dΩasymp(

2times 13times 4times etimes 16times 10minus19 C

4π times 89times 10minus12 Fmtimes 4times 100times 106 eV

1

sin2(π180 times 302)

)2

asymp 2times 10minus30 m2sr = 20 mbsr

(1)

e il numero di particelle visto dal rivelatore vale se indichiamo con nAl = ρAlNAAAl

la densitagrave numero di atomi di alluminio e con d lo spessore del rivelatore

dNrivelate

dt= ∆Ω

dΩnAld

dNi

dt

asymp 1times 10minus4 srtimes 2times 10minus30 m2srtimes 1times 104 cm2m2 times 27 gcm3 6times 1023 molminus1

27 gmol

= 120 Hz

Esercizio 37 Sezione drsquourtoUn bersaglio di idrogeno liquido di densitagrave ρ = 0071 gcm3 e volume V =

125 cm3 egrave bombardato da un fascio monoenergetico di pioni negativi con unflusso φ = 2times 107 mminus2sminus1 e si osserva la reazione

πminus + prarr π0 + n

rivelando i fotoni del decadimento π0 rarr γγ che avviene nel 988 dei casi Sela sezione drsquourto di quellrsquointerazione egrave σ = 40 mb quanti fotoni sono emessi ognisecondo

19

Esercizio 38 Sezione drsquourtoLa sezione drsquourto dellrsquoeffetto fotoelettrico per raggi X di 10 keV in carbonio

egrave 40 b per atomo Data una lastra di carbonio di 4 mm di spessore si calcoli1 il numero di bersagli per unitagrave di volume2 il coefficiente di assorbimento per effetto fotoelettrico dei raggi X di tale

energia3 la probabilitagrave che un raggio X incidente sulla lastra produca un elettrone

per effetto fotoelettrico

Esercizio 39 Sezioni drsquourtoUn bersaglio drsquooro di densitagrave superficiale ρS = 097 mgcm2 e superficie

SB = 1 cm2 viene colpito da un fascio di particelle α la cui sezione trasversaegrave contenuta completamente nellrsquoarea del bersaglio Sul bersaglio impattano37times 104 αs La sezione drsquourto di diffusione elastica ad un certo angolo θ valedσ

ddΩ = 1 bsr Calcolare1 la densitagrave di atomi del bersaglio per unitagrave di superficie2 il numero di particelle α rivelate in unrsquoora da un rivelatore di superficie

SR = 2 cm2 posto allrsquoangolo θ e a distanza DR = 01 m dal bersaglio3 Se il fascio di particelle viene sostituito da una sorgente radioattiva di

particelle α con distribuzione isotropa su tutto lrsquoangolo solido che vieneposta lungo la stessa linea del fascio a distanza DB = 20 cm dal bersaglioquanto tempo egrave necessario per rivelare con lo stesso rivelatore lo stessonumero di particelle calcolato sopra a paritagrave di sezione drsquourto

Esercizio 40 CinematicaUn fascio di positroni di 100 MeV di energia annichila su una targhetta fissa

di elettroni producendo due fotoni tramite il processo

e+ + eminus rarr γ + γ

Se uno dei due fotoni egrave emesso a 30 nel sistema del centro di massa1 quanto vale lrsquoenergia dei due fotoni nel sistema del centro di massa2 che energia e direzione avranno nel sistema del laboratorio

Esercizio 41 Sezione drsquourtoGli antineutrini νe prodotti da un reattore nucleare con potenza P = 16 GW

attraversano un bersaglio da 2000 l drsquoacqua posto a 50 m di distanza dal reat-tore

1 Supponendo che per ogni fissione sia prodotta unrsquoenergia termica di200 MeV e vengano emessi 6 antineutrini calcolare il numero medio direazioni

νe + prarr e+ + n

prodotte al giorno se la sezione drsquourto egrave σ = 1times 10minus43 cm2

20

2 Qual egrave la minima energia del neutrino per cui la reazione egrave permessa

6 Lezione 7 (8 maggio 2020)

Esercizio 42Un protone interagisce con un bersaglio producendo un pione di energia E =300 MeV Con un tracciatore posto a distanza d = 6 cm dal bersaglio egrave possi-bile rivelare la traiettoria del pione e risalire al punto di produzione del pionesul bersaglio Determinare lrsquoerrore sulla misura della posizione di tale puntocausato dalla presenza di un piano di alluminio di spessore L = 05 mm postoimmediatamente davanti al tracciatore (quindi a distanza d dal bersaglio) nellrsquoi-potesi che gli angoli delle tracce rispetto alla normale alle superfici del bersaglioe del piano di alluminio siano piccoli [mπ = 1396 MeV Al (Z = 13 A = 27ρ = 27 gcm3)]

Soluzione dellrsquoesercizio 42La deviazione standard dellrsquoangolo di diffusione coulombiana multipla vale

radic〈θ2〉 = 21 MeV

z

βc|p|

radicx

X0= 21 MeV

z

βc|p|

radicLρ

X0

dove p =radicE2 minusm2

π = 266 MeV β = pE = 0885 z = 1 Per lrsquoalluminioabbiamo

X0 = 24 gcm2 rarr X0ρ = 89 cm

per cui radic〈θ2〉 asymp 00067

e lo spostamento medio corrispondente rispetto alla posizione nominale egrave

δs = tan(00067)times 6 cm = 04 mm

Esercizio 43Un fascio misto di elettroni e antiprotoni passa attraverso una regione con

B = 2 T e dopo aver curvato per R = 3 m raggiunge una regione senza campomagnetico

1 Qual egrave lrsquoimpulso delle particelle selezionate2 Come discriminereste fra i due tipi di particelle3 Se usaste due scintillatori posti a 15 m di distanza quale risoluzione

temporale sarebbe necessaria per discriminare i due tipi di particelle4 Se i due scintillatori fossero spessi 2 cm e avessero una lunghezza di ra-

diazione X0 = 40 cm quanta energia perderebbero elettroni e protoninellrsquoattraversarli Si assuma una perdita di energia per ionizzazione di2 MeVcm e 25 MeVcm per protoni ed elettroni rispettivamente

21

5 Che indice di rifrazione dovrebbe avere un rivelatore a luce Cherenkovper discriminare elettroni e protoni

Soluzione dellrsquoesercizio 43Dalla definizione di forza

F =dpdt

= qvtimesB

|F| = mv2

R

per cui se il campo magnetico egrave ortogonale alla traiettoria

pc = qcBR

che poicheacute c = 03 mns possiamo esprimere in maniera piugrave conveniente come

pc[GeV] = 03B[T]R[m]

Ne segue che nel nostro esercizio (e tornando a c = 1)

p = 18 GeV

Trascurando le perdite di energia e ricordando che me mp i tempi divolo di elettroni e protoni valgono rispettivamente

te =L

βecasymp L

casymp 50 ns

etp =

L

βpc=

Lpradic

p2+m2p

casymp 56 ns

per cui serve una risoluzione dellrsquoordine del ns per discriminare elettroni e muonidal tempo di volo

Se gli scintillatori sono spessi 2 cm i protoni perderanno in ciascuno di essi2 cm times 2 MeVcm = 4 MeV per ionizzazione Gli elettroni perderanno invecenel primo rivelatore 5 MeV per ionizzazione e (me asymp 0)

18 GeV times(

1minus exp

(minus 2 cm

40 cm

))asymp 88 MeV

cioegrave dopo il primo rivelatore avranno 18 GeVminus93 MeV di energia e nel secondone perderanno altri

5 MeV + (18 GeV minus 93 MeV)

(1minus exp

(minus 2 cm

40 cm

))asymp 88 MeV

Lrsquoangolo di emissione di luce Cherenkov egrave dato da

cos θc =1

βnle 1

22

e poicheacute βe asymp 1 βp asymp 0887 per avere luce Cherenkov solo al passaggio deglielettroni (e non dei protoni) egrave necessario che

1 lt n lt 113

Esercizio 44Come misurereste la massa di una particella carica E quella di una parti-

cella neutra

Esercizio 45Un muone di energia E = 400 GeV penetra verticalmente nel mare Attra-

verso quale processo puograve essere rivelato A quale profonditagrave arriva prima didecadere

Esercizio 46Avete a disposizione dei tubi fotomoltiplicatori sensibili a lunghezze drsquoonda

fra 300 nm e 500 nm e volete rivelare la luce Cherenkov prodotta dal passaggio diun elettrone di 1 MeV di energia in un metro drsquoacqua Quanti fotoni vi aspettateche vengano prodotti Confrontate con il numero di elettroni di ionizzazioneche produrrebbe una particella α di 5 keV nello stesso rivelatore

Esercizio 47Determinare quali sono i processi piugrave probabili (cioegrave quelli di sezione drsquourto

piugrave alta) nellrsquointerazione fra1 fotoni di 1 MeV e atomi di alluminio2 fotoni di 100 keV e H23 fotoni di 100 keV e atomi di ferro4 fotoni di 10 MeV e atomi di carbonio5 fotoni di 10 MeV e atomi di piombo

Esercizio 48Volete misurare lrsquoimpulso di una particella carica che attraversa un campo

magnetico B ortogonale alla sua traiettoria Avete a disposizione tre rivelatori diposizione della stessa risoluzione spaziale δx come li disponete Che risoluzionein impulso vi aspettate di ottenere

23

  • Lezione 1 (13 marzo 2020)
  • Lezione 2 (20 marzo 2020)
  • Lezione 3 (27 marzo 2020)
  • Lezione 4 (3 aprile 2020)
  • Lezione 5 (16 aprile 2020)
  • Lezione 7 (8 maggio 2020)
Page 4: Esercizi per casa (risolti)ippolitv/pdf/fns1_2020_solutions.pdfEsercizi per casa (risolti) Valerio Ippolito 13 maggio 2020 1 Lezione 1 (13 marzo 2020) Esercizio1 Dilatazionedeitempi

in quanto le due astronavi si avvicinano lrsquouna allrsquoaltraEsprimiamo quindi la velocitagrave di B nel sistema di riferimento di A che egrave

nota in funzione della velocitagrave di B nel sistema di riferimento del laboratorioche egrave ignota tenendo opportunamente conto dei segni

v(A)B =

v(lab)B minus v(lab)

A

1minus v(lab)A

c

v(lab)B

c

=2x

1 + x2

c

= a

da cui

x =2plusmn

radic4minus 4a

2

c2

2ac=c

a

(1plusmn

radic1minus

(ac

)2)asymp 040c

dove abbiamo scartato la soluzione con x gt c Viste dal sistema di riferimentodel laboratorio le astronavi viaggiano quindi a +040c e minus040c

Esercizio 6 Leggi di trasformazioneIn relativitagrave speciale come si trasforma il volume E la densitagrave

Soluzione dellrsquoesercizio 6Il volume di un corpo in moto egrave il prodotto di tre componenti due trasverse e

una parallela al boost di Lorentz Perciograve il volume misurato a terra egrave V = V primeγLe densitagrave invece aumentano di un fattore γ

Esercizio 7 Intervallo invarianteSi considerino anzitutto due punti A e B nello spazio euclideo (rappresentato

per semplicitagrave in due dimensioni) Quale dei due cammini in figura egrave piugrave breve

x

y

A

B

x

y

A

B

Si consideri ora lo spazio di Minkowski (pseudo-euclideo) con lrsquousuale defi-nizione di distanza ds2 = c2dt2 minus dx2 minus dy2 minus dz2 Quale dei due tragitti egrave piugravebreve Si assuma che tutti i segmenti di curva che congiungono A e B siano ditipo tempo

x

ct

A

B

x

ct

A

B

4

Soluzione dellrsquoesercizio 7Nello spazio euclideo la linea retta tra due punti egrave quella che minimizza

la distanza dx Nello spazio di Minkowski a causa del segno negativo allecomponenti spaziali di ds2 la distanza ds egrave invece massima per il cammino inlinea retta

2 Lezione 2 (20 marzo 2020)

Esercizio 8 QuadrivelocitagraveQual egrave la quadrivelocitagrave di una particella che si muove di moto rettilineo uni-forme lungo lrsquoasse x con velocitagrave v = 3

5cSoluzione dellrsquoesercizio 8

La quadrivelocitagrave di una particella egrave rappresentata dal quadrivettore con-travariante

ηmicro equiv dxmicro

dτ=

(c

dt

dx

dy

dz

)

dove τ egrave il tempo proprio della particella ovvero il tempo misurato nel suo siste-ma di riferimento Il problema ci fornisce la velocitagrave nel sistema di riferimentodel laboratorio perciograve ci conviene esprimere il tempo proprio in funzione deltempo misurato nel laboratorio t = γτ

ηmicro =

(γc γ

dx

dt γ

dy

dt γ

dz

dt

)= γ(c v 0 0) =

1radic1minus

(35

)2(c

3

5c

)=c

4(5 3 0 0)

La norma della quadrivelocitagrave vale

ηmicroηmicro = gmicroνηmicroην =

c2

42(52 minus 32 minus 0minus 0) = c2

che egrave manifestamente invariante

Esercizio 9 Decadimento e dilatazione dei tempiMetagrave dei muoni di un fascio composto da muoni di energia fissata sopravvive

dopo aver viaggiato l = 600 m nel sistema di riferimento del laboratorio Qualegrave la velocitagrave dei muoni

Soluzione dellrsquoesercizio 9Da

N

N0= exp

(minus tτ

)= exp

(minus vtvτ

)= exp

(minus l

βcγτ0

)=

1

2

segue che

minus log1

2=

l

βγcτ0

βγ =βradic

1minus β2= minus l

log 12cτ0

equiv λ

5

ed elevando al quadrato

β =

radicλ2

1 + λ2asymp 080

Esercizio 10 Calcolo tensorialeUn tensore le cui componenti contravarianti si indicano come amicroν egrave la ge-

neralizzazione a due indici di un quadrivettore Mentre un quadrivettore ha 4componenti un tensore a due indici ne ha 4times 4 = 16 Se nel passaggio da unsistema di riferimento O in quiete a uno Oprime in moto lungo lrsquoasse x con velocitagraveV = βc le coordinate di un quadrivettore trasformano secondo la legge

xmicroprime = Λmicroνxν

dove le Λmicroν sono le componenti della matrice

Λ =

γ minusβγ 0 0minusβγ γ 0 0

0 0 1 00 0 0 1

analogamente un tensore trasforma secondo

amicroν prime = ΛmicroρΛνσaρσ

Analogamente a quanto accade per i quadrivettori le coordinate contravariantidi un tensore sono legate a quelle covarianti dalla relazione

amicroν = gmicroρgνσaρσ

Un tensore si dice simmetrico se le sue componenti sono uguali sotto scambiodegli indici (amicroν = aνmicro) e antisimmetrico se sono uguali ma di segno opposto(amicroν = minusaνmicro)

1 Quanti elementi indipendenti ci sono in un tensore simmetrico (siconsiderano dipendenti ad esempio a12 e a21 = a12)

2 E in un tensore antisimmetrico3 Simmetria e antisimmetria sono caratteristiche che si mantengono sotto

trasformazione di Lorentz4 Se un tensore amicroν egrave simmetrico lo egrave anche la sua versione covariante amicroν

E se egrave antisimmetrico5 Se amicroν egrave un tensore simmetrico e bmicroν un tensore antisimmetrico quanto

vale amicroνbmicroν6 Scomporre un tensore generico amicroν nella somma di due tensori uno

simmetrico e lrsquoaltro antisimmetricoSoluzione dellrsquoesercizio 10

6

1 Un tensore simmetrico ha 10 componenti indipendenti (a11 a12 a13 a14 a22 a23 a24 a33 a34 a44)

2 Un tensore antisimmetrico ha 6 componenti indipendenti (a12 a13 a14 a23 a24 a34)

3 Se un tensore egrave simmetrico significa che

amicroν = aνmicro

La versione trasformata del termine a sinistra egrave

amicroν prime = ΛmicroρΛνσaρσ

mentre quella del termine di destra egrave

aνmicroprime = ΛνρΛmicroσaρσ

ma applicando la definizione di tensore simmetrico nel riferimento di par-tenza (coordinate senza apici) e applicando la proprietagrave commutativa aitermini della matrice di Lorentz si ha

aνmicroprime = ΛνρΛmicroσaρσ = ΛνρΛmicroσa

σρ = ΛmicroσΛνρaσρ = ΛmicroρΛνσa

ρσ = amicroν prime

dove abbiamo ndash nellrsquoultimo passaggio ndash usato il fatto che sia ρ che σ sonoindici muti Ne concludiamo che un tensore simmetrico rimane tale dopotrasformazione di Lorentz

Se il tensore egrave antisimmetrico analogamente significa che

amicroν = minusaνmicro

La versione trasformata del termine a sinistra egrave sempre

amicroν prime = ΛmicroρΛνσaρσ

Applicando ancora una volta la definizione di tensore antisimmetrico nelriferimento di partenza e la proprietagrave commutativa si ha

aνmicroprime = ΛνρΛmicroσaρσ = ΛνρΛmicroσ (minusaσρ) = minusΛmicroσΛνρa

σρ = minusΛmicroρΛνσaρσ = minusamicroν prime

cioegrave anche lrsquoantisimmetria egrave mantenuta dopo trasformazione di Lorentz

4 Per il tensore simmetrico

amicroν = gmicroρgνσaρσ = gmicroρgνσa

σρ = gνσgmicroρaσρ = amicroν

Nel caso antisimmetrico

amicroν = gmicroρgνσaρσ = gmicroρgνσ (minusaσρ) = minusgνσgmicroρaσρ = minusamicroν

7

5 Vale

amicroνbmicroν = aνmicrob

νmicro = amicroν (minusbmicroν) = minus (amicroνbmicroν)rarr amicroνbmicroν = 0

dove al primo passaggio abbiamo semplicemente rinominato gli indici mu-ti e al secondo abbiamo usato le proprietagrave di simmetriaantisimmetriadei tensori di partenza Ne segue dunque che il prodotto fra un tensoresimmetrico e uno antisimmetrico egrave zero

Suggerimento almeno una volta nella vita conviene esplicitare la som-ma implicita nella notazione di Einstein per comprendere davvero questoformalismo cosigrave compatto

6 Basta costruire due somme che per costruzione allo scambio di indicisono lrsquouna simmetrica

amicroν equiv 1

2(tmicroν + tνmicro)

e lrsquoaltra antisimmetrica cioegrave

amicroν equiv 1

2(tmicroν minus tνmicro)

Egrave evidente che questi due tensori sono simmetriciantisimmetrici sottoscambio degli indici

Esercizio 11 Relazione fra forza e accelerazioneIn relativitagrave speciale forza e accelerazione sono in generale proporzionali fra

loro Usare la definizione di forza F = dpdt

Suggerimento scomporre lrsquoaccelerazione nella somma di un termine paral-lelo e un termine ortogonale alla direzione del moto (cioegrave alla velocitagrave)

Soluzione dellrsquoesercizio 11Scomponiamo lrsquoaccelerazione a lungo le direzioni ortogonale e parallela al

moto della particella (cioegrave a v)

a equiv aperp + a

Usando il fatto che

dt=

dv

dv

dt=

d

[(1minus v2

c2

)minus 12

]dt

dv

dt= minus1

2

(minus2v

c2

)(1minus v2

c2

)minus 32 dv

dt=

v

c2γ3 dv

dt

e che dalla definizione di prodotto scalare fra vettori

vdv

dt= v middot a

8

otteniamo che

F =dpdt

=d(mγv)

dt= mγ

dvdt

+mdγ

dtv

= mγa +mγ3 (v middot a) vc2

= mγ(aperp + a) +mγ3(v middot aperp + v middot a)v

= mγaperp +mγ3

(1

γ2+v2

c2

)a

= mγaperp +mγ3

(1minus v2

c2+v2

c2

)a

= mγaperp +mγ3a

Si noti come in relativitagrave speciale la forza non egrave in generale proporzionaleallrsquoaccelerazione

Esercizio 12 Classificazione dei quadrivettoriIl quadrimpulso egrave un quadrivettore di tipo spazio tempo o luce

Soluzione dellrsquoesercizio 12Poicheacute P 2 = E2

c2 minus p2 = m2 ge 0 saragrave di tipo tempo per particelle massive (e

di tipo luce per particelle senza massa)

Esercizio 13 Energia cineticaQuanto lavoro bisogna compiere per aumentare la velocitagrave di un elettrone

(m = 511 keVc2) dalla posizione di riposo a1 050c2 0990c3 09990c

Soluzione dellrsquoesercizio 13A questi elettroni dovremo dare una certa energia cinetica T in modo da

far passare lrsquoenergia totale da quella a riposo (γ = 0) cioegrave

Ei = mc2

aEf = T +mc2

Dalla relazioneEf = mγc2

segue

T = Ef minusmc2 = m(γ minus 1)c2 = m

1radic1minus v2

c2

minus 1

c2

per cui nei tre casi indicati servono rispettivamente 79 keV 31 MeV e 109 MeV

9

Esercizio 14 Energia di sogliaSupponiamo di far scontrare un fascio di protoni con un protone a riposo

Qual egrave lrsquoenergia minima che devono avere i protoni del fascio percheacute la reazione

p+ prarr p+ p+ p+ p

sia permessa (La massa del protone egrave pari a quella dellrsquoantiprotone p e vale938 MeVc2)

Esercizio 15 Diffusione elasticaChiamiamo elastico un urto (ldquoscatteringrdquo) in cui le particelle dello stato ini-

ziale e dello stato finale sono le stesse Si consideri un urto elastico fra unaparticella di massa nulla e una particella di massa m (bersaglio) che si trovaa riposo nel sistema di riferimento del laboratorio qual egrave la massima energiatrasferita dalla particella incidente al bersaglio Suggerimento si lavori nelsistema di riferimento del laboratorio e si espliciti il prodotto scalare fra gliimpulsi spaziali della particella di massa nulla prima e dopo lrsquourto in funzionedellrsquoangolo sempre nel sistema di riferimento del laboratorio fra la direzioneiniziale e finale della particella incidente

Se la particella incidente egrave un fotone e il bersaglio egrave un elettrone atomico ariposo di quanto varia la lunghezza drsquoonda del fotone fra prima e dopo lrsquourto

Soluzione dellrsquoesercizio 15Per scattering elastico intendiamo un processo in cui le particelle dello stato

iniziale sono le stesse di quelle dello stato finaleDenotiamo con k e P i quadrimpulsi della particella incidente e del bersa-

glio prima dellrsquourto e indichiamo con lrsquoapice le stesse quantitagrave dopo lrsquourto ilproblema ci dice che

k = (Ek)

kprime = (Eprimekrsquo)P = (mc0)

Partiamo dalla conservazione del quadrimpulso durante lrsquourto isoliamo la quan-titagrave che non misuriamo direttamente ndash cioegrave il quadrimpulso del bersaglio dopolrsquourto P prime ndash ed eleviamo al quadrato

k + P = kprime + P prime

P prime = k + P minus kprimem2c2 = 0 +m2c2 + 0 + 2Emminus 2(EEprime minus k middot krsquo)minus 2mEprime

e se indichiamo con θprime lrsquoangolo ndash nel riferimento del laboratorio ndash fra la direzioneiniziale e finale della particella incidente e usiamo il fatto che |k|c = E e |krsquo|c =

10

Eprime

0 = 2mc2(E minus Eprime)minus 2(EEprime minus EEprime cos θprime)

mc2(Eprime minus E) = minusEEprime(1minus cos θprime)

Eprime(mc2 + E(1minus cos θprime)) = mc2E

Eprime =E

1 + Emc2 (1minus cos θprime)

Il bersaglio rinculeragrave di una energia E minus Eprime massima per θ = π Il valoremassimo di questrsquoenergia di rinculo

E minus E

1 + 2 Emc2

= E2Emc2

1 + 2Emc2

prende il nome ndash nel caso dello scattering Compton in cui la particella incidenteegrave un fotone e il bersaglio egrave un elettrone atomico ndash di picco Compton

Cosa cambia fra un fotone di energia E ed uno di energia Eprime Dalla mecca-nica quantistica

E = hν =hc

λ

cioegrave cambia la lunghezza drsquoonda del fotone

Eprime =hc

λprime=

hcλ

1 +hcλ

mc2 (1minus cos θprime)

1

λprime=

1 +hcλ

mc2 (1minus cos θprime)

λprime = λ

(1 +

hc

λmc2(1minus cos θprime)

)

λprime = λ+h

mc(1minus cos θprime) equiv λ+ λc(1minus cos θprime)

dove abbiamo definito la lunghezza drsquoonda Compton dellrsquoelettrone λc che rap-presenta la scala di lunghezza sotto la quale gli effetti della meccanica quantisticarelativistica divengono importanti

3 Lezione 3 (27 marzo 2020)

Esercizio 16 Trasformazione delle velocitagraveUn oggetto si muove di moto rettilineo uniforme con velocitagrave costante αc versoun secondo oggetto immobile A che velocitagrave dobbiamo muoverci noi lungo lastessa direzione per vedere entrambi gli oggetti muoversi con velocitagrave uguali eopposte

11

Soluzione dellrsquoesercizio 16Se indichiamo con lrsquoapice la velocitagrave nel nostro sistema di riferimento che

egrave in moto rispetto al sistema di riferimento del secondo oggetto la legge ditrasformazione delle velocitagrave ci dice che

vprime1 =v1 minus V

1minus v1cVc

=αcminus V1minus αVc

vprime2 =v2 minus V

1minus v2cVc

= minusV

Stiamo cercando V la nostra velocitagrave nel sistema di riferimento del secondooggetto tale che le velocitagrave dei due oggetti nel nostro sistema di riferimentosiano uguali e opposte

vprime1 = minusvprime2 = V

perciograve la richiesta egrave

αcminus V1minus αVc

= V

αcminus V minus V + αV 2

c= 0

(αc)V 2 + (minus2)V + (α)c = 0

V =1minusradic

1minus α2

αc

dove abbiamo scelto la soluzione dellrsquoequazione di secondo grado con V le c

Esercizio 17 Conseguenze della relativitagraveUna navicella spaziale in moto rettilineo uniforme con velocitagrave 05c in al-

lontanamento dalla Terra egrave in orbita verso Plutone che si trova a 75times 109 kmdi distanza dalla Terra Non appena raggiunto il pianeta la comandante inviaun segnale radio alla base a Houston per chiedere lrsquoautorizzazione allrsquoatter-raggio Quanto tempo impiega la richiesta a raggiungere la base secondo lacomandante E secondo i suoi colleghi a Houston

Soluzione dellrsquoesercizio 17Lrsquoonda radio viaggia a velocitagrave c secondo ogni sistema di riferimento Per

lrsquoosservatore lrsquoonda radio percorreragrave il tragitto Plutone-Terra in un tempo

tTerra =L

c=

75times 109 km

3times 108 msasymp 25 000 s

mentre per la comandante saragrave passato un tempo inferiore che dipende dal fat-tore γ dellrsquoastronave nel sistema di riferimento della Terra secondo la relazione

tcomandante =tTerra

γasymp 25 000 stimes

radic1minus 052 asymp 21 650 s

12

Esercizio 18 Energia cineticaHa piugrave energia un protone che viaggia a 09999999896c o un Boeing 747 al

decollo

Esercizio 19 Dilatazione dei tempiUna scienziata misura che un fascio di particelle selezionate con impulso di

10 GeVc si degrada dellrsquo84 dopo aver percorso 1 m Se la massa di questeparticelle egrave 498 MeVc2 qual egrave la loro vita media

Soluzione dellrsquoesercizio 19La distanza misurata dalla scienziata egrave chiaramente riferita al suo sistema

di riferimento quello del laboratorio ed egrave legata alla vita media delle particelledel fascio τ dalla relazione

Ldecay = 1 m = βγcτ =pc

E

E

mc2cτ =

p

mccτ

e dalla legge del decadimento se indichiamo con N0 il numero di particelleinizialmente presenti nel fascio e con N il numero di particelle misurato si haN

N0= 1minus084 = exp

(minus L

Ldecay

)= exp

(minus L

pmccτ

)rarr τ =

mcL

pc log(N0

N

) asymp 9times 10minus11 s

Esercizio 20 Dilatazione dei tempiVi trovate a dover studiare un fascio di particelle di cui conoscete lrsquoenergia

ndash 2 GeV ndash ma non la massa a quanto ne sapete potrebbero essere composti daelettroni (di massa 511 keVc2) o protoni (938 MeVc2) Avete a disposizionedue rivelatori identici in grado di registrare con precisione il tempo in cui unaparticella li attraversa Come potete utilizzarli per determinare se il vostrofascio contiene elettroni o protoni

Soluzione dellrsquoesercizio 20Disponendo i due rivelatori lungo la direzione del fascio a distanza ∆L

lrsquouno dallrsquoaltro si puograve misurare il tempo impiegato dalle particelle per passaredallrsquouno allrsquoaltro e quindi la loro massa infatti se ∆t egrave la distanza temporalefra i segnali dei due rivelatori

∆L = v∆t = βc∆t =pc2

E∆t

Questo principio egrave usato nei cosiddetti rivelatori di time of flight che sono usatiper discriminare diversi tipi di particelle una particella di tipo 1 e una di tipo2 infatti percorreranno la distanza ∆L in tempi diversi legati alle rispettivemasse dalla relazione

∆L1 = ∆L2

p1c2

E1∆t1 =

radicE2 minusm2

1c2c

E∆t1 =

radicE2 minusm2

2c2c

E∆t2

13

Esercizio 21 Energia di sogliaDue fisici delle particelle vogliono produrre il bosone Z una particella di

carica neutra e di massamZ = 91 GeVc2 e discutono come fare Alice proponedi far scontrare fasci di elettroni e positroni di energia identica che viaggianodunque con impulso spaziale uguale in modulo e direzione ma di verso oppostoproducendo Z tramite il processo

e+ + eminus rarr Z

mentre Bob preferisce scontrare un fascio di protoni su un bersaglio fisso diidrogeno tramite il processo

p+ prarr Z + p+ p

Chi dei due avragrave bisogno di fasci di particelle di energia piugrave alta La massa delprotone egrave di 9383 MeVc2 quella dellrsquoelettrone di 511 keVc2

Esercizio 22 Leggi di conservazioneUn fotone (particella di massa nulla) puograve decadere in un elettrone e in un

positrone (entrambi di massa 511 keVc2) tramite il processo

γ rarr e+ + eminus

Esercizio 23 Decadimento βminus

Quali sono lrsquoenergia minima e massima dellrsquoelettrone nel decadimento

nrarr p+ eminus + νe

se il neutrone decade da fermo La massa del neutrone egrave di 9396 MeVc2 quelladel protone di 9383 MeVc2 e quella dellrsquoelettrone di 511 keVc2 si assuma chelrsquoantineutrino elettronico νe abbia massa nulla

Esercizio 24 Leggi di conservazioneIl decadimento

prarr n+ e+ + νe

egrave permesso

4 Lezione 4 (3 aprile 2020)

Esercizio 25Usando il fatto che hc = 1973 MeVfm si dimostri che in un sistema di unitagrave dimisura in cui h = c = 1 vale

14

1 1 GeVminus2 = 0389 mb

2 1 m = 5068times 1015 GeVminus1

3 1 s = 15times 1024 GeVminus1

Ricordiamo che 1 b = 1times 10minus28 m2 e che

[hc] = [Jsms] = [E][L]

Soluzione dellrsquoesercizio 25Lrsquoidea egrave di capire per quale potenza di hc e c va moltiplicato il termine a

sinistra di ciascuna equazione per ottenere il termine di destraPer cui

bull [1 GeVminus2][hc]α = [E]minus2[E]α[L]α = [0389 mb] = [L]2 da cui segue α = 2

e 1 GeVminus2(hc)2 = 1973 MeVfm1 GeV =

(01973times 10minus15 GeVm

1 GeV

)2

= 0389 mb

bull [1 m][hc]α = [L][E]α[L]α = [5068times 1015 GeVminus1] = [E]minus1 da cui segueα = minus1 e 1 m(hc)minus1 = 1 m

1973 MeVfm = 1 m01973times 10minus15 GeVm

= 5068times 1015 GeVminus1

bull [1 s][hc]α[c]β = [T ][E]α[L]α[L]β [T ]minusβ = [T ][E]α[L]α+β [T ]minusβ = [15times 1024 GeVminus1] =[E]minus1 da cui segue α = minus1 β = 1 e 1 s(hc)minus1c = 1 s

1973 MeVfm299 792 458 ms =299 792 458 m

01973times 10minus15 GeVm= 15times 1024 GeVminus1

Esercizio 26 Massa invarianteTre protoni (mp = 938 MeVc2) hanno impulsi uguali in modulo (p =

3 GeVc) e che formano angoli di 120 lrsquouno con lrsquoaltro Qual egrave la massainvariante del sistema

Esercizio 27 Energia di sogliaSi consideri il processo

γ + prarr p+ π0

dove il fotone ha massa nulla il protone ha massa di 938 MeVc2 e il π0 hamassa di 135 MeVc2

1 Se il protone egrave a riposo qual egrave lrsquoenergia minima che deve avere il fotoneincidente percheacute la reazione abbia luogo

2 La stessa reazione puograve avvenire nellrsquouniverso in cui un protone dei raggicosmici di alta energia puograve collidere con uno dei fotoni della radiazionecosmica di fondo di energia dellrsquoordine di 1 meV Qual egrave in questo casolrsquoenergia minima che deve avere il protone percheacute la reazione abbia luogo

Esercizio 28 Conseguenze della relativitagraveUn misterioso supereroe pattuglia a velocitagrave molto elevata la periferia roma-

na Allrsquoincrocio con via di Tor Bella Monaca incontra un semaforo e ndash vedendolo

15

verde ndash attraversa senza rallentare Una pattuglia della polizia municipale loferma e lo multa per esser passato col rosso Assumendo sia il supereroe che ivigili siano nel giusto a che velocitagrave viaggiava il supereroe

Soluzione dellrsquoesercizio 28Per la polizia municipale il semaforo emette fotoni di energia

E0 = hν0 =hc

λrossoasymp hc

630 nm

mentre il supereoe vede fotoni di energia

E = hν =hc

λverdeasymp hc

490 nm

e dalle trasformazioni di Lorentz indicando con γ e β le variabili calcolateusando la velocitagrave del supereroe misurata dalla municipale possiamo scrivere

E = γ(E0minusβp0) = γ(E0minusβE0) =1minus βradic1minus β2

E0 =1minus βradic

(1 + β)(1minus β)E0 =

radic1minus βradic1 + β

E0

dove abbiamo usato il fatto che i fotoni hanno massa nulla Perciograve

E

E0=

hcλverdehc

λrosso

=λrosso

λverde=

radic1minus βradic1 + β

rarr v = βc = 025c

Esercizio 29 Conseguenze della relativitagraveI neutrini sono particelle di massa molto piccola e al momento ignota Uno

dei modi con cui egrave stato possibile dedurre un limite superiore al suo valore egravestata lrsquoosservazione nel 1987 di neutrini prodotti dalla supernova 1987A1 chesi trova a 168000 anni luce dalla Terra Sono stati osservati due segnali dineutrini che possono essere schematizzati come segue si egrave osservato prima unneutrino di 35 MeV di energia seguito a 9 s di distanza da un secondo segnaledi 13 MeV Si assuma che questo ritardo sia dovuto al fatto che la massa delneutrino non egrave nulla e si calcoli questrsquoultima

Esercizio 30 DecadimentoAnimali e piante assumono dallrsquoatmosfera diversi composti contenenti car-

bonio Il carbonio presente nellrsquoatmosfera egrave predominantemente 126 C ma sono

presenti piccole concentrazioni del suo isotopo 146 C (un atomo ogni 1012) che

decade con emissione di elettroni attraverso il processo2

146 C rarr14

7 N + eminus + νe

con un tempo di dimezzamento di 5700 anni1httpsenwikipediaorgwikiSN_1987ANeutrino_emissions2Di altro non si tratta che del decadimento βminus nrarr p+ e+ νe

16

1 Qual egrave la concentrazione di 146 C dopo 11400 anni

2 Animali e piante assumono in vita proporzioni fisse di 146 C e 12

6 C mentrealla loro morte la quantitagrave di 14

6 C inizia a diminuire Avete a portata dimano un relitto di legno per cui misurate una emissione di elettroni daldecadimento di 14

6 C pari al 61 di quella di un pezzo di legno vivodella stessa massa quanti anni ha il manufatto

Esercizio 31 Energia cinetica e trasformazioni di LorentzDue particelle identiche di massa m ed energia cinetica T collidono frontal-

mente Qual egrave la loro energia cinetica relativa (ossia lrsquoenergia cinetica di unaparticella misurata nel sistema di riferimento dellrsquoaltra particella)

Esercizio 32 Energia nel centro di massaLa reazione

πminusprarr K0Λ0

avviene con unrsquoenergia nel centro di massa diradics = 3 GeV La massa del πminus egrave

di 1396 MeVc2 la massa del protone egrave di 938 MeVc2 la massa del Kminus egrave di498 MeVc2 e quella della Λ0 di 11 GeVc2

1 Calcolare lrsquoimpulso di πminus e Λ0 nel sistema di riferimento del centro dimassa

2 Se il protone egrave a riposo il K puograve essere emesso allrsquoindietro nel sistemadi riferimento del laboratorio

Esercizio 33 DecadimentoIl mesone φ0 egrave una particella neutra3 di circa 1 GeVc2 di massa che puograve

decadere in una coppia di particelle

φ0 rarr K+ +Kminus

di massa identicamK = 494 MeVc2 Si assuma di produrre φ0 di impulso notoegrave possibile che uno dei due K sia prodotto a riposo nel sistema di riferimentodel laboratorio

Esercizio 34 DecadimentoUn fascio di anti-neutrini muonici νmicro si puograve generare selezionando pioni

o kaoni π+ e K+ e facendoli passare in un lungo tubo in cui egrave stato fattoil vuoto4 in modo che dopo un certo tragitto L una buona parte di loro saragrave

3Il collisore DAFNE ai Laboratori Nazionali di Frascati produce specificatamente parti-celle di questo tipo tramite il processo e+ + eminus rarr φ0 httpswwwyoutubecomwatchv=L5yB9gDGKms

4Una tecnica di questo tipo egrave stata usata per inviare ai Laboratori Nazionali del Gran Sassodei fasci di neutrini prodotti al CERN di Ginevra httpsvideoscernchrecord985892

17

decaduta tramite i processi

π+ rarr microminus + νmicro

K+ rarr microminus + νmicro

Se lrsquoimpulso di pioni e kaoni egrave di 200 GeVc e la loro vita media di 26 ns e12 ns rispettivamente

1 Quanto a lungo viaggiano nel laboratorio i due tipi di particelle2 Se L = 1000 m quale saragrave la frazione di pioni e kaoni che saragrave decaduta

alla fine del tubo3 Qual egrave lrsquoenergia massima dei neutrini che egrave possibile misurare nel sistema

di riferimento del laboratorio nei due casi

5 Lezione 5 (16 aprile 2020)

Esercizio 35 Energia cineticaTra le eccellenze con sede in territorio elvetico primeggiano indiscutibilmente lacioccolata al latte e il Large Hadron Collider (LHC) Questrsquoultimo egrave un collisoredi particelle situato al CERN di Ginevra che fa scontrare due fasci identici diprotoni di impulso uguale in modulo e direzione ma verso opposto I fasci sonocomposti da circa 2800 gruppi (pacchetti) di 1011 particelle ciascuno Sapendoche lrsquoenergia nel centro di massa della collisione fra due protoni egrave

radics = 13 TeV

quanta cioccolata al latte dovete mangiare per assumere un numero di caloriepari allrsquoenergia cinetica di ciascun fascio di protoni di LHC

Soluzione dellrsquoesercizio 35Poicheacute si tratta di un collider e le energie in gioco sono molto maggiori della

massa del protone lrsquoenergia di ciascun protone egrave diradics2 = 65 TeV e quella

totale di un fascio egrave

2800times 1011 times 65 middot 1012 times 16 middot 10minus19J asymp 300 MJ

Secondo Google 100 g di cioccolata al latte apportano circa 500 cal = 500 times4184 J asymp 2 MJ Lrsquoenergia di un fascio di LHC corrisponde quindi a quella dicirca 15 kg di cioccolata

Esercizio 36 Scattering RutherfordUn fascio di particelle α di 100 MeV di energia e 032 nA di corrente5 collide

contro un bersaglio fisso di alluminio spesso 1 cm Una sperimentatrice prendeun rivelatore di 1 cm times 1 cm di superficie e lo posiziona ad un angolo di 30

5Per una spiegazione breve su come (e percheacute) si misura la corrente di un fascio di particel-le vedi httpswwwlhc-closerestaking_a_closer_look_at_lhc0beam_current Unatrattazione piugrave completa egrave data ad esempio da httpscdscernchrecord1213275filesp141pdf

18

rispetto al fascio di particelle a 1 m di distanza dal bersaglio Quante particelleα incideranno sul rivelatore ogni secondo

Soluzione dellrsquoesercizio 36Lrsquoalluminio ha una densitagrave di 27 gcm3 numero atomico 13 e massa atomica

27 uPoicheacute le particelle α sono nuclei di elio hanno carica 2e e la corrente di

032 nA corrisponde a un miliardo di particelle incidenti al secondo

dNidt

=032 nCs

2times 16times 10minus19 C= 1times 109 sminus1

Il rivelatore vede un angolo solido di

∆Ω equiv superficieraggio

2

=1 cm2

(1 m)2= 1times 10minus4 sr

Si tratta di uno scattering alla Rutherford per cui la sezione drsquourto per unitagravedi angolo solido rilevata ad un certo angolo θ vale

dΩ=

(zαzAle

2

4πε04E

1

sin2(θ2)

)2

pari a

dΩasymp(

2times 13times 4times etimes 16times 10minus19 C

4π times 89times 10minus12 Fmtimes 4times 100times 106 eV

1

sin2(π180 times 302)

)2

asymp 2times 10minus30 m2sr = 20 mbsr

(1)

e il numero di particelle visto dal rivelatore vale se indichiamo con nAl = ρAlNAAAl

la densitagrave numero di atomi di alluminio e con d lo spessore del rivelatore

dNrivelate

dt= ∆Ω

dΩnAld

dNi

dt

asymp 1times 10minus4 srtimes 2times 10minus30 m2srtimes 1times 104 cm2m2 times 27 gcm3 6times 1023 molminus1

27 gmol

= 120 Hz

Esercizio 37 Sezione drsquourtoUn bersaglio di idrogeno liquido di densitagrave ρ = 0071 gcm3 e volume V =

125 cm3 egrave bombardato da un fascio monoenergetico di pioni negativi con unflusso φ = 2times 107 mminus2sminus1 e si osserva la reazione

πminus + prarr π0 + n

rivelando i fotoni del decadimento π0 rarr γγ che avviene nel 988 dei casi Sela sezione drsquourto di quellrsquointerazione egrave σ = 40 mb quanti fotoni sono emessi ognisecondo

19

Esercizio 38 Sezione drsquourtoLa sezione drsquourto dellrsquoeffetto fotoelettrico per raggi X di 10 keV in carbonio

egrave 40 b per atomo Data una lastra di carbonio di 4 mm di spessore si calcoli1 il numero di bersagli per unitagrave di volume2 il coefficiente di assorbimento per effetto fotoelettrico dei raggi X di tale

energia3 la probabilitagrave che un raggio X incidente sulla lastra produca un elettrone

per effetto fotoelettrico

Esercizio 39 Sezioni drsquourtoUn bersaglio drsquooro di densitagrave superficiale ρS = 097 mgcm2 e superficie

SB = 1 cm2 viene colpito da un fascio di particelle α la cui sezione trasversaegrave contenuta completamente nellrsquoarea del bersaglio Sul bersaglio impattano37times 104 αs La sezione drsquourto di diffusione elastica ad un certo angolo θ valedσ

ddΩ = 1 bsr Calcolare1 la densitagrave di atomi del bersaglio per unitagrave di superficie2 il numero di particelle α rivelate in unrsquoora da un rivelatore di superficie

SR = 2 cm2 posto allrsquoangolo θ e a distanza DR = 01 m dal bersaglio3 Se il fascio di particelle viene sostituito da una sorgente radioattiva di

particelle α con distribuzione isotropa su tutto lrsquoangolo solido che vieneposta lungo la stessa linea del fascio a distanza DB = 20 cm dal bersaglioquanto tempo egrave necessario per rivelare con lo stesso rivelatore lo stessonumero di particelle calcolato sopra a paritagrave di sezione drsquourto

Esercizio 40 CinematicaUn fascio di positroni di 100 MeV di energia annichila su una targhetta fissa

di elettroni producendo due fotoni tramite il processo

e+ + eminus rarr γ + γ

Se uno dei due fotoni egrave emesso a 30 nel sistema del centro di massa1 quanto vale lrsquoenergia dei due fotoni nel sistema del centro di massa2 che energia e direzione avranno nel sistema del laboratorio

Esercizio 41 Sezione drsquourtoGli antineutrini νe prodotti da un reattore nucleare con potenza P = 16 GW

attraversano un bersaglio da 2000 l drsquoacqua posto a 50 m di distanza dal reat-tore

1 Supponendo che per ogni fissione sia prodotta unrsquoenergia termica di200 MeV e vengano emessi 6 antineutrini calcolare il numero medio direazioni

νe + prarr e+ + n

prodotte al giorno se la sezione drsquourto egrave σ = 1times 10minus43 cm2

20

2 Qual egrave la minima energia del neutrino per cui la reazione egrave permessa

6 Lezione 7 (8 maggio 2020)

Esercizio 42Un protone interagisce con un bersaglio producendo un pione di energia E =300 MeV Con un tracciatore posto a distanza d = 6 cm dal bersaglio egrave possi-bile rivelare la traiettoria del pione e risalire al punto di produzione del pionesul bersaglio Determinare lrsquoerrore sulla misura della posizione di tale puntocausato dalla presenza di un piano di alluminio di spessore L = 05 mm postoimmediatamente davanti al tracciatore (quindi a distanza d dal bersaglio) nellrsquoi-potesi che gli angoli delle tracce rispetto alla normale alle superfici del bersaglioe del piano di alluminio siano piccoli [mπ = 1396 MeV Al (Z = 13 A = 27ρ = 27 gcm3)]

Soluzione dellrsquoesercizio 42La deviazione standard dellrsquoangolo di diffusione coulombiana multipla vale

radic〈θ2〉 = 21 MeV

z

βc|p|

radicx

X0= 21 MeV

z

βc|p|

radicLρ

X0

dove p =radicE2 minusm2

π = 266 MeV β = pE = 0885 z = 1 Per lrsquoalluminioabbiamo

X0 = 24 gcm2 rarr X0ρ = 89 cm

per cui radic〈θ2〉 asymp 00067

e lo spostamento medio corrispondente rispetto alla posizione nominale egrave

δs = tan(00067)times 6 cm = 04 mm

Esercizio 43Un fascio misto di elettroni e antiprotoni passa attraverso una regione con

B = 2 T e dopo aver curvato per R = 3 m raggiunge una regione senza campomagnetico

1 Qual egrave lrsquoimpulso delle particelle selezionate2 Come discriminereste fra i due tipi di particelle3 Se usaste due scintillatori posti a 15 m di distanza quale risoluzione

temporale sarebbe necessaria per discriminare i due tipi di particelle4 Se i due scintillatori fossero spessi 2 cm e avessero una lunghezza di ra-

diazione X0 = 40 cm quanta energia perderebbero elettroni e protoninellrsquoattraversarli Si assuma una perdita di energia per ionizzazione di2 MeVcm e 25 MeVcm per protoni ed elettroni rispettivamente

21

5 Che indice di rifrazione dovrebbe avere un rivelatore a luce Cherenkovper discriminare elettroni e protoni

Soluzione dellrsquoesercizio 43Dalla definizione di forza

F =dpdt

= qvtimesB

|F| = mv2

R

per cui se il campo magnetico egrave ortogonale alla traiettoria

pc = qcBR

che poicheacute c = 03 mns possiamo esprimere in maniera piugrave conveniente come

pc[GeV] = 03B[T]R[m]

Ne segue che nel nostro esercizio (e tornando a c = 1)

p = 18 GeV

Trascurando le perdite di energia e ricordando che me mp i tempi divolo di elettroni e protoni valgono rispettivamente

te =L

βecasymp L

casymp 50 ns

etp =

L

βpc=

Lpradic

p2+m2p

casymp 56 ns

per cui serve una risoluzione dellrsquoordine del ns per discriminare elettroni e muonidal tempo di volo

Se gli scintillatori sono spessi 2 cm i protoni perderanno in ciascuno di essi2 cm times 2 MeVcm = 4 MeV per ionizzazione Gli elettroni perderanno invecenel primo rivelatore 5 MeV per ionizzazione e (me asymp 0)

18 GeV times(

1minus exp

(minus 2 cm

40 cm

))asymp 88 MeV

cioegrave dopo il primo rivelatore avranno 18 GeVminus93 MeV di energia e nel secondone perderanno altri

5 MeV + (18 GeV minus 93 MeV)

(1minus exp

(minus 2 cm

40 cm

))asymp 88 MeV

Lrsquoangolo di emissione di luce Cherenkov egrave dato da

cos θc =1

βnle 1

22

e poicheacute βe asymp 1 βp asymp 0887 per avere luce Cherenkov solo al passaggio deglielettroni (e non dei protoni) egrave necessario che

1 lt n lt 113

Esercizio 44Come misurereste la massa di una particella carica E quella di una parti-

cella neutra

Esercizio 45Un muone di energia E = 400 GeV penetra verticalmente nel mare Attra-

verso quale processo puograve essere rivelato A quale profonditagrave arriva prima didecadere

Esercizio 46Avete a disposizione dei tubi fotomoltiplicatori sensibili a lunghezze drsquoonda

fra 300 nm e 500 nm e volete rivelare la luce Cherenkov prodotta dal passaggio diun elettrone di 1 MeV di energia in un metro drsquoacqua Quanti fotoni vi aspettateche vengano prodotti Confrontate con il numero di elettroni di ionizzazioneche produrrebbe una particella α di 5 keV nello stesso rivelatore

Esercizio 47Determinare quali sono i processi piugrave probabili (cioegrave quelli di sezione drsquourto

piugrave alta) nellrsquointerazione fra1 fotoni di 1 MeV e atomi di alluminio2 fotoni di 100 keV e H23 fotoni di 100 keV e atomi di ferro4 fotoni di 10 MeV e atomi di carbonio5 fotoni di 10 MeV e atomi di piombo

Esercizio 48Volete misurare lrsquoimpulso di una particella carica che attraversa un campo

magnetico B ortogonale alla sua traiettoria Avete a disposizione tre rivelatori diposizione della stessa risoluzione spaziale δx come li disponete Che risoluzionein impulso vi aspettate di ottenere

23

  • Lezione 1 (13 marzo 2020)
  • Lezione 2 (20 marzo 2020)
  • Lezione 3 (27 marzo 2020)
  • Lezione 4 (3 aprile 2020)
  • Lezione 5 (16 aprile 2020)
  • Lezione 7 (8 maggio 2020)
Page 5: Esercizi per casa (risolti)ippolitv/pdf/fns1_2020_solutions.pdfEsercizi per casa (risolti) Valerio Ippolito 13 maggio 2020 1 Lezione 1 (13 marzo 2020) Esercizio1 Dilatazionedeitempi

Soluzione dellrsquoesercizio 7Nello spazio euclideo la linea retta tra due punti egrave quella che minimizza

la distanza dx Nello spazio di Minkowski a causa del segno negativo allecomponenti spaziali di ds2 la distanza ds egrave invece massima per il cammino inlinea retta

2 Lezione 2 (20 marzo 2020)

Esercizio 8 QuadrivelocitagraveQual egrave la quadrivelocitagrave di una particella che si muove di moto rettilineo uni-forme lungo lrsquoasse x con velocitagrave v = 3

5cSoluzione dellrsquoesercizio 8

La quadrivelocitagrave di una particella egrave rappresentata dal quadrivettore con-travariante

ηmicro equiv dxmicro

dτ=

(c

dt

dx

dy

dz

)

dove τ egrave il tempo proprio della particella ovvero il tempo misurato nel suo siste-ma di riferimento Il problema ci fornisce la velocitagrave nel sistema di riferimentodel laboratorio perciograve ci conviene esprimere il tempo proprio in funzione deltempo misurato nel laboratorio t = γτ

ηmicro =

(γc γ

dx

dt γ

dy

dt γ

dz

dt

)= γ(c v 0 0) =

1radic1minus

(35

)2(c

3

5c

)=c

4(5 3 0 0)

La norma della quadrivelocitagrave vale

ηmicroηmicro = gmicroνηmicroην =

c2

42(52 minus 32 minus 0minus 0) = c2

che egrave manifestamente invariante

Esercizio 9 Decadimento e dilatazione dei tempiMetagrave dei muoni di un fascio composto da muoni di energia fissata sopravvive

dopo aver viaggiato l = 600 m nel sistema di riferimento del laboratorio Qualegrave la velocitagrave dei muoni

Soluzione dellrsquoesercizio 9Da

N

N0= exp

(minus tτ

)= exp

(minus vtvτ

)= exp

(minus l

βcγτ0

)=

1

2

segue che

minus log1

2=

l

βγcτ0

βγ =βradic

1minus β2= minus l

log 12cτ0

equiv λ

5

ed elevando al quadrato

β =

radicλ2

1 + λ2asymp 080

Esercizio 10 Calcolo tensorialeUn tensore le cui componenti contravarianti si indicano come amicroν egrave la ge-

neralizzazione a due indici di un quadrivettore Mentre un quadrivettore ha 4componenti un tensore a due indici ne ha 4times 4 = 16 Se nel passaggio da unsistema di riferimento O in quiete a uno Oprime in moto lungo lrsquoasse x con velocitagraveV = βc le coordinate di un quadrivettore trasformano secondo la legge

xmicroprime = Λmicroνxν

dove le Λmicroν sono le componenti della matrice

Λ =

γ minusβγ 0 0minusβγ γ 0 0

0 0 1 00 0 0 1

analogamente un tensore trasforma secondo

amicroν prime = ΛmicroρΛνσaρσ

Analogamente a quanto accade per i quadrivettori le coordinate contravariantidi un tensore sono legate a quelle covarianti dalla relazione

amicroν = gmicroρgνσaρσ

Un tensore si dice simmetrico se le sue componenti sono uguali sotto scambiodegli indici (amicroν = aνmicro) e antisimmetrico se sono uguali ma di segno opposto(amicroν = minusaνmicro)

1 Quanti elementi indipendenti ci sono in un tensore simmetrico (siconsiderano dipendenti ad esempio a12 e a21 = a12)

2 E in un tensore antisimmetrico3 Simmetria e antisimmetria sono caratteristiche che si mantengono sotto

trasformazione di Lorentz4 Se un tensore amicroν egrave simmetrico lo egrave anche la sua versione covariante amicroν

E se egrave antisimmetrico5 Se amicroν egrave un tensore simmetrico e bmicroν un tensore antisimmetrico quanto

vale amicroνbmicroν6 Scomporre un tensore generico amicroν nella somma di due tensori uno

simmetrico e lrsquoaltro antisimmetricoSoluzione dellrsquoesercizio 10

6

1 Un tensore simmetrico ha 10 componenti indipendenti (a11 a12 a13 a14 a22 a23 a24 a33 a34 a44)

2 Un tensore antisimmetrico ha 6 componenti indipendenti (a12 a13 a14 a23 a24 a34)

3 Se un tensore egrave simmetrico significa che

amicroν = aνmicro

La versione trasformata del termine a sinistra egrave

amicroν prime = ΛmicroρΛνσaρσ

mentre quella del termine di destra egrave

aνmicroprime = ΛνρΛmicroσaρσ

ma applicando la definizione di tensore simmetrico nel riferimento di par-tenza (coordinate senza apici) e applicando la proprietagrave commutativa aitermini della matrice di Lorentz si ha

aνmicroprime = ΛνρΛmicroσaρσ = ΛνρΛmicroσa

σρ = ΛmicroσΛνρaσρ = ΛmicroρΛνσa

ρσ = amicroν prime

dove abbiamo ndash nellrsquoultimo passaggio ndash usato il fatto che sia ρ che σ sonoindici muti Ne concludiamo che un tensore simmetrico rimane tale dopotrasformazione di Lorentz

Se il tensore egrave antisimmetrico analogamente significa che

amicroν = minusaνmicro

La versione trasformata del termine a sinistra egrave sempre

amicroν prime = ΛmicroρΛνσaρσ

Applicando ancora una volta la definizione di tensore antisimmetrico nelriferimento di partenza e la proprietagrave commutativa si ha

aνmicroprime = ΛνρΛmicroσaρσ = ΛνρΛmicroσ (minusaσρ) = minusΛmicroσΛνρa

σρ = minusΛmicroρΛνσaρσ = minusamicroν prime

cioegrave anche lrsquoantisimmetria egrave mantenuta dopo trasformazione di Lorentz

4 Per il tensore simmetrico

amicroν = gmicroρgνσaρσ = gmicroρgνσa

σρ = gνσgmicroρaσρ = amicroν

Nel caso antisimmetrico

amicroν = gmicroρgνσaρσ = gmicroρgνσ (minusaσρ) = minusgνσgmicroρaσρ = minusamicroν

7

5 Vale

amicroνbmicroν = aνmicrob

νmicro = amicroν (minusbmicroν) = minus (amicroνbmicroν)rarr amicroνbmicroν = 0

dove al primo passaggio abbiamo semplicemente rinominato gli indici mu-ti e al secondo abbiamo usato le proprietagrave di simmetriaantisimmetriadei tensori di partenza Ne segue dunque che il prodotto fra un tensoresimmetrico e uno antisimmetrico egrave zero

Suggerimento almeno una volta nella vita conviene esplicitare la som-ma implicita nella notazione di Einstein per comprendere davvero questoformalismo cosigrave compatto

6 Basta costruire due somme che per costruzione allo scambio di indicisono lrsquouna simmetrica

amicroν equiv 1

2(tmicroν + tνmicro)

e lrsquoaltra antisimmetrica cioegrave

amicroν equiv 1

2(tmicroν minus tνmicro)

Egrave evidente che questi due tensori sono simmetriciantisimmetrici sottoscambio degli indici

Esercizio 11 Relazione fra forza e accelerazioneIn relativitagrave speciale forza e accelerazione sono in generale proporzionali fra

loro Usare la definizione di forza F = dpdt

Suggerimento scomporre lrsquoaccelerazione nella somma di un termine paral-lelo e un termine ortogonale alla direzione del moto (cioegrave alla velocitagrave)

Soluzione dellrsquoesercizio 11Scomponiamo lrsquoaccelerazione a lungo le direzioni ortogonale e parallela al

moto della particella (cioegrave a v)

a equiv aperp + a

Usando il fatto che

dt=

dv

dv

dt=

d

[(1minus v2

c2

)minus 12

]dt

dv

dt= minus1

2

(minus2v

c2

)(1minus v2

c2

)minus 32 dv

dt=

v

c2γ3 dv

dt

e che dalla definizione di prodotto scalare fra vettori

vdv

dt= v middot a

8

otteniamo che

F =dpdt

=d(mγv)

dt= mγ

dvdt

+mdγ

dtv

= mγa +mγ3 (v middot a) vc2

= mγ(aperp + a) +mγ3(v middot aperp + v middot a)v

= mγaperp +mγ3

(1

γ2+v2

c2

)a

= mγaperp +mγ3

(1minus v2

c2+v2

c2

)a

= mγaperp +mγ3a

Si noti come in relativitagrave speciale la forza non egrave in generale proporzionaleallrsquoaccelerazione

Esercizio 12 Classificazione dei quadrivettoriIl quadrimpulso egrave un quadrivettore di tipo spazio tempo o luce

Soluzione dellrsquoesercizio 12Poicheacute P 2 = E2

c2 minus p2 = m2 ge 0 saragrave di tipo tempo per particelle massive (e

di tipo luce per particelle senza massa)

Esercizio 13 Energia cineticaQuanto lavoro bisogna compiere per aumentare la velocitagrave di un elettrone

(m = 511 keVc2) dalla posizione di riposo a1 050c2 0990c3 09990c

Soluzione dellrsquoesercizio 13A questi elettroni dovremo dare una certa energia cinetica T in modo da

far passare lrsquoenergia totale da quella a riposo (γ = 0) cioegrave

Ei = mc2

aEf = T +mc2

Dalla relazioneEf = mγc2

segue

T = Ef minusmc2 = m(γ minus 1)c2 = m

1radic1minus v2

c2

minus 1

c2

per cui nei tre casi indicati servono rispettivamente 79 keV 31 MeV e 109 MeV

9

Esercizio 14 Energia di sogliaSupponiamo di far scontrare un fascio di protoni con un protone a riposo

Qual egrave lrsquoenergia minima che devono avere i protoni del fascio percheacute la reazione

p+ prarr p+ p+ p+ p

sia permessa (La massa del protone egrave pari a quella dellrsquoantiprotone p e vale938 MeVc2)

Esercizio 15 Diffusione elasticaChiamiamo elastico un urto (ldquoscatteringrdquo) in cui le particelle dello stato ini-

ziale e dello stato finale sono le stesse Si consideri un urto elastico fra unaparticella di massa nulla e una particella di massa m (bersaglio) che si trovaa riposo nel sistema di riferimento del laboratorio qual egrave la massima energiatrasferita dalla particella incidente al bersaglio Suggerimento si lavori nelsistema di riferimento del laboratorio e si espliciti il prodotto scalare fra gliimpulsi spaziali della particella di massa nulla prima e dopo lrsquourto in funzionedellrsquoangolo sempre nel sistema di riferimento del laboratorio fra la direzioneiniziale e finale della particella incidente

Se la particella incidente egrave un fotone e il bersaglio egrave un elettrone atomico ariposo di quanto varia la lunghezza drsquoonda del fotone fra prima e dopo lrsquourto

Soluzione dellrsquoesercizio 15Per scattering elastico intendiamo un processo in cui le particelle dello stato

iniziale sono le stesse di quelle dello stato finaleDenotiamo con k e P i quadrimpulsi della particella incidente e del bersa-

glio prima dellrsquourto e indichiamo con lrsquoapice le stesse quantitagrave dopo lrsquourto ilproblema ci dice che

k = (Ek)

kprime = (Eprimekrsquo)P = (mc0)

Partiamo dalla conservazione del quadrimpulso durante lrsquourto isoliamo la quan-titagrave che non misuriamo direttamente ndash cioegrave il quadrimpulso del bersaglio dopolrsquourto P prime ndash ed eleviamo al quadrato

k + P = kprime + P prime

P prime = k + P minus kprimem2c2 = 0 +m2c2 + 0 + 2Emminus 2(EEprime minus k middot krsquo)minus 2mEprime

e se indichiamo con θprime lrsquoangolo ndash nel riferimento del laboratorio ndash fra la direzioneiniziale e finale della particella incidente e usiamo il fatto che |k|c = E e |krsquo|c =

10

Eprime

0 = 2mc2(E minus Eprime)minus 2(EEprime minus EEprime cos θprime)

mc2(Eprime minus E) = minusEEprime(1minus cos θprime)

Eprime(mc2 + E(1minus cos θprime)) = mc2E

Eprime =E

1 + Emc2 (1minus cos θprime)

Il bersaglio rinculeragrave di una energia E minus Eprime massima per θ = π Il valoremassimo di questrsquoenergia di rinculo

E minus E

1 + 2 Emc2

= E2Emc2

1 + 2Emc2

prende il nome ndash nel caso dello scattering Compton in cui la particella incidenteegrave un fotone e il bersaglio egrave un elettrone atomico ndash di picco Compton

Cosa cambia fra un fotone di energia E ed uno di energia Eprime Dalla mecca-nica quantistica

E = hν =hc

λ

cioegrave cambia la lunghezza drsquoonda del fotone

Eprime =hc

λprime=

hcλ

1 +hcλ

mc2 (1minus cos θprime)

1

λprime=

1 +hcλ

mc2 (1minus cos θprime)

λprime = λ

(1 +

hc

λmc2(1minus cos θprime)

)

λprime = λ+h

mc(1minus cos θprime) equiv λ+ λc(1minus cos θprime)

dove abbiamo definito la lunghezza drsquoonda Compton dellrsquoelettrone λc che rap-presenta la scala di lunghezza sotto la quale gli effetti della meccanica quantisticarelativistica divengono importanti

3 Lezione 3 (27 marzo 2020)

Esercizio 16 Trasformazione delle velocitagraveUn oggetto si muove di moto rettilineo uniforme con velocitagrave costante αc versoun secondo oggetto immobile A che velocitagrave dobbiamo muoverci noi lungo lastessa direzione per vedere entrambi gli oggetti muoversi con velocitagrave uguali eopposte

11

Soluzione dellrsquoesercizio 16Se indichiamo con lrsquoapice la velocitagrave nel nostro sistema di riferimento che

egrave in moto rispetto al sistema di riferimento del secondo oggetto la legge ditrasformazione delle velocitagrave ci dice che

vprime1 =v1 minus V

1minus v1cVc

=αcminus V1minus αVc

vprime2 =v2 minus V

1minus v2cVc

= minusV

Stiamo cercando V la nostra velocitagrave nel sistema di riferimento del secondooggetto tale che le velocitagrave dei due oggetti nel nostro sistema di riferimentosiano uguali e opposte

vprime1 = minusvprime2 = V

perciograve la richiesta egrave

αcminus V1minus αVc

= V

αcminus V minus V + αV 2

c= 0

(αc)V 2 + (minus2)V + (α)c = 0

V =1minusradic

1minus α2

αc

dove abbiamo scelto la soluzione dellrsquoequazione di secondo grado con V le c

Esercizio 17 Conseguenze della relativitagraveUna navicella spaziale in moto rettilineo uniforme con velocitagrave 05c in al-

lontanamento dalla Terra egrave in orbita verso Plutone che si trova a 75times 109 kmdi distanza dalla Terra Non appena raggiunto il pianeta la comandante inviaun segnale radio alla base a Houston per chiedere lrsquoautorizzazione allrsquoatter-raggio Quanto tempo impiega la richiesta a raggiungere la base secondo lacomandante E secondo i suoi colleghi a Houston

Soluzione dellrsquoesercizio 17Lrsquoonda radio viaggia a velocitagrave c secondo ogni sistema di riferimento Per

lrsquoosservatore lrsquoonda radio percorreragrave il tragitto Plutone-Terra in un tempo

tTerra =L

c=

75times 109 km

3times 108 msasymp 25 000 s

mentre per la comandante saragrave passato un tempo inferiore che dipende dal fat-tore γ dellrsquoastronave nel sistema di riferimento della Terra secondo la relazione

tcomandante =tTerra

γasymp 25 000 stimes

radic1minus 052 asymp 21 650 s

12

Esercizio 18 Energia cineticaHa piugrave energia un protone che viaggia a 09999999896c o un Boeing 747 al

decollo

Esercizio 19 Dilatazione dei tempiUna scienziata misura che un fascio di particelle selezionate con impulso di

10 GeVc si degrada dellrsquo84 dopo aver percorso 1 m Se la massa di questeparticelle egrave 498 MeVc2 qual egrave la loro vita media

Soluzione dellrsquoesercizio 19La distanza misurata dalla scienziata egrave chiaramente riferita al suo sistema

di riferimento quello del laboratorio ed egrave legata alla vita media delle particelledel fascio τ dalla relazione

Ldecay = 1 m = βγcτ =pc

E

E

mc2cτ =

p

mccτ

e dalla legge del decadimento se indichiamo con N0 il numero di particelleinizialmente presenti nel fascio e con N il numero di particelle misurato si haN

N0= 1minus084 = exp

(minus L

Ldecay

)= exp

(minus L

pmccτ

)rarr τ =

mcL

pc log(N0

N

) asymp 9times 10minus11 s

Esercizio 20 Dilatazione dei tempiVi trovate a dover studiare un fascio di particelle di cui conoscete lrsquoenergia

ndash 2 GeV ndash ma non la massa a quanto ne sapete potrebbero essere composti daelettroni (di massa 511 keVc2) o protoni (938 MeVc2) Avete a disposizionedue rivelatori identici in grado di registrare con precisione il tempo in cui unaparticella li attraversa Come potete utilizzarli per determinare se il vostrofascio contiene elettroni o protoni

Soluzione dellrsquoesercizio 20Disponendo i due rivelatori lungo la direzione del fascio a distanza ∆L

lrsquouno dallrsquoaltro si puograve misurare il tempo impiegato dalle particelle per passaredallrsquouno allrsquoaltro e quindi la loro massa infatti se ∆t egrave la distanza temporalefra i segnali dei due rivelatori

∆L = v∆t = βc∆t =pc2

E∆t

Questo principio egrave usato nei cosiddetti rivelatori di time of flight che sono usatiper discriminare diversi tipi di particelle una particella di tipo 1 e una di tipo2 infatti percorreranno la distanza ∆L in tempi diversi legati alle rispettivemasse dalla relazione

∆L1 = ∆L2

p1c2

E1∆t1 =

radicE2 minusm2

1c2c

E∆t1 =

radicE2 minusm2

2c2c

E∆t2

13

Esercizio 21 Energia di sogliaDue fisici delle particelle vogliono produrre il bosone Z una particella di

carica neutra e di massamZ = 91 GeVc2 e discutono come fare Alice proponedi far scontrare fasci di elettroni e positroni di energia identica che viaggianodunque con impulso spaziale uguale in modulo e direzione ma di verso oppostoproducendo Z tramite il processo

e+ + eminus rarr Z

mentre Bob preferisce scontrare un fascio di protoni su un bersaglio fisso diidrogeno tramite il processo

p+ prarr Z + p+ p

Chi dei due avragrave bisogno di fasci di particelle di energia piugrave alta La massa delprotone egrave di 9383 MeVc2 quella dellrsquoelettrone di 511 keVc2

Esercizio 22 Leggi di conservazioneUn fotone (particella di massa nulla) puograve decadere in un elettrone e in un

positrone (entrambi di massa 511 keVc2) tramite il processo

γ rarr e+ + eminus

Esercizio 23 Decadimento βminus

Quali sono lrsquoenergia minima e massima dellrsquoelettrone nel decadimento

nrarr p+ eminus + νe

se il neutrone decade da fermo La massa del neutrone egrave di 9396 MeVc2 quelladel protone di 9383 MeVc2 e quella dellrsquoelettrone di 511 keVc2 si assuma chelrsquoantineutrino elettronico νe abbia massa nulla

Esercizio 24 Leggi di conservazioneIl decadimento

prarr n+ e+ + νe

egrave permesso

4 Lezione 4 (3 aprile 2020)

Esercizio 25Usando il fatto che hc = 1973 MeVfm si dimostri che in un sistema di unitagrave dimisura in cui h = c = 1 vale

14

1 1 GeVminus2 = 0389 mb

2 1 m = 5068times 1015 GeVminus1

3 1 s = 15times 1024 GeVminus1

Ricordiamo che 1 b = 1times 10minus28 m2 e che

[hc] = [Jsms] = [E][L]

Soluzione dellrsquoesercizio 25Lrsquoidea egrave di capire per quale potenza di hc e c va moltiplicato il termine a

sinistra di ciascuna equazione per ottenere il termine di destraPer cui

bull [1 GeVminus2][hc]α = [E]minus2[E]α[L]α = [0389 mb] = [L]2 da cui segue α = 2

e 1 GeVminus2(hc)2 = 1973 MeVfm1 GeV =

(01973times 10minus15 GeVm

1 GeV

)2

= 0389 mb

bull [1 m][hc]α = [L][E]α[L]α = [5068times 1015 GeVminus1] = [E]minus1 da cui segueα = minus1 e 1 m(hc)minus1 = 1 m

1973 MeVfm = 1 m01973times 10minus15 GeVm

= 5068times 1015 GeVminus1

bull [1 s][hc]α[c]β = [T ][E]α[L]α[L]β [T ]minusβ = [T ][E]α[L]α+β [T ]minusβ = [15times 1024 GeVminus1] =[E]minus1 da cui segue α = minus1 β = 1 e 1 s(hc)minus1c = 1 s

1973 MeVfm299 792 458 ms =299 792 458 m

01973times 10minus15 GeVm= 15times 1024 GeVminus1

Esercizio 26 Massa invarianteTre protoni (mp = 938 MeVc2) hanno impulsi uguali in modulo (p =

3 GeVc) e che formano angoli di 120 lrsquouno con lrsquoaltro Qual egrave la massainvariante del sistema

Esercizio 27 Energia di sogliaSi consideri il processo

γ + prarr p+ π0

dove il fotone ha massa nulla il protone ha massa di 938 MeVc2 e il π0 hamassa di 135 MeVc2

1 Se il protone egrave a riposo qual egrave lrsquoenergia minima che deve avere il fotoneincidente percheacute la reazione abbia luogo

2 La stessa reazione puograve avvenire nellrsquouniverso in cui un protone dei raggicosmici di alta energia puograve collidere con uno dei fotoni della radiazionecosmica di fondo di energia dellrsquoordine di 1 meV Qual egrave in questo casolrsquoenergia minima che deve avere il protone percheacute la reazione abbia luogo

Esercizio 28 Conseguenze della relativitagraveUn misterioso supereroe pattuglia a velocitagrave molto elevata la periferia roma-

na Allrsquoincrocio con via di Tor Bella Monaca incontra un semaforo e ndash vedendolo

15

verde ndash attraversa senza rallentare Una pattuglia della polizia municipale loferma e lo multa per esser passato col rosso Assumendo sia il supereroe che ivigili siano nel giusto a che velocitagrave viaggiava il supereroe

Soluzione dellrsquoesercizio 28Per la polizia municipale il semaforo emette fotoni di energia

E0 = hν0 =hc

λrossoasymp hc

630 nm

mentre il supereoe vede fotoni di energia

E = hν =hc

λverdeasymp hc

490 nm

e dalle trasformazioni di Lorentz indicando con γ e β le variabili calcolateusando la velocitagrave del supereroe misurata dalla municipale possiamo scrivere

E = γ(E0minusβp0) = γ(E0minusβE0) =1minus βradic1minus β2

E0 =1minus βradic

(1 + β)(1minus β)E0 =

radic1minus βradic1 + β

E0

dove abbiamo usato il fatto che i fotoni hanno massa nulla Perciograve

E

E0=

hcλverdehc

λrosso

=λrosso

λverde=

radic1minus βradic1 + β

rarr v = βc = 025c

Esercizio 29 Conseguenze della relativitagraveI neutrini sono particelle di massa molto piccola e al momento ignota Uno

dei modi con cui egrave stato possibile dedurre un limite superiore al suo valore egravestata lrsquoosservazione nel 1987 di neutrini prodotti dalla supernova 1987A1 chesi trova a 168000 anni luce dalla Terra Sono stati osservati due segnali dineutrini che possono essere schematizzati come segue si egrave osservato prima unneutrino di 35 MeV di energia seguito a 9 s di distanza da un secondo segnaledi 13 MeV Si assuma che questo ritardo sia dovuto al fatto che la massa delneutrino non egrave nulla e si calcoli questrsquoultima

Esercizio 30 DecadimentoAnimali e piante assumono dallrsquoatmosfera diversi composti contenenti car-

bonio Il carbonio presente nellrsquoatmosfera egrave predominantemente 126 C ma sono

presenti piccole concentrazioni del suo isotopo 146 C (un atomo ogni 1012) che

decade con emissione di elettroni attraverso il processo2

146 C rarr14

7 N + eminus + νe

con un tempo di dimezzamento di 5700 anni1httpsenwikipediaorgwikiSN_1987ANeutrino_emissions2Di altro non si tratta che del decadimento βminus nrarr p+ e+ νe

16

1 Qual egrave la concentrazione di 146 C dopo 11400 anni

2 Animali e piante assumono in vita proporzioni fisse di 146 C e 12

6 C mentrealla loro morte la quantitagrave di 14

6 C inizia a diminuire Avete a portata dimano un relitto di legno per cui misurate una emissione di elettroni daldecadimento di 14

6 C pari al 61 di quella di un pezzo di legno vivodella stessa massa quanti anni ha il manufatto

Esercizio 31 Energia cinetica e trasformazioni di LorentzDue particelle identiche di massa m ed energia cinetica T collidono frontal-

mente Qual egrave la loro energia cinetica relativa (ossia lrsquoenergia cinetica di unaparticella misurata nel sistema di riferimento dellrsquoaltra particella)

Esercizio 32 Energia nel centro di massaLa reazione

πminusprarr K0Λ0

avviene con unrsquoenergia nel centro di massa diradics = 3 GeV La massa del πminus egrave

di 1396 MeVc2 la massa del protone egrave di 938 MeVc2 la massa del Kminus egrave di498 MeVc2 e quella della Λ0 di 11 GeVc2

1 Calcolare lrsquoimpulso di πminus e Λ0 nel sistema di riferimento del centro dimassa

2 Se il protone egrave a riposo il K puograve essere emesso allrsquoindietro nel sistemadi riferimento del laboratorio

Esercizio 33 DecadimentoIl mesone φ0 egrave una particella neutra3 di circa 1 GeVc2 di massa che puograve

decadere in una coppia di particelle

φ0 rarr K+ +Kminus

di massa identicamK = 494 MeVc2 Si assuma di produrre φ0 di impulso notoegrave possibile che uno dei due K sia prodotto a riposo nel sistema di riferimentodel laboratorio

Esercizio 34 DecadimentoUn fascio di anti-neutrini muonici νmicro si puograve generare selezionando pioni

o kaoni π+ e K+ e facendoli passare in un lungo tubo in cui egrave stato fattoil vuoto4 in modo che dopo un certo tragitto L una buona parte di loro saragrave

3Il collisore DAFNE ai Laboratori Nazionali di Frascati produce specificatamente parti-celle di questo tipo tramite il processo e+ + eminus rarr φ0 httpswwwyoutubecomwatchv=L5yB9gDGKms

4Una tecnica di questo tipo egrave stata usata per inviare ai Laboratori Nazionali del Gran Sassodei fasci di neutrini prodotti al CERN di Ginevra httpsvideoscernchrecord985892

17

decaduta tramite i processi

π+ rarr microminus + νmicro

K+ rarr microminus + νmicro

Se lrsquoimpulso di pioni e kaoni egrave di 200 GeVc e la loro vita media di 26 ns e12 ns rispettivamente

1 Quanto a lungo viaggiano nel laboratorio i due tipi di particelle2 Se L = 1000 m quale saragrave la frazione di pioni e kaoni che saragrave decaduta

alla fine del tubo3 Qual egrave lrsquoenergia massima dei neutrini che egrave possibile misurare nel sistema

di riferimento del laboratorio nei due casi

5 Lezione 5 (16 aprile 2020)

Esercizio 35 Energia cineticaTra le eccellenze con sede in territorio elvetico primeggiano indiscutibilmente lacioccolata al latte e il Large Hadron Collider (LHC) Questrsquoultimo egrave un collisoredi particelle situato al CERN di Ginevra che fa scontrare due fasci identici diprotoni di impulso uguale in modulo e direzione ma verso opposto I fasci sonocomposti da circa 2800 gruppi (pacchetti) di 1011 particelle ciascuno Sapendoche lrsquoenergia nel centro di massa della collisione fra due protoni egrave

radics = 13 TeV

quanta cioccolata al latte dovete mangiare per assumere un numero di caloriepari allrsquoenergia cinetica di ciascun fascio di protoni di LHC

Soluzione dellrsquoesercizio 35Poicheacute si tratta di un collider e le energie in gioco sono molto maggiori della

massa del protone lrsquoenergia di ciascun protone egrave diradics2 = 65 TeV e quella

totale di un fascio egrave

2800times 1011 times 65 middot 1012 times 16 middot 10minus19J asymp 300 MJ

Secondo Google 100 g di cioccolata al latte apportano circa 500 cal = 500 times4184 J asymp 2 MJ Lrsquoenergia di un fascio di LHC corrisponde quindi a quella dicirca 15 kg di cioccolata

Esercizio 36 Scattering RutherfordUn fascio di particelle α di 100 MeV di energia e 032 nA di corrente5 collide

contro un bersaglio fisso di alluminio spesso 1 cm Una sperimentatrice prendeun rivelatore di 1 cm times 1 cm di superficie e lo posiziona ad un angolo di 30

5Per una spiegazione breve su come (e percheacute) si misura la corrente di un fascio di particel-le vedi httpswwwlhc-closerestaking_a_closer_look_at_lhc0beam_current Unatrattazione piugrave completa egrave data ad esempio da httpscdscernchrecord1213275filesp141pdf

18

rispetto al fascio di particelle a 1 m di distanza dal bersaglio Quante particelleα incideranno sul rivelatore ogni secondo

Soluzione dellrsquoesercizio 36Lrsquoalluminio ha una densitagrave di 27 gcm3 numero atomico 13 e massa atomica

27 uPoicheacute le particelle α sono nuclei di elio hanno carica 2e e la corrente di

032 nA corrisponde a un miliardo di particelle incidenti al secondo

dNidt

=032 nCs

2times 16times 10minus19 C= 1times 109 sminus1

Il rivelatore vede un angolo solido di

∆Ω equiv superficieraggio

2

=1 cm2

(1 m)2= 1times 10minus4 sr

Si tratta di uno scattering alla Rutherford per cui la sezione drsquourto per unitagravedi angolo solido rilevata ad un certo angolo θ vale

dΩ=

(zαzAle

2

4πε04E

1

sin2(θ2)

)2

pari a

dΩasymp(

2times 13times 4times etimes 16times 10minus19 C

4π times 89times 10minus12 Fmtimes 4times 100times 106 eV

1

sin2(π180 times 302)

)2

asymp 2times 10minus30 m2sr = 20 mbsr

(1)

e il numero di particelle visto dal rivelatore vale se indichiamo con nAl = ρAlNAAAl

la densitagrave numero di atomi di alluminio e con d lo spessore del rivelatore

dNrivelate

dt= ∆Ω

dΩnAld

dNi

dt

asymp 1times 10minus4 srtimes 2times 10minus30 m2srtimes 1times 104 cm2m2 times 27 gcm3 6times 1023 molminus1

27 gmol

= 120 Hz

Esercizio 37 Sezione drsquourtoUn bersaglio di idrogeno liquido di densitagrave ρ = 0071 gcm3 e volume V =

125 cm3 egrave bombardato da un fascio monoenergetico di pioni negativi con unflusso φ = 2times 107 mminus2sminus1 e si osserva la reazione

πminus + prarr π0 + n

rivelando i fotoni del decadimento π0 rarr γγ che avviene nel 988 dei casi Sela sezione drsquourto di quellrsquointerazione egrave σ = 40 mb quanti fotoni sono emessi ognisecondo

19

Esercizio 38 Sezione drsquourtoLa sezione drsquourto dellrsquoeffetto fotoelettrico per raggi X di 10 keV in carbonio

egrave 40 b per atomo Data una lastra di carbonio di 4 mm di spessore si calcoli1 il numero di bersagli per unitagrave di volume2 il coefficiente di assorbimento per effetto fotoelettrico dei raggi X di tale

energia3 la probabilitagrave che un raggio X incidente sulla lastra produca un elettrone

per effetto fotoelettrico

Esercizio 39 Sezioni drsquourtoUn bersaglio drsquooro di densitagrave superficiale ρS = 097 mgcm2 e superficie

SB = 1 cm2 viene colpito da un fascio di particelle α la cui sezione trasversaegrave contenuta completamente nellrsquoarea del bersaglio Sul bersaglio impattano37times 104 αs La sezione drsquourto di diffusione elastica ad un certo angolo θ valedσ

ddΩ = 1 bsr Calcolare1 la densitagrave di atomi del bersaglio per unitagrave di superficie2 il numero di particelle α rivelate in unrsquoora da un rivelatore di superficie

SR = 2 cm2 posto allrsquoangolo θ e a distanza DR = 01 m dal bersaglio3 Se il fascio di particelle viene sostituito da una sorgente radioattiva di

particelle α con distribuzione isotropa su tutto lrsquoangolo solido che vieneposta lungo la stessa linea del fascio a distanza DB = 20 cm dal bersaglioquanto tempo egrave necessario per rivelare con lo stesso rivelatore lo stessonumero di particelle calcolato sopra a paritagrave di sezione drsquourto

Esercizio 40 CinematicaUn fascio di positroni di 100 MeV di energia annichila su una targhetta fissa

di elettroni producendo due fotoni tramite il processo

e+ + eminus rarr γ + γ

Se uno dei due fotoni egrave emesso a 30 nel sistema del centro di massa1 quanto vale lrsquoenergia dei due fotoni nel sistema del centro di massa2 che energia e direzione avranno nel sistema del laboratorio

Esercizio 41 Sezione drsquourtoGli antineutrini νe prodotti da un reattore nucleare con potenza P = 16 GW

attraversano un bersaglio da 2000 l drsquoacqua posto a 50 m di distanza dal reat-tore

1 Supponendo che per ogni fissione sia prodotta unrsquoenergia termica di200 MeV e vengano emessi 6 antineutrini calcolare il numero medio direazioni

νe + prarr e+ + n

prodotte al giorno se la sezione drsquourto egrave σ = 1times 10minus43 cm2

20

2 Qual egrave la minima energia del neutrino per cui la reazione egrave permessa

6 Lezione 7 (8 maggio 2020)

Esercizio 42Un protone interagisce con un bersaglio producendo un pione di energia E =300 MeV Con un tracciatore posto a distanza d = 6 cm dal bersaglio egrave possi-bile rivelare la traiettoria del pione e risalire al punto di produzione del pionesul bersaglio Determinare lrsquoerrore sulla misura della posizione di tale puntocausato dalla presenza di un piano di alluminio di spessore L = 05 mm postoimmediatamente davanti al tracciatore (quindi a distanza d dal bersaglio) nellrsquoi-potesi che gli angoli delle tracce rispetto alla normale alle superfici del bersaglioe del piano di alluminio siano piccoli [mπ = 1396 MeV Al (Z = 13 A = 27ρ = 27 gcm3)]

Soluzione dellrsquoesercizio 42La deviazione standard dellrsquoangolo di diffusione coulombiana multipla vale

radic〈θ2〉 = 21 MeV

z

βc|p|

radicx

X0= 21 MeV

z

βc|p|

radicLρ

X0

dove p =radicE2 minusm2

π = 266 MeV β = pE = 0885 z = 1 Per lrsquoalluminioabbiamo

X0 = 24 gcm2 rarr X0ρ = 89 cm

per cui radic〈θ2〉 asymp 00067

e lo spostamento medio corrispondente rispetto alla posizione nominale egrave

δs = tan(00067)times 6 cm = 04 mm

Esercizio 43Un fascio misto di elettroni e antiprotoni passa attraverso una regione con

B = 2 T e dopo aver curvato per R = 3 m raggiunge una regione senza campomagnetico

1 Qual egrave lrsquoimpulso delle particelle selezionate2 Come discriminereste fra i due tipi di particelle3 Se usaste due scintillatori posti a 15 m di distanza quale risoluzione

temporale sarebbe necessaria per discriminare i due tipi di particelle4 Se i due scintillatori fossero spessi 2 cm e avessero una lunghezza di ra-

diazione X0 = 40 cm quanta energia perderebbero elettroni e protoninellrsquoattraversarli Si assuma una perdita di energia per ionizzazione di2 MeVcm e 25 MeVcm per protoni ed elettroni rispettivamente

21

5 Che indice di rifrazione dovrebbe avere un rivelatore a luce Cherenkovper discriminare elettroni e protoni

Soluzione dellrsquoesercizio 43Dalla definizione di forza

F =dpdt

= qvtimesB

|F| = mv2

R

per cui se il campo magnetico egrave ortogonale alla traiettoria

pc = qcBR

che poicheacute c = 03 mns possiamo esprimere in maniera piugrave conveniente come

pc[GeV] = 03B[T]R[m]

Ne segue che nel nostro esercizio (e tornando a c = 1)

p = 18 GeV

Trascurando le perdite di energia e ricordando che me mp i tempi divolo di elettroni e protoni valgono rispettivamente

te =L

βecasymp L

casymp 50 ns

etp =

L

βpc=

Lpradic

p2+m2p

casymp 56 ns

per cui serve una risoluzione dellrsquoordine del ns per discriminare elettroni e muonidal tempo di volo

Se gli scintillatori sono spessi 2 cm i protoni perderanno in ciascuno di essi2 cm times 2 MeVcm = 4 MeV per ionizzazione Gli elettroni perderanno invecenel primo rivelatore 5 MeV per ionizzazione e (me asymp 0)

18 GeV times(

1minus exp

(minus 2 cm

40 cm

))asymp 88 MeV

cioegrave dopo il primo rivelatore avranno 18 GeVminus93 MeV di energia e nel secondone perderanno altri

5 MeV + (18 GeV minus 93 MeV)

(1minus exp

(minus 2 cm

40 cm

))asymp 88 MeV

Lrsquoangolo di emissione di luce Cherenkov egrave dato da

cos θc =1

βnle 1

22

e poicheacute βe asymp 1 βp asymp 0887 per avere luce Cherenkov solo al passaggio deglielettroni (e non dei protoni) egrave necessario che

1 lt n lt 113

Esercizio 44Come misurereste la massa di una particella carica E quella di una parti-

cella neutra

Esercizio 45Un muone di energia E = 400 GeV penetra verticalmente nel mare Attra-

verso quale processo puograve essere rivelato A quale profonditagrave arriva prima didecadere

Esercizio 46Avete a disposizione dei tubi fotomoltiplicatori sensibili a lunghezze drsquoonda

fra 300 nm e 500 nm e volete rivelare la luce Cherenkov prodotta dal passaggio diun elettrone di 1 MeV di energia in un metro drsquoacqua Quanti fotoni vi aspettateche vengano prodotti Confrontate con il numero di elettroni di ionizzazioneche produrrebbe una particella α di 5 keV nello stesso rivelatore

Esercizio 47Determinare quali sono i processi piugrave probabili (cioegrave quelli di sezione drsquourto

piugrave alta) nellrsquointerazione fra1 fotoni di 1 MeV e atomi di alluminio2 fotoni di 100 keV e H23 fotoni di 100 keV e atomi di ferro4 fotoni di 10 MeV e atomi di carbonio5 fotoni di 10 MeV e atomi di piombo

Esercizio 48Volete misurare lrsquoimpulso di una particella carica che attraversa un campo

magnetico B ortogonale alla sua traiettoria Avete a disposizione tre rivelatori diposizione della stessa risoluzione spaziale δx come li disponete Che risoluzionein impulso vi aspettate di ottenere

23

  • Lezione 1 (13 marzo 2020)
  • Lezione 2 (20 marzo 2020)
  • Lezione 3 (27 marzo 2020)
  • Lezione 4 (3 aprile 2020)
  • Lezione 5 (16 aprile 2020)
  • Lezione 7 (8 maggio 2020)
Page 6: Esercizi per casa (risolti)ippolitv/pdf/fns1_2020_solutions.pdfEsercizi per casa (risolti) Valerio Ippolito 13 maggio 2020 1 Lezione 1 (13 marzo 2020) Esercizio1 Dilatazionedeitempi

ed elevando al quadrato

β =

radicλ2

1 + λ2asymp 080

Esercizio 10 Calcolo tensorialeUn tensore le cui componenti contravarianti si indicano come amicroν egrave la ge-

neralizzazione a due indici di un quadrivettore Mentre un quadrivettore ha 4componenti un tensore a due indici ne ha 4times 4 = 16 Se nel passaggio da unsistema di riferimento O in quiete a uno Oprime in moto lungo lrsquoasse x con velocitagraveV = βc le coordinate di un quadrivettore trasformano secondo la legge

xmicroprime = Λmicroνxν

dove le Λmicroν sono le componenti della matrice

Λ =

γ minusβγ 0 0minusβγ γ 0 0

0 0 1 00 0 0 1

analogamente un tensore trasforma secondo

amicroν prime = ΛmicroρΛνσaρσ

Analogamente a quanto accade per i quadrivettori le coordinate contravariantidi un tensore sono legate a quelle covarianti dalla relazione

amicroν = gmicroρgνσaρσ

Un tensore si dice simmetrico se le sue componenti sono uguali sotto scambiodegli indici (amicroν = aνmicro) e antisimmetrico se sono uguali ma di segno opposto(amicroν = minusaνmicro)

1 Quanti elementi indipendenti ci sono in un tensore simmetrico (siconsiderano dipendenti ad esempio a12 e a21 = a12)

2 E in un tensore antisimmetrico3 Simmetria e antisimmetria sono caratteristiche che si mantengono sotto

trasformazione di Lorentz4 Se un tensore amicroν egrave simmetrico lo egrave anche la sua versione covariante amicroν

E se egrave antisimmetrico5 Se amicroν egrave un tensore simmetrico e bmicroν un tensore antisimmetrico quanto

vale amicroνbmicroν6 Scomporre un tensore generico amicroν nella somma di due tensori uno

simmetrico e lrsquoaltro antisimmetricoSoluzione dellrsquoesercizio 10

6

1 Un tensore simmetrico ha 10 componenti indipendenti (a11 a12 a13 a14 a22 a23 a24 a33 a34 a44)

2 Un tensore antisimmetrico ha 6 componenti indipendenti (a12 a13 a14 a23 a24 a34)

3 Se un tensore egrave simmetrico significa che

amicroν = aνmicro

La versione trasformata del termine a sinistra egrave

amicroν prime = ΛmicroρΛνσaρσ

mentre quella del termine di destra egrave

aνmicroprime = ΛνρΛmicroσaρσ

ma applicando la definizione di tensore simmetrico nel riferimento di par-tenza (coordinate senza apici) e applicando la proprietagrave commutativa aitermini della matrice di Lorentz si ha

aνmicroprime = ΛνρΛmicroσaρσ = ΛνρΛmicroσa

σρ = ΛmicroσΛνρaσρ = ΛmicroρΛνσa

ρσ = amicroν prime

dove abbiamo ndash nellrsquoultimo passaggio ndash usato il fatto che sia ρ che σ sonoindici muti Ne concludiamo che un tensore simmetrico rimane tale dopotrasformazione di Lorentz

Se il tensore egrave antisimmetrico analogamente significa che

amicroν = minusaνmicro

La versione trasformata del termine a sinistra egrave sempre

amicroν prime = ΛmicroρΛνσaρσ

Applicando ancora una volta la definizione di tensore antisimmetrico nelriferimento di partenza e la proprietagrave commutativa si ha

aνmicroprime = ΛνρΛmicroσaρσ = ΛνρΛmicroσ (minusaσρ) = minusΛmicroσΛνρa

σρ = minusΛmicroρΛνσaρσ = minusamicroν prime

cioegrave anche lrsquoantisimmetria egrave mantenuta dopo trasformazione di Lorentz

4 Per il tensore simmetrico

amicroν = gmicroρgνσaρσ = gmicroρgνσa

σρ = gνσgmicroρaσρ = amicroν

Nel caso antisimmetrico

amicroν = gmicroρgνσaρσ = gmicroρgνσ (minusaσρ) = minusgνσgmicroρaσρ = minusamicroν

7

5 Vale

amicroνbmicroν = aνmicrob

νmicro = amicroν (minusbmicroν) = minus (amicroνbmicroν)rarr amicroνbmicroν = 0

dove al primo passaggio abbiamo semplicemente rinominato gli indici mu-ti e al secondo abbiamo usato le proprietagrave di simmetriaantisimmetriadei tensori di partenza Ne segue dunque che il prodotto fra un tensoresimmetrico e uno antisimmetrico egrave zero

Suggerimento almeno una volta nella vita conviene esplicitare la som-ma implicita nella notazione di Einstein per comprendere davvero questoformalismo cosigrave compatto

6 Basta costruire due somme che per costruzione allo scambio di indicisono lrsquouna simmetrica

amicroν equiv 1

2(tmicroν + tνmicro)

e lrsquoaltra antisimmetrica cioegrave

amicroν equiv 1

2(tmicroν minus tνmicro)

Egrave evidente che questi due tensori sono simmetriciantisimmetrici sottoscambio degli indici

Esercizio 11 Relazione fra forza e accelerazioneIn relativitagrave speciale forza e accelerazione sono in generale proporzionali fra

loro Usare la definizione di forza F = dpdt

Suggerimento scomporre lrsquoaccelerazione nella somma di un termine paral-lelo e un termine ortogonale alla direzione del moto (cioegrave alla velocitagrave)

Soluzione dellrsquoesercizio 11Scomponiamo lrsquoaccelerazione a lungo le direzioni ortogonale e parallela al

moto della particella (cioegrave a v)

a equiv aperp + a

Usando il fatto che

dt=

dv

dv

dt=

d

[(1minus v2

c2

)minus 12

]dt

dv

dt= minus1

2

(minus2v

c2

)(1minus v2

c2

)minus 32 dv

dt=

v

c2γ3 dv

dt

e che dalla definizione di prodotto scalare fra vettori

vdv

dt= v middot a

8

otteniamo che

F =dpdt

=d(mγv)

dt= mγ

dvdt

+mdγ

dtv

= mγa +mγ3 (v middot a) vc2

= mγ(aperp + a) +mγ3(v middot aperp + v middot a)v

= mγaperp +mγ3

(1

γ2+v2

c2

)a

= mγaperp +mγ3

(1minus v2

c2+v2

c2

)a

= mγaperp +mγ3a

Si noti come in relativitagrave speciale la forza non egrave in generale proporzionaleallrsquoaccelerazione

Esercizio 12 Classificazione dei quadrivettoriIl quadrimpulso egrave un quadrivettore di tipo spazio tempo o luce

Soluzione dellrsquoesercizio 12Poicheacute P 2 = E2

c2 minus p2 = m2 ge 0 saragrave di tipo tempo per particelle massive (e

di tipo luce per particelle senza massa)

Esercizio 13 Energia cineticaQuanto lavoro bisogna compiere per aumentare la velocitagrave di un elettrone

(m = 511 keVc2) dalla posizione di riposo a1 050c2 0990c3 09990c

Soluzione dellrsquoesercizio 13A questi elettroni dovremo dare una certa energia cinetica T in modo da

far passare lrsquoenergia totale da quella a riposo (γ = 0) cioegrave

Ei = mc2

aEf = T +mc2

Dalla relazioneEf = mγc2

segue

T = Ef minusmc2 = m(γ minus 1)c2 = m

1radic1minus v2

c2

minus 1

c2

per cui nei tre casi indicati servono rispettivamente 79 keV 31 MeV e 109 MeV

9

Esercizio 14 Energia di sogliaSupponiamo di far scontrare un fascio di protoni con un protone a riposo

Qual egrave lrsquoenergia minima che devono avere i protoni del fascio percheacute la reazione

p+ prarr p+ p+ p+ p

sia permessa (La massa del protone egrave pari a quella dellrsquoantiprotone p e vale938 MeVc2)

Esercizio 15 Diffusione elasticaChiamiamo elastico un urto (ldquoscatteringrdquo) in cui le particelle dello stato ini-

ziale e dello stato finale sono le stesse Si consideri un urto elastico fra unaparticella di massa nulla e una particella di massa m (bersaglio) che si trovaa riposo nel sistema di riferimento del laboratorio qual egrave la massima energiatrasferita dalla particella incidente al bersaglio Suggerimento si lavori nelsistema di riferimento del laboratorio e si espliciti il prodotto scalare fra gliimpulsi spaziali della particella di massa nulla prima e dopo lrsquourto in funzionedellrsquoangolo sempre nel sistema di riferimento del laboratorio fra la direzioneiniziale e finale della particella incidente

Se la particella incidente egrave un fotone e il bersaglio egrave un elettrone atomico ariposo di quanto varia la lunghezza drsquoonda del fotone fra prima e dopo lrsquourto

Soluzione dellrsquoesercizio 15Per scattering elastico intendiamo un processo in cui le particelle dello stato

iniziale sono le stesse di quelle dello stato finaleDenotiamo con k e P i quadrimpulsi della particella incidente e del bersa-

glio prima dellrsquourto e indichiamo con lrsquoapice le stesse quantitagrave dopo lrsquourto ilproblema ci dice che

k = (Ek)

kprime = (Eprimekrsquo)P = (mc0)

Partiamo dalla conservazione del quadrimpulso durante lrsquourto isoliamo la quan-titagrave che non misuriamo direttamente ndash cioegrave il quadrimpulso del bersaglio dopolrsquourto P prime ndash ed eleviamo al quadrato

k + P = kprime + P prime

P prime = k + P minus kprimem2c2 = 0 +m2c2 + 0 + 2Emminus 2(EEprime minus k middot krsquo)minus 2mEprime

e se indichiamo con θprime lrsquoangolo ndash nel riferimento del laboratorio ndash fra la direzioneiniziale e finale della particella incidente e usiamo il fatto che |k|c = E e |krsquo|c =

10

Eprime

0 = 2mc2(E minus Eprime)minus 2(EEprime minus EEprime cos θprime)

mc2(Eprime minus E) = minusEEprime(1minus cos θprime)

Eprime(mc2 + E(1minus cos θprime)) = mc2E

Eprime =E

1 + Emc2 (1minus cos θprime)

Il bersaglio rinculeragrave di una energia E minus Eprime massima per θ = π Il valoremassimo di questrsquoenergia di rinculo

E minus E

1 + 2 Emc2

= E2Emc2

1 + 2Emc2

prende il nome ndash nel caso dello scattering Compton in cui la particella incidenteegrave un fotone e il bersaglio egrave un elettrone atomico ndash di picco Compton

Cosa cambia fra un fotone di energia E ed uno di energia Eprime Dalla mecca-nica quantistica

E = hν =hc

λ

cioegrave cambia la lunghezza drsquoonda del fotone

Eprime =hc

λprime=

hcλ

1 +hcλ

mc2 (1minus cos θprime)

1

λprime=

1 +hcλ

mc2 (1minus cos θprime)

λprime = λ

(1 +

hc

λmc2(1minus cos θprime)

)

λprime = λ+h

mc(1minus cos θprime) equiv λ+ λc(1minus cos θprime)

dove abbiamo definito la lunghezza drsquoonda Compton dellrsquoelettrone λc che rap-presenta la scala di lunghezza sotto la quale gli effetti della meccanica quantisticarelativistica divengono importanti

3 Lezione 3 (27 marzo 2020)

Esercizio 16 Trasformazione delle velocitagraveUn oggetto si muove di moto rettilineo uniforme con velocitagrave costante αc versoun secondo oggetto immobile A che velocitagrave dobbiamo muoverci noi lungo lastessa direzione per vedere entrambi gli oggetti muoversi con velocitagrave uguali eopposte

11

Soluzione dellrsquoesercizio 16Se indichiamo con lrsquoapice la velocitagrave nel nostro sistema di riferimento che

egrave in moto rispetto al sistema di riferimento del secondo oggetto la legge ditrasformazione delle velocitagrave ci dice che

vprime1 =v1 minus V

1minus v1cVc

=αcminus V1minus αVc

vprime2 =v2 minus V

1minus v2cVc

= minusV

Stiamo cercando V la nostra velocitagrave nel sistema di riferimento del secondooggetto tale che le velocitagrave dei due oggetti nel nostro sistema di riferimentosiano uguali e opposte

vprime1 = minusvprime2 = V

perciograve la richiesta egrave

αcminus V1minus αVc

= V

αcminus V minus V + αV 2

c= 0

(αc)V 2 + (minus2)V + (α)c = 0

V =1minusradic

1minus α2

αc

dove abbiamo scelto la soluzione dellrsquoequazione di secondo grado con V le c

Esercizio 17 Conseguenze della relativitagraveUna navicella spaziale in moto rettilineo uniforme con velocitagrave 05c in al-

lontanamento dalla Terra egrave in orbita verso Plutone che si trova a 75times 109 kmdi distanza dalla Terra Non appena raggiunto il pianeta la comandante inviaun segnale radio alla base a Houston per chiedere lrsquoautorizzazione allrsquoatter-raggio Quanto tempo impiega la richiesta a raggiungere la base secondo lacomandante E secondo i suoi colleghi a Houston

Soluzione dellrsquoesercizio 17Lrsquoonda radio viaggia a velocitagrave c secondo ogni sistema di riferimento Per

lrsquoosservatore lrsquoonda radio percorreragrave il tragitto Plutone-Terra in un tempo

tTerra =L

c=

75times 109 km

3times 108 msasymp 25 000 s

mentre per la comandante saragrave passato un tempo inferiore che dipende dal fat-tore γ dellrsquoastronave nel sistema di riferimento della Terra secondo la relazione

tcomandante =tTerra

γasymp 25 000 stimes

radic1minus 052 asymp 21 650 s

12

Esercizio 18 Energia cineticaHa piugrave energia un protone che viaggia a 09999999896c o un Boeing 747 al

decollo

Esercizio 19 Dilatazione dei tempiUna scienziata misura che un fascio di particelle selezionate con impulso di

10 GeVc si degrada dellrsquo84 dopo aver percorso 1 m Se la massa di questeparticelle egrave 498 MeVc2 qual egrave la loro vita media

Soluzione dellrsquoesercizio 19La distanza misurata dalla scienziata egrave chiaramente riferita al suo sistema

di riferimento quello del laboratorio ed egrave legata alla vita media delle particelledel fascio τ dalla relazione

Ldecay = 1 m = βγcτ =pc

E

E

mc2cτ =

p

mccτ

e dalla legge del decadimento se indichiamo con N0 il numero di particelleinizialmente presenti nel fascio e con N il numero di particelle misurato si haN

N0= 1minus084 = exp

(minus L

Ldecay

)= exp

(minus L

pmccτ

)rarr τ =

mcL

pc log(N0

N

) asymp 9times 10minus11 s

Esercizio 20 Dilatazione dei tempiVi trovate a dover studiare un fascio di particelle di cui conoscete lrsquoenergia

ndash 2 GeV ndash ma non la massa a quanto ne sapete potrebbero essere composti daelettroni (di massa 511 keVc2) o protoni (938 MeVc2) Avete a disposizionedue rivelatori identici in grado di registrare con precisione il tempo in cui unaparticella li attraversa Come potete utilizzarli per determinare se il vostrofascio contiene elettroni o protoni

Soluzione dellrsquoesercizio 20Disponendo i due rivelatori lungo la direzione del fascio a distanza ∆L

lrsquouno dallrsquoaltro si puograve misurare il tempo impiegato dalle particelle per passaredallrsquouno allrsquoaltro e quindi la loro massa infatti se ∆t egrave la distanza temporalefra i segnali dei due rivelatori

∆L = v∆t = βc∆t =pc2

E∆t

Questo principio egrave usato nei cosiddetti rivelatori di time of flight che sono usatiper discriminare diversi tipi di particelle una particella di tipo 1 e una di tipo2 infatti percorreranno la distanza ∆L in tempi diversi legati alle rispettivemasse dalla relazione

∆L1 = ∆L2

p1c2

E1∆t1 =

radicE2 minusm2

1c2c

E∆t1 =

radicE2 minusm2

2c2c

E∆t2

13

Esercizio 21 Energia di sogliaDue fisici delle particelle vogliono produrre il bosone Z una particella di

carica neutra e di massamZ = 91 GeVc2 e discutono come fare Alice proponedi far scontrare fasci di elettroni e positroni di energia identica che viaggianodunque con impulso spaziale uguale in modulo e direzione ma di verso oppostoproducendo Z tramite il processo

e+ + eminus rarr Z

mentre Bob preferisce scontrare un fascio di protoni su un bersaglio fisso diidrogeno tramite il processo

p+ prarr Z + p+ p

Chi dei due avragrave bisogno di fasci di particelle di energia piugrave alta La massa delprotone egrave di 9383 MeVc2 quella dellrsquoelettrone di 511 keVc2

Esercizio 22 Leggi di conservazioneUn fotone (particella di massa nulla) puograve decadere in un elettrone e in un

positrone (entrambi di massa 511 keVc2) tramite il processo

γ rarr e+ + eminus

Esercizio 23 Decadimento βminus

Quali sono lrsquoenergia minima e massima dellrsquoelettrone nel decadimento

nrarr p+ eminus + νe

se il neutrone decade da fermo La massa del neutrone egrave di 9396 MeVc2 quelladel protone di 9383 MeVc2 e quella dellrsquoelettrone di 511 keVc2 si assuma chelrsquoantineutrino elettronico νe abbia massa nulla

Esercizio 24 Leggi di conservazioneIl decadimento

prarr n+ e+ + νe

egrave permesso

4 Lezione 4 (3 aprile 2020)

Esercizio 25Usando il fatto che hc = 1973 MeVfm si dimostri che in un sistema di unitagrave dimisura in cui h = c = 1 vale

14

1 1 GeVminus2 = 0389 mb

2 1 m = 5068times 1015 GeVminus1

3 1 s = 15times 1024 GeVminus1

Ricordiamo che 1 b = 1times 10minus28 m2 e che

[hc] = [Jsms] = [E][L]

Soluzione dellrsquoesercizio 25Lrsquoidea egrave di capire per quale potenza di hc e c va moltiplicato il termine a

sinistra di ciascuna equazione per ottenere il termine di destraPer cui

bull [1 GeVminus2][hc]α = [E]minus2[E]α[L]α = [0389 mb] = [L]2 da cui segue α = 2

e 1 GeVminus2(hc)2 = 1973 MeVfm1 GeV =

(01973times 10minus15 GeVm

1 GeV

)2

= 0389 mb

bull [1 m][hc]α = [L][E]α[L]α = [5068times 1015 GeVminus1] = [E]minus1 da cui segueα = minus1 e 1 m(hc)minus1 = 1 m

1973 MeVfm = 1 m01973times 10minus15 GeVm

= 5068times 1015 GeVminus1

bull [1 s][hc]α[c]β = [T ][E]α[L]α[L]β [T ]minusβ = [T ][E]α[L]α+β [T ]minusβ = [15times 1024 GeVminus1] =[E]minus1 da cui segue α = minus1 β = 1 e 1 s(hc)minus1c = 1 s

1973 MeVfm299 792 458 ms =299 792 458 m

01973times 10minus15 GeVm= 15times 1024 GeVminus1

Esercizio 26 Massa invarianteTre protoni (mp = 938 MeVc2) hanno impulsi uguali in modulo (p =

3 GeVc) e che formano angoli di 120 lrsquouno con lrsquoaltro Qual egrave la massainvariante del sistema

Esercizio 27 Energia di sogliaSi consideri il processo

γ + prarr p+ π0

dove il fotone ha massa nulla il protone ha massa di 938 MeVc2 e il π0 hamassa di 135 MeVc2

1 Se il protone egrave a riposo qual egrave lrsquoenergia minima che deve avere il fotoneincidente percheacute la reazione abbia luogo

2 La stessa reazione puograve avvenire nellrsquouniverso in cui un protone dei raggicosmici di alta energia puograve collidere con uno dei fotoni della radiazionecosmica di fondo di energia dellrsquoordine di 1 meV Qual egrave in questo casolrsquoenergia minima che deve avere il protone percheacute la reazione abbia luogo

Esercizio 28 Conseguenze della relativitagraveUn misterioso supereroe pattuglia a velocitagrave molto elevata la periferia roma-

na Allrsquoincrocio con via di Tor Bella Monaca incontra un semaforo e ndash vedendolo

15

verde ndash attraversa senza rallentare Una pattuglia della polizia municipale loferma e lo multa per esser passato col rosso Assumendo sia il supereroe che ivigili siano nel giusto a che velocitagrave viaggiava il supereroe

Soluzione dellrsquoesercizio 28Per la polizia municipale il semaforo emette fotoni di energia

E0 = hν0 =hc

λrossoasymp hc

630 nm

mentre il supereoe vede fotoni di energia

E = hν =hc

λverdeasymp hc

490 nm

e dalle trasformazioni di Lorentz indicando con γ e β le variabili calcolateusando la velocitagrave del supereroe misurata dalla municipale possiamo scrivere

E = γ(E0minusβp0) = γ(E0minusβE0) =1minus βradic1minus β2

E0 =1minus βradic

(1 + β)(1minus β)E0 =

radic1minus βradic1 + β

E0

dove abbiamo usato il fatto che i fotoni hanno massa nulla Perciograve

E

E0=

hcλverdehc

λrosso

=λrosso

λverde=

radic1minus βradic1 + β

rarr v = βc = 025c

Esercizio 29 Conseguenze della relativitagraveI neutrini sono particelle di massa molto piccola e al momento ignota Uno

dei modi con cui egrave stato possibile dedurre un limite superiore al suo valore egravestata lrsquoosservazione nel 1987 di neutrini prodotti dalla supernova 1987A1 chesi trova a 168000 anni luce dalla Terra Sono stati osservati due segnali dineutrini che possono essere schematizzati come segue si egrave osservato prima unneutrino di 35 MeV di energia seguito a 9 s di distanza da un secondo segnaledi 13 MeV Si assuma che questo ritardo sia dovuto al fatto che la massa delneutrino non egrave nulla e si calcoli questrsquoultima

Esercizio 30 DecadimentoAnimali e piante assumono dallrsquoatmosfera diversi composti contenenti car-

bonio Il carbonio presente nellrsquoatmosfera egrave predominantemente 126 C ma sono

presenti piccole concentrazioni del suo isotopo 146 C (un atomo ogni 1012) che

decade con emissione di elettroni attraverso il processo2

146 C rarr14

7 N + eminus + νe

con un tempo di dimezzamento di 5700 anni1httpsenwikipediaorgwikiSN_1987ANeutrino_emissions2Di altro non si tratta che del decadimento βminus nrarr p+ e+ νe

16

1 Qual egrave la concentrazione di 146 C dopo 11400 anni

2 Animali e piante assumono in vita proporzioni fisse di 146 C e 12

6 C mentrealla loro morte la quantitagrave di 14

6 C inizia a diminuire Avete a portata dimano un relitto di legno per cui misurate una emissione di elettroni daldecadimento di 14

6 C pari al 61 di quella di un pezzo di legno vivodella stessa massa quanti anni ha il manufatto

Esercizio 31 Energia cinetica e trasformazioni di LorentzDue particelle identiche di massa m ed energia cinetica T collidono frontal-

mente Qual egrave la loro energia cinetica relativa (ossia lrsquoenergia cinetica di unaparticella misurata nel sistema di riferimento dellrsquoaltra particella)

Esercizio 32 Energia nel centro di massaLa reazione

πminusprarr K0Λ0

avviene con unrsquoenergia nel centro di massa diradics = 3 GeV La massa del πminus egrave

di 1396 MeVc2 la massa del protone egrave di 938 MeVc2 la massa del Kminus egrave di498 MeVc2 e quella della Λ0 di 11 GeVc2

1 Calcolare lrsquoimpulso di πminus e Λ0 nel sistema di riferimento del centro dimassa

2 Se il protone egrave a riposo il K puograve essere emesso allrsquoindietro nel sistemadi riferimento del laboratorio

Esercizio 33 DecadimentoIl mesone φ0 egrave una particella neutra3 di circa 1 GeVc2 di massa che puograve

decadere in una coppia di particelle

φ0 rarr K+ +Kminus

di massa identicamK = 494 MeVc2 Si assuma di produrre φ0 di impulso notoegrave possibile che uno dei due K sia prodotto a riposo nel sistema di riferimentodel laboratorio

Esercizio 34 DecadimentoUn fascio di anti-neutrini muonici νmicro si puograve generare selezionando pioni

o kaoni π+ e K+ e facendoli passare in un lungo tubo in cui egrave stato fattoil vuoto4 in modo che dopo un certo tragitto L una buona parte di loro saragrave

3Il collisore DAFNE ai Laboratori Nazionali di Frascati produce specificatamente parti-celle di questo tipo tramite il processo e+ + eminus rarr φ0 httpswwwyoutubecomwatchv=L5yB9gDGKms

4Una tecnica di questo tipo egrave stata usata per inviare ai Laboratori Nazionali del Gran Sassodei fasci di neutrini prodotti al CERN di Ginevra httpsvideoscernchrecord985892

17

decaduta tramite i processi

π+ rarr microminus + νmicro

K+ rarr microminus + νmicro

Se lrsquoimpulso di pioni e kaoni egrave di 200 GeVc e la loro vita media di 26 ns e12 ns rispettivamente

1 Quanto a lungo viaggiano nel laboratorio i due tipi di particelle2 Se L = 1000 m quale saragrave la frazione di pioni e kaoni che saragrave decaduta

alla fine del tubo3 Qual egrave lrsquoenergia massima dei neutrini che egrave possibile misurare nel sistema

di riferimento del laboratorio nei due casi

5 Lezione 5 (16 aprile 2020)

Esercizio 35 Energia cineticaTra le eccellenze con sede in territorio elvetico primeggiano indiscutibilmente lacioccolata al latte e il Large Hadron Collider (LHC) Questrsquoultimo egrave un collisoredi particelle situato al CERN di Ginevra che fa scontrare due fasci identici diprotoni di impulso uguale in modulo e direzione ma verso opposto I fasci sonocomposti da circa 2800 gruppi (pacchetti) di 1011 particelle ciascuno Sapendoche lrsquoenergia nel centro di massa della collisione fra due protoni egrave

radics = 13 TeV

quanta cioccolata al latte dovete mangiare per assumere un numero di caloriepari allrsquoenergia cinetica di ciascun fascio di protoni di LHC

Soluzione dellrsquoesercizio 35Poicheacute si tratta di un collider e le energie in gioco sono molto maggiori della

massa del protone lrsquoenergia di ciascun protone egrave diradics2 = 65 TeV e quella

totale di un fascio egrave

2800times 1011 times 65 middot 1012 times 16 middot 10minus19J asymp 300 MJ

Secondo Google 100 g di cioccolata al latte apportano circa 500 cal = 500 times4184 J asymp 2 MJ Lrsquoenergia di un fascio di LHC corrisponde quindi a quella dicirca 15 kg di cioccolata

Esercizio 36 Scattering RutherfordUn fascio di particelle α di 100 MeV di energia e 032 nA di corrente5 collide

contro un bersaglio fisso di alluminio spesso 1 cm Una sperimentatrice prendeun rivelatore di 1 cm times 1 cm di superficie e lo posiziona ad un angolo di 30

5Per una spiegazione breve su come (e percheacute) si misura la corrente di un fascio di particel-le vedi httpswwwlhc-closerestaking_a_closer_look_at_lhc0beam_current Unatrattazione piugrave completa egrave data ad esempio da httpscdscernchrecord1213275filesp141pdf

18

rispetto al fascio di particelle a 1 m di distanza dal bersaglio Quante particelleα incideranno sul rivelatore ogni secondo

Soluzione dellrsquoesercizio 36Lrsquoalluminio ha una densitagrave di 27 gcm3 numero atomico 13 e massa atomica

27 uPoicheacute le particelle α sono nuclei di elio hanno carica 2e e la corrente di

032 nA corrisponde a un miliardo di particelle incidenti al secondo

dNidt

=032 nCs

2times 16times 10minus19 C= 1times 109 sminus1

Il rivelatore vede un angolo solido di

∆Ω equiv superficieraggio

2

=1 cm2

(1 m)2= 1times 10minus4 sr

Si tratta di uno scattering alla Rutherford per cui la sezione drsquourto per unitagravedi angolo solido rilevata ad un certo angolo θ vale

dΩ=

(zαzAle

2

4πε04E

1

sin2(θ2)

)2

pari a

dΩasymp(

2times 13times 4times etimes 16times 10minus19 C

4π times 89times 10minus12 Fmtimes 4times 100times 106 eV

1

sin2(π180 times 302)

)2

asymp 2times 10minus30 m2sr = 20 mbsr

(1)

e il numero di particelle visto dal rivelatore vale se indichiamo con nAl = ρAlNAAAl

la densitagrave numero di atomi di alluminio e con d lo spessore del rivelatore

dNrivelate

dt= ∆Ω

dΩnAld

dNi

dt

asymp 1times 10minus4 srtimes 2times 10minus30 m2srtimes 1times 104 cm2m2 times 27 gcm3 6times 1023 molminus1

27 gmol

= 120 Hz

Esercizio 37 Sezione drsquourtoUn bersaglio di idrogeno liquido di densitagrave ρ = 0071 gcm3 e volume V =

125 cm3 egrave bombardato da un fascio monoenergetico di pioni negativi con unflusso φ = 2times 107 mminus2sminus1 e si osserva la reazione

πminus + prarr π0 + n

rivelando i fotoni del decadimento π0 rarr γγ che avviene nel 988 dei casi Sela sezione drsquourto di quellrsquointerazione egrave σ = 40 mb quanti fotoni sono emessi ognisecondo

19

Esercizio 38 Sezione drsquourtoLa sezione drsquourto dellrsquoeffetto fotoelettrico per raggi X di 10 keV in carbonio

egrave 40 b per atomo Data una lastra di carbonio di 4 mm di spessore si calcoli1 il numero di bersagli per unitagrave di volume2 il coefficiente di assorbimento per effetto fotoelettrico dei raggi X di tale

energia3 la probabilitagrave che un raggio X incidente sulla lastra produca un elettrone

per effetto fotoelettrico

Esercizio 39 Sezioni drsquourtoUn bersaglio drsquooro di densitagrave superficiale ρS = 097 mgcm2 e superficie

SB = 1 cm2 viene colpito da un fascio di particelle α la cui sezione trasversaegrave contenuta completamente nellrsquoarea del bersaglio Sul bersaglio impattano37times 104 αs La sezione drsquourto di diffusione elastica ad un certo angolo θ valedσ

ddΩ = 1 bsr Calcolare1 la densitagrave di atomi del bersaglio per unitagrave di superficie2 il numero di particelle α rivelate in unrsquoora da un rivelatore di superficie

SR = 2 cm2 posto allrsquoangolo θ e a distanza DR = 01 m dal bersaglio3 Se il fascio di particelle viene sostituito da una sorgente radioattiva di

particelle α con distribuzione isotropa su tutto lrsquoangolo solido che vieneposta lungo la stessa linea del fascio a distanza DB = 20 cm dal bersaglioquanto tempo egrave necessario per rivelare con lo stesso rivelatore lo stessonumero di particelle calcolato sopra a paritagrave di sezione drsquourto

Esercizio 40 CinematicaUn fascio di positroni di 100 MeV di energia annichila su una targhetta fissa

di elettroni producendo due fotoni tramite il processo

e+ + eminus rarr γ + γ

Se uno dei due fotoni egrave emesso a 30 nel sistema del centro di massa1 quanto vale lrsquoenergia dei due fotoni nel sistema del centro di massa2 che energia e direzione avranno nel sistema del laboratorio

Esercizio 41 Sezione drsquourtoGli antineutrini νe prodotti da un reattore nucleare con potenza P = 16 GW

attraversano un bersaglio da 2000 l drsquoacqua posto a 50 m di distanza dal reat-tore

1 Supponendo che per ogni fissione sia prodotta unrsquoenergia termica di200 MeV e vengano emessi 6 antineutrini calcolare il numero medio direazioni

νe + prarr e+ + n

prodotte al giorno se la sezione drsquourto egrave σ = 1times 10minus43 cm2

20

2 Qual egrave la minima energia del neutrino per cui la reazione egrave permessa

6 Lezione 7 (8 maggio 2020)

Esercizio 42Un protone interagisce con un bersaglio producendo un pione di energia E =300 MeV Con un tracciatore posto a distanza d = 6 cm dal bersaglio egrave possi-bile rivelare la traiettoria del pione e risalire al punto di produzione del pionesul bersaglio Determinare lrsquoerrore sulla misura della posizione di tale puntocausato dalla presenza di un piano di alluminio di spessore L = 05 mm postoimmediatamente davanti al tracciatore (quindi a distanza d dal bersaglio) nellrsquoi-potesi che gli angoli delle tracce rispetto alla normale alle superfici del bersaglioe del piano di alluminio siano piccoli [mπ = 1396 MeV Al (Z = 13 A = 27ρ = 27 gcm3)]

Soluzione dellrsquoesercizio 42La deviazione standard dellrsquoangolo di diffusione coulombiana multipla vale

radic〈θ2〉 = 21 MeV

z

βc|p|

radicx

X0= 21 MeV

z

βc|p|

radicLρ

X0

dove p =radicE2 minusm2

π = 266 MeV β = pE = 0885 z = 1 Per lrsquoalluminioabbiamo

X0 = 24 gcm2 rarr X0ρ = 89 cm

per cui radic〈θ2〉 asymp 00067

e lo spostamento medio corrispondente rispetto alla posizione nominale egrave

δs = tan(00067)times 6 cm = 04 mm

Esercizio 43Un fascio misto di elettroni e antiprotoni passa attraverso una regione con

B = 2 T e dopo aver curvato per R = 3 m raggiunge una regione senza campomagnetico

1 Qual egrave lrsquoimpulso delle particelle selezionate2 Come discriminereste fra i due tipi di particelle3 Se usaste due scintillatori posti a 15 m di distanza quale risoluzione

temporale sarebbe necessaria per discriminare i due tipi di particelle4 Se i due scintillatori fossero spessi 2 cm e avessero una lunghezza di ra-

diazione X0 = 40 cm quanta energia perderebbero elettroni e protoninellrsquoattraversarli Si assuma una perdita di energia per ionizzazione di2 MeVcm e 25 MeVcm per protoni ed elettroni rispettivamente

21

5 Che indice di rifrazione dovrebbe avere un rivelatore a luce Cherenkovper discriminare elettroni e protoni

Soluzione dellrsquoesercizio 43Dalla definizione di forza

F =dpdt

= qvtimesB

|F| = mv2

R

per cui se il campo magnetico egrave ortogonale alla traiettoria

pc = qcBR

che poicheacute c = 03 mns possiamo esprimere in maniera piugrave conveniente come

pc[GeV] = 03B[T]R[m]

Ne segue che nel nostro esercizio (e tornando a c = 1)

p = 18 GeV

Trascurando le perdite di energia e ricordando che me mp i tempi divolo di elettroni e protoni valgono rispettivamente

te =L

βecasymp L

casymp 50 ns

etp =

L

βpc=

Lpradic

p2+m2p

casymp 56 ns

per cui serve una risoluzione dellrsquoordine del ns per discriminare elettroni e muonidal tempo di volo

Se gli scintillatori sono spessi 2 cm i protoni perderanno in ciascuno di essi2 cm times 2 MeVcm = 4 MeV per ionizzazione Gli elettroni perderanno invecenel primo rivelatore 5 MeV per ionizzazione e (me asymp 0)

18 GeV times(

1minus exp

(minus 2 cm

40 cm

))asymp 88 MeV

cioegrave dopo il primo rivelatore avranno 18 GeVminus93 MeV di energia e nel secondone perderanno altri

5 MeV + (18 GeV minus 93 MeV)

(1minus exp

(minus 2 cm

40 cm

))asymp 88 MeV

Lrsquoangolo di emissione di luce Cherenkov egrave dato da

cos θc =1

βnle 1

22

e poicheacute βe asymp 1 βp asymp 0887 per avere luce Cherenkov solo al passaggio deglielettroni (e non dei protoni) egrave necessario che

1 lt n lt 113

Esercizio 44Come misurereste la massa di una particella carica E quella di una parti-

cella neutra

Esercizio 45Un muone di energia E = 400 GeV penetra verticalmente nel mare Attra-

verso quale processo puograve essere rivelato A quale profonditagrave arriva prima didecadere

Esercizio 46Avete a disposizione dei tubi fotomoltiplicatori sensibili a lunghezze drsquoonda

fra 300 nm e 500 nm e volete rivelare la luce Cherenkov prodotta dal passaggio diun elettrone di 1 MeV di energia in un metro drsquoacqua Quanti fotoni vi aspettateche vengano prodotti Confrontate con il numero di elettroni di ionizzazioneche produrrebbe una particella α di 5 keV nello stesso rivelatore

Esercizio 47Determinare quali sono i processi piugrave probabili (cioegrave quelli di sezione drsquourto

piugrave alta) nellrsquointerazione fra1 fotoni di 1 MeV e atomi di alluminio2 fotoni di 100 keV e H23 fotoni di 100 keV e atomi di ferro4 fotoni di 10 MeV e atomi di carbonio5 fotoni di 10 MeV e atomi di piombo

Esercizio 48Volete misurare lrsquoimpulso di una particella carica che attraversa un campo

magnetico B ortogonale alla sua traiettoria Avete a disposizione tre rivelatori diposizione della stessa risoluzione spaziale δx come li disponete Che risoluzionein impulso vi aspettate di ottenere

23

  • Lezione 1 (13 marzo 2020)
  • Lezione 2 (20 marzo 2020)
  • Lezione 3 (27 marzo 2020)
  • Lezione 4 (3 aprile 2020)
  • Lezione 5 (16 aprile 2020)
  • Lezione 7 (8 maggio 2020)
Page 7: Esercizi per casa (risolti)ippolitv/pdf/fns1_2020_solutions.pdfEsercizi per casa (risolti) Valerio Ippolito 13 maggio 2020 1 Lezione 1 (13 marzo 2020) Esercizio1 Dilatazionedeitempi

1 Un tensore simmetrico ha 10 componenti indipendenti (a11 a12 a13 a14 a22 a23 a24 a33 a34 a44)

2 Un tensore antisimmetrico ha 6 componenti indipendenti (a12 a13 a14 a23 a24 a34)

3 Se un tensore egrave simmetrico significa che

amicroν = aνmicro

La versione trasformata del termine a sinistra egrave

amicroν prime = ΛmicroρΛνσaρσ

mentre quella del termine di destra egrave

aνmicroprime = ΛνρΛmicroσaρσ

ma applicando la definizione di tensore simmetrico nel riferimento di par-tenza (coordinate senza apici) e applicando la proprietagrave commutativa aitermini della matrice di Lorentz si ha

aνmicroprime = ΛνρΛmicroσaρσ = ΛνρΛmicroσa

σρ = ΛmicroσΛνρaσρ = ΛmicroρΛνσa

ρσ = amicroν prime

dove abbiamo ndash nellrsquoultimo passaggio ndash usato il fatto che sia ρ che σ sonoindici muti Ne concludiamo che un tensore simmetrico rimane tale dopotrasformazione di Lorentz

Se il tensore egrave antisimmetrico analogamente significa che

amicroν = minusaνmicro

La versione trasformata del termine a sinistra egrave sempre

amicroν prime = ΛmicroρΛνσaρσ

Applicando ancora una volta la definizione di tensore antisimmetrico nelriferimento di partenza e la proprietagrave commutativa si ha

aνmicroprime = ΛνρΛmicroσaρσ = ΛνρΛmicroσ (minusaσρ) = minusΛmicroσΛνρa

σρ = minusΛmicroρΛνσaρσ = minusamicroν prime

cioegrave anche lrsquoantisimmetria egrave mantenuta dopo trasformazione di Lorentz

4 Per il tensore simmetrico

amicroν = gmicroρgνσaρσ = gmicroρgνσa

σρ = gνσgmicroρaσρ = amicroν

Nel caso antisimmetrico

amicroν = gmicroρgνσaρσ = gmicroρgνσ (minusaσρ) = minusgνσgmicroρaσρ = minusamicroν

7

5 Vale

amicroνbmicroν = aνmicrob

νmicro = amicroν (minusbmicroν) = minus (amicroνbmicroν)rarr amicroνbmicroν = 0

dove al primo passaggio abbiamo semplicemente rinominato gli indici mu-ti e al secondo abbiamo usato le proprietagrave di simmetriaantisimmetriadei tensori di partenza Ne segue dunque che il prodotto fra un tensoresimmetrico e uno antisimmetrico egrave zero

Suggerimento almeno una volta nella vita conviene esplicitare la som-ma implicita nella notazione di Einstein per comprendere davvero questoformalismo cosigrave compatto

6 Basta costruire due somme che per costruzione allo scambio di indicisono lrsquouna simmetrica

amicroν equiv 1

2(tmicroν + tνmicro)

e lrsquoaltra antisimmetrica cioegrave

amicroν equiv 1

2(tmicroν minus tνmicro)

Egrave evidente che questi due tensori sono simmetriciantisimmetrici sottoscambio degli indici

Esercizio 11 Relazione fra forza e accelerazioneIn relativitagrave speciale forza e accelerazione sono in generale proporzionali fra

loro Usare la definizione di forza F = dpdt

Suggerimento scomporre lrsquoaccelerazione nella somma di un termine paral-lelo e un termine ortogonale alla direzione del moto (cioegrave alla velocitagrave)

Soluzione dellrsquoesercizio 11Scomponiamo lrsquoaccelerazione a lungo le direzioni ortogonale e parallela al

moto della particella (cioegrave a v)

a equiv aperp + a

Usando il fatto che

dt=

dv

dv

dt=

d

[(1minus v2

c2

)minus 12

]dt

dv

dt= minus1

2

(minus2v

c2

)(1minus v2

c2

)minus 32 dv

dt=

v

c2γ3 dv

dt

e che dalla definizione di prodotto scalare fra vettori

vdv

dt= v middot a

8

otteniamo che

F =dpdt

=d(mγv)

dt= mγ

dvdt

+mdγ

dtv

= mγa +mγ3 (v middot a) vc2

= mγ(aperp + a) +mγ3(v middot aperp + v middot a)v

= mγaperp +mγ3

(1

γ2+v2

c2

)a

= mγaperp +mγ3

(1minus v2

c2+v2

c2

)a

= mγaperp +mγ3a

Si noti come in relativitagrave speciale la forza non egrave in generale proporzionaleallrsquoaccelerazione

Esercizio 12 Classificazione dei quadrivettoriIl quadrimpulso egrave un quadrivettore di tipo spazio tempo o luce

Soluzione dellrsquoesercizio 12Poicheacute P 2 = E2

c2 minus p2 = m2 ge 0 saragrave di tipo tempo per particelle massive (e

di tipo luce per particelle senza massa)

Esercizio 13 Energia cineticaQuanto lavoro bisogna compiere per aumentare la velocitagrave di un elettrone

(m = 511 keVc2) dalla posizione di riposo a1 050c2 0990c3 09990c

Soluzione dellrsquoesercizio 13A questi elettroni dovremo dare una certa energia cinetica T in modo da

far passare lrsquoenergia totale da quella a riposo (γ = 0) cioegrave

Ei = mc2

aEf = T +mc2

Dalla relazioneEf = mγc2

segue

T = Ef minusmc2 = m(γ minus 1)c2 = m

1radic1minus v2

c2

minus 1

c2

per cui nei tre casi indicati servono rispettivamente 79 keV 31 MeV e 109 MeV

9

Esercizio 14 Energia di sogliaSupponiamo di far scontrare un fascio di protoni con un protone a riposo

Qual egrave lrsquoenergia minima che devono avere i protoni del fascio percheacute la reazione

p+ prarr p+ p+ p+ p

sia permessa (La massa del protone egrave pari a quella dellrsquoantiprotone p e vale938 MeVc2)

Esercizio 15 Diffusione elasticaChiamiamo elastico un urto (ldquoscatteringrdquo) in cui le particelle dello stato ini-

ziale e dello stato finale sono le stesse Si consideri un urto elastico fra unaparticella di massa nulla e una particella di massa m (bersaglio) che si trovaa riposo nel sistema di riferimento del laboratorio qual egrave la massima energiatrasferita dalla particella incidente al bersaglio Suggerimento si lavori nelsistema di riferimento del laboratorio e si espliciti il prodotto scalare fra gliimpulsi spaziali della particella di massa nulla prima e dopo lrsquourto in funzionedellrsquoangolo sempre nel sistema di riferimento del laboratorio fra la direzioneiniziale e finale della particella incidente

Se la particella incidente egrave un fotone e il bersaglio egrave un elettrone atomico ariposo di quanto varia la lunghezza drsquoonda del fotone fra prima e dopo lrsquourto

Soluzione dellrsquoesercizio 15Per scattering elastico intendiamo un processo in cui le particelle dello stato

iniziale sono le stesse di quelle dello stato finaleDenotiamo con k e P i quadrimpulsi della particella incidente e del bersa-

glio prima dellrsquourto e indichiamo con lrsquoapice le stesse quantitagrave dopo lrsquourto ilproblema ci dice che

k = (Ek)

kprime = (Eprimekrsquo)P = (mc0)

Partiamo dalla conservazione del quadrimpulso durante lrsquourto isoliamo la quan-titagrave che non misuriamo direttamente ndash cioegrave il quadrimpulso del bersaglio dopolrsquourto P prime ndash ed eleviamo al quadrato

k + P = kprime + P prime

P prime = k + P minus kprimem2c2 = 0 +m2c2 + 0 + 2Emminus 2(EEprime minus k middot krsquo)minus 2mEprime

e se indichiamo con θprime lrsquoangolo ndash nel riferimento del laboratorio ndash fra la direzioneiniziale e finale della particella incidente e usiamo il fatto che |k|c = E e |krsquo|c =

10

Eprime

0 = 2mc2(E minus Eprime)minus 2(EEprime minus EEprime cos θprime)

mc2(Eprime minus E) = minusEEprime(1minus cos θprime)

Eprime(mc2 + E(1minus cos θprime)) = mc2E

Eprime =E

1 + Emc2 (1minus cos θprime)

Il bersaglio rinculeragrave di una energia E minus Eprime massima per θ = π Il valoremassimo di questrsquoenergia di rinculo

E minus E

1 + 2 Emc2

= E2Emc2

1 + 2Emc2

prende il nome ndash nel caso dello scattering Compton in cui la particella incidenteegrave un fotone e il bersaglio egrave un elettrone atomico ndash di picco Compton

Cosa cambia fra un fotone di energia E ed uno di energia Eprime Dalla mecca-nica quantistica

E = hν =hc

λ

cioegrave cambia la lunghezza drsquoonda del fotone

Eprime =hc

λprime=

hcλ

1 +hcλ

mc2 (1minus cos θprime)

1

λprime=

1 +hcλ

mc2 (1minus cos θprime)

λprime = λ

(1 +

hc

λmc2(1minus cos θprime)

)

λprime = λ+h

mc(1minus cos θprime) equiv λ+ λc(1minus cos θprime)

dove abbiamo definito la lunghezza drsquoonda Compton dellrsquoelettrone λc che rap-presenta la scala di lunghezza sotto la quale gli effetti della meccanica quantisticarelativistica divengono importanti

3 Lezione 3 (27 marzo 2020)

Esercizio 16 Trasformazione delle velocitagraveUn oggetto si muove di moto rettilineo uniforme con velocitagrave costante αc versoun secondo oggetto immobile A che velocitagrave dobbiamo muoverci noi lungo lastessa direzione per vedere entrambi gli oggetti muoversi con velocitagrave uguali eopposte

11

Soluzione dellrsquoesercizio 16Se indichiamo con lrsquoapice la velocitagrave nel nostro sistema di riferimento che

egrave in moto rispetto al sistema di riferimento del secondo oggetto la legge ditrasformazione delle velocitagrave ci dice che

vprime1 =v1 minus V

1minus v1cVc

=αcminus V1minus αVc

vprime2 =v2 minus V

1minus v2cVc

= minusV

Stiamo cercando V la nostra velocitagrave nel sistema di riferimento del secondooggetto tale che le velocitagrave dei due oggetti nel nostro sistema di riferimentosiano uguali e opposte

vprime1 = minusvprime2 = V

perciograve la richiesta egrave

αcminus V1minus αVc

= V

αcminus V minus V + αV 2

c= 0

(αc)V 2 + (minus2)V + (α)c = 0

V =1minusradic

1minus α2

αc

dove abbiamo scelto la soluzione dellrsquoequazione di secondo grado con V le c

Esercizio 17 Conseguenze della relativitagraveUna navicella spaziale in moto rettilineo uniforme con velocitagrave 05c in al-

lontanamento dalla Terra egrave in orbita verso Plutone che si trova a 75times 109 kmdi distanza dalla Terra Non appena raggiunto il pianeta la comandante inviaun segnale radio alla base a Houston per chiedere lrsquoautorizzazione allrsquoatter-raggio Quanto tempo impiega la richiesta a raggiungere la base secondo lacomandante E secondo i suoi colleghi a Houston

Soluzione dellrsquoesercizio 17Lrsquoonda radio viaggia a velocitagrave c secondo ogni sistema di riferimento Per

lrsquoosservatore lrsquoonda radio percorreragrave il tragitto Plutone-Terra in un tempo

tTerra =L

c=

75times 109 km

3times 108 msasymp 25 000 s

mentre per la comandante saragrave passato un tempo inferiore che dipende dal fat-tore γ dellrsquoastronave nel sistema di riferimento della Terra secondo la relazione

tcomandante =tTerra

γasymp 25 000 stimes

radic1minus 052 asymp 21 650 s

12

Esercizio 18 Energia cineticaHa piugrave energia un protone che viaggia a 09999999896c o un Boeing 747 al

decollo

Esercizio 19 Dilatazione dei tempiUna scienziata misura che un fascio di particelle selezionate con impulso di

10 GeVc si degrada dellrsquo84 dopo aver percorso 1 m Se la massa di questeparticelle egrave 498 MeVc2 qual egrave la loro vita media

Soluzione dellrsquoesercizio 19La distanza misurata dalla scienziata egrave chiaramente riferita al suo sistema

di riferimento quello del laboratorio ed egrave legata alla vita media delle particelledel fascio τ dalla relazione

Ldecay = 1 m = βγcτ =pc

E

E

mc2cτ =

p

mccτ

e dalla legge del decadimento se indichiamo con N0 il numero di particelleinizialmente presenti nel fascio e con N il numero di particelle misurato si haN

N0= 1minus084 = exp

(minus L

Ldecay

)= exp

(minus L

pmccτ

)rarr τ =

mcL

pc log(N0

N

) asymp 9times 10minus11 s

Esercizio 20 Dilatazione dei tempiVi trovate a dover studiare un fascio di particelle di cui conoscete lrsquoenergia

ndash 2 GeV ndash ma non la massa a quanto ne sapete potrebbero essere composti daelettroni (di massa 511 keVc2) o protoni (938 MeVc2) Avete a disposizionedue rivelatori identici in grado di registrare con precisione il tempo in cui unaparticella li attraversa Come potete utilizzarli per determinare se il vostrofascio contiene elettroni o protoni

Soluzione dellrsquoesercizio 20Disponendo i due rivelatori lungo la direzione del fascio a distanza ∆L

lrsquouno dallrsquoaltro si puograve misurare il tempo impiegato dalle particelle per passaredallrsquouno allrsquoaltro e quindi la loro massa infatti se ∆t egrave la distanza temporalefra i segnali dei due rivelatori

∆L = v∆t = βc∆t =pc2

E∆t

Questo principio egrave usato nei cosiddetti rivelatori di time of flight che sono usatiper discriminare diversi tipi di particelle una particella di tipo 1 e una di tipo2 infatti percorreranno la distanza ∆L in tempi diversi legati alle rispettivemasse dalla relazione

∆L1 = ∆L2

p1c2

E1∆t1 =

radicE2 minusm2

1c2c

E∆t1 =

radicE2 minusm2

2c2c

E∆t2

13

Esercizio 21 Energia di sogliaDue fisici delle particelle vogliono produrre il bosone Z una particella di

carica neutra e di massamZ = 91 GeVc2 e discutono come fare Alice proponedi far scontrare fasci di elettroni e positroni di energia identica che viaggianodunque con impulso spaziale uguale in modulo e direzione ma di verso oppostoproducendo Z tramite il processo

e+ + eminus rarr Z

mentre Bob preferisce scontrare un fascio di protoni su un bersaglio fisso diidrogeno tramite il processo

p+ prarr Z + p+ p

Chi dei due avragrave bisogno di fasci di particelle di energia piugrave alta La massa delprotone egrave di 9383 MeVc2 quella dellrsquoelettrone di 511 keVc2

Esercizio 22 Leggi di conservazioneUn fotone (particella di massa nulla) puograve decadere in un elettrone e in un

positrone (entrambi di massa 511 keVc2) tramite il processo

γ rarr e+ + eminus

Esercizio 23 Decadimento βminus

Quali sono lrsquoenergia minima e massima dellrsquoelettrone nel decadimento

nrarr p+ eminus + νe

se il neutrone decade da fermo La massa del neutrone egrave di 9396 MeVc2 quelladel protone di 9383 MeVc2 e quella dellrsquoelettrone di 511 keVc2 si assuma chelrsquoantineutrino elettronico νe abbia massa nulla

Esercizio 24 Leggi di conservazioneIl decadimento

prarr n+ e+ + νe

egrave permesso

4 Lezione 4 (3 aprile 2020)

Esercizio 25Usando il fatto che hc = 1973 MeVfm si dimostri che in un sistema di unitagrave dimisura in cui h = c = 1 vale

14

1 1 GeVminus2 = 0389 mb

2 1 m = 5068times 1015 GeVminus1

3 1 s = 15times 1024 GeVminus1

Ricordiamo che 1 b = 1times 10minus28 m2 e che

[hc] = [Jsms] = [E][L]

Soluzione dellrsquoesercizio 25Lrsquoidea egrave di capire per quale potenza di hc e c va moltiplicato il termine a

sinistra di ciascuna equazione per ottenere il termine di destraPer cui

bull [1 GeVminus2][hc]α = [E]minus2[E]α[L]α = [0389 mb] = [L]2 da cui segue α = 2

e 1 GeVminus2(hc)2 = 1973 MeVfm1 GeV =

(01973times 10minus15 GeVm

1 GeV

)2

= 0389 mb

bull [1 m][hc]α = [L][E]α[L]α = [5068times 1015 GeVminus1] = [E]minus1 da cui segueα = minus1 e 1 m(hc)minus1 = 1 m

1973 MeVfm = 1 m01973times 10minus15 GeVm

= 5068times 1015 GeVminus1

bull [1 s][hc]α[c]β = [T ][E]α[L]α[L]β [T ]minusβ = [T ][E]α[L]α+β [T ]minusβ = [15times 1024 GeVminus1] =[E]minus1 da cui segue α = minus1 β = 1 e 1 s(hc)minus1c = 1 s

1973 MeVfm299 792 458 ms =299 792 458 m

01973times 10minus15 GeVm= 15times 1024 GeVminus1

Esercizio 26 Massa invarianteTre protoni (mp = 938 MeVc2) hanno impulsi uguali in modulo (p =

3 GeVc) e che formano angoli di 120 lrsquouno con lrsquoaltro Qual egrave la massainvariante del sistema

Esercizio 27 Energia di sogliaSi consideri il processo

γ + prarr p+ π0

dove il fotone ha massa nulla il protone ha massa di 938 MeVc2 e il π0 hamassa di 135 MeVc2

1 Se il protone egrave a riposo qual egrave lrsquoenergia minima che deve avere il fotoneincidente percheacute la reazione abbia luogo

2 La stessa reazione puograve avvenire nellrsquouniverso in cui un protone dei raggicosmici di alta energia puograve collidere con uno dei fotoni della radiazionecosmica di fondo di energia dellrsquoordine di 1 meV Qual egrave in questo casolrsquoenergia minima che deve avere il protone percheacute la reazione abbia luogo

Esercizio 28 Conseguenze della relativitagraveUn misterioso supereroe pattuglia a velocitagrave molto elevata la periferia roma-

na Allrsquoincrocio con via di Tor Bella Monaca incontra un semaforo e ndash vedendolo

15

verde ndash attraversa senza rallentare Una pattuglia della polizia municipale loferma e lo multa per esser passato col rosso Assumendo sia il supereroe che ivigili siano nel giusto a che velocitagrave viaggiava il supereroe

Soluzione dellrsquoesercizio 28Per la polizia municipale il semaforo emette fotoni di energia

E0 = hν0 =hc

λrossoasymp hc

630 nm

mentre il supereoe vede fotoni di energia

E = hν =hc

λverdeasymp hc

490 nm

e dalle trasformazioni di Lorentz indicando con γ e β le variabili calcolateusando la velocitagrave del supereroe misurata dalla municipale possiamo scrivere

E = γ(E0minusβp0) = γ(E0minusβE0) =1minus βradic1minus β2

E0 =1minus βradic

(1 + β)(1minus β)E0 =

radic1minus βradic1 + β

E0

dove abbiamo usato il fatto che i fotoni hanno massa nulla Perciograve

E

E0=

hcλverdehc

λrosso

=λrosso

λverde=

radic1minus βradic1 + β

rarr v = βc = 025c

Esercizio 29 Conseguenze della relativitagraveI neutrini sono particelle di massa molto piccola e al momento ignota Uno

dei modi con cui egrave stato possibile dedurre un limite superiore al suo valore egravestata lrsquoosservazione nel 1987 di neutrini prodotti dalla supernova 1987A1 chesi trova a 168000 anni luce dalla Terra Sono stati osservati due segnali dineutrini che possono essere schematizzati come segue si egrave osservato prima unneutrino di 35 MeV di energia seguito a 9 s di distanza da un secondo segnaledi 13 MeV Si assuma che questo ritardo sia dovuto al fatto che la massa delneutrino non egrave nulla e si calcoli questrsquoultima

Esercizio 30 DecadimentoAnimali e piante assumono dallrsquoatmosfera diversi composti contenenti car-

bonio Il carbonio presente nellrsquoatmosfera egrave predominantemente 126 C ma sono

presenti piccole concentrazioni del suo isotopo 146 C (un atomo ogni 1012) che

decade con emissione di elettroni attraverso il processo2

146 C rarr14

7 N + eminus + νe

con un tempo di dimezzamento di 5700 anni1httpsenwikipediaorgwikiSN_1987ANeutrino_emissions2Di altro non si tratta che del decadimento βminus nrarr p+ e+ νe

16

1 Qual egrave la concentrazione di 146 C dopo 11400 anni

2 Animali e piante assumono in vita proporzioni fisse di 146 C e 12

6 C mentrealla loro morte la quantitagrave di 14

6 C inizia a diminuire Avete a portata dimano un relitto di legno per cui misurate una emissione di elettroni daldecadimento di 14

6 C pari al 61 di quella di un pezzo di legno vivodella stessa massa quanti anni ha il manufatto

Esercizio 31 Energia cinetica e trasformazioni di LorentzDue particelle identiche di massa m ed energia cinetica T collidono frontal-

mente Qual egrave la loro energia cinetica relativa (ossia lrsquoenergia cinetica di unaparticella misurata nel sistema di riferimento dellrsquoaltra particella)

Esercizio 32 Energia nel centro di massaLa reazione

πminusprarr K0Λ0

avviene con unrsquoenergia nel centro di massa diradics = 3 GeV La massa del πminus egrave

di 1396 MeVc2 la massa del protone egrave di 938 MeVc2 la massa del Kminus egrave di498 MeVc2 e quella della Λ0 di 11 GeVc2

1 Calcolare lrsquoimpulso di πminus e Λ0 nel sistema di riferimento del centro dimassa

2 Se il protone egrave a riposo il K puograve essere emesso allrsquoindietro nel sistemadi riferimento del laboratorio

Esercizio 33 DecadimentoIl mesone φ0 egrave una particella neutra3 di circa 1 GeVc2 di massa che puograve

decadere in una coppia di particelle

φ0 rarr K+ +Kminus

di massa identicamK = 494 MeVc2 Si assuma di produrre φ0 di impulso notoegrave possibile che uno dei due K sia prodotto a riposo nel sistema di riferimentodel laboratorio

Esercizio 34 DecadimentoUn fascio di anti-neutrini muonici νmicro si puograve generare selezionando pioni

o kaoni π+ e K+ e facendoli passare in un lungo tubo in cui egrave stato fattoil vuoto4 in modo che dopo un certo tragitto L una buona parte di loro saragrave

3Il collisore DAFNE ai Laboratori Nazionali di Frascati produce specificatamente parti-celle di questo tipo tramite il processo e+ + eminus rarr φ0 httpswwwyoutubecomwatchv=L5yB9gDGKms

4Una tecnica di questo tipo egrave stata usata per inviare ai Laboratori Nazionali del Gran Sassodei fasci di neutrini prodotti al CERN di Ginevra httpsvideoscernchrecord985892

17

decaduta tramite i processi

π+ rarr microminus + νmicro

K+ rarr microminus + νmicro

Se lrsquoimpulso di pioni e kaoni egrave di 200 GeVc e la loro vita media di 26 ns e12 ns rispettivamente

1 Quanto a lungo viaggiano nel laboratorio i due tipi di particelle2 Se L = 1000 m quale saragrave la frazione di pioni e kaoni che saragrave decaduta

alla fine del tubo3 Qual egrave lrsquoenergia massima dei neutrini che egrave possibile misurare nel sistema

di riferimento del laboratorio nei due casi

5 Lezione 5 (16 aprile 2020)

Esercizio 35 Energia cineticaTra le eccellenze con sede in territorio elvetico primeggiano indiscutibilmente lacioccolata al latte e il Large Hadron Collider (LHC) Questrsquoultimo egrave un collisoredi particelle situato al CERN di Ginevra che fa scontrare due fasci identici diprotoni di impulso uguale in modulo e direzione ma verso opposto I fasci sonocomposti da circa 2800 gruppi (pacchetti) di 1011 particelle ciascuno Sapendoche lrsquoenergia nel centro di massa della collisione fra due protoni egrave

radics = 13 TeV

quanta cioccolata al latte dovete mangiare per assumere un numero di caloriepari allrsquoenergia cinetica di ciascun fascio di protoni di LHC

Soluzione dellrsquoesercizio 35Poicheacute si tratta di un collider e le energie in gioco sono molto maggiori della

massa del protone lrsquoenergia di ciascun protone egrave diradics2 = 65 TeV e quella

totale di un fascio egrave

2800times 1011 times 65 middot 1012 times 16 middot 10minus19J asymp 300 MJ

Secondo Google 100 g di cioccolata al latte apportano circa 500 cal = 500 times4184 J asymp 2 MJ Lrsquoenergia di un fascio di LHC corrisponde quindi a quella dicirca 15 kg di cioccolata

Esercizio 36 Scattering RutherfordUn fascio di particelle α di 100 MeV di energia e 032 nA di corrente5 collide

contro un bersaglio fisso di alluminio spesso 1 cm Una sperimentatrice prendeun rivelatore di 1 cm times 1 cm di superficie e lo posiziona ad un angolo di 30

5Per una spiegazione breve su come (e percheacute) si misura la corrente di un fascio di particel-le vedi httpswwwlhc-closerestaking_a_closer_look_at_lhc0beam_current Unatrattazione piugrave completa egrave data ad esempio da httpscdscernchrecord1213275filesp141pdf

18

rispetto al fascio di particelle a 1 m di distanza dal bersaglio Quante particelleα incideranno sul rivelatore ogni secondo

Soluzione dellrsquoesercizio 36Lrsquoalluminio ha una densitagrave di 27 gcm3 numero atomico 13 e massa atomica

27 uPoicheacute le particelle α sono nuclei di elio hanno carica 2e e la corrente di

032 nA corrisponde a un miliardo di particelle incidenti al secondo

dNidt

=032 nCs

2times 16times 10minus19 C= 1times 109 sminus1

Il rivelatore vede un angolo solido di

∆Ω equiv superficieraggio

2

=1 cm2

(1 m)2= 1times 10minus4 sr

Si tratta di uno scattering alla Rutherford per cui la sezione drsquourto per unitagravedi angolo solido rilevata ad un certo angolo θ vale

dΩ=

(zαzAle

2

4πε04E

1

sin2(θ2)

)2

pari a

dΩasymp(

2times 13times 4times etimes 16times 10minus19 C

4π times 89times 10minus12 Fmtimes 4times 100times 106 eV

1

sin2(π180 times 302)

)2

asymp 2times 10minus30 m2sr = 20 mbsr

(1)

e il numero di particelle visto dal rivelatore vale se indichiamo con nAl = ρAlNAAAl

la densitagrave numero di atomi di alluminio e con d lo spessore del rivelatore

dNrivelate

dt= ∆Ω

dΩnAld

dNi

dt

asymp 1times 10minus4 srtimes 2times 10minus30 m2srtimes 1times 104 cm2m2 times 27 gcm3 6times 1023 molminus1

27 gmol

= 120 Hz

Esercizio 37 Sezione drsquourtoUn bersaglio di idrogeno liquido di densitagrave ρ = 0071 gcm3 e volume V =

125 cm3 egrave bombardato da un fascio monoenergetico di pioni negativi con unflusso φ = 2times 107 mminus2sminus1 e si osserva la reazione

πminus + prarr π0 + n

rivelando i fotoni del decadimento π0 rarr γγ che avviene nel 988 dei casi Sela sezione drsquourto di quellrsquointerazione egrave σ = 40 mb quanti fotoni sono emessi ognisecondo

19

Esercizio 38 Sezione drsquourtoLa sezione drsquourto dellrsquoeffetto fotoelettrico per raggi X di 10 keV in carbonio

egrave 40 b per atomo Data una lastra di carbonio di 4 mm di spessore si calcoli1 il numero di bersagli per unitagrave di volume2 il coefficiente di assorbimento per effetto fotoelettrico dei raggi X di tale

energia3 la probabilitagrave che un raggio X incidente sulla lastra produca un elettrone

per effetto fotoelettrico

Esercizio 39 Sezioni drsquourtoUn bersaglio drsquooro di densitagrave superficiale ρS = 097 mgcm2 e superficie

SB = 1 cm2 viene colpito da un fascio di particelle α la cui sezione trasversaegrave contenuta completamente nellrsquoarea del bersaglio Sul bersaglio impattano37times 104 αs La sezione drsquourto di diffusione elastica ad un certo angolo θ valedσ

ddΩ = 1 bsr Calcolare1 la densitagrave di atomi del bersaglio per unitagrave di superficie2 il numero di particelle α rivelate in unrsquoora da un rivelatore di superficie

SR = 2 cm2 posto allrsquoangolo θ e a distanza DR = 01 m dal bersaglio3 Se il fascio di particelle viene sostituito da una sorgente radioattiva di

particelle α con distribuzione isotropa su tutto lrsquoangolo solido che vieneposta lungo la stessa linea del fascio a distanza DB = 20 cm dal bersaglioquanto tempo egrave necessario per rivelare con lo stesso rivelatore lo stessonumero di particelle calcolato sopra a paritagrave di sezione drsquourto

Esercizio 40 CinematicaUn fascio di positroni di 100 MeV di energia annichila su una targhetta fissa

di elettroni producendo due fotoni tramite il processo

e+ + eminus rarr γ + γ

Se uno dei due fotoni egrave emesso a 30 nel sistema del centro di massa1 quanto vale lrsquoenergia dei due fotoni nel sistema del centro di massa2 che energia e direzione avranno nel sistema del laboratorio

Esercizio 41 Sezione drsquourtoGli antineutrini νe prodotti da un reattore nucleare con potenza P = 16 GW

attraversano un bersaglio da 2000 l drsquoacqua posto a 50 m di distanza dal reat-tore

1 Supponendo che per ogni fissione sia prodotta unrsquoenergia termica di200 MeV e vengano emessi 6 antineutrini calcolare il numero medio direazioni

νe + prarr e+ + n

prodotte al giorno se la sezione drsquourto egrave σ = 1times 10minus43 cm2

20

2 Qual egrave la minima energia del neutrino per cui la reazione egrave permessa

6 Lezione 7 (8 maggio 2020)

Esercizio 42Un protone interagisce con un bersaglio producendo un pione di energia E =300 MeV Con un tracciatore posto a distanza d = 6 cm dal bersaglio egrave possi-bile rivelare la traiettoria del pione e risalire al punto di produzione del pionesul bersaglio Determinare lrsquoerrore sulla misura della posizione di tale puntocausato dalla presenza di un piano di alluminio di spessore L = 05 mm postoimmediatamente davanti al tracciatore (quindi a distanza d dal bersaglio) nellrsquoi-potesi che gli angoli delle tracce rispetto alla normale alle superfici del bersaglioe del piano di alluminio siano piccoli [mπ = 1396 MeV Al (Z = 13 A = 27ρ = 27 gcm3)]

Soluzione dellrsquoesercizio 42La deviazione standard dellrsquoangolo di diffusione coulombiana multipla vale

radic〈θ2〉 = 21 MeV

z

βc|p|

radicx

X0= 21 MeV

z

βc|p|

radicLρ

X0

dove p =radicE2 minusm2

π = 266 MeV β = pE = 0885 z = 1 Per lrsquoalluminioabbiamo

X0 = 24 gcm2 rarr X0ρ = 89 cm

per cui radic〈θ2〉 asymp 00067

e lo spostamento medio corrispondente rispetto alla posizione nominale egrave

δs = tan(00067)times 6 cm = 04 mm

Esercizio 43Un fascio misto di elettroni e antiprotoni passa attraverso una regione con

B = 2 T e dopo aver curvato per R = 3 m raggiunge una regione senza campomagnetico

1 Qual egrave lrsquoimpulso delle particelle selezionate2 Come discriminereste fra i due tipi di particelle3 Se usaste due scintillatori posti a 15 m di distanza quale risoluzione

temporale sarebbe necessaria per discriminare i due tipi di particelle4 Se i due scintillatori fossero spessi 2 cm e avessero una lunghezza di ra-

diazione X0 = 40 cm quanta energia perderebbero elettroni e protoninellrsquoattraversarli Si assuma una perdita di energia per ionizzazione di2 MeVcm e 25 MeVcm per protoni ed elettroni rispettivamente

21

5 Che indice di rifrazione dovrebbe avere un rivelatore a luce Cherenkovper discriminare elettroni e protoni

Soluzione dellrsquoesercizio 43Dalla definizione di forza

F =dpdt

= qvtimesB

|F| = mv2

R

per cui se il campo magnetico egrave ortogonale alla traiettoria

pc = qcBR

che poicheacute c = 03 mns possiamo esprimere in maniera piugrave conveniente come

pc[GeV] = 03B[T]R[m]

Ne segue che nel nostro esercizio (e tornando a c = 1)

p = 18 GeV

Trascurando le perdite di energia e ricordando che me mp i tempi divolo di elettroni e protoni valgono rispettivamente

te =L

βecasymp L

casymp 50 ns

etp =

L

βpc=

Lpradic

p2+m2p

casymp 56 ns

per cui serve una risoluzione dellrsquoordine del ns per discriminare elettroni e muonidal tempo di volo

Se gli scintillatori sono spessi 2 cm i protoni perderanno in ciascuno di essi2 cm times 2 MeVcm = 4 MeV per ionizzazione Gli elettroni perderanno invecenel primo rivelatore 5 MeV per ionizzazione e (me asymp 0)

18 GeV times(

1minus exp

(minus 2 cm

40 cm

))asymp 88 MeV

cioegrave dopo il primo rivelatore avranno 18 GeVminus93 MeV di energia e nel secondone perderanno altri

5 MeV + (18 GeV minus 93 MeV)

(1minus exp

(minus 2 cm

40 cm

))asymp 88 MeV

Lrsquoangolo di emissione di luce Cherenkov egrave dato da

cos θc =1

βnle 1

22

e poicheacute βe asymp 1 βp asymp 0887 per avere luce Cherenkov solo al passaggio deglielettroni (e non dei protoni) egrave necessario che

1 lt n lt 113

Esercizio 44Come misurereste la massa di una particella carica E quella di una parti-

cella neutra

Esercizio 45Un muone di energia E = 400 GeV penetra verticalmente nel mare Attra-

verso quale processo puograve essere rivelato A quale profonditagrave arriva prima didecadere

Esercizio 46Avete a disposizione dei tubi fotomoltiplicatori sensibili a lunghezze drsquoonda

fra 300 nm e 500 nm e volete rivelare la luce Cherenkov prodotta dal passaggio diun elettrone di 1 MeV di energia in un metro drsquoacqua Quanti fotoni vi aspettateche vengano prodotti Confrontate con il numero di elettroni di ionizzazioneche produrrebbe una particella α di 5 keV nello stesso rivelatore

Esercizio 47Determinare quali sono i processi piugrave probabili (cioegrave quelli di sezione drsquourto

piugrave alta) nellrsquointerazione fra1 fotoni di 1 MeV e atomi di alluminio2 fotoni di 100 keV e H23 fotoni di 100 keV e atomi di ferro4 fotoni di 10 MeV e atomi di carbonio5 fotoni di 10 MeV e atomi di piombo

Esercizio 48Volete misurare lrsquoimpulso di una particella carica che attraversa un campo

magnetico B ortogonale alla sua traiettoria Avete a disposizione tre rivelatori diposizione della stessa risoluzione spaziale δx come li disponete Che risoluzionein impulso vi aspettate di ottenere

23

  • Lezione 1 (13 marzo 2020)
  • Lezione 2 (20 marzo 2020)
  • Lezione 3 (27 marzo 2020)
  • Lezione 4 (3 aprile 2020)
  • Lezione 5 (16 aprile 2020)
  • Lezione 7 (8 maggio 2020)
Page 8: Esercizi per casa (risolti)ippolitv/pdf/fns1_2020_solutions.pdfEsercizi per casa (risolti) Valerio Ippolito 13 maggio 2020 1 Lezione 1 (13 marzo 2020) Esercizio1 Dilatazionedeitempi

5 Vale

amicroνbmicroν = aνmicrob

νmicro = amicroν (minusbmicroν) = minus (amicroνbmicroν)rarr amicroνbmicroν = 0

dove al primo passaggio abbiamo semplicemente rinominato gli indici mu-ti e al secondo abbiamo usato le proprietagrave di simmetriaantisimmetriadei tensori di partenza Ne segue dunque che il prodotto fra un tensoresimmetrico e uno antisimmetrico egrave zero

Suggerimento almeno una volta nella vita conviene esplicitare la som-ma implicita nella notazione di Einstein per comprendere davvero questoformalismo cosigrave compatto

6 Basta costruire due somme che per costruzione allo scambio di indicisono lrsquouna simmetrica

amicroν equiv 1

2(tmicroν + tνmicro)

e lrsquoaltra antisimmetrica cioegrave

amicroν equiv 1

2(tmicroν minus tνmicro)

Egrave evidente che questi due tensori sono simmetriciantisimmetrici sottoscambio degli indici

Esercizio 11 Relazione fra forza e accelerazioneIn relativitagrave speciale forza e accelerazione sono in generale proporzionali fra

loro Usare la definizione di forza F = dpdt

Suggerimento scomporre lrsquoaccelerazione nella somma di un termine paral-lelo e un termine ortogonale alla direzione del moto (cioegrave alla velocitagrave)

Soluzione dellrsquoesercizio 11Scomponiamo lrsquoaccelerazione a lungo le direzioni ortogonale e parallela al

moto della particella (cioegrave a v)

a equiv aperp + a

Usando il fatto che

dt=

dv

dv

dt=

d

[(1minus v2

c2

)minus 12

]dt

dv

dt= minus1

2

(minus2v

c2

)(1minus v2

c2

)minus 32 dv

dt=

v

c2γ3 dv

dt

e che dalla definizione di prodotto scalare fra vettori

vdv

dt= v middot a

8

otteniamo che

F =dpdt

=d(mγv)

dt= mγ

dvdt

+mdγ

dtv

= mγa +mγ3 (v middot a) vc2

= mγ(aperp + a) +mγ3(v middot aperp + v middot a)v

= mγaperp +mγ3

(1

γ2+v2

c2

)a

= mγaperp +mγ3

(1minus v2

c2+v2

c2

)a

= mγaperp +mγ3a

Si noti come in relativitagrave speciale la forza non egrave in generale proporzionaleallrsquoaccelerazione

Esercizio 12 Classificazione dei quadrivettoriIl quadrimpulso egrave un quadrivettore di tipo spazio tempo o luce

Soluzione dellrsquoesercizio 12Poicheacute P 2 = E2

c2 minus p2 = m2 ge 0 saragrave di tipo tempo per particelle massive (e

di tipo luce per particelle senza massa)

Esercizio 13 Energia cineticaQuanto lavoro bisogna compiere per aumentare la velocitagrave di un elettrone

(m = 511 keVc2) dalla posizione di riposo a1 050c2 0990c3 09990c

Soluzione dellrsquoesercizio 13A questi elettroni dovremo dare una certa energia cinetica T in modo da

far passare lrsquoenergia totale da quella a riposo (γ = 0) cioegrave

Ei = mc2

aEf = T +mc2

Dalla relazioneEf = mγc2

segue

T = Ef minusmc2 = m(γ minus 1)c2 = m

1radic1minus v2

c2

minus 1

c2

per cui nei tre casi indicati servono rispettivamente 79 keV 31 MeV e 109 MeV

9

Esercizio 14 Energia di sogliaSupponiamo di far scontrare un fascio di protoni con un protone a riposo

Qual egrave lrsquoenergia minima che devono avere i protoni del fascio percheacute la reazione

p+ prarr p+ p+ p+ p

sia permessa (La massa del protone egrave pari a quella dellrsquoantiprotone p e vale938 MeVc2)

Esercizio 15 Diffusione elasticaChiamiamo elastico un urto (ldquoscatteringrdquo) in cui le particelle dello stato ini-

ziale e dello stato finale sono le stesse Si consideri un urto elastico fra unaparticella di massa nulla e una particella di massa m (bersaglio) che si trovaa riposo nel sistema di riferimento del laboratorio qual egrave la massima energiatrasferita dalla particella incidente al bersaglio Suggerimento si lavori nelsistema di riferimento del laboratorio e si espliciti il prodotto scalare fra gliimpulsi spaziali della particella di massa nulla prima e dopo lrsquourto in funzionedellrsquoangolo sempre nel sistema di riferimento del laboratorio fra la direzioneiniziale e finale della particella incidente

Se la particella incidente egrave un fotone e il bersaglio egrave un elettrone atomico ariposo di quanto varia la lunghezza drsquoonda del fotone fra prima e dopo lrsquourto

Soluzione dellrsquoesercizio 15Per scattering elastico intendiamo un processo in cui le particelle dello stato

iniziale sono le stesse di quelle dello stato finaleDenotiamo con k e P i quadrimpulsi della particella incidente e del bersa-

glio prima dellrsquourto e indichiamo con lrsquoapice le stesse quantitagrave dopo lrsquourto ilproblema ci dice che

k = (Ek)

kprime = (Eprimekrsquo)P = (mc0)

Partiamo dalla conservazione del quadrimpulso durante lrsquourto isoliamo la quan-titagrave che non misuriamo direttamente ndash cioegrave il quadrimpulso del bersaglio dopolrsquourto P prime ndash ed eleviamo al quadrato

k + P = kprime + P prime

P prime = k + P minus kprimem2c2 = 0 +m2c2 + 0 + 2Emminus 2(EEprime minus k middot krsquo)minus 2mEprime

e se indichiamo con θprime lrsquoangolo ndash nel riferimento del laboratorio ndash fra la direzioneiniziale e finale della particella incidente e usiamo il fatto che |k|c = E e |krsquo|c =

10

Eprime

0 = 2mc2(E minus Eprime)minus 2(EEprime minus EEprime cos θprime)

mc2(Eprime minus E) = minusEEprime(1minus cos θprime)

Eprime(mc2 + E(1minus cos θprime)) = mc2E

Eprime =E

1 + Emc2 (1minus cos θprime)

Il bersaglio rinculeragrave di una energia E minus Eprime massima per θ = π Il valoremassimo di questrsquoenergia di rinculo

E minus E

1 + 2 Emc2

= E2Emc2

1 + 2Emc2

prende il nome ndash nel caso dello scattering Compton in cui la particella incidenteegrave un fotone e il bersaglio egrave un elettrone atomico ndash di picco Compton

Cosa cambia fra un fotone di energia E ed uno di energia Eprime Dalla mecca-nica quantistica

E = hν =hc

λ

cioegrave cambia la lunghezza drsquoonda del fotone

Eprime =hc

λprime=

hcλ

1 +hcλ

mc2 (1minus cos θprime)

1

λprime=

1 +hcλ

mc2 (1minus cos θprime)

λprime = λ

(1 +

hc

λmc2(1minus cos θprime)

)

λprime = λ+h

mc(1minus cos θprime) equiv λ+ λc(1minus cos θprime)

dove abbiamo definito la lunghezza drsquoonda Compton dellrsquoelettrone λc che rap-presenta la scala di lunghezza sotto la quale gli effetti della meccanica quantisticarelativistica divengono importanti

3 Lezione 3 (27 marzo 2020)

Esercizio 16 Trasformazione delle velocitagraveUn oggetto si muove di moto rettilineo uniforme con velocitagrave costante αc versoun secondo oggetto immobile A che velocitagrave dobbiamo muoverci noi lungo lastessa direzione per vedere entrambi gli oggetti muoversi con velocitagrave uguali eopposte

11

Soluzione dellrsquoesercizio 16Se indichiamo con lrsquoapice la velocitagrave nel nostro sistema di riferimento che

egrave in moto rispetto al sistema di riferimento del secondo oggetto la legge ditrasformazione delle velocitagrave ci dice che

vprime1 =v1 minus V

1minus v1cVc

=αcminus V1minus αVc

vprime2 =v2 minus V

1minus v2cVc

= minusV

Stiamo cercando V la nostra velocitagrave nel sistema di riferimento del secondooggetto tale che le velocitagrave dei due oggetti nel nostro sistema di riferimentosiano uguali e opposte

vprime1 = minusvprime2 = V

perciograve la richiesta egrave

αcminus V1minus αVc

= V

αcminus V minus V + αV 2

c= 0

(αc)V 2 + (minus2)V + (α)c = 0

V =1minusradic

1minus α2

αc

dove abbiamo scelto la soluzione dellrsquoequazione di secondo grado con V le c

Esercizio 17 Conseguenze della relativitagraveUna navicella spaziale in moto rettilineo uniforme con velocitagrave 05c in al-

lontanamento dalla Terra egrave in orbita verso Plutone che si trova a 75times 109 kmdi distanza dalla Terra Non appena raggiunto il pianeta la comandante inviaun segnale radio alla base a Houston per chiedere lrsquoautorizzazione allrsquoatter-raggio Quanto tempo impiega la richiesta a raggiungere la base secondo lacomandante E secondo i suoi colleghi a Houston

Soluzione dellrsquoesercizio 17Lrsquoonda radio viaggia a velocitagrave c secondo ogni sistema di riferimento Per

lrsquoosservatore lrsquoonda radio percorreragrave il tragitto Plutone-Terra in un tempo

tTerra =L

c=

75times 109 km

3times 108 msasymp 25 000 s

mentre per la comandante saragrave passato un tempo inferiore che dipende dal fat-tore γ dellrsquoastronave nel sistema di riferimento della Terra secondo la relazione

tcomandante =tTerra

γasymp 25 000 stimes

radic1minus 052 asymp 21 650 s

12

Esercizio 18 Energia cineticaHa piugrave energia un protone che viaggia a 09999999896c o un Boeing 747 al

decollo

Esercizio 19 Dilatazione dei tempiUna scienziata misura che un fascio di particelle selezionate con impulso di

10 GeVc si degrada dellrsquo84 dopo aver percorso 1 m Se la massa di questeparticelle egrave 498 MeVc2 qual egrave la loro vita media

Soluzione dellrsquoesercizio 19La distanza misurata dalla scienziata egrave chiaramente riferita al suo sistema

di riferimento quello del laboratorio ed egrave legata alla vita media delle particelledel fascio τ dalla relazione

Ldecay = 1 m = βγcτ =pc

E

E

mc2cτ =

p

mccτ

e dalla legge del decadimento se indichiamo con N0 il numero di particelleinizialmente presenti nel fascio e con N il numero di particelle misurato si haN

N0= 1minus084 = exp

(minus L

Ldecay

)= exp

(minus L

pmccτ

)rarr τ =

mcL

pc log(N0

N

) asymp 9times 10minus11 s

Esercizio 20 Dilatazione dei tempiVi trovate a dover studiare un fascio di particelle di cui conoscete lrsquoenergia

ndash 2 GeV ndash ma non la massa a quanto ne sapete potrebbero essere composti daelettroni (di massa 511 keVc2) o protoni (938 MeVc2) Avete a disposizionedue rivelatori identici in grado di registrare con precisione il tempo in cui unaparticella li attraversa Come potete utilizzarli per determinare se il vostrofascio contiene elettroni o protoni

Soluzione dellrsquoesercizio 20Disponendo i due rivelatori lungo la direzione del fascio a distanza ∆L

lrsquouno dallrsquoaltro si puograve misurare il tempo impiegato dalle particelle per passaredallrsquouno allrsquoaltro e quindi la loro massa infatti se ∆t egrave la distanza temporalefra i segnali dei due rivelatori

∆L = v∆t = βc∆t =pc2

E∆t

Questo principio egrave usato nei cosiddetti rivelatori di time of flight che sono usatiper discriminare diversi tipi di particelle una particella di tipo 1 e una di tipo2 infatti percorreranno la distanza ∆L in tempi diversi legati alle rispettivemasse dalla relazione

∆L1 = ∆L2

p1c2

E1∆t1 =

radicE2 minusm2

1c2c

E∆t1 =

radicE2 minusm2

2c2c

E∆t2

13

Esercizio 21 Energia di sogliaDue fisici delle particelle vogliono produrre il bosone Z una particella di

carica neutra e di massamZ = 91 GeVc2 e discutono come fare Alice proponedi far scontrare fasci di elettroni e positroni di energia identica che viaggianodunque con impulso spaziale uguale in modulo e direzione ma di verso oppostoproducendo Z tramite il processo

e+ + eminus rarr Z

mentre Bob preferisce scontrare un fascio di protoni su un bersaglio fisso diidrogeno tramite il processo

p+ prarr Z + p+ p

Chi dei due avragrave bisogno di fasci di particelle di energia piugrave alta La massa delprotone egrave di 9383 MeVc2 quella dellrsquoelettrone di 511 keVc2

Esercizio 22 Leggi di conservazioneUn fotone (particella di massa nulla) puograve decadere in un elettrone e in un

positrone (entrambi di massa 511 keVc2) tramite il processo

γ rarr e+ + eminus

Esercizio 23 Decadimento βminus

Quali sono lrsquoenergia minima e massima dellrsquoelettrone nel decadimento

nrarr p+ eminus + νe

se il neutrone decade da fermo La massa del neutrone egrave di 9396 MeVc2 quelladel protone di 9383 MeVc2 e quella dellrsquoelettrone di 511 keVc2 si assuma chelrsquoantineutrino elettronico νe abbia massa nulla

Esercizio 24 Leggi di conservazioneIl decadimento

prarr n+ e+ + νe

egrave permesso

4 Lezione 4 (3 aprile 2020)

Esercizio 25Usando il fatto che hc = 1973 MeVfm si dimostri che in un sistema di unitagrave dimisura in cui h = c = 1 vale

14

1 1 GeVminus2 = 0389 mb

2 1 m = 5068times 1015 GeVminus1

3 1 s = 15times 1024 GeVminus1

Ricordiamo che 1 b = 1times 10minus28 m2 e che

[hc] = [Jsms] = [E][L]

Soluzione dellrsquoesercizio 25Lrsquoidea egrave di capire per quale potenza di hc e c va moltiplicato il termine a

sinistra di ciascuna equazione per ottenere il termine di destraPer cui

bull [1 GeVminus2][hc]α = [E]minus2[E]α[L]α = [0389 mb] = [L]2 da cui segue α = 2

e 1 GeVminus2(hc)2 = 1973 MeVfm1 GeV =

(01973times 10minus15 GeVm

1 GeV

)2

= 0389 mb

bull [1 m][hc]α = [L][E]α[L]α = [5068times 1015 GeVminus1] = [E]minus1 da cui segueα = minus1 e 1 m(hc)minus1 = 1 m

1973 MeVfm = 1 m01973times 10minus15 GeVm

= 5068times 1015 GeVminus1

bull [1 s][hc]α[c]β = [T ][E]α[L]α[L]β [T ]minusβ = [T ][E]α[L]α+β [T ]minusβ = [15times 1024 GeVminus1] =[E]minus1 da cui segue α = minus1 β = 1 e 1 s(hc)minus1c = 1 s

1973 MeVfm299 792 458 ms =299 792 458 m

01973times 10minus15 GeVm= 15times 1024 GeVminus1

Esercizio 26 Massa invarianteTre protoni (mp = 938 MeVc2) hanno impulsi uguali in modulo (p =

3 GeVc) e che formano angoli di 120 lrsquouno con lrsquoaltro Qual egrave la massainvariante del sistema

Esercizio 27 Energia di sogliaSi consideri il processo

γ + prarr p+ π0

dove il fotone ha massa nulla il protone ha massa di 938 MeVc2 e il π0 hamassa di 135 MeVc2

1 Se il protone egrave a riposo qual egrave lrsquoenergia minima che deve avere il fotoneincidente percheacute la reazione abbia luogo

2 La stessa reazione puograve avvenire nellrsquouniverso in cui un protone dei raggicosmici di alta energia puograve collidere con uno dei fotoni della radiazionecosmica di fondo di energia dellrsquoordine di 1 meV Qual egrave in questo casolrsquoenergia minima che deve avere il protone percheacute la reazione abbia luogo

Esercizio 28 Conseguenze della relativitagraveUn misterioso supereroe pattuglia a velocitagrave molto elevata la periferia roma-

na Allrsquoincrocio con via di Tor Bella Monaca incontra un semaforo e ndash vedendolo

15

verde ndash attraversa senza rallentare Una pattuglia della polizia municipale loferma e lo multa per esser passato col rosso Assumendo sia il supereroe che ivigili siano nel giusto a che velocitagrave viaggiava il supereroe

Soluzione dellrsquoesercizio 28Per la polizia municipale il semaforo emette fotoni di energia

E0 = hν0 =hc

λrossoasymp hc

630 nm

mentre il supereoe vede fotoni di energia

E = hν =hc

λverdeasymp hc

490 nm

e dalle trasformazioni di Lorentz indicando con γ e β le variabili calcolateusando la velocitagrave del supereroe misurata dalla municipale possiamo scrivere

E = γ(E0minusβp0) = γ(E0minusβE0) =1minus βradic1minus β2

E0 =1minus βradic

(1 + β)(1minus β)E0 =

radic1minus βradic1 + β

E0

dove abbiamo usato il fatto che i fotoni hanno massa nulla Perciograve

E

E0=

hcλverdehc

λrosso

=λrosso

λverde=

radic1minus βradic1 + β

rarr v = βc = 025c

Esercizio 29 Conseguenze della relativitagraveI neutrini sono particelle di massa molto piccola e al momento ignota Uno

dei modi con cui egrave stato possibile dedurre un limite superiore al suo valore egravestata lrsquoosservazione nel 1987 di neutrini prodotti dalla supernova 1987A1 chesi trova a 168000 anni luce dalla Terra Sono stati osservati due segnali dineutrini che possono essere schematizzati come segue si egrave osservato prima unneutrino di 35 MeV di energia seguito a 9 s di distanza da un secondo segnaledi 13 MeV Si assuma che questo ritardo sia dovuto al fatto che la massa delneutrino non egrave nulla e si calcoli questrsquoultima

Esercizio 30 DecadimentoAnimali e piante assumono dallrsquoatmosfera diversi composti contenenti car-

bonio Il carbonio presente nellrsquoatmosfera egrave predominantemente 126 C ma sono

presenti piccole concentrazioni del suo isotopo 146 C (un atomo ogni 1012) che

decade con emissione di elettroni attraverso il processo2

146 C rarr14

7 N + eminus + νe

con un tempo di dimezzamento di 5700 anni1httpsenwikipediaorgwikiSN_1987ANeutrino_emissions2Di altro non si tratta che del decadimento βminus nrarr p+ e+ νe

16

1 Qual egrave la concentrazione di 146 C dopo 11400 anni

2 Animali e piante assumono in vita proporzioni fisse di 146 C e 12

6 C mentrealla loro morte la quantitagrave di 14

6 C inizia a diminuire Avete a portata dimano un relitto di legno per cui misurate una emissione di elettroni daldecadimento di 14

6 C pari al 61 di quella di un pezzo di legno vivodella stessa massa quanti anni ha il manufatto

Esercizio 31 Energia cinetica e trasformazioni di LorentzDue particelle identiche di massa m ed energia cinetica T collidono frontal-

mente Qual egrave la loro energia cinetica relativa (ossia lrsquoenergia cinetica di unaparticella misurata nel sistema di riferimento dellrsquoaltra particella)

Esercizio 32 Energia nel centro di massaLa reazione

πminusprarr K0Λ0

avviene con unrsquoenergia nel centro di massa diradics = 3 GeV La massa del πminus egrave

di 1396 MeVc2 la massa del protone egrave di 938 MeVc2 la massa del Kminus egrave di498 MeVc2 e quella della Λ0 di 11 GeVc2

1 Calcolare lrsquoimpulso di πminus e Λ0 nel sistema di riferimento del centro dimassa

2 Se il protone egrave a riposo il K puograve essere emesso allrsquoindietro nel sistemadi riferimento del laboratorio

Esercizio 33 DecadimentoIl mesone φ0 egrave una particella neutra3 di circa 1 GeVc2 di massa che puograve

decadere in una coppia di particelle

φ0 rarr K+ +Kminus

di massa identicamK = 494 MeVc2 Si assuma di produrre φ0 di impulso notoegrave possibile che uno dei due K sia prodotto a riposo nel sistema di riferimentodel laboratorio

Esercizio 34 DecadimentoUn fascio di anti-neutrini muonici νmicro si puograve generare selezionando pioni

o kaoni π+ e K+ e facendoli passare in un lungo tubo in cui egrave stato fattoil vuoto4 in modo che dopo un certo tragitto L una buona parte di loro saragrave

3Il collisore DAFNE ai Laboratori Nazionali di Frascati produce specificatamente parti-celle di questo tipo tramite il processo e+ + eminus rarr φ0 httpswwwyoutubecomwatchv=L5yB9gDGKms

4Una tecnica di questo tipo egrave stata usata per inviare ai Laboratori Nazionali del Gran Sassodei fasci di neutrini prodotti al CERN di Ginevra httpsvideoscernchrecord985892

17

decaduta tramite i processi

π+ rarr microminus + νmicro

K+ rarr microminus + νmicro

Se lrsquoimpulso di pioni e kaoni egrave di 200 GeVc e la loro vita media di 26 ns e12 ns rispettivamente

1 Quanto a lungo viaggiano nel laboratorio i due tipi di particelle2 Se L = 1000 m quale saragrave la frazione di pioni e kaoni che saragrave decaduta

alla fine del tubo3 Qual egrave lrsquoenergia massima dei neutrini che egrave possibile misurare nel sistema

di riferimento del laboratorio nei due casi

5 Lezione 5 (16 aprile 2020)

Esercizio 35 Energia cineticaTra le eccellenze con sede in territorio elvetico primeggiano indiscutibilmente lacioccolata al latte e il Large Hadron Collider (LHC) Questrsquoultimo egrave un collisoredi particelle situato al CERN di Ginevra che fa scontrare due fasci identici diprotoni di impulso uguale in modulo e direzione ma verso opposto I fasci sonocomposti da circa 2800 gruppi (pacchetti) di 1011 particelle ciascuno Sapendoche lrsquoenergia nel centro di massa della collisione fra due protoni egrave

radics = 13 TeV

quanta cioccolata al latte dovete mangiare per assumere un numero di caloriepari allrsquoenergia cinetica di ciascun fascio di protoni di LHC

Soluzione dellrsquoesercizio 35Poicheacute si tratta di un collider e le energie in gioco sono molto maggiori della

massa del protone lrsquoenergia di ciascun protone egrave diradics2 = 65 TeV e quella

totale di un fascio egrave

2800times 1011 times 65 middot 1012 times 16 middot 10minus19J asymp 300 MJ

Secondo Google 100 g di cioccolata al latte apportano circa 500 cal = 500 times4184 J asymp 2 MJ Lrsquoenergia di un fascio di LHC corrisponde quindi a quella dicirca 15 kg di cioccolata

Esercizio 36 Scattering RutherfordUn fascio di particelle α di 100 MeV di energia e 032 nA di corrente5 collide

contro un bersaglio fisso di alluminio spesso 1 cm Una sperimentatrice prendeun rivelatore di 1 cm times 1 cm di superficie e lo posiziona ad un angolo di 30

5Per una spiegazione breve su come (e percheacute) si misura la corrente di un fascio di particel-le vedi httpswwwlhc-closerestaking_a_closer_look_at_lhc0beam_current Unatrattazione piugrave completa egrave data ad esempio da httpscdscernchrecord1213275filesp141pdf

18

rispetto al fascio di particelle a 1 m di distanza dal bersaglio Quante particelleα incideranno sul rivelatore ogni secondo

Soluzione dellrsquoesercizio 36Lrsquoalluminio ha una densitagrave di 27 gcm3 numero atomico 13 e massa atomica

27 uPoicheacute le particelle α sono nuclei di elio hanno carica 2e e la corrente di

032 nA corrisponde a un miliardo di particelle incidenti al secondo

dNidt

=032 nCs

2times 16times 10minus19 C= 1times 109 sminus1

Il rivelatore vede un angolo solido di

∆Ω equiv superficieraggio

2

=1 cm2

(1 m)2= 1times 10minus4 sr

Si tratta di uno scattering alla Rutherford per cui la sezione drsquourto per unitagravedi angolo solido rilevata ad un certo angolo θ vale

dΩ=

(zαzAle

2

4πε04E

1

sin2(θ2)

)2

pari a

dΩasymp(

2times 13times 4times etimes 16times 10minus19 C

4π times 89times 10minus12 Fmtimes 4times 100times 106 eV

1

sin2(π180 times 302)

)2

asymp 2times 10minus30 m2sr = 20 mbsr

(1)

e il numero di particelle visto dal rivelatore vale se indichiamo con nAl = ρAlNAAAl

la densitagrave numero di atomi di alluminio e con d lo spessore del rivelatore

dNrivelate

dt= ∆Ω

dΩnAld

dNi

dt

asymp 1times 10minus4 srtimes 2times 10minus30 m2srtimes 1times 104 cm2m2 times 27 gcm3 6times 1023 molminus1

27 gmol

= 120 Hz

Esercizio 37 Sezione drsquourtoUn bersaglio di idrogeno liquido di densitagrave ρ = 0071 gcm3 e volume V =

125 cm3 egrave bombardato da un fascio monoenergetico di pioni negativi con unflusso φ = 2times 107 mminus2sminus1 e si osserva la reazione

πminus + prarr π0 + n

rivelando i fotoni del decadimento π0 rarr γγ che avviene nel 988 dei casi Sela sezione drsquourto di quellrsquointerazione egrave σ = 40 mb quanti fotoni sono emessi ognisecondo

19

Esercizio 38 Sezione drsquourtoLa sezione drsquourto dellrsquoeffetto fotoelettrico per raggi X di 10 keV in carbonio

egrave 40 b per atomo Data una lastra di carbonio di 4 mm di spessore si calcoli1 il numero di bersagli per unitagrave di volume2 il coefficiente di assorbimento per effetto fotoelettrico dei raggi X di tale

energia3 la probabilitagrave che un raggio X incidente sulla lastra produca un elettrone

per effetto fotoelettrico

Esercizio 39 Sezioni drsquourtoUn bersaglio drsquooro di densitagrave superficiale ρS = 097 mgcm2 e superficie

SB = 1 cm2 viene colpito da un fascio di particelle α la cui sezione trasversaegrave contenuta completamente nellrsquoarea del bersaglio Sul bersaglio impattano37times 104 αs La sezione drsquourto di diffusione elastica ad un certo angolo θ valedσ

ddΩ = 1 bsr Calcolare1 la densitagrave di atomi del bersaglio per unitagrave di superficie2 il numero di particelle α rivelate in unrsquoora da un rivelatore di superficie

SR = 2 cm2 posto allrsquoangolo θ e a distanza DR = 01 m dal bersaglio3 Se il fascio di particelle viene sostituito da una sorgente radioattiva di

particelle α con distribuzione isotropa su tutto lrsquoangolo solido che vieneposta lungo la stessa linea del fascio a distanza DB = 20 cm dal bersaglioquanto tempo egrave necessario per rivelare con lo stesso rivelatore lo stessonumero di particelle calcolato sopra a paritagrave di sezione drsquourto

Esercizio 40 CinematicaUn fascio di positroni di 100 MeV di energia annichila su una targhetta fissa

di elettroni producendo due fotoni tramite il processo

e+ + eminus rarr γ + γ

Se uno dei due fotoni egrave emesso a 30 nel sistema del centro di massa1 quanto vale lrsquoenergia dei due fotoni nel sistema del centro di massa2 che energia e direzione avranno nel sistema del laboratorio

Esercizio 41 Sezione drsquourtoGli antineutrini νe prodotti da un reattore nucleare con potenza P = 16 GW

attraversano un bersaglio da 2000 l drsquoacqua posto a 50 m di distanza dal reat-tore

1 Supponendo che per ogni fissione sia prodotta unrsquoenergia termica di200 MeV e vengano emessi 6 antineutrini calcolare il numero medio direazioni

νe + prarr e+ + n

prodotte al giorno se la sezione drsquourto egrave σ = 1times 10minus43 cm2

20

2 Qual egrave la minima energia del neutrino per cui la reazione egrave permessa

6 Lezione 7 (8 maggio 2020)

Esercizio 42Un protone interagisce con un bersaglio producendo un pione di energia E =300 MeV Con un tracciatore posto a distanza d = 6 cm dal bersaglio egrave possi-bile rivelare la traiettoria del pione e risalire al punto di produzione del pionesul bersaglio Determinare lrsquoerrore sulla misura della posizione di tale puntocausato dalla presenza di un piano di alluminio di spessore L = 05 mm postoimmediatamente davanti al tracciatore (quindi a distanza d dal bersaglio) nellrsquoi-potesi che gli angoli delle tracce rispetto alla normale alle superfici del bersaglioe del piano di alluminio siano piccoli [mπ = 1396 MeV Al (Z = 13 A = 27ρ = 27 gcm3)]

Soluzione dellrsquoesercizio 42La deviazione standard dellrsquoangolo di diffusione coulombiana multipla vale

radic〈θ2〉 = 21 MeV

z

βc|p|

radicx

X0= 21 MeV

z

βc|p|

radicLρ

X0

dove p =radicE2 minusm2

π = 266 MeV β = pE = 0885 z = 1 Per lrsquoalluminioabbiamo

X0 = 24 gcm2 rarr X0ρ = 89 cm

per cui radic〈θ2〉 asymp 00067

e lo spostamento medio corrispondente rispetto alla posizione nominale egrave

δs = tan(00067)times 6 cm = 04 mm

Esercizio 43Un fascio misto di elettroni e antiprotoni passa attraverso una regione con

B = 2 T e dopo aver curvato per R = 3 m raggiunge una regione senza campomagnetico

1 Qual egrave lrsquoimpulso delle particelle selezionate2 Come discriminereste fra i due tipi di particelle3 Se usaste due scintillatori posti a 15 m di distanza quale risoluzione

temporale sarebbe necessaria per discriminare i due tipi di particelle4 Se i due scintillatori fossero spessi 2 cm e avessero una lunghezza di ra-

diazione X0 = 40 cm quanta energia perderebbero elettroni e protoninellrsquoattraversarli Si assuma una perdita di energia per ionizzazione di2 MeVcm e 25 MeVcm per protoni ed elettroni rispettivamente

21

5 Che indice di rifrazione dovrebbe avere un rivelatore a luce Cherenkovper discriminare elettroni e protoni

Soluzione dellrsquoesercizio 43Dalla definizione di forza

F =dpdt

= qvtimesB

|F| = mv2

R

per cui se il campo magnetico egrave ortogonale alla traiettoria

pc = qcBR

che poicheacute c = 03 mns possiamo esprimere in maniera piugrave conveniente come

pc[GeV] = 03B[T]R[m]

Ne segue che nel nostro esercizio (e tornando a c = 1)

p = 18 GeV

Trascurando le perdite di energia e ricordando che me mp i tempi divolo di elettroni e protoni valgono rispettivamente

te =L

βecasymp L

casymp 50 ns

etp =

L

βpc=

Lpradic

p2+m2p

casymp 56 ns

per cui serve una risoluzione dellrsquoordine del ns per discriminare elettroni e muonidal tempo di volo

Se gli scintillatori sono spessi 2 cm i protoni perderanno in ciascuno di essi2 cm times 2 MeVcm = 4 MeV per ionizzazione Gli elettroni perderanno invecenel primo rivelatore 5 MeV per ionizzazione e (me asymp 0)

18 GeV times(

1minus exp

(minus 2 cm

40 cm

))asymp 88 MeV

cioegrave dopo il primo rivelatore avranno 18 GeVminus93 MeV di energia e nel secondone perderanno altri

5 MeV + (18 GeV minus 93 MeV)

(1minus exp

(minus 2 cm

40 cm

))asymp 88 MeV

Lrsquoangolo di emissione di luce Cherenkov egrave dato da

cos θc =1

βnle 1

22

e poicheacute βe asymp 1 βp asymp 0887 per avere luce Cherenkov solo al passaggio deglielettroni (e non dei protoni) egrave necessario che

1 lt n lt 113

Esercizio 44Come misurereste la massa di una particella carica E quella di una parti-

cella neutra

Esercizio 45Un muone di energia E = 400 GeV penetra verticalmente nel mare Attra-

verso quale processo puograve essere rivelato A quale profonditagrave arriva prima didecadere

Esercizio 46Avete a disposizione dei tubi fotomoltiplicatori sensibili a lunghezze drsquoonda

fra 300 nm e 500 nm e volete rivelare la luce Cherenkov prodotta dal passaggio diun elettrone di 1 MeV di energia in un metro drsquoacqua Quanti fotoni vi aspettateche vengano prodotti Confrontate con il numero di elettroni di ionizzazioneche produrrebbe una particella α di 5 keV nello stesso rivelatore

Esercizio 47Determinare quali sono i processi piugrave probabili (cioegrave quelli di sezione drsquourto

piugrave alta) nellrsquointerazione fra1 fotoni di 1 MeV e atomi di alluminio2 fotoni di 100 keV e H23 fotoni di 100 keV e atomi di ferro4 fotoni di 10 MeV e atomi di carbonio5 fotoni di 10 MeV e atomi di piombo

Esercizio 48Volete misurare lrsquoimpulso di una particella carica che attraversa un campo

magnetico B ortogonale alla sua traiettoria Avete a disposizione tre rivelatori diposizione della stessa risoluzione spaziale δx come li disponete Che risoluzionein impulso vi aspettate di ottenere

23

  • Lezione 1 (13 marzo 2020)
  • Lezione 2 (20 marzo 2020)
  • Lezione 3 (27 marzo 2020)
  • Lezione 4 (3 aprile 2020)
  • Lezione 5 (16 aprile 2020)
  • Lezione 7 (8 maggio 2020)
Page 9: Esercizi per casa (risolti)ippolitv/pdf/fns1_2020_solutions.pdfEsercizi per casa (risolti) Valerio Ippolito 13 maggio 2020 1 Lezione 1 (13 marzo 2020) Esercizio1 Dilatazionedeitempi

otteniamo che

F =dpdt

=d(mγv)

dt= mγ

dvdt

+mdγ

dtv

= mγa +mγ3 (v middot a) vc2

= mγ(aperp + a) +mγ3(v middot aperp + v middot a)v

= mγaperp +mγ3

(1

γ2+v2

c2

)a

= mγaperp +mγ3

(1minus v2

c2+v2

c2

)a

= mγaperp +mγ3a

Si noti come in relativitagrave speciale la forza non egrave in generale proporzionaleallrsquoaccelerazione

Esercizio 12 Classificazione dei quadrivettoriIl quadrimpulso egrave un quadrivettore di tipo spazio tempo o luce

Soluzione dellrsquoesercizio 12Poicheacute P 2 = E2

c2 minus p2 = m2 ge 0 saragrave di tipo tempo per particelle massive (e

di tipo luce per particelle senza massa)

Esercizio 13 Energia cineticaQuanto lavoro bisogna compiere per aumentare la velocitagrave di un elettrone

(m = 511 keVc2) dalla posizione di riposo a1 050c2 0990c3 09990c

Soluzione dellrsquoesercizio 13A questi elettroni dovremo dare una certa energia cinetica T in modo da

far passare lrsquoenergia totale da quella a riposo (γ = 0) cioegrave

Ei = mc2

aEf = T +mc2

Dalla relazioneEf = mγc2

segue

T = Ef minusmc2 = m(γ minus 1)c2 = m

1radic1minus v2

c2

minus 1

c2

per cui nei tre casi indicati servono rispettivamente 79 keV 31 MeV e 109 MeV

9

Esercizio 14 Energia di sogliaSupponiamo di far scontrare un fascio di protoni con un protone a riposo

Qual egrave lrsquoenergia minima che devono avere i protoni del fascio percheacute la reazione

p+ prarr p+ p+ p+ p

sia permessa (La massa del protone egrave pari a quella dellrsquoantiprotone p e vale938 MeVc2)

Esercizio 15 Diffusione elasticaChiamiamo elastico un urto (ldquoscatteringrdquo) in cui le particelle dello stato ini-

ziale e dello stato finale sono le stesse Si consideri un urto elastico fra unaparticella di massa nulla e una particella di massa m (bersaglio) che si trovaa riposo nel sistema di riferimento del laboratorio qual egrave la massima energiatrasferita dalla particella incidente al bersaglio Suggerimento si lavori nelsistema di riferimento del laboratorio e si espliciti il prodotto scalare fra gliimpulsi spaziali della particella di massa nulla prima e dopo lrsquourto in funzionedellrsquoangolo sempre nel sistema di riferimento del laboratorio fra la direzioneiniziale e finale della particella incidente

Se la particella incidente egrave un fotone e il bersaglio egrave un elettrone atomico ariposo di quanto varia la lunghezza drsquoonda del fotone fra prima e dopo lrsquourto

Soluzione dellrsquoesercizio 15Per scattering elastico intendiamo un processo in cui le particelle dello stato

iniziale sono le stesse di quelle dello stato finaleDenotiamo con k e P i quadrimpulsi della particella incidente e del bersa-

glio prima dellrsquourto e indichiamo con lrsquoapice le stesse quantitagrave dopo lrsquourto ilproblema ci dice che

k = (Ek)

kprime = (Eprimekrsquo)P = (mc0)

Partiamo dalla conservazione del quadrimpulso durante lrsquourto isoliamo la quan-titagrave che non misuriamo direttamente ndash cioegrave il quadrimpulso del bersaglio dopolrsquourto P prime ndash ed eleviamo al quadrato

k + P = kprime + P prime

P prime = k + P minus kprimem2c2 = 0 +m2c2 + 0 + 2Emminus 2(EEprime minus k middot krsquo)minus 2mEprime

e se indichiamo con θprime lrsquoangolo ndash nel riferimento del laboratorio ndash fra la direzioneiniziale e finale della particella incidente e usiamo il fatto che |k|c = E e |krsquo|c =

10

Eprime

0 = 2mc2(E minus Eprime)minus 2(EEprime minus EEprime cos θprime)

mc2(Eprime minus E) = minusEEprime(1minus cos θprime)

Eprime(mc2 + E(1minus cos θprime)) = mc2E

Eprime =E

1 + Emc2 (1minus cos θprime)

Il bersaglio rinculeragrave di una energia E minus Eprime massima per θ = π Il valoremassimo di questrsquoenergia di rinculo

E minus E

1 + 2 Emc2

= E2Emc2

1 + 2Emc2

prende il nome ndash nel caso dello scattering Compton in cui la particella incidenteegrave un fotone e il bersaglio egrave un elettrone atomico ndash di picco Compton

Cosa cambia fra un fotone di energia E ed uno di energia Eprime Dalla mecca-nica quantistica

E = hν =hc

λ

cioegrave cambia la lunghezza drsquoonda del fotone

Eprime =hc

λprime=

hcλ

1 +hcλ

mc2 (1minus cos θprime)

1

λprime=

1 +hcλ

mc2 (1minus cos θprime)

λprime = λ

(1 +

hc

λmc2(1minus cos θprime)

)

λprime = λ+h

mc(1minus cos θprime) equiv λ+ λc(1minus cos θprime)

dove abbiamo definito la lunghezza drsquoonda Compton dellrsquoelettrone λc che rap-presenta la scala di lunghezza sotto la quale gli effetti della meccanica quantisticarelativistica divengono importanti

3 Lezione 3 (27 marzo 2020)

Esercizio 16 Trasformazione delle velocitagraveUn oggetto si muove di moto rettilineo uniforme con velocitagrave costante αc versoun secondo oggetto immobile A che velocitagrave dobbiamo muoverci noi lungo lastessa direzione per vedere entrambi gli oggetti muoversi con velocitagrave uguali eopposte

11

Soluzione dellrsquoesercizio 16Se indichiamo con lrsquoapice la velocitagrave nel nostro sistema di riferimento che

egrave in moto rispetto al sistema di riferimento del secondo oggetto la legge ditrasformazione delle velocitagrave ci dice che

vprime1 =v1 minus V

1minus v1cVc

=αcminus V1minus αVc

vprime2 =v2 minus V

1minus v2cVc

= minusV

Stiamo cercando V la nostra velocitagrave nel sistema di riferimento del secondooggetto tale che le velocitagrave dei due oggetti nel nostro sistema di riferimentosiano uguali e opposte

vprime1 = minusvprime2 = V

perciograve la richiesta egrave

αcminus V1minus αVc

= V

αcminus V minus V + αV 2

c= 0

(αc)V 2 + (minus2)V + (α)c = 0

V =1minusradic

1minus α2

αc

dove abbiamo scelto la soluzione dellrsquoequazione di secondo grado con V le c

Esercizio 17 Conseguenze della relativitagraveUna navicella spaziale in moto rettilineo uniforme con velocitagrave 05c in al-

lontanamento dalla Terra egrave in orbita verso Plutone che si trova a 75times 109 kmdi distanza dalla Terra Non appena raggiunto il pianeta la comandante inviaun segnale radio alla base a Houston per chiedere lrsquoautorizzazione allrsquoatter-raggio Quanto tempo impiega la richiesta a raggiungere la base secondo lacomandante E secondo i suoi colleghi a Houston

Soluzione dellrsquoesercizio 17Lrsquoonda radio viaggia a velocitagrave c secondo ogni sistema di riferimento Per

lrsquoosservatore lrsquoonda radio percorreragrave il tragitto Plutone-Terra in un tempo

tTerra =L

c=

75times 109 km

3times 108 msasymp 25 000 s

mentre per la comandante saragrave passato un tempo inferiore che dipende dal fat-tore γ dellrsquoastronave nel sistema di riferimento della Terra secondo la relazione

tcomandante =tTerra

γasymp 25 000 stimes

radic1minus 052 asymp 21 650 s

12

Esercizio 18 Energia cineticaHa piugrave energia un protone che viaggia a 09999999896c o un Boeing 747 al

decollo

Esercizio 19 Dilatazione dei tempiUna scienziata misura che un fascio di particelle selezionate con impulso di

10 GeVc si degrada dellrsquo84 dopo aver percorso 1 m Se la massa di questeparticelle egrave 498 MeVc2 qual egrave la loro vita media

Soluzione dellrsquoesercizio 19La distanza misurata dalla scienziata egrave chiaramente riferita al suo sistema

di riferimento quello del laboratorio ed egrave legata alla vita media delle particelledel fascio τ dalla relazione

Ldecay = 1 m = βγcτ =pc

E

E

mc2cτ =

p

mccτ

e dalla legge del decadimento se indichiamo con N0 il numero di particelleinizialmente presenti nel fascio e con N il numero di particelle misurato si haN

N0= 1minus084 = exp

(minus L

Ldecay

)= exp

(minus L

pmccτ

)rarr τ =

mcL

pc log(N0

N

) asymp 9times 10minus11 s

Esercizio 20 Dilatazione dei tempiVi trovate a dover studiare un fascio di particelle di cui conoscete lrsquoenergia

ndash 2 GeV ndash ma non la massa a quanto ne sapete potrebbero essere composti daelettroni (di massa 511 keVc2) o protoni (938 MeVc2) Avete a disposizionedue rivelatori identici in grado di registrare con precisione il tempo in cui unaparticella li attraversa Come potete utilizzarli per determinare se il vostrofascio contiene elettroni o protoni

Soluzione dellrsquoesercizio 20Disponendo i due rivelatori lungo la direzione del fascio a distanza ∆L

lrsquouno dallrsquoaltro si puograve misurare il tempo impiegato dalle particelle per passaredallrsquouno allrsquoaltro e quindi la loro massa infatti se ∆t egrave la distanza temporalefra i segnali dei due rivelatori

∆L = v∆t = βc∆t =pc2

E∆t

Questo principio egrave usato nei cosiddetti rivelatori di time of flight che sono usatiper discriminare diversi tipi di particelle una particella di tipo 1 e una di tipo2 infatti percorreranno la distanza ∆L in tempi diversi legati alle rispettivemasse dalla relazione

∆L1 = ∆L2

p1c2

E1∆t1 =

radicE2 minusm2

1c2c

E∆t1 =

radicE2 minusm2

2c2c

E∆t2

13

Esercizio 21 Energia di sogliaDue fisici delle particelle vogliono produrre il bosone Z una particella di

carica neutra e di massamZ = 91 GeVc2 e discutono come fare Alice proponedi far scontrare fasci di elettroni e positroni di energia identica che viaggianodunque con impulso spaziale uguale in modulo e direzione ma di verso oppostoproducendo Z tramite il processo

e+ + eminus rarr Z

mentre Bob preferisce scontrare un fascio di protoni su un bersaglio fisso diidrogeno tramite il processo

p+ prarr Z + p+ p

Chi dei due avragrave bisogno di fasci di particelle di energia piugrave alta La massa delprotone egrave di 9383 MeVc2 quella dellrsquoelettrone di 511 keVc2

Esercizio 22 Leggi di conservazioneUn fotone (particella di massa nulla) puograve decadere in un elettrone e in un

positrone (entrambi di massa 511 keVc2) tramite il processo

γ rarr e+ + eminus

Esercizio 23 Decadimento βminus

Quali sono lrsquoenergia minima e massima dellrsquoelettrone nel decadimento

nrarr p+ eminus + νe

se il neutrone decade da fermo La massa del neutrone egrave di 9396 MeVc2 quelladel protone di 9383 MeVc2 e quella dellrsquoelettrone di 511 keVc2 si assuma chelrsquoantineutrino elettronico νe abbia massa nulla

Esercizio 24 Leggi di conservazioneIl decadimento

prarr n+ e+ + νe

egrave permesso

4 Lezione 4 (3 aprile 2020)

Esercizio 25Usando il fatto che hc = 1973 MeVfm si dimostri che in un sistema di unitagrave dimisura in cui h = c = 1 vale

14

1 1 GeVminus2 = 0389 mb

2 1 m = 5068times 1015 GeVminus1

3 1 s = 15times 1024 GeVminus1

Ricordiamo che 1 b = 1times 10minus28 m2 e che

[hc] = [Jsms] = [E][L]

Soluzione dellrsquoesercizio 25Lrsquoidea egrave di capire per quale potenza di hc e c va moltiplicato il termine a

sinistra di ciascuna equazione per ottenere il termine di destraPer cui

bull [1 GeVminus2][hc]α = [E]minus2[E]α[L]α = [0389 mb] = [L]2 da cui segue α = 2

e 1 GeVminus2(hc)2 = 1973 MeVfm1 GeV =

(01973times 10minus15 GeVm

1 GeV

)2

= 0389 mb

bull [1 m][hc]α = [L][E]α[L]α = [5068times 1015 GeVminus1] = [E]minus1 da cui segueα = minus1 e 1 m(hc)minus1 = 1 m

1973 MeVfm = 1 m01973times 10minus15 GeVm

= 5068times 1015 GeVminus1

bull [1 s][hc]α[c]β = [T ][E]α[L]α[L]β [T ]minusβ = [T ][E]α[L]α+β [T ]minusβ = [15times 1024 GeVminus1] =[E]minus1 da cui segue α = minus1 β = 1 e 1 s(hc)minus1c = 1 s

1973 MeVfm299 792 458 ms =299 792 458 m

01973times 10minus15 GeVm= 15times 1024 GeVminus1

Esercizio 26 Massa invarianteTre protoni (mp = 938 MeVc2) hanno impulsi uguali in modulo (p =

3 GeVc) e che formano angoli di 120 lrsquouno con lrsquoaltro Qual egrave la massainvariante del sistema

Esercizio 27 Energia di sogliaSi consideri il processo

γ + prarr p+ π0

dove il fotone ha massa nulla il protone ha massa di 938 MeVc2 e il π0 hamassa di 135 MeVc2

1 Se il protone egrave a riposo qual egrave lrsquoenergia minima che deve avere il fotoneincidente percheacute la reazione abbia luogo

2 La stessa reazione puograve avvenire nellrsquouniverso in cui un protone dei raggicosmici di alta energia puograve collidere con uno dei fotoni della radiazionecosmica di fondo di energia dellrsquoordine di 1 meV Qual egrave in questo casolrsquoenergia minima che deve avere il protone percheacute la reazione abbia luogo

Esercizio 28 Conseguenze della relativitagraveUn misterioso supereroe pattuglia a velocitagrave molto elevata la periferia roma-

na Allrsquoincrocio con via di Tor Bella Monaca incontra un semaforo e ndash vedendolo

15

verde ndash attraversa senza rallentare Una pattuglia della polizia municipale loferma e lo multa per esser passato col rosso Assumendo sia il supereroe che ivigili siano nel giusto a che velocitagrave viaggiava il supereroe

Soluzione dellrsquoesercizio 28Per la polizia municipale il semaforo emette fotoni di energia

E0 = hν0 =hc

λrossoasymp hc

630 nm

mentre il supereoe vede fotoni di energia

E = hν =hc

λverdeasymp hc

490 nm

e dalle trasformazioni di Lorentz indicando con γ e β le variabili calcolateusando la velocitagrave del supereroe misurata dalla municipale possiamo scrivere

E = γ(E0minusβp0) = γ(E0minusβE0) =1minus βradic1minus β2

E0 =1minus βradic

(1 + β)(1minus β)E0 =

radic1minus βradic1 + β

E0

dove abbiamo usato il fatto che i fotoni hanno massa nulla Perciograve

E

E0=

hcλverdehc

λrosso

=λrosso

λverde=

radic1minus βradic1 + β

rarr v = βc = 025c

Esercizio 29 Conseguenze della relativitagraveI neutrini sono particelle di massa molto piccola e al momento ignota Uno

dei modi con cui egrave stato possibile dedurre un limite superiore al suo valore egravestata lrsquoosservazione nel 1987 di neutrini prodotti dalla supernova 1987A1 chesi trova a 168000 anni luce dalla Terra Sono stati osservati due segnali dineutrini che possono essere schematizzati come segue si egrave osservato prima unneutrino di 35 MeV di energia seguito a 9 s di distanza da un secondo segnaledi 13 MeV Si assuma che questo ritardo sia dovuto al fatto che la massa delneutrino non egrave nulla e si calcoli questrsquoultima

Esercizio 30 DecadimentoAnimali e piante assumono dallrsquoatmosfera diversi composti contenenti car-

bonio Il carbonio presente nellrsquoatmosfera egrave predominantemente 126 C ma sono

presenti piccole concentrazioni del suo isotopo 146 C (un atomo ogni 1012) che

decade con emissione di elettroni attraverso il processo2

146 C rarr14

7 N + eminus + νe

con un tempo di dimezzamento di 5700 anni1httpsenwikipediaorgwikiSN_1987ANeutrino_emissions2Di altro non si tratta che del decadimento βminus nrarr p+ e+ νe

16

1 Qual egrave la concentrazione di 146 C dopo 11400 anni

2 Animali e piante assumono in vita proporzioni fisse di 146 C e 12

6 C mentrealla loro morte la quantitagrave di 14

6 C inizia a diminuire Avete a portata dimano un relitto di legno per cui misurate una emissione di elettroni daldecadimento di 14

6 C pari al 61 di quella di un pezzo di legno vivodella stessa massa quanti anni ha il manufatto

Esercizio 31 Energia cinetica e trasformazioni di LorentzDue particelle identiche di massa m ed energia cinetica T collidono frontal-

mente Qual egrave la loro energia cinetica relativa (ossia lrsquoenergia cinetica di unaparticella misurata nel sistema di riferimento dellrsquoaltra particella)

Esercizio 32 Energia nel centro di massaLa reazione

πminusprarr K0Λ0

avviene con unrsquoenergia nel centro di massa diradics = 3 GeV La massa del πminus egrave

di 1396 MeVc2 la massa del protone egrave di 938 MeVc2 la massa del Kminus egrave di498 MeVc2 e quella della Λ0 di 11 GeVc2

1 Calcolare lrsquoimpulso di πminus e Λ0 nel sistema di riferimento del centro dimassa

2 Se il protone egrave a riposo il K puograve essere emesso allrsquoindietro nel sistemadi riferimento del laboratorio

Esercizio 33 DecadimentoIl mesone φ0 egrave una particella neutra3 di circa 1 GeVc2 di massa che puograve

decadere in una coppia di particelle

φ0 rarr K+ +Kminus

di massa identicamK = 494 MeVc2 Si assuma di produrre φ0 di impulso notoegrave possibile che uno dei due K sia prodotto a riposo nel sistema di riferimentodel laboratorio

Esercizio 34 DecadimentoUn fascio di anti-neutrini muonici νmicro si puograve generare selezionando pioni

o kaoni π+ e K+ e facendoli passare in un lungo tubo in cui egrave stato fattoil vuoto4 in modo che dopo un certo tragitto L una buona parte di loro saragrave

3Il collisore DAFNE ai Laboratori Nazionali di Frascati produce specificatamente parti-celle di questo tipo tramite il processo e+ + eminus rarr φ0 httpswwwyoutubecomwatchv=L5yB9gDGKms

4Una tecnica di questo tipo egrave stata usata per inviare ai Laboratori Nazionali del Gran Sassodei fasci di neutrini prodotti al CERN di Ginevra httpsvideoscernchrecord985892

17

decaduta tramite i processi

π+ rarr microminus + νmicro

K+ rarr microminus + νmicro

Se lrsquoimpulso di pioni e kaoni egrave di 200 GeVc e la loro vita media di 26 ns e12 ns rispettivamente

1 Quanto a lungo viaggiano nel laboratorio i due tipi di particelle2 Se L = 1000 m quale saragrave la frazione di pioni e kaoni che saragrave decaduta

alla fine del tubo3 Qual egrave lrsquoenergia massima dei neutrini che egrave possibile misurare nel sistema

di riferimento del laboratorio nei due casi

5 Lezione 5 (16 aprile 2020)

Esercizio 35 Energia cineticaTra le eccellenze con sede in territorio elvetico primeggiano indiscutibilmente lacioccolata al latte e il Large Hadron Collider (LHC) Questrsquoultimo egrave un collisoredi particelle situato al CERN di Ginevra che fa scontrare due fasci identici diprotoni di impulso uguale in modulo e direzione ma verso opposto I fasci sonocomposti da circa 2800 gruppi (pacchetti) di 1011 particelle ciascuno Sapendoche lrsquoenergia nel centro di massa della collisione fra due protoni egrave

radics = 13 TeV

quanta cioccolata al latte dovete mangiare per assumere un numero di caloriepari allrsquoenergia cinetica di ciascun fascio di protoni di LHC

Soluzione dellrsquoesercizio 35Poicheacute si tratta di un collider e le energie in gioco sono molto maggiori della

massa del protone lrsquoenergia di ciascun protone egrave diradics2 = 65 TeV e quella

totale di un fascio egrave

2800times 1011 times 65 middot 1012 times 16 middot 10minus19J asymp 300 MJ

Secondo Google 100 g di cioccolata al latte apportano circa 500 cal = 500 times4184 J asymp 2 MJ Lrsquoenergia di un fascio di LHC corrisponde quindi a quella dicirca 15 kg di cioccolata

Esercizio 36 Scattering RutherfordUn fascio di particelle α di 100 MeV di energia e 032 nA di corrente5 collide

contro un bersaglio fisso di alluminio spesso 1 cm Una sperimentatrice prendeun rivelatore di 1 cm times 1 cm di superficie e lo posiziona ad un angolo di 30

5Per una spiegazione breve su come (e percheacute) si misura la corrente di un fascio di particel-le vedi httpswwwlhc-closerestaking_a_closer_look_at_lhc0beam_current Unatrattazione piugrave completa egrave data ad esempio da httpscdscernchrecord1213275filesp141pdf

18

rispetto al fascio di particelle a 1 m di distanza dal bersaglio Quante particelleα incideranno sul rivelatore ogni secondo

Soluzione dellrsquoesercizio 36Lrsquoalluminio ha una densitagrave di 27 gcm3 numero atomico 13 e massa atomica

27 uPoicheacute le particelle α sono nuclei di elio hanno carica 2e e la corrente di

032 nA corrisponde a un miliardo di particelle incidenti al secondo

dNidt

=032 nCs

2times 16times 10minus19 C= 1times 109 sminus1

Il rivelatore vede un angolo solido di

∆Ω equiv superficieraggio

2

=1 cm2

(1 m)2= 1times 10minus4 sr

Si tratta di uno scattering alla Rutherford per cui la sezione drsquourto per unitagravedi angolo solido rilevata ad un certo angolo θ vale

dΩ=

(zαzAle

2

4πε04E

1

sin2(θ2)

)2

pari a

dΩasymp(

2times 13times 4times etimes 16times 10minus19 C

4π times 89times 10minus12 Fmtimes 4times 100times 106 eV

1

sin2(π180 times 302)

)2

asymp 2times 10minus30 m2sr = 20 mbsr

(1)

e il numero di particelle visto dal rivelatore vale se indichiamo con nAl = ρAlNAAAl

la densitagrave numero di atomi di alluminio e con d lo spessore del rivelatore

dNrivelate

dt= ∆Ω

dΩnAld

dNi

dt

asymp 1times 10minus4 srtimes 2times 10minus30 m2srtimes 1times 104 cm2m2 times 27 gcm3 6times 1023 molminus1

27 gmol

= 120 Hz

Esercizio 37 Sezione drsquourtoUn bersaglio di idrogeno liquido di densitagrave ρ = 0071 gcm3 e volume V =

125 cm3 egrave bombardato da un fascio monoenergetico di pioni negativi con unflusso φ = 2times 107 mminus2sminus1 e si osserva la reazione

πminus + prarr π0 + n

rivelando i fotoni del decadimento π0 rarr γγ che avviene nel 988 dei casi Sela sezione drsquourto di quellrsquointerazione egrave σ = 40 mb quanti fotoni sono emessi ognisecondo

19

Esercizio 38 Sezione drsquourtoLa sezione drsquourto dellrsquoeffetto fotoelettrico per raggi X di 10 keV in carbonio

egrave 40 b per atomo Data una lastra di carbonio di 4 mm di spessore si calcoli1 il numero di bersagli per unitagrave di volume2 il coefficiente di assorbimento per effetto fotoelettrico dei raggi X di tale

energia3 la probabilitagrave che un raggio X incidente sulla lastra produca un elettrone

per effetto fotoelettrico

Esercizio 39 Sezioni drsquourtoUn bersaglio drsquooro di densitagrave superficiale ρS = 097 mgcm2 e superficie

SB = 1 cm2 viene colpito da un fascio di particelle α la cui sezione trasversaegrave contenuta completamente nellrsquoarea del bersaglio Sul bersaglio impattano37times 104 αs La sezione drsquourto di diffusione elastica ad un certo angolo θ valedσ

ddΩ = 1 bsr Calcolare1 la densitagrave di atomi del bersaglio per unitagrave di superficie2 il numero di particelle α rivelate in unrsquoora da un rivelatore di superficie

SR = 2 cm2 posto allrsquoangolo θ e a distanza DR = 01 m dal bersaglio3 Se il fascio di particelle viene sostituito da una sorgente radioattiva di

particelle α con distribuzione isotropa su tutto lrsquoangolo solido che vieneposta lungo la stessa linea del fascio a distanza DB = 20 cm dal bersaglioquanto tempo egrave necessario per rivelare con lo stesso rivelatore lo stessonumero di particelle calcolato sopra a paritagrave di sezione drsquourto

Esercizio 40 CinematicaUn fascio di positroni di 100 MeV di energia annichila su una targhetta fissa

di elettroni producendo due fotoni tramite il processo

e+ + eminus rarr γ + γ

Se uno dei due fotoni egrave emesso a 30 nel sistema del centro di massa1 quanto vale lrsquoenergia dei due fotoni nel sistema del centro di massa2 che energia e direzione avranno nel sistema del laboratorio

Esercizio 41 Sezione drsquourtoGli antineutrini νe prodotti da un reattore nucleare con potenza P = 16 GW

attraversano un bersaglio da 2000 l drsquoacqua posto a 50 m di distanza dal reat-tore

1 Supponendo che per ogni fissione sia prodotta unrsquoenergia termica di200 MeV e vengano emessi 6 antineutrini calcolare il numero medio direazioni

νe + prarr e+ + n

prodotte al giorno se la sezione drsquourto egrave σ = 1times 10minus43 cm2

20

2 Qual egrave la minima energia del neutrino per cui la reazione egrave permessa

6 Lezione 7 (8 maggio 2020)

Esercizio 42Un protone interagisce con un bersaglio producendo un pione di energia E =300 MeV Con un tracciatore posto a distanza d = 6 cm dal bersaglio egrave possi-bile rivelare la traiettoria del pione e risalire al punto di produzione del pionesul bersaglio Determinare lrsquoerrore sulla misura della posizione di tale puntocausato dalla presenza di un piano di alluminio di spessore L = 05 mm postoimmediatamente davanti al tracciatore (quindi a distanza d dal bersaglio) nellrsquoi-potesi che gli angoli delle tracce rispetto alla normale alle superfici del bersaglioe del piano di alluminio siano piccoli [mπ = 1396 MeV Al (Z = 13 A = 27ρ = 27 gcm3)]

Soluzione dellrsquoesercizio 42La deviazione standard dellrsquoangolo di diffusione coulombiana multipla vale

radic〈θ2〉 = 21 MeV

z

βc|p|

radicx

X0= 21 MeV

z

βc|p|

radicLρ

X0

dove p =radicE2 minusm2

π = 266 MeV β = pE = 0885 z = 1 Per lrsquoalluminioabbiamo

X0 = 24 gcm2 rarr X0ρ = 89 cm

per cui radic〈θ2〉 asymp 00067

e lo spostamento medio corrispondente rispetto alla posizione nominale egrave

δs = tan(00067)times 6 cm = 04 mm

Esercizio 43Un fascio misto di elettroni e antiprotoni passa attraverso una regione con

B = 2 T e dopo aver curvato per R = 3 m raggiunge una regione senza campomagnetico

1 Qual egrave lrsquoimpulso delle particelle selezionate2 Come discriminereste fra i due tipi di particelle3 Se usaste due scintillatori posti a 15 m di distanza quale risoluzione

temporale sarebbe necessaria per discriminare i due tipi di particelle4 Se i due scintillatori fossero spessi 2 cm e avessero una lunghezza di ra-

diazione X0 = 40 cm quanta energia perderebbero elettroni e protoninellrsquoattraversarli Si assuma una perdita di energia per ionizzazione di2 MeVcm e 25 MeVcm per protoni ed elettroni rispettivamente

21

5 Che indice di rifrazione dovrebbe avere un rivelatore a luce Cherenkovper discriminare elettroni e protoni

Soluzione dellrsquoesercizio 43Dalla definizione di forza

F =dpdt

= qvtimesB

|F| = mv2

R

per cui se il campo magnetico egrave ortogonale alla traiettoria

pc = qcBR

che poicheacute c = 03 mns possiamo esprimere in maniera piugrave conveniente come

pc[GeV] = 03B[T]R[m]

Ne segue che nel nostro esercizio (e tornando a c = 1)

p = 18 GeV

Trascurando le perdite di energia e ricordando che me mp i tempi divolo di elettroni e protoni valgono rispettivamente

te =L

βecasymp L

casymp 50 ns

etp =

L

βpc=

Lpradic

p2+m2p

casymp 56 ns

per cui serve una risoluzione dellrsquoordine del ns per discriminare elettroni e muonidal tempo di volo

Se gli scintillatori sono spessi 2 cm i protoni perderanno in ciascuno di essi2 cm times 2 MeVcm = 4 MeV per ionizzazione Gli elettroni perderanno invecenel primo rivelatore 5 MeV per ionizzazione e (me asymp 0)

18 GeV times(

1minus exp

(minus 2 cm

40 cm

))asymp 88 MeV

cioegrave dopo il primo rivelatore avranno 18 GeVminus93 MeV di energia e nel secondone perderanno altri

5 MeV + (18 GeV minus 93 MeV)

(1minus exp

(minus 2 cm

40 cm

))asymp 88 MeV

Lrsquoangolo di emissione di luce Cherenkov egrave dato da

cos θc =1

βnle 1

22

e poicheacute βe asymp 1 βp asymp 0887 per avere luce Cherenkov solo al passaggio deglielettroni (e non dei protoni) egrave necessario che

1 lt n lt 113

Esercizio 44Come misurereste la massa di una particella carica E quella di una parti-

cella neutra

Esercizio 45Un muone di energia E = 400 GeV penetra verticalmente nel mare Attra-

verso quale processo puograve essere rivelato A quale profonditagrave arriva prima didecadere

Esercizio 46Avete a disposizione dei tubi fotomoltiplicatori sensibili a lunghezze drsquoonda

fra 300 nm e 500 nm e volete rivelare la luce Cherenkov prodotta dal passaggio diun elettrone di 1 MeV di energia in un metro drsquoacqua Quanti fotoni vi aspettateche vengano prodotti Confrontate con il numero di elettroni di ionizzazioneche produrrebbe una particella α di 5 keV nello stesso rivelatore

Esercizio 47Determinare quali sono i processi piugrave probabili (cioegrave quelli di sezione drsquourto

piugrave alta) nellrsquointerazione fra1 fotoni di 1 MeV e atomi di alluminio2 fotoni di 100 keV e H23 fotoni di 100 keV e atomi di ferro4 fotoni di 10 MeV e atomi di carbonio5 fotoni di 10 MeV e atomi di piombo

Esercizio 48Volete misurare lrsquoimpulso di una particella carica che attraversa un campo

magnetico B ortogonale alla sua traiettoria Avete a disposizione tre rivelatori diposizione della stessa risoluzione spaziale δx come li disponete Che risoluzionein impulso vi aspettate di ottenere

23

  • Lezione 1 (13 marzo 2020)
  • Lezione 2 (20 marzo 2020)
  • Lezione 3 (27 marzo 2020)
  • Lezione 4 (3 aprile 2020)
  • Lezione 5 (16 aprile 2020)
  • Lezione 7 (8 maggio 2020)
Page 10: Esercizi per casa (risolti)ippolitv/pdf/fns1_2020_solutions.pdfEsercizi per casa (risolti) Valerio Ippolito 13 maggio 2020 1 Lezione 1 (13 marzo 2020) Esercizio1 Dilatazionedeitempi

Esercizio 14 Energia di sogliaSupponiamo di far scontrare un fascio di protoni con un protone a riposo

Qual egrave lrsquoenergia minima che devono avere i protoni del fascio percheacute la reazione

p+ prarr p+ p+ p+ p

sia permessa (La massa del protone egrave pari a quella dellrsquoantiprotone p e vale938 MeVc2)

Esercizio 15 Diffusione elasticaChiamiamo elastico un urto (ldquoscatteringrdquo) in cui le particelle dello stato ini-

ziale e dello stato finale sono le stesse Si consideri un urto elastico fra unaparticella di massa nulla e una particella di massa m (bersaglio) che si trovaa riposo nel sistema di riferimento del laboratorio qual egrave la massima energiatrasferita dalla particella incidente al bersaglio Suggerimento si lavori nelsistema di riferimento del laboratorio e si espliciti il prodotto scalare fra gliimpulsi spaziali della particella di massa nulla prima e dopo lrsquourto in funzionedellrsquoangolo sempre nel sistema di riferimento del laboratorio fra la direzioneiniziale e finale della particella incidente

Se la particella incidente egrave un fotone e il bersaglio egrave un elettrone atomico ariposo di quanto varia la lunghezza drsquoonda del fotone fra prima e dopo lrsquourto

Soluzione dellrsquoesercizio 15Per scattering elastico intendiamo un processo in cui le particelle dello stato

iniziale sono le stesse di quelle dello stato finaleDenotiamo con k e P i quadrimpulsi della particella incidente e del bersa-

glio prima dellrsquourto e indichiamo con lrsquoapice le stesse quantitagrave dopo lrsquourto ilproblema ci dice che

k = (Ek)

kprime = (Eprimekrsquo)P = (mc0)

Partiamo dalla conservazione del quadrimpulso durante lrsquourto isoliamo la quan-titagrave che non misuriamo direttamente ndash cioegrave il quadrimpulso del bersaglio dopolrsquourto P prime ndash ed eleviamo al quadrato

k + P = kprime + P prime

P prime = k + P minus kprimem2c2 = 0 +m2c2 + 0 + 2Emminus 2(EEprime minus k middot krsquo)minus 2mEprime

e se indichiamo con θprime lrsquoangolo ndash nel riferimento del laboratorio ndash fra la direzioneiniziale e finale della particella incidente e usiamo il fatto che |k|c = E e |krsquo|c =

10

Eprime

0 = 2mc2(E minus Eprime)minus 2(EEprime minus EEprime cos θprime)

mc2(Eprime minus E) = minusEEprime(1minus cos θprime)

Eprime(mc2 + E(1minus cos θprime)) = mc2E

Eprime =E

1 + Emc2 (1minus cos θprime)

Il bersaglio rinculeragrave di una energia E minus Eprime massima per θ = π Il valoremassimo di questrsquoenergia di rinculo

E minus E

1 + 2 Emc2

= E2Emc2

1 + 2Emc2

prende il nome ndash nel caso dello scattering Compton in cui la particella incidenteegrave un fotone e il bersaglio egrave un elettrone atomico ndash di picco Compton

Cosa cambia fra un fotone di energia E ed uno di energia Eprime Dalla mecca-nica quantistica

E = hν =hc

λ

cioegrave cambia la lunghezza drsquoonda del fotone

Eprime =hc

λprime=

hcλ

1 +hcλ

mc2 (1minus cos θprime)

1

λprime=

1 +hcλ

mc2 (1minus cos θprime)

λprime = λ

(1 +

hc

λmc2(1minus cos θprime)

)

λprime = λ+h

mc(1minus cos θprime) equiv λ+ λc(1minus cos θprime)

dove abbiamo definito la lunghezza drsquoonda Compton dellrsquoelettrone λc che rap-presenta la scala di lunghezza sotto la quale gli effetti della meccanica quantisticarelativistica divengono importanti

3 Lezione 3 (27 marzo 2020)

Esercizio 16 Trasformazione delle velocitagraveUn oggetto si muove di moto rettilineo uniforme con velocitagrave costante αc versoun secondo oggetto immobile A che velocitagrave dobbiamo muoverci noi lungo lastessa direzione per vedere entrambi gli oggetti muoversi con velocitagrave uguali eopposte

11

Soluzione dellrsquoesercizio 16Se indichiamo con lrsquoapice la velocitagrave nel nostro sistema di riferimento che

egrave in moto rispetto al sistema di riferimento del secondo oggetto la legge ditrasformazione delle velocitagrave ci dice che

vprime1 =v1 minus V

1minus v1cVc

=αcminus V1minus αVc

vprime2 =v2 minus V

1minus v2cVc

= minusV

Stiamo cercando V la nostra velocitagrave nel sistema di riferimento del secondooggetto tale che le velocitagrave dei due oggetti nel nostro sistema di riferimentosiano uguali e opposte

vprime1 = minusvprime2 = V

perciograve la richiesta egrave

αcminus V1minus αVc

= V

αcminus V minus V + αV 2

c= 0

(αc)V 2 + (minus2)V + (α)c = 0

V =1minusradic

1minus α2

αc

dove abbiamo scelto la soluzione dellrsquoequazione di secondo grado con V le c

Esercizio 17 Conseguenze della relativitagraveUna navicella spaziale in moto rettilineo uniforme con velocitagrave 05c in al-

lontanamento dalla Terra egrave in orbita verso Plutone che si trova a 75times 109 kmdi distanza dalla Terra Non appena raggiunto il pianeta la comandante inviaun segnale radio alla base a Houston per chiedere lrsquoautorizzazione allrsquoatter-raggio Quanto tempo impiega la richiesta a raggiungere la base secondo lacomandante E secondo i suoi colleghi a Houston

Soluzione dellrsquoesercizio 17Lrsquoonda radio viaggia a velocitagrave c secondo ogni sistema di riferimento Per

lrsquoosservatore lrsquoonda radio percorreragrave il tragitto Plutone-Terra in un tempo

tTerra =L

c=

75times 109 km

3times 108 msasymp 25 000 s

mentre per la comandante saragrave passato un tempo inferiore che dipende dal fat-tore γ dellrsquoastronave nel sistema di riferimento della Terra secondo la relazione

tcomandante =tTerra

γasymp 25 000 stimes

radic1minus 052 asymp 21 650 s

12

Esercizio 18 Energia cineticaHa piugrave energia un protone che viaggia a 09999999896c o un Boeing 747 al

decollo

Esercizio 19 Dilatazione dei tempiUna scienziata misura che un fascio di particelle selezionate con impulso di

10 GeVc si degrada dellrsquo84 dopo aver percorso 1 m Se la massa di questeparticelle egrave 498 MeVc2 qual egrave la loro vita media

Soluzione dellrsquoesercizio 19La distanza misurata dalla scienziata egrave chiaramente riferita al suo sistema

di riferimento quello del laboratorio ed egrave legata alla vita media delle particelledel fascio τ dalla relazione

Ldecay = 1 m = βγcτ =pc

E

E

mc2cτ =

p

mccτ

e dalla legge del decadimento se indichiamo con N0 il numero di particelleinizialmente presenti nel fascio e con N il numero di particelle misurato si haN

N0= 1minus084 = exp

(minus L

Ldecay

)= exp

(minus L

pmccτ

)rarr τ =

mcL

pc log(N0

N

) asymp 9times 10minus11 s

Esercizio 20 Dilatazione dei tempiVi trovate a dover studiare un fascio di particelle di cui conoscete lrsquoenergia

ndash 2 GeV ndash ma non la massa a quanto ne sapete potrebbero essere composti daelettroni (di massa 511 keVc2) o protoni (938 MeVc2) Avete a disposizionedue rivelatori identici in grado di registrare con precisione il tempo in cui unaparticella li attraversa Come potete utilizzarli per determinare se il vostrofascio contiene elettroni o protoni

Soluzione dellrsquoesercizio 20Disponendo i due rivelatori lungo la direzione del fascio a distanza ∆L

lrsquouno dallrsquoaltro si puograve misurare il tempo impiegato dalle particelle per passaredallrsquouno allrsquoaltro e quindi la loro massa infatti se ∆t egrave la distanza temporalefra i segnali dei due rivelatori

∆L = v∆t = βc∆t =pc2

E∆t

Questo principio egrave usato nei cosiddetti rivelatori di time of flight che sono usatiper discriminare diversi tipi di particelle una particella di tipo 1 e una di tipo2 infatti percorreranno la distanza ∆L in tempi diversi legati alle rispettivemasse dalla relazione

∆L1 = ∆L2

p1c2

E1∆t1 =

radicE2 minusm2

1c2c

E∆t1 =

radicE2 minusm2

2c2c

E∆t2

13

Esercizio 21 Energia di sogliaDue fisici delle particelle vogliono produrre il bosone Z una particella di

carica neutra e di massamZ = 91 GeVc2 e discutono come fare Alice proponedi far scontrare fasci di elettroni e positroni di energia identica che viaggianodunque con impulso spaziale uguale in modulo e direzione ma di verso oppostoproducendo Z tramite il processo

e+ + eminus rarr Z

mentre Bob preferisce scontrare un fascio di protoni su un bersaglio fisso diidrogeno tramite il processo

p+ prarr Z + p+ p

Chi dei due avragrave bisogno di fasci di particelle di energia piugrave alta La massa delprotone egrave di 9383 MeVc2 quella dellrsquoelettrone di 511 keVc2

Esercizio 22 Leggi di conservazioneUn fotone (particella di massa nulla) puograve decadere in un elettrone e in un

positrone (entrambi di massa 511 keVc2) tramite il processo

γ rarr e+ + eminus

Esercizio 23 Decadimento βminus

Quali sono lrsquoenergia minima e massima dellrsquoelettrone nel decadimento

nrarr p+ eminus + νe

se il neutrone decade da fermo La massa del neutrone egrave di 9396 MeVc2 quelladel protone di 9383 MeVc2 e quella dellrsquoelettrone di 511 keVc2 si assuma chelrsquoantineutrino elettronico νe abbia massa nulla

Esercizio 24 Leggi di conservazioneIl decadimento

prarr n+ e+ + νe

egrave permesso

4 Lezione 4 (3 aprile 2020)

Esercizio 25Usando il fatto che hc = 1973 MeVfm si dimostri che in un sistema di unitagrave dimisura in cui h = c = 1 vale

14

1 1 GeVminus2 = 0389 mb

2 1 m = 5068times 1015 GeVminus1

3 1 s = 15times 1024 GeVminus1

Ricordiamo che 1 b = 1times 10minus28 m2 e che

[hc] = [Jsms] = [E][L]

Soluzione dellrsquoesercizio 25Lrsquoidea egrave di capire per quale potenza di hc e c va moltiplicato il termine a

sinistra di ciascuna equazione per ottenere il termine di destraPer cui

bull [1 GeVminus2][hc]α = [E]minus2[E]α[L]α = [0389 mb] = [L]2 da cui segue α = 2

e 1 GeVminus2(hc)2 = 1973 MeVfm1 GeV =

(01973times 10minus15 GeVm

1 GeV

)2

= 0389 mb

bull [1 m][hc]α = [L][E]α[L]α = [5068times 1015 GeVminus1] = [E]minus1 da cui segueα = minus1 e 1 m(hc)minus1 = 1 m

1973 MeVfm = 1 m01973times 10minus15 GeVm

= 5068times 1015 GeVminus1

bull [1 s][hc]α[c]β = [T ][E]α[L]α[L]β [T ]minusβ = [T ][E]α[L]α+β [T ]minusβ = [15times 1024 GeVminus1] =[E]minus1 da cui segue α = minus1 β = 1 e 1 s(hc)minus1c = 1 s

1973 MeVfm299 792 458 ms =299 792 458 m

01973times 10minus15 GeVm= 15times 1024 GeVminus1

Esercizio 26 Massa invarianteTre protoni (mp = 938 MeVc2) hanno impulsi uguali in modulo (p =

3 GeVc) e che formano angoli di 120 lrsquouno con lrsquoaltro Qual egrave la massainvariante del sistema

Esercizio 27 Energia di sogliaSi consideri il processo

γ + prarr p+ π0

dove il fotone ha massa nulla il protone ha massa di 938 MeVc2 e il π0 hamassa di 135 MeVc2

1 Se il protone egrave a riposo qual egrave lrsquoenergia minima che deve avere il fotoneincidente percheacute la reazione abbia luogo

2 La stessa reazione puograve avvenire nellrsquouniverso in cui un protone dei raggicosmici di alta energia puograve collidere con uno dei fotoni della radiazionecosmica di fondo di energia dellrsquoordine di 1 meV Qual egrave in questo casolrsquoenergia minima che deve avere il protone percheacute la reazione abbia luogo

Esercizio 28 Conseguenze della relativitagraveUn misterioso supereroe pattuglia a velocitagrave molto elevata la periferia roma-

na Allrsquoincrocio con via di Tor Bella Monaca incontra un semaforo e ndash vedendolo

15

verde ndash attraversa senza rallentare Una pattuglia della polizia municipale loferma e lo multa per esser passato col rosso Assumendo sia il supereroe che ivigili siano nel giusto a che velocitagrave viaggiava il supereroe

Soluzione dellrsquoesercizio 28Per la polizia municipale il semaforo emette fotoni di energia

E0 = hν0 =hc

λrossoasymp hc

630 nm

mentre il supereoe vede fotoni di energia

E = hν =hc

λverdeasymp hc

490 nm

e dalle trasformazioni di Lorentz indicando con γ e β le variabili calcolateusando la velocitagrave del supereroe misurata dalla municipale possiamo scrivere

E = γ(E0minusβp0) = γ(E0minusβE0) =1minus βradic1minus β2

E0 =1minus βradic

(1 + β)(1minus β)E0 =

radic1minus βradic1 + β

E0

dove abbiamo usato il fatto che i fotoni hanno massa nulla Perciograve

E

E0=

hcλverdehc

λrosso

=λrosso

λverde=

radic1minus βradic1 + β

rarr v = βc = 025c

Esercizio 29 Conseguenze della relativitagraveI neutrini sono particelle di massa molto piccola e al momento ignota Uno

dei modi con cui egrave stato possibile dedurre un limite superiore al suo valore egravestata lrsquoosservazione nel 1987 di neutrini prodotti dalla supernova 1987A1 chesi trova a 168000 anni luce dalla Terra Sono stati osservati due segnali dineutrini che possono essere schematizzati come segue si egrave osservato prima unneutrino di 35 MeV di energia seguito a 9 s di distanza da un secondo segnaledi 13 MeV Si assuma che questo ritardo sia dovuto al fatto che la massa delneutrino non egrave nulla e si calcoli questrsquoultima

Esercizio 30 DecadimentoAnimali e piante assumono dallrsquoatmosfera diversi composti contenenti car-

bonio Il carbonio presente nellrsquoatmosfera egrave predominantemente 126 C ma sono

presenti piccole concentrazioni del suo isotopo 146 C (un atomo ogni 1012) che

decade con emissione di elettroni attraverso il processo2

146 C rarr14

7 N + eminus + νe

con un tempo di dimezzamento di 5700 anni1httpsenwikipediaorgwikiSN_1987ANeutrino_emissions2Di altro non si tratta che del decadimento βminus nrarr p+ e+ νe

16

1 Qual egrave la concentrazione di 146 C dopo 11400 anni

2 Animali e piante assumono in vita proporzioni fisse di 146 C e 12

6 C mentrealla loro morte la quantitagrave di 14

6 C inizia a diminuire Avete a portata dimano un relitto di legno per cui misurate una emissione di elettroni daldecadimento di 14

6 C pari al 61 di quella di un pezzo di legno vivodella stessa massa quanti anni ha il manufatto

Esercizio 31 Energia cinetica e trasformazioni di LorentzDue particelle identiche di massa m ed energia cinetica T collidono frontal-

mente Qual egrave la loro energia cinetica relativa (ossia lrsquoenergia cinetica di unaparticella misurata nel sistema di riferimento dellrsquoaltra particella)

Esercizio 32 Energia nel centro di massaLa reazione

πminusprarr K0Λ0

avviene con unrsquoenergia nel centro di massa diradics = 3 GeV La massa del πminus egrave

di 1396 MeVc2 la massa del protone egrave di 938 MeVc2 la massa del Kminus egrave di498 MeVc2 e quella della Λ0 di 11 GeVc2

1 Calcolare lrsquoimpulso di πminus e Λ0 nel sistema di riferimento del centro dimassa

2 Se il protone egrave a riposo il K puograve essere emesso allrsquoindietro nel sistemadi riferimento del laboratorio

Esercizio 33 DecadimentoIl mesone φ0 egrave una particella neutra3 di circa 1 GeVc2 di massa che puograve

decadere in una coppia di particelle

φ0 rarr K+ +Kminus

di massa identicamK = 494 MeVc2 Si assuma di produrre φ0 di impulso notoegrave possibile che uno dei due K sia prodotto a riposo nel sistema di riferimentodel laboratorio

Esercizio 34 DecadimentoUn fascio di anti-neutrini muonici νmicro si puograve generare selezionando pioni

o kaoni π+ e K+ e facendoli passare in un lungo tubo in cui egrave stato fattoil vuoto4 in modo che dopo un certo tragitto L una buona parte di loro saragrave

3Il collisore DAFNE ai Laboratori Nazionali di Frascati produce specificatamente parti-celle di questo tipo tramite il processo e+ + eminus rarr φ0 httpswwwyoutubecomwatchv=L5yB9gDGKms

4Una tecnica di questo tipo egrave stata usata per inviare ai Laboratori Nazionali del Gran Sassodei fasci di neutrini prodotti al CERN di Ginevra httpsvideoscernchrecord985892

17

decaduta tramite i processi

π+ rarr microminus + νmicro

K+ rarr microminus + νmicro

Se lrsquoimpulso di pioni e kaoni egrave di 200 GeVc e la loro vita media di 26 ns e12 ns rispettivamente

1 Quanto a lungo viaggiano nel laboratorio i due tipi di particelle2 Se L = 1000 m quale saragrave la frazione di pioni e kaoni che saragrave decaduta

alla fine del tubo3 Qual egrave lrsquoenergia massima dei neutrini che egrave possibile misurare nel sistema

di riferimento del laboratorio nei due casi

5 Lezione 5 (16 aprile 2020)

Esercizio 35 Energia cineticaTra le eccellenze con sede in territorio elvetico primeggiano indiscutibilmente lacioccolata al latte e il Large Hadron Collider (LHC) Questrsquoultimo egrave un collisoredi particelle situato al CERN di Ginevra che fa scontrare due fasci identici diprotoni di impulso uguale in modulo e direzione ma verso opposto I fasci sonocomposti da circa 2800 gruppi (pacchetti) di 1011 particelle ciascuno Sapendoche lrsquoenergia nel centro di massa della collisione fra due protoni egrave

radics = 13 TeV

quanta cioccolata al latte dovete mangiare per assumere un numero di caloriepari allrsquoenergia cinetica di ciascun fascio di protoni di LHC

Soluzione dellrsquoesercizio 35Poicheacute si tratta di un collider e le energie in gioco sono molto maggiori della

massa del protone lrsquoenergia di ciascun protone egrave diradics2 = 65 TeV e quella

totale di un fascio egrave

2800times 1011 times 65 middot 1012 times 16 middot 10minus19J asymp 300 MJ

Secondo Google 100 g di cioccolata al latte apportano circa 500 cal = 500 times4184 J asymp 2 MJ Lrsquoenergia di un fascio di LHC corrisponde quindi a quella dicirca 15 kg di cioccolata

Esercizio 36 Scattering RutherfordUn fascio di particelle α di 100 MeV di energia e 032 nA di corrente5 collide

contro un bersaglio fisso di alluminio spesso 1 cm Una sperimentatrice prendeun rivelatore di 1 cm times 1 cm di superficie e lo posiziona ad un angolo di 30

5Per una spiegazione breve su come (e percheacute) si misura la corrente di un fascio di particel-le vedi httpswwwlhc-closerestaking_a_closer_look_at_lhc0beam_current Unatrattazione piugrave completa egrave data ad esempio da httpscdscernchrecord1213275filesp141pdf

18

rispetto al fascio di particelle a 1 m di distanza dal bersaglio Quante particelleα incideranno sul rivelatore ogni secondo

Soluzione dellrsquoesercizio 36Lrsquoalluminio ha una densitagrave di 27 gcm3 numero atomico 13 e massa atomica

27 uPoicheacute le particelle α sono nuclei di elio hanno carica 2e e la corrente di

032 nA corrisponde a un miliardo di particelle incidenti al secondo

dNidt

=032 nCs

2times 16times 10minus19 C= 1times 109 sminus1

Il rivelatore vede un angolo solido di

∆Ω equiv superficieraggio

2

=1 cm2

(1 m)2= 1times 10minus4 sr

Si tratta di uno scattering alla Rutherford per cui la sezione drsquourto per unitagravedi angolo solido rilevata ad un certo angolo θ vale

dΩ=

(zαzAle

2

4πε04E

1

sin2(θ2)

)2

pari a

dΩasymp(

2times 13times 4times etimes 16times 10minus19 C

4π times 89times 10minus12 Fmtimes 4times 100times 106 eV

1

sin2(π180 times 302)

)2

asymp 2times 10minus30 m2sr = 20 mbsr

(1)

e il numero di particelle visto dal rivelatore vale se indichiamo con nAl = ρAlNAAAl

la densitagrave numero di atomi di alluminio e con d lo spessore del rivelatore

dNrivelate

dt= ∆Ω

dΩnAld

dNi

dt

asymp 1times 10minus4 srtimes 2times 10minus30 m2srtimes 1times 104 cm2m2 times 27 gcm3 6times 1023 molminus1

27 gmol

= 120 Hz

Esercizio 37 Sezione drsquourtoUn bersaglio di idrogeno liquido di densitagrave ρ = 0071 gcm3 e volume V =

125 cm3 egrave bombardato da un fascio monoenergetico di pioni negativi con unflusso φ = 2times 107 mminus2sminus1 e si osserva la reazione

πminus + prarr π0 + n

rivelando i fotoni del decadimento π0 rarr γγ che avviene nel 988 dei casi Sela sezione drsquourto di quellrsquointerazione egrave σ = 40 mb quanti fotoni sono emessi ognisecondo

19

Esercizio 38 Sezione drsquourtoLa sezione drsquourto dellrsquoeffetto fotoelettrico per raggi X di 10 keV in carbonio

egrave 40 b per atomo Data una lastra di carbonio di 4 mm di spessore si calcoli1 il numero di bersagli per unitagrave di volume2 il coefficiente di assorbimento per effetto fotoelettrico dei raggi X di tale

energia3 la probabilitagrave che un raggio X incidente sulla lastra produca un elettrone

per effetto fotoelettrico

Esercizio 39 Sezioni drsquourtoUn bersaglio drsquooro di densitagrave superficiale ρS = 097 mgcm2 e superficie

SB = 1 cm2 viene colpito da un fascio di particelle α la cui sezione trasversaegrave contenuta completamente nellrsquoarea del bersaglio Sul bersaglio impattano37times 104 αs La sezione drsquourto di diffusione elastica ad un certo angolo θ valedσ

ddΩ = 1 bsr Calcolare1 la densitagrave di atomi del bersaglio per unitagrave di superficie2 il numero di particelle α rivelate in unrsquoora da un rivelatore di superficie

SR = 2 cm2 posto allrsquoangolo θ e a distanza DR = 01 m dal bersaglio3 Se il fascio di particelle viene sostituito da una sorgente radioattiva di

particelle α con distribuzione isotropa su tutto lrsquoangolo solido che vieneposta lungo la stessa linea del fascio a distanza DB = 20 cm dal bersaglioquanto tempo egrave necessario per rivelare con lo stesso rivelatore lo stessonumero di particelle calcolato sopra a paritagrave di sezione drsquourto

Esercizio 40 CinematicaUn fascio di positroni di 100 MeV di energia annichila su una targhetta fissa

di elettroni producendo due fotoni tramite il processo

e+ + eminus rarr γ + γ

Se uno dei due fotoni egrave emesso a 30 nel sistema del centro di massa1 quanto vale lrsquoenergia dei due fotoni nel sistema del centro di massa2 che energia e direzione avranno nel sistema del laboratorio

Esercizio 41 Sezione drsquourtoGli antineutrini νe prodotti da un reattore nucleare con potenza P = 16 GW

attraversano un bersaglio da 2000 l drsquoacqua posto a 50 m di distanza dal reat-tore

1 Supponendo che per ogni fissione sia prodotta unrsquoenergia termica di200 MeV e vengano emessi 6 antineutrini calcolare il numero medio direazioni

νe + prarr e+ + n

prodotte al giorno se la sezione drsquourto egrave σ = 1times 10minus43 cm2

20

2 Qual egrave la minima energia del neutrino per cui la reazione egrave permessa

6 Lezione 7 (8 maggio 2020)

Esercizio 42Un protone interagisce con un bersaglio producendo un pione di energia E =300 MeV Con un tracciatore posto a distanza d = 6 cm dal bersaglio egrave possi-bile rivelare la traiettoria del pione e risalire al punto di produzione del pionesul bersaglio Determinare lrsquoerrore sulla misura della posizione di tale puntocausato dalla presenza di un piano di alluminio di spessore L = 05 mm postoimmediatamente davanti al tracciatore (quindi a distanza d dal bersaglio) nellrsquoi-potesi che gli angoli delle tracce rispetto alla normale alle superfici del bersaglioe del piano di alluminio siano piccoli [mπ = 1396 MeV Al (Z = 13 A = 27ρ = 27 gcm3)]

Soluzione dellrsquoesercizio 42La deviazione standard dellrsquoangolo di diffusione coulombiana multipla vale

radic〈θ2〉 = 21 MeV

z

βc|p|

radicx

X0= 21 MeV

z

βc|p|

radicLρ

X0

dove p =radicE2 minusm2

π = 266 MeV β = pE = 0885 z = 1 Per lrsquoalluminioabbiamo

X0 = 24 gcm2 rarr X0ρ = 89 cm

per cui radic〈θ2〉 asymp 00067

e lo spostamento medio corrispondente rispetto alla posizione nominale egrave

δs = tan(00067)times 6 cm = 04 mm

Esercizio 43Un fascio misto di elettroni e antiprotoni passa attraverso una regione con

B = 2 T e dopo aver curvato per R = 3 m raggiunge una regione senza campomagnetico

1 Qual egrave lrsquoimpulso delle particelle selezionate2 Come discriminereste fra i due tipi di particelle3 Se usaste due scintillatori posti a 15 m di distanza quale risoluzione

temporale sarebbe necessaria per discriminare i due tipi di particelle4 Se i due scintillatori fossero spessi 2 cm e avessero una lunghezza di ra-

diazione X0 = 40 cm quanta energia perderebbero elettroni e protoninellrsquoattraversarli Si assuma una perdita di energia per ionizzazione di2 MeVcm e 25 MeVcm per protoni ed elettroni rispettivamente

21

5 Che indice di rifrazione dovrebbe avere un rivelatore a luce Cherenkovper discriminare elettroni e protoni

Soluzione dellrsquoesercizio 43Dalla definizione di forza

F =dpdt

= qvtimesB

|F| = mv2

R

per cui se il campo magnetico egrave ortogonale alla traiettoria

pc = qcBR

che poicheacute c = 03 mns possiamo esprimere in maniera piugrave conveniente come

pc[GeV] = 03B[T]R[m]

Ne segue che nel nostro esercizio (e tornando a c = 1)

p = 18 GeV

Trascurando le perdite di energia e ricordando che me mp i tempi divolo di elettroni e protoni valgono rispettivamente

te =L

βecasymp L

casymp 50 ns

etp =

L

βpc=

Lpradic

p2+m2p

casymp 56 ns

per cui serve una risoluzione dellrsquoordine del ns per discriminare elettroni e muonidal tempo di volo

Se gli scintillatori sono spessi 2 cm i protoni perderanno in ciascuno di essi2 cm times 2 MeVcm = 4 MeV per ionizzazione Gli elettroni perderanno invecenel primo rivelatore 5 MeV per ionizzazione e (me asymp 0)

18 GeV times(

1minus exp

(minus 2 cm

40 cm

))asymp 88 MeV

cioegrave dopo il primo rivelatore avranno 18 GeVminus93 MeV di energia e nel secondone perderanno altri

5 MeV + (18 GeV minus 93 MeV)

(1minus exp

(minus 2 cm

40 cm

))asymp 88 MeV

Lrsquoangolo di emissione di luce Cherenkov egrave dato da

cos θc =1

βnle 1

22

e poicheacute βe asymp 1 βp asymp 0887 per avere luce Cherenkov solo al passaggio deglielettroni (e non dei protoni) egrave necessario che

1 lt n lt 113

Esercizio 44Come misurereste la massa di una particella carica E quella di una parti-

cella neutra

Esercizio 45Un muone di energia E = 400 GeV penetra verticalmente nel mare Attra-

verso quale processo puograve essere rivelato A quale profonditagrave arriva prima didecadere

Esercizio 46Avete a disposizione dei tubi fotomoltiplicatori sensibili a lunghezze drsquoonda

fra 300 nm e 500 nm e volete rivelare la luce Cherenkov prodotta dal passaggio diun elettrone di 1 MeV di energia in un metro drsquoacqua Quanti fotoni vi aspettateche vengano prodotti Confrontate con il numero di elettroni di ionizzazioneche produrrebbe una particella α di 5 keV nello stesso rivelatore

Esercizio 47Determinare quali sono i processi piugrave probabili (cioegrave quelli di sezione drsquourto

piugrave alta) nellrsquointerazione fra1 fotoni di 1 MeV e atomi di alluminio2 fotoni di 100 keV e H23 fotoni di 100 keV e atomi di ferro4 fotoni di 10 MeV e atomi di carbonio5 fotoni di 10 MeV e atomi di piombo

Esercizio 48Volete misurare lrsquoimpulso di una particella carica che attraversa un campo

magnetico B ortogonale alla sua traiettoria Avete a disposizione tre rivelatori diposizione della stessa risoluzione spaziale δx come li disponete Che risoluzionein impulso vi aspettate di ottenere

23

  • Lezione 1 (13 marzo 2020)
  • Lezione 2 (20 marzo 2020)
  • Lezione 3 (27 marzo 2020)
  • Lezione 4 (3 aprile 2020)
  • Lezione 5 (16 aprile 2020)
  • Lezione 7 (8 maggio 2020)
Page 11: Esercizi per casa (risolti)ippolitv/pdf/fns1_2020_solutions.pdfEsercizi per casa (risolti) Valerio Ippolito 13 maggio 2020 1 Lezione 1 (13 marzo 2020) Esercizio1 Dilatazionedeitempi

Eprime

0 = 2mc2(E minus Eprime)minus 2(EEprime minus EEprime cos θprime)

mc2(Eprime minus E) = minusEEprime(1minus cos θprime)

Eprime(mc2 + E(1minus cos θprime)) = mc2E

Eprime =E

1 + Emc2 (1minus cos θprime)

Il bersaglio rinculeragrave di una energia E minus Eprime massima per θ = π Il valoremassimo di questrsquoenergia di rinculo

E minus E

1 + 2 Emc2

= E2Emc2

1 + 2Emc2

prende il nome ndash nel caso dello scattering Compton in cui la particella incidenteegrave un fotone e il bersaglio egrave un elettrone atomico ndash di picco Compton

Cosa cambia fra un fotone di energia E ed uno di energia Eprime Dalla mecca-nica quantistica

E = hν =hc

λ

cioegrave cambia la lunghezza drsquoonda del fotone

Eprime =hc

λprime=

hcλ

1 +hcλ

mc2 (1minus cos θprime)

1

λprime=

1 +hcλ

mc2 (1minus cos θprime)

λprime = λ

(1 +

hc

λmc2(1minus cos θprime)

)

λprime = λ+h

mc(1minus cos θprime) equiv λ+ λc(1minus cos θprime)

dove abbiamo definito la lunghezza drsquoonda Compton dellrsquoelettrone λc che rap-presenta la scala di lunghezza sotto la quale gli effetti della meccanica quantisticarelativistica divengono importanti

3 Lezione 3 (27 marzo 2020)

Esercizio 16 Trasformazione delle velocitagraveUn oggetto si muove di moto rettilineo uniforme con velocitagrave costante αc versoun secondo oggetto immobile A che velocitagrave dobbiamo muoverci noi lungo lastessa direzione per vedere entrambi gli oggetti muoversi con velocitagrave uguali eopposte

11

Soluzione dellrsquoesercizio 16Se indichiamo con lrsquoapice la velocitagrave nel nostro sistema di riferimento che

egrave in moto rispetto al sistema di riferimento del secondo oggetto la legge ditrasformazione delle velocitagrave ci dice che

vprime1 =v1 minus V

1minus v1cVc

=αcminus V1minus αVc

vprime2 =v2 minus V

1minus v2cVc

= minusV

Stiamo cercando V la nostra velocitagrave nel sistema di riferimento del secondooggetto tale che le velocitagrave dei due oggetti nel nostro sistema di riferimentosiano uguali e opposte

vprime1 = minusvprime2 = V

perciograve la richiesta egrave

αcminus V1minus αVc

= V

αcminus V minus V + αV 2

c= 0

(αc)V 2 + (minus2)V + (α)c = 0

V =1minusradic

1minus α2

αc

dove abbiamo scelto la soluzione dellrsquoequazione di secondo grado con V le c

Esercizio 17 Conseguenze della relativitagraveUna navicella spaziale in moto rettilineo uniforme con velocitagrave 05c in al-

lontanamento dalla Terra egrave in orbita verso Plutone che si trova a 75times 109 kmdi distanza dalla Terra Non appena raggiunto il pianeta la comandante inviaun segnale radio alla base a Houston per chiedere lrsquoautorizzazione allrsquoatter-raggio Quanto tempo impiega la richiesta a raggiungere la base secondo lacomandante E secondo i suoi colleghi a Houston

Soluzione dellrsquoesercizio 17Lrsquoonda radio viaggia a velocitagrave c secondo ogni sistema di riferimento Per

lrsquoosservatore lrsquoonda radio percorreragrave il tragitto Plutone-Terra in un tempo

tTerra =L

c=

75times 109 km

3times 108 msasymp 25 000 s

mentre per la comandante saragrave passato un tempo inferiore che dipende dal fat-tore γ dellrsquoastronave nel sistema di riferimento della Terra secondo la relazione

tcomandante =tTerra

γasymp 25 000 stimes

radic1minus 052 asymp 21 650 s

12

Esercizio 18 Energia cineticaHa piugrave energia un protone che viaggia a 09999999896c o un Boeing 747 al

decollo

Esercizio 19 Dilatazione dei tempiUna scienziata misura che un fascio di particelle selezionate con impulso di

10 GeVc si degrada dellrsquo84 dopo aver percorso 1 m Se la massa di questeparticelle egrave 498 MeVc2 qual egrave la loro vita media

Soluzione dellrsquoesercizio 19La distanza misurata dalla scienziata egrave chiaramente riferita al suo sistema

di riferimento quello del laboratorio ed egrave legata alla vita media delle particelledel fascio τ dalla relazione

Ldecay = 1 m = βγcτ =pc

E

E

mc2cτ =

p

mccτ

e dalla legge del decadimento se indichiamo con N0 il numero di particelleinizialmente presenti nel fascio e con N il numero di particelle misurato si haN

N0= 1minus084 = exp

(minus L

Ldecay

)= exp

(minus L

pmccτ

)rarr τ =

mcL

pc log(N0

N

) asymp 9times 10minus11 s

Esercizio 20 Dilatazione dei tempiVi trovate a dover studiare un fascio di particelle di cui conoscete lrsquoenergia

ndash 2 GeV ndash ma non la massa a quanto ne sapete potrebbero essere composti daelettroni (di massa 511 keVc2) o protoni (938 MeVc2) Avete a disposizionedue rivelatori identici in grado di registrare con precisione il tempo in cui unaparticella li attraversa Come potete utilizzarli per determinare se il vostrofascio contiene elettroni o protoni

Soluzione dellrsquoesercizio 20Disponendo i due rivelatori lungo la direzione del fascio a distanza ∆L

lrsquouno dallrsquoaltro si puograve misurare il tempo impiegato dalle particelle per passaredallrsquouno allrsquoaltro e quindi la loro massa infatti se ∆t egrave la distanza temporalefra i segnali dei due rivelatori

∆L = v∆t = βc∆t =pc2

E∆t

Questo principio egrave usato nei cosiddetti rivelatori di time of flight che sono usatiper discriminare diversi tipi di particelle una particella di tipo 1 e una di tipo2 infatti percorreranno la distanza ∆L in tempi diversi legati alle rispettivemasse dalla relazione

∆L1 = ∆L2

p1c2

E1∆t1 =

radicE2 minusm2

1c2c

E∆t1 =

radicE2 minusm2

2c2c

E∆t2

13

Esercizio 21 Energia di sogliaDue fisici delle particelle vogliono produrre il bosone Z una particella di

carica neutra e di massamZ = 91 GeVc2 e discutono come fare Alice proponedi far scontrare fasci di elettroni e positroni di energia identica che viaggianodunque con impulso spaziale uguale in modulo e direzione ma di verso oppostoproducendo Z tramite il processo

e+ + eminus rarr Z

mentre Bob preferisce scontrare un fascio di protoni su un bersaglio fisso diidrogeno tramite il processo

p+ prarr Z + p+ p

Chi dei due avragrave bisogno di fasci di particelle di energia piugrave alta La massa delprotone egrave di 9383 MeVc2 quella dellrsquoelettrone di 511 keVc2

Esercizio 22 Leggi di conservazioneUn fotone (particella di massa nulla) puograve decadere in un elettrone e in un

positrone (entrambi di massa 511 keVc2) tramite il processo

γ rarr e+ + eminus

Esercizio 23 Decadimento βminus

Quali sono lrsquoenergia minima e massima dellrsquoelettrone nel decadimento

nrarr p+ eminus + νe

se il neutrone decade da fermo La massa del neutrone egrave di 9396 MeVc2 quelladel protone di 9383 MeVc2 e quella dellrsquoelettrone di 511 keVc2 si assuma chelrsquoantineutrino elettronico νe abbia massa nulla

Esercizio 24 Leggi di conservazioneIl decadimento

prarr n+ e+ + νe

egrave permesso

4 Lezione 4 (3 aprile 2020)

Esercizio 25Usando il fatto che hc = 1973 MeVfm si dimostri che in un sistema di unitagrave dimisura in cui h = c = 1 vale

14

1 1 GeVminus2 = 0389 mb

2 1 m = 5068times 1015 GeVminus1

3 1 s = 15times 1024 GeVminus1

Ricordiamo che 1 b = 1times 10minus28 m2 e che

[hc] = [Jsms] = [E][L]

Soluzione dellrsquoesercizio 25Lrsquoidea egrave di capire per quale potenza di hc e c va moltiplicato il termine a

sinistra di ciascuna equazione per ottenere il termine di destraPer cui

bull [1 GeVminus2][hc]α = [E]minus2[E]α[L]α = [0389 mb] = [L]2 da cui segue α = 2

e 1 GeVminus2(hc)2 = 1973 MeVfm1 GeV =

(01973times 10minus15 GeVm

1 GeV

)2

= 0389 mb

bull [1 m][hc]α = [L][E]α[L]α = [5068times 1015 GeVminus1] = [E]minus1 da cui segueα = minus1 e 1 m(hc)minus1 = 1 m

1973 MeVfm = 1 m01973times 10minus15 GeVm

= 5068times 1015 GeVminus1

bull [1 s][hc]α[c]β = [T ][E]α[L]α[L]β [T ]minusβ = [T ][E]α[L]α+β [T ]minusβ = [15times 1024 GeVminus1] =[E]minus1 da cui segue α = minus1 β = 1 e 1 s(hc)minus1c = 1 s

1973 MeVfm299 792 458 ms =299 792 458 m

01973times 10minus15 GeVm= 15times 1024 GeVminus1

Esercizio 26 Massa invarianteTre protoni (mp = 938 MeVc2) hanno impulsi uguali in modulo (p =

3 GeVc) e che formano angoli di 120 lrsquouno con lrsquoaltro Qual egrave la massainvariante del sistema

Esercizio 27 Energia di sogliaSi consideri il processo

γ + prarr p+ π0

dove il fotone ha massa nulla il protone ha massa di 938 MeVc2 e il π0 hamassa di 135 MeVc2

1 Se il protone egrave a riposo qual egrave lrsquoenergia minima che deve avere il fotoneincidente percheacute la reazione abbia luogo

2 La stessa reazione puograve avvenire nellrsquouniverso in cui un protone dei raggicosmici di alta energia puograve collidere con uno dei fotoni della radiazionecosmica di fondo di energia dellrsquoordine di 1 meV Qual egrave in questo casolrsquoenergia minima che deve avere il protone percheacute la reazione abbia luogo

Esercizio 28 Conseguenze della relativitagraveUn misterioso supereroe pattuglia a velocitagrave molto elevata la periferia roma-

na Allrsquoincrocio con via di Tor Bella Monaca incontra un semaforo e ndash vedendolo

15

verde ndash attraversa senza rallentare Una pattuglia della polizia municipale loferma e lo multa per esser passato col rosso Assumendo sia il supereroe che ivigili siano nel giusto a che velocitagrave viaggiava il supereroe

Soluzione dellrsquoesercizio 28Per la polizia municipale il semaforo emette fotoni di energia

E0 = hν0 =hc

λrossoasymp hc

630 nm

mentre il supereoe vede fotoni di energia

E = hν =hc

λverdeasymp hc

490 nm

e dalle trasformazioni di Lorentz indicando con γ e β le variabili calcolateusando la velocitagrave del supereroe misurata dalla municipale possiamo scrivere

E = γ(E0minusβp0) = γ(E0minusβE0) =1minus βradic1minus β2

E0 =1minus βradic

(1 + β)(1minus β)E0 =

radic1minus βradic1 + β

E0

dove abbiamo usato il fatto che i fotoni hanno massa nulla Perciograve

E

E0=

hcλverdehc

λrosso

=λrosso

λverde=

radic1minus βradic1 + β

rarr v = βc = 025c

Esercizio 29 Conseguenze della relativitagraveI neutrini sono particelle di massa molto piccola e al momento ignota Uno

dei modi con cui egrave stato possibile dedurre un limite superiore al suo valore egravestata lrsquoosservazione nel 1987 di neutrini prodotti dalla supernova 1987A1 chesi trova a 168000 anni luce dalla Terra Sono stati osservati due segnali dineutrini che possono essere schematizzati come segue si egrave osservato prima unneutrino di 35 MeV di energia seguito a 9 s di distanza da un secondo segnaledi 13 MeV Si assuma che questo ritardo sia dovuto al fatto che la massa delneutrino non egrave nulla e si calcoli questrsquoultima

Esercizio 30 DecadimentoAnimali e piante assumono dallrsquoatmosfera diversi composti contenenti car-

bonio Il carbonio presente nellrsquoatmosfera egrave predominantemente 126 C ma sono

presenti piccole concentrazioni del suo isotopo 146 C (un atomo ogni 1012) che

decade con emissione di elettroni attraverso il processo2

146 C rarr14

7 N + eminus + νe

con un tempo di dimezzamento di 5700 anni1httpsenwikipediaorgwikiSN_1987ANeutrino_emissions2Di altro non si tratta che del decadimento βminus nrarr p+ e+ νe

16

1 Qual egrave la concentrazione di 146 C dopo 11400 anni

2 Animali e piante assumono in vita proporzioni fisse di 146 C e 12

6 C mentrealla loro morte la quantitagrave di 14

6 C inizia a diminuire Avete a portata dimano un relitto di legno per cui misurate una emissione di elettroni daldecadimento di 14

6 C pari al 61 di quella di un pezzo di legno vivodella stessa massa quanti anni ha il manufatto

Esercizio 31 Energia cinetica e trasformazioni di LorentzDue particelle identiche di massa m ed energia cinetica T collidono frontal-

mente Qual egrave la loro energia cinetica relativa (ossia lrsquoenergia cinetica di unaparticella misurata nel sistema di riferimento dellrsquoaltra particella)

Esercizio 32 Energia nel centro di massaLa reazione

πminusprarr K0Λ0

avviene con unrsquoenergia nel centro di massa diradics = 3 GeV La massa del πminus egrave

di 1396 MeVc2 la massa del protone egrave di 938 MeVc2 la massa del Kminus egrave di498 MeVc2 e quella della Λ0 di 11 GeVc2

1 Calcolare lrsquoimpulso di πminus e Λ0 nel sistema di riferimento del centro dimassa

2 Se il protone egrave a riposo il K puograve essere emesso allrsquoindietro nel sistemadi riferimento del laboratorio

Esercizio 33 DecadimentoIl mesone φ0 egrave una particella neutra3 di circa 1 GeVc2 di massa che puograve

decadere in una coppia di particelle

φ0 rarr K+ +Kminus

di massa identicamK = 494 MeVc2 Si assuma di produrre φ0 di impulso notoegrave possibile che uno dei due K sia prodotto a riposo nel sistema di riferimentodel laboratorio

Esercizio 34 DecadimentoUn fascio di anti-neutrini muonici νmicro si puograve generare selezionando pioni

o kaoni π+ e K+ e facendoli passare in un lungo tubo in cui egrave stato fattoil vuoto4 in modo che dopo un certo tragitto L una buona parte di loro saragrave

3Il collisore DAFNE ai Laboratori Nazionali di Frascati produce specificatamente parti-celle di questo tipo tramite il processo e+ + eminus rarr φ0 httpswwwyoutubecomwatchv=L5yB9gDGKms

4Una tecnica di questo tipo egrave stata usata per inviare ai Laboratori Nazionali del Gran Sassodei fasci di neutrini prodotti al CERN di Ginevra httpsvideoscernchrecord985892

17

decaduta tramite i processi

π+ rarr microminus + νmicro

K+ rarr microminus + νmicro

Se lrsquoimpulso di pioni e kaoni egrave di 200 GeVc e la loro vita media di 26 ns e12 ns rispettivamente

1 Quanto a lungo viaggiano nel laboratorio i due tipi di particelle2 Se L = 1000 m quale saragrave la frazione di pioni e kaoni che saragrave decaduta

alla fine del tubo3 Qual egrave lrsquoenergia massima dei neutrini che egrave possibile misurare nel sistema

di riferimento del laboratorio nei due casi

5 Lezione 5 (16 aprile 2020)

Esercizio 35 Energia cineticaTra le eccellenze con sede in territorio elvetico primeggiano indiscutibilmente lacioccolata al latte e il Large Hadron Collider (LHC) Questrsquoultimo egrave un collisoredi particelle situato al CERN di Ginevra che fa scontrare due fasci identici diprotoni di impulso uguale in modulo e direzione ma verso opposto I fasci sonocomposti da circa 2800 gruppi (pacchetti) di 1011 particelle ciascuno Sapendoche lrsquoenergia nel centro di massa della collisione fra due protoni egrave

radics = 13 TeV

quanta cioccolata al latte dovete mangiare per assumere un numero di caloriepari allrsquoenergia cinetica di ciascun fascio di protoni di LHC

Soluzione dellrsquoesercizio 35Poicheacute si tratta di un collider e le energie in gioco sono molto maggiori della

massa del protone lrsquoenergia di ciascun protone egrave diradics2 = 65 TeV e quella

totale di un fascio egrave

2800times 1011 times 65 middot 1012 times 16 middot 10minus19J asymp 300 MJ

Secondo Google 100 g di cioccolata al latte apportano circa 500 cal = 500 times4184 J asymp 2 MJ Lrsquoenergia di un fascio di LHC corrisponde quindi a quella dicirca 15 kg di cioccolata

Esercizio 36 Scattering RutherfordUn fascio di particelle α di 100 MeV di energia e 032 nA di corrente5 collide

contro un bersaglio fisso di alluminio spesso 1 cm Una sperimentatrice prendeun rivelatore di 1 cm times 1 cm di superficie e lo posiziona ad un angolo di 30

5Per una spiegazione breve su come (e percheacute) si misura la corrente di un fascio di particel-le vedi httpswwwlhc-closerestaking_a_closer_look_at_lhc0beam_current Unatrattazione piugrave completa egrave data ad esempio da httpscdscernchrecord1213275filesp141pdf

18

rispetto al fascio di particelle a 1 m di distanza dal bersaglio Quante particelleα incideranno sul rivelatore ogni secondo

Soluzione dellrsquoesercizio 36Lrsquoalluminio ha una densitagrave di 27 gcm3 numero atomico 13 e massa atomica

27 uPoicheacute le particelle α sono nuclei di elio hanno carica 2e e la corrente di

032 nA corrisponde a un miliardo di particelle incidenti al secondo

dNidt

=032 nCs

2times 16times 10minus19 C= 1times 109 sminus1

Il rivelatore vede un angolo solido di

∆Ω equiv superficieraggio

2

=1 cm2

(1 m)2= 1times 10minus4 sr

Si tratta di uno scattering alla Rutherford per cui la sezione drsquourto per unitagravedi angolo solido rilevata ad un certo angolo θ vale

dΩ=

(zαzAle

2

4πε04E

1

sin2(θ2)

)2

pari a

dΩasymp(

2times 13times 4times etimes 16times 10minus19 C

4π times 89times 10minus12 Fmtimes 4times 100times 106 eV

1

sin2(π180 times 302)

)2

asymp 2times 10minus30 m2sr = 20 mbsr

(1)

e il numero di particelle visto dal rivelatore vale se indichiamo con nAl = ρAlNAAAl

la densitagrave numero di atomi di alluminio e con d lo spessore del rivelatore

dNrivelate

dt= ∆Ω

dΩnAld

dNi

dt

asymp 1times 10minus4 srtimes 2times 10minus30 m2srtimes 1times 104 cm2m2 times 27 gcm3 6times 1023 molminus1

27 gmol

= 120 Hz

Esercizio 37 Sezione drsquourtoUn bersaglio di idrogeno liquido di densitagrave ρ = 0071 gcm3 e volume V =

125 cm3 egrave bombardato da un fascio monoenergetico di pioni negativi con unflusso φ = 2times 107 mminus2sminus1 e si osserva la reazione

πminus + prarr π0 + n

rivelando i fotoni del decadimento π0 rarr γγ che avviene nel 988 dei casi Sela sezione drsquourto di quellrsquointerazione egrave σ = 40 mb quanti fotoni sono emessi ognisecondo

19

Esercizio 38 Sezione drsquourtoLa sezione drsquourto dellrsquoeffetto fotoelettrico per raggi X di 10 keV in carbonio

egrave 40 b per atomo Data una lastra di carbonio di 4 mm di spessore si calcoli1 il numero di bersagli per unitagrave di volume2 il coefficiente di assorbimento per effetto fotoelettrico dei raggi X di tale

energia3 la probabilitagrave che un raggio X incidente sulla lastra produca un elettrone

per effetto fotoelettrico

Esercizio 39 Sezioni drsquourtoUn bersaglio drsquooro di densitagrave superficiale ρS = 097 mgcm2 e superficie

SB = 1 cm2 viene colpito da un fascio di particelle α la cui sezione trasversaegrave contenuta completamente nellrsquoarea del bersaglio Sul bersaglio impattano37times 104 αs La sezione drsquourto di diffusione elastica ad un certo angolo θ valedσ

ddΩ = 1 bsr Calcolare1 la densitagrave di atomi del bersaglio per unitagrave di superficie2 il numero di particelle α rivelate in unrsquoora da un rivelatore di superficie

SR = 2 cm2 posto allrsquoangolo θ e a distanza DR = 01 m dal bersaglio3 Se il fascio di particelle viene sostituito da una sorgente radioattiva di

particelle α con distribuzione isotropa su tutto lrsquoangolo solido che vieneposta lungo la stessa linea del fascio a distanza DB = 20 cm dal bersaglioquanto tempo egrave necessario per rivelare con lo stesso rivelatore lo stessonumero di particelle calcolato sopra a paritagrave di sezione drsquourto

Esercizio 40 CinematicaUn fascio di positroni di 100 MeV di energia annichila su una targhetta fissa

di elettroni producendo due fotoni tramite il processo

e+ + eminus rarr γ + γ

Se uno dei due fotoni egrave emesso a 30 nel sistema del centro di massa1 quanto vale lrsquoenergia dei due fotoni nel sistema del centro di massa2 che energia e direzione avranno nel sistema del laboratorio

Esercizio 41 Sezione drsquourtoGli antineutrini νe prodotti da un reattore nucleare con potenza P = 16 GW

attraversano un bersaglio da 2000 l drsquoacqua posto a 50 m di distanza dal reat-tore

1 Supponendo che per ogni fissione sia prodotta unrsquoenergia termica di200 MeV e vengano emessi 6 antineutrini calcolare il numero medio direazioni

νe + prarr e+ + n

prodotte al giorno se la sezione drsquourto egrave σ = 1times 10minus43 cm2

20

2 Qual egrave la minima energia del neutrino per cui la reazione egrave permessa

6 Lezione 7 (8 maggio 2020)

Esercizio 42Un protone interagisce con un bersaglio producendo un pione di energia E =300 MeV Con un tracciatore posto a distanza d = 6 cm dal bersaglio egrave possi-bile rivelare la traiettoria del pione e risalire al punto di produzione del pionesul bersaglio Determinare lrsquoerrore sulla misura della posizione di tale puntocausato dalla presenza di un piano di alluminio di spessore L = 05 mm postoimmediatamente davanti al tracciatore (quindi a distanza d dal bersaglio) nellrsquoi-potesi che gli angoli delle tracce rispetto alla normale alle superfici del bersaglioe del piano di alluminio siano piccoli [mπ = 1396 MeV Al (Z = 13 A = 27ρ = 27 gcm3)]

Soluzione dellrsquoesercizio 42La deviazione standard dellrsquoangolo di diffusione coulombiana multipla vale

radic〈θ2〉 = 21 MeV

z

βc|p|

radicx

X0= 21 MeV

z

βc|p|

radicLρ

X0

dove p =radicE2 minusm2

π = 266 MeV β = pE = 0885 z = 1 Per lrsquoalluminioabbiamo

X0 = 24 gcm2 rarr X0ρ = 89 cm

per cui radic〈θ2〉 asymp 00067

e lo spostamento medio corrispondente rispetto alla posizione nominale egrave

δs = tan(00067)times 6 cm = 04 mm

Esercizio 43Un fascio misto di elettroni e antiprotoni passa attraverso una regione con

B = 2 T e dopo aver curvato per R = 3 m raggiunge una regione senza campomagnetico

1 Qual egrave lrsquoimpulso delle particelle selezionate2 Come discriminereste fra i due tipi di particelle3 Se usaste due scintillatori posti a 15 m di distanza quale risoluzione

temporale sarebbe necessaria per discriminare i due tipi di particelle4 Se i due scintillatori fossero spessi 2 cm e avessero una lunghezza di ra-

diazione X0 = 40 cm quanta energia perderebbero elettroni e protoninellrsquoattraversarli Si assuma una perdita di energia per ionizzazione di2 MeVcm e 25 MeVcm per protoni ed elettroni rispettivamente

21

5 Che indice di rifrazione dovrebbe avere un rivelatore a luce Cherenkovper discriminare elettroni e protoni

Soluzione dellrsquoesercizio 43Dalla definizione di forza

F =dpdt

= qvtimesB

|F| = mv2

R

per cui se il campo magnetico egrave ortogonale alla traiettoria

pc = qcBR

che poicheacute c = 03 mns possiamo esprimere in maniera piugrave conveniente come

pc[GeV] = 03B[T]R[m]

Ne segue che nel nostro esercizio (e tornando a c = 1)

p = 18 GeV

Trascurando le perdite di energia e ricordando che me mp i tempi divolo di elettroni e protoni valgono rispettivamente

te =L

βecasymp L

casymp 50 ns

etp =

L

βpc=

Lpradic

p2+m2p

casymp 56 ns

per cui serve una risoluzione dellrsquoordine del ns per discriminare elettroni e muonidal tempo di volo

Se gli scintillatori sono spessi 2 cm i protoni perderanno in ciascuno di essi2 cm times 2 MeVcm = 4 MeV per ionizzazione Gli elettroni perderanno invecenel primo rivelatore 5 MeV per ionizzazione e (me asymp 0)

18 GeV times(

1minus exp

(minus 2 cm

40 cm

))asymp 88 MeV

cioegrave dopo il primo rivelatore avranno 18 GeVminus93 MeV di energia e nel secondone perderanno altri

5 MeV + (18 GeV minus 93 MeV)

(1minus exp

(minus 2 cm

40 cm

))asymp 88 MeV

Lrsquoangolo di emissione di luce Cherenkov egrave dato da

cos θc =1

βnle 1

22

e poicheacute βe asymp 1 βp asymp 0887 per avere luce Cherenkov solo al passaggio deglielettroni (e non dei protoni) egrave necessario che

1 lt n lt 113

Esercizio 44Come misurereste la massa di una particella carica E quella di una parti-

cella neutra

Esercizio 45Un muone di energia E = 400 GeV penetra verticalmente nel mare Attra-

verso quale processo puograve essere rivelato A quale profonditagrave arriva prima didecadere

Esercizio 46Avete a disposizione dei tubi fotomoltiplicatori sensibili a lunghezze drsquoonda

fra 300 nm e 500 nm e volete rivelare la luce Cherenkov prodotta dal passaggio diun elettrone di 1 MeV di energia in un metro drsquoacqua Quanti fotoni vi aspettateche vengano prodotti Confrontate con il numero di elettroni di ionizzazioneche produrrebbe una particella α di 5 keV nello stesso rivelatore

Esercizio 47Determinare quali sono i processi piugrave probabili (cioegrave quelli di sezione drsquourto

piugrave alta) nellrsquointerazione fra1 fotoni di 1 MeV e atomi di alluminio2 fotoni di 100 keV e H23 fotoni di 100 keV e atomi di ferro4 fotoni di 10 MeV e atomi di carbonio5 fotoni di 10 MeV e atomi di piombo

Esercizio 48Volete misurare lrsquoimpulso di una particella carica che attraversa un campo

magnetico B ortogonale alla sua traiettoria Avete a disposizione tre rivelatori diposizione della stessa risoluzione spaziale δx come li disponete Che risoluzionein impulso vi aspettate di ottenere

23

  • Lezione 1 (13 marzo 2020)
  • Lezione 2 (20 marzo 2020)
  • Lezione 3 (27 marzo 2020)
  • Lezione 4 (3 aprile 2020)
  • Lezione 5 (16 aprile 2020)
  • Lezione 7 (8 maggio 2020)
Page 12: Esercizi per casa (risolti)ippolitv/pdf/fns1_2020_solutions.pdfEsercizi per casa (risolti) Valerio Ippolito 13 maggio 2020 1 Lezione 1 (13 marzo 2020) Esercizio1 Dilatazionedeitempi

Soluzione dellrsquoesercizio 16Se indichiamo con lrsquoapice la velocitagrave nel nostro sistema di riferimento che

egrave in moto rispetto al sistema di riferimento del secondo oggetto la legge ditrasformazione delle velocitagrave ci dice che

vprime1 =v1 minus V

1minus v1cVc

=αcminus V1minus αVc

vprime2 =v2 minus V

1minus v2cVc

= minusV

Stiamo cercando V la nostra velocitagrave nel sistema di riferimento del secondooggetto tale che le velocitagrave dei due oggetti nel nostro sistema di riferimentosiano uguali e opposte

vprime1 = minusvprime2 = V

perciograve la richiesta egrave

αcminus V1minus αVc

= V

αcminus V minus V + αV 2

c= 0

(αc)V 2 + (minus2)V + (α)c = 0

V =1minusradic

1minus α2

αc

dove abbiamo scelto la soluzione dellrsquoequazione di secondo grado con V le c

Esercizio 17 Conseguenze della relativitagraveUna navicella spaziale in moto rettilineo uniforme con velocitagrave 05c in al-

lontanamento dalla Terra egrave in orbita verso Plutone che si trova a 75times 109 kmdi distanza dalla Terra Non appena raggiunto il pianeta la comandante inviaun segnale radio alla base a Houston per chiedere lrsquoautorizzazione allrsquoatter-raggio Quanto tempo impiega la richiesta a raggiungere la base secondo lacomandante E secondo i suoi colleghi a Houston

Soluzione dellrsquoesercizio 17Lrsquoonda radio viaggia a velocitagrave c secondo ogni sistema di riferimento Per

lrsquoosservatore lrsquoonda radio percorreragrave il tragitto Plutone-Terra in un tempo

tTerra =L

c=

75times 109 km

3times 108 msasymp 25 000 s

mentre per la comandante saragrave passato un tempo inferiore che dipende dal fat-tore γ dellrsquoastronave nel sistema di riferimento della Terra secondo la relazione

tcomandante =tTerra

γasymp 25 000 stimes

radic1minus 052 asymp 21 650 s

12

Esercizio 18 Energia cineticaHa piugrave energia un protone che viaggia a 09999999896c o un Boeing 747 al

decollo

Esercizio 19 Dilatazione dei tempiUna scienziata misura che un fascio di particelle selezionate con impulso di

10 GeVc si degrada dellrsquo84 dopo aver percorso 1 m Se la massa di questeparticelle egrave 498 MeVc2 qual egrave la loro vita media

Soluzione dellrsquoesercizio 19La distanza misurata dalla scienziata egrave chiaramente riferita al suo sistema

di riferimento quello del laboratorio ed egrave legata alla vita media delle particelledel fascio τ dalla relazione

Ldecay = 1 m = βγcτ =pc

E

E

mc2cτ =

p

mccτ

e dalla legge del decadimento se indichiamo con N0 il numero di particelleinizialmente presenti nel fascio e con N il numero di particelle misurato si haN

N0= 1minus084 = exp

(minus L

Ldecay

)= exp

(minus L

pmccτ

)rarr τ =

mcL

pc log(N0

N

) asymp 9times 10minus11 s

Esercizio 20 Dilatazione dei tempiVi trovate a dover studiare un fascio di particelle di cui conoscete lrsquoenergia

ndash 2 GeV ndash ma non la massa a quanto ne sapete potrebbero essere composti daelettroni (di massa 511 keVc2) o protoni (938 MeVc2) Avete a disposizionedue rivelatori identici in grado di registrare con precisione il tempo in cui unaparticella li attraversa Come potete utilizzarli per determinare se il vostrofascio contiene elettroni o protoni

Soluzione dellrsquoesercizio 20Disponendo i due rivelatori lungo la direzione del fascio a distanza ∆L

lrsquouno dallrsquoaltro si puograve misurare il tempo impiegato dalle particelle per passaredallrsquouno allrsquoaltro e quindi la loro massa infatti se ∆t egrave la distanza temporalefra i segnali dei due rivelatori

∆L = v∆t = βc∆t =pc2

E∆t

Questo principio egrave usato nei cosiddetti rivelatori di time of flight che sono usatiper discriminare diversi tipi di particelle una particella di tipo 1 e una di tipo2 infatti percorreranno la distanza ∆L in tempi diversi legati alle rispettivemasse dalla relazione

∆L1 = ∆L2

p1c2

E1∆t1 =

radicE2 minusm2

1c2c

E∆t1 =

radicE2 minusm2

2c2c

E∆t2

13

Esercizio 21 Energia di sogliaDue fisici delle particelle vogliono produrre il bosone Z una particella di

carica neutra e di massamZ = 91 GeVc2 e discutono come fare Alice proponedi far scontrare fasci di elettroni e positroni di energia identica che viaggianodunque con impulso spaziale uguale in modulo e direzione ma di verso oppostoproducendo Z tramite il processo

e+ + eminus rarr Z

mentre Bob preferisce scontrare un fascio di protoni su un bersaglio fisso diidrogeno tramite il processo

p+ prarr Z + p+ p

Chi dei due avragrave bisogno di fasci di particelle di energia piugrave alta La massa delprotone egrave di 9383 MeVc2 quella dellrsquoelettrone di 511 keVc2

Esercizio 22 Leggi di conservazioneUn fotone (particella di massa nulla) puograve decadere in un elettrone e in un

positrone (entrambi di massa 511 keVc2) tramite il processo

γ rarr e+ + eminus

Esercizio 23 Decadimento βminus

Quali sono lrsquoenergia minima e massima dellrsquoelettrone nel decadimento

nrarr p+ eminus + νe

se il neutrone decade da fermo La massa del neutrone egrave di 9396 MeVc2 quelladel protone di 9383 MeVc2 e quella dellrsquoelettrone di 511 keVc2 si assuma chelrsquoantineutrino elettronico νe abbia massa nulla

Esercizio 24 Leggi di conservazioneIl decadimento

prarr n+ e+ + νe

egrave permesso

4 Lezione 4 (3 aprile 2020)

Esercizio 25Usando il fatto che hc = 1973 MeVfm si dimostri che in un sistema di unitagrave dimisura in cui h = c = 1 vale

14

1 1 GeVminus2 = 0389 mb

2 1 m = 5068times 1015 GeVminus1

3 1 s = 15times 1024 GeVminus1

Ricordiamo che 1 b = 1times 10minus28 m2 e che

[hc] = [Jsms] = [E][L]

Soluzione dellrsquoesercizio 25Lrsquoidea egrave di capire per quale potenza di hc e c va moltiplicato il termine a

sinistra di ciascuna equazione per ottenere il termine di destraPer cui

bull [1 GeVminus2][hc]α = [E]minus2[E]α[L]α = [0389 mb] = [L]2 da cui segue α = 2

e 1 GeVminus2(hc)2 = 1973 MeVfm1 GeV =

(01973times 10minus15 GeVm

1 GeV

)2

= 0389 mb

bull [1 m][hc]α = [L][E]α[L]α = [5068times 1015 GeVminus1] = [E]minus1 da cui segueα = minus1 e 1 m(hc)minus1 = 1 m

1973 MeVfm = 1 m01973times 10minus15 GeVm

= 5068times 1015 GeVminus1

bull [1 s][hc]α[c]β = [T ][E]α[L]α[L]β [T ]minusβ = [T ][E]α[L]α+β [T ]minusβ = [15times 1024 GeVminus1] =[E]minus1 da cui segue α = minus1 β = 1 e 1 s(hc)minus1c = 1 s

1973 MeVfm299 792 458 ms =299 792 458 m

01973times 10minus15 GeVm= 15times 1024 GeVminus1

Esercizio 26 Massa invarianteTre protoni (mp = 938 MeVc2) hanno impulsi uguali in modulo (p =

3 GeVc) e che formano angoli di 120 lrsquouno con lrsquoaltro Qual egrave la massainvariante del sistema

Esercizio 27 Energia di sogliaSi consideri il processo

γ + prarr p+ π0

dove il fotone ha massa nulla il protone ha massa di 938 MeVc2 e il π0 hamassa di 135 MeVc2

1 Se il protone egrave a riposo qual egrave lrsquoenergia minima che deve avere il fotoneincidente percheacute la reazione abbia luogo

2 La stessa reazione puograve avvenire nellrsquouniverso in cui un protone dei raggicosmici di alta energia puograve collidere con uno dei fotoni della radiazionecosmica di fondo di energia dellrsquoordine di 1 meV Qual egrave in questo casolrsquoenergia minima che deve avere il protone percheacute la reazione abbia luogo

Esercizio 28 Conseguenze della relativitagraveUn misterioso supereroe pattuglia a velocitagrave molto elevata la periferia roma-

na Allrsquoincrocio con via di Tor Bella Monaca incontra un semaforo e ndash vedendolo

15

verde ndash attraversa senza rallentare Una pattuglia della polizia municipale loferma e lo multa per esser passato col rosso Assumendo sia il supereroe che ivigili siano nel giusto a che velocitagrave viaggiava il supereroe

Soluzione dellrsquoesercizio 28Per la polizia municipale il semaforo emette fotoni di energia

E0 = hν0 =hc

λrossoasymp hc

630 nm

mentre il supereoe vede fotoni di energia

E = hν =hc

λverdeasymp hc

490 nm

e dalle trasformazioni di Lorentz indicando con γ e β le variabili calcolateusando la velocitagrave del supereroe misurata dalla municipale possiamo scrivere

E = γ(E0minusβp0) = γ(E0minusβE0) =1minus βradic1minus β2

E0 =1minus βradic

(1 + β)(1minus β)E0 =

radic1minus βradic1 + β

E0

dove abbiamo usato il fatto che i fotoni hanno massa nulla Perciograve

E

E0=

hcλverdehc

λrosso

=λrosso

λverde=

radic1minus βradic1 + β

rarr v = βc = 025c

Esercizio 29 Conseguenze della relativitagraveI neutrini sono particelle di massa molto piccola e al momento ignota Uno

dei modi con cui egrave stato possibile dedurre un limite superiore al suo valore egravestata lrsquoosservazione nel 1987 di neutrini prodotti dalla supernova 1987A1 chesi trova a 168000 anni luce dalla Terra Sono stati osservati due segnali dineutrini che possono essere schematizzati come segue si egrave osservato prima unneutrino di 35 MeV di energia seguito a 9 s di distanza da un secondo segnaledi 13 MeV Si assuma che questo ritardo sia dovuto al fatto che la massa delneutrino non egrave nulla e si calcoli questrsquoultima

Esercizio 30 DecadimentoAnimali e piante assumono dallrsquoatmosfera diversi composti contenenti car-

bonio Il carbonio presente nellrsquoatmosfera egrave predominantemente 126 C ma sono

presenti piccole concentrazioni del suo isotopo 146 C (un atomo ogni 1012) che

decade con emissione di elettroni attraverso il processo2

146 C rarr14

7 N + eminus + νe

con un tempo di dimezzamento di 5700 anni1httpsenwikipediaorgwikiSN_1987ANeutrino_emissions2Di altro non si tratta che del decadimento βminus nrarr p+ e+ νe

16

1 Qual egrave la concentrazione di 146 C dopo 11400 anni

2 Animali e piante assumono in vita proporzioni fisse di 146 C e 12

6 C mentrealla loro morte la quantitagrave di 14

6 C inizia a diminuire Avete a portata dimano un relitto di legno per cui misurate una emissione di elettroni daldecadimento di 14

6 C pari al 61 di quella di un pezzo di legno vivodella stessa massa quanti anni ha il manufatto

Esercizio 31 Energia cinetica e trasformazioni di LorentzDue particelle identiche di massa m ed energia cinetica T collidono frontal-

mente Qual egrave la loro energia cinetica relativa (ossia lrsquoenergia cinetica di unaparticella misurata nel sistema di riferimento dellrsquoaltra particella)

Esercizio 32 Energia nel centro di massaLa reazione

πminusprarr K0Λ0

avviene con unrsquoenergia nel centro di massa diradics = 3 GeV La massa del πminus egrave

di 1396 MeVc2 la massa del protone egrave di 938 MeVc2 la massa del Kminus egrave di498 MeVc2 e quella della Λ0 di 11 GeVc2

1 Calcolare lrsquoimpulso di πminus e Λ0 nel sistema di riferimento del centro dimassa

2 Se il protone egrave a riposo il K puograve essere emesso allrsquoindietro nel sistemadi riferimento del laboratorio

Esercizio 33 DecadimentoIl mesone φ0 egrave una particella neutra3 di circa 1 GeVc2 di massa che puograve

decadere in una coppia di particelle

φ0 rarr K+ +Kminus

di massa identicamK = 494 MeVc2 Si assuma di produrre φ0 di impulso notoegrave possibile che uno dei due K sia prodotto a riposo nel sistema di riferimentodel laboratorio

Esercizio 34 DecadimentoUn fascio di anti-neutrini muonici νmicro si puograve generare selezionando pioni

o kaoni π+ e K+ e facendoli passare in un lungo tubo in cui egrave stato fattoil vuoto4 in modo che dopo un certo tragitto L una buona parte di loro saragrave

3Il collisore DAFNE ai Laboratori Nazionali di Frascati produce specificatamente parti-celle di questo tipo tramite il processo e+ + eminus rarr φ0 httpswwwyoutubecomwatchv=L5yB9gDGKms

4Una tecnica di questo tipo egrave stata usata per inviare ai Laboratori Nazionali del Gran Sassodei fasci di neutrini prodotti al CERN di Ginevra httpsvideoscernchrecord985892

17

decaduta tramite i processi

π+ rarr microminus + νmicro

K+ rarr microminus + νmicro

Se lrsquoimpulso di pioni e kaoni egrave di 200 GeVc e la loro vita media di 26 ns e12 ns rispettivamente

1 Quanto a lungo viaggiano nel laboratorio i due tipi di particelle2 Se L = 1000 m quale saragrave la frazione di pioni e kaoni che saragrave decaduta

alla fine del tubo3 Qual egrave lrsquoenergia massima dei neutrini che egrave possibile misurare nel sistema

di riferimento del laboratorio nei due casi

5 Lezione 5 (16 aprile 2020)

Esercizio 35 Energia cineticaTra le eccellenze con sede in territorio elvetico primeggiano indiscutibilmente lacioccolata al latte e il Large Hadron Collider (LHC) Questrsquoultimo egrave un collisoredi particelle situato al CERN di Ginevra che fa scontrare due fasci identici diprotoni di impulso uguale in modulo e direzione ma verso opposto I fasci sonocomposti da circa 2800 gruppi (pacchetti) di 1011 particelle ciascuno Sapendoche lrsquoenergia nel centro di massa della collisione fra due protoni egrave

radics = 13 TeV

quanta cioccolata al latte dovete mangiare per assumere un numero di caloriepari allrsquoenergia cinetica di ciascun fascio di protoni di LHC

Soluzione dellrsquoesercizio 35Poicheacute si tratta di un collider e le energie in gioco sono molto maggiori della

massa del protone lrsquoenergia di ciascun protone egrave diradics2 = 65 TeV e quella

totale di un fascio egrave

2800times 1011 times 65 middot 1012 times 16 middot 10minus19J asymp 300 MJ

Secondo Google 100 g di cioccolata al latte apportano circa 500 cal = 500 times4184 J asymp 2 MJ Lrsquoenergia di un fascio di LHC corrisponde quindi a quella dicirca 15 kg di cioccolata

Esercizio 36 Scattering RutherfordUn fascio di particelle α di 100 MeV di energia e 032 nA di corrente5 collide

contro un bersaglio fisso di alluminio spesso 1 cm Una sperimentatrice prendeun rivelatore di 1 cm times 1 cm di superficie e lo posiziona ad un angolo di 30

5Per una spiegazione breve su come (e percheacute) si misura la corrente di un fascio di particel-le vedi httpswwwlhc-closerestaking_a_closer_look_at_lhc0beam_current Unatrattazione piugrave completa egrave data ad esempio da httpscdscernchrecord1213275filesp141pdf

18

rispetto al fascio di particelle a 1 m di distanza dal bersaglio Quante particelleα incideranno sul rivelatore ogni secondo

Soluzione dellrsquoesercizio 36Lrsquoalluminio ha una densitagrave di 27 gcm3 numero atomico 13 e massa atomica

27 uPoicheacute le particelle α sono nuclei di elio hanno carica 2e e la corrente di

032 nA corrisponde a un miliardo di particelle incidenti al secondo

dNidt

=032 nCs

2times 16times 10minus19 C= 1times 109 sminus1

Il rivelatore vede un angolo solido di

∆Ω equiv superficieraggio

2

=1 cm2

(1 m)2= 1times 10minus4 sr

Si tratta di uno scattering alla Rutherford per cui la sezione drsquourto per unitagravedi angolo solido rilevata ad un certo angolo θ vale

dΩ=

(zαzAle

2

4πε04E

1

sin2(θ2)

)2

pari a

dΩasymp(

2times 13times 4times etimes 16times 10minus19 C

4π times 89times 10minus12 Fmtimes 4times 100times 106 eV

1

sin2(π180 times 302)

)2

asymp 2times 10minus30 m2sr = 20 mbsr

(1)

e il numero di particelle visto dal rivelatore vale se indichiamo con nAl = ρAlNAAAl

la densitagrave numero di atomi di alluminio e con d lo spessore del rivelatore

dNrivelate

dt= ∆Ω

dΩnAld

dNi

dt

asymp 1times 10minus4 srtimes 2times 10minus30 m2srtimes 1times 104 cm2m2 times 27 gcm3 6times 1023 molminus1

27 gmol

= 120 Hz

Esercizio 37 Sezione drsquourtoUn bersaglio di idrogeno liquido di densitagrave ρ = 0071 gcm3 e volume V =

125 cm3 egrave bombardato da un fascio monoenergetico di pioni negativi con unflusso φ = 2times 107 mminus2sminus1 e si osserva la reazione

πminus + prarr π0 + n

rivelando i fotoni del decadimento π0 rarr γγ che avviene nel 988 dei casi Sela sezione drsquourto di quellrsquointerazione egrave σ = 40 mb quanti fotoni sono emessi ognisecondo

19

Esercizio 38 Sezione drsquourtoLa sezione drsquourto dellrsquoeffetto fotoelettrico per raggi X di 10 keV in carbonio

egrave 40 b per atomo Data una lastra di carbonio di 4 mm di spessore si calcoli1 il numero di bersagli per unitagrave di volume2 il coefficiente di assorbimento per effetto fotoelettrico dei raggi X di tale

energia3 la probabilitagrave che un raggio X incidente sulla lastra produca un elettrone

per effetto fotoelettrico

Esercizio 39 Sezioni drsquourtoUn bersaglio drsquooro di densitagrave superficiale ρS = 097 mgcm2 e superficie

SB = 1 cm2 viene colpito da un fascio di particelle α la cui sezione trasversaegrave contenuta completamente nellrsquoarea del bersaglio Sul bersaglio impattano37times 104 αs La sezione drsquourto di diffusione elastica ad un certo angolo θ valedσ

ddΩ = 1 bsr Calcolare1 la densitagrave di atomi del bersaglio per unitagrave di superficie2 il numero di particelle α rivelate in unrsquoora da un rivelatore di superficie

SR = 2 cm2 posto allrsquoangolo θ e a distanza DR = 01 m dal bersaglio3 Se il fascio di particelle viene sostituito da una sorgente radioattiva di

particelle α con distribuzione isotropa su tutto lrsquoangolo solido che vieneposta lungo la stessa linea del fascio a distanza DB = 20 cm dal bersaglioquanto tempo egrave necessario per rivelare con lo stesso rivelatore lo stessonumero di particelle calcolato sopra a paritagrave di sezione drsquourto

Esercizio 40 CinematicaUn fascio di positroni di 100 MeV di energia annichila su una targhetta fissa

di elettroni producendo due fotoni tramite il processo

e+ + eminus rarr γ + γ

Se uno dei due fotoni egrave emesso a 30 nel sistema del centro di massa1 quanto vale lrsquoenergia dei due fotoni nel sistema del centro di massa2 che energia e direzione avranno nel sistema del laboratorio

Esercizio 41 Sezione drsquourtoGli antineutrini νe prodotti da un reattore nucleare con potenza P = 16 GW

attraversano un bersaglio da 2000 l drsquoacqua posto a 50 m di distanza dal reat-tore

1 Supponendo che per ogni fissione sia prodotta unrsquoenergia termica di200 MeV e vengano emessi 6 antineutrini calcolare il numero medio direazioni

νe + prarr e+ + n

prodotte al giorno se la sezione drsquourto egrave σ = 1times 10minus43 cm2

20

2 Qual egrave la minima energia del neutrino per cui la reazione egrave permessa

6 Lezione 7 (8 maggio 2020)

Esercizio 42Un protone interagisce con un bersaglio producendo un pione di energia E =300 MeV Con un tracciatore posto a distanza d = 6 cm dal bersaglio egrave possi-bile rivelare la traiettoria del pione e risalire al punto di produzione del pionesul bersaglio Determinare lrsquoerrore sulla misura della posizione di tale puntocausato dalla presenza di un piano di alluminio di spessore L = 05 mm postoimmediatamente davanti al tracciatore (quindi a distanza d dal bersaglio) nellrsquoi-potesi che gli angoli delle tracce rispetto alla normale alle superfici del bersaglioe del piano di alluminio siano piccoli [mπ = 1396 MeV Al (Z = 13 A = 27ρ = 27 gcm3)]

Soluzione dellrsquoesercizio 42La deviazione standard dellrsquoangolo di diffusione coulombiana multipla vale

radic〈θ2〉 = 21 MeV

z

βc|p|

radicx

X0= 21 MeV

z

βc|p|

radicLρ

X0

dove p =radicE2 minusm2

π = 266 MeV β = pE = 0885 z = 1 Per lrsquoalluminioabbiamo

X0 = 24 gcm2 rarr X0ρ = 89 cm

per cui radic〈θ2〉 asymp 00067

e lo spostamento medio corrispondente rispetto alla posizione nominale egrave

δs = tan(00067)times 6 cm = 04 mm

Esercizio 43Un fascio misto di elettroni e antiprotoni passa attraverso una regione con

B = 2 T e dopo aver curvato per R = 3 m raggiunge una regione senza campomagnetico

1 Qual egrave lrsquoimpulso delle particelle selezionate2 Come discriminereste fra i due tipi di particelle3 Se usaste due scintillatori posti a 15 m di distanza quale risoluzione

temporale sarebbe necessaria per discriminare i due tipi di particelle4 Se i due scintillatori fossero spessi 2 cm e avessero una lunghezza di ra-

diazione X0 = 40 cm quanta energia perderebbero elettroni e protoninellrsquoattraversarli Si assuma una perdita di energia per ionizzazione di2 MeVcm e 25 MeVcm per protoni ed elettroni rispettivamente

21

5 Che indice di rifrazione dovrebbe avere un rivelatore a luce Cherenkovper discriminare elettroni e protoni

Soluzione dellrsquoesercizio 43Dalla definizione di forza

F =dpdt

= qvtimesB

|F| = mv2

R

per cui se il campo magnetico egrave ortogonale alla traiettoria

pc = qcBR

che poicheacute c = 03 mns possiamo esprimere in maniera piugrave conveniente come

pc[GeV] = 03B[T]R[m]

Ne segue che nel nostro esercizio (e tornando a c = 1)

p = 18 GeV

Trascurando le perdite di energia e ricordando che me mp i tempi divolo di elettroni e protoni valgono rispettivamente

te =L

βecasymp L

casymp 50 ns

etp =

L

βpc=

Lpradic

p2+m2p

casymp 56 ns

per cui serve una risoluzione dellrsquoordine del ns per discriminare elettroni e muonidal tempo di volo

Se gli scintillatori sono spessi 2 cm i protoni perderanno in ciascuno di essi2 cm times 2 MeVcm = 4 MeV per ionizzazione Gli elettroni perderanno invecenel primo rivelatore 5 MeV per ionizzazione e (me asymp 0)

18 GeV times(

1minus exp

(minus 2 cm

40 cm

))asymp 88 MeV

cioegrave dopo il primo rivelatore avranno 18 GeVminus93 MeV di energia e nel secondone perderanno altri

5 MeV + (18 GeV minus 93 MeV)

(1minus exp

(minus 2 cm

40 cm

))asymp 88 MeV

Lrsquoangolo di emissione di luce Cherenkov egrave dato da

cos θc =1

βnle 1

22

e poicheacute βe asymp 1 βp asymp 0887 per avere luce Cherenkov solo al passaggio deglielettroni (e non dei protoni) egrave necessario che

1 lt n lt 113

Esercizio 44Come misurereste la massa di una particella carica E quella di una parti-

cella neutra

Esercizio 45Un muone di energia E = 400 GeV penetra verticalmente nel mare Attra-

verso quale processo puograve essere rivelato A quale profonditagrave arriva prima didecadere

Esercizio 46Avete a disposizione dei tubi fotomoltiplicatori sensibili a lunghezze drsquoonda

fra 300 nm e 500 nm e volete rivelare la luce Cherenkov prodotta dal passaggio diun elettrone di 1 MeV di energia in un metro drsquoacqua Quanti fotoni vi aspettateche vengano prodotti Confrontate con il numero di elettroni di ionizzazioneche produrrebbe una particella α di 5 keV nello stesso rivelatore

Esercizio 47Determinare quali sono i processi piugrave probabili (cioegrave quelli di sezione drsquourto

piugrave alta) nellrsquointerazione fra1 fotoni di 1 MeV e atomi di alluminio2 fotoni di 100 keV e H23 fotoni di 100 keV e atomi di ferro4 fotoni di 10 MeV e atomi di carbonio5 fotoni di 10 MeV e atomi di piombo

Esercizio 48Volete misurare lrsquoimpulso di una particella carica che attraversa un campo

magnetico B ortogonale alla sua traiettoria Avete a disposizione tre rivelatori diposizione della stessa risoluzione spaziale δx come li disponete Che risoluzionein impulso vi aspettate di ottenere

23

  • Lezione 1 (13 marzo 2020)
  • Lezione 2 (20 marzo 2020)
  • Lezione 3 (27 marzo 2020)
  • Lezione 4 (3 aprile 2020)
  • Lezione 5 (16 aprile 2020)
  • Lezione 7 (8 maggio 2020)
Page 13: Esercizi per casa (risolti)ippolitv/pdf/fns1_2020_solutions.pdfEsercizi per casa (risolti) Valerio Ippolito 13 maggio 2020 1 Lezione 1 (13 marzo 2020) Esercizio1 Dilatazionedeitempi

Esercizio 18 Energia cineticaHa piugrave energia un protone che viaggia a 09999999896c o un Boeing 747 al

decollo

Esercizio 19 Dilatazione dei tempiUna scienziata misura che un fascio di particelle selezionate con impulso di

10 GeVc si degrada dellrsquo84 dopo aver percorso 1 m Se la massa di questeparticelle egrave 498 MeVc2 qual egrave la loro vita media

Soluzione dellrsquoesercizio 19La distanza misurata dalla scienziata egrave chiaramente riferita al suo sistema

di riferimento quello del laboratorio ed egrave legata alla vita media delle particelledel fascio τ dalla relazione

Ldecay = 1 m = βγcτ =pc

E

E

mc2cτ =

p

mccτ

e dalla legge del decadimento se indichiamo con N0 il numero di particelleinizialmente presenti nel fascio e con N il numero di particelle misurato si haN

N0= 1minus084 = exp

(minus L

Ldecay

)= exp

(minus L

pmccτ

)rarr τ =

mcL

pc log(N0

N

) asymp 9times 10minus11 s

Esercizio 20 Dilatazione dei tempiVi trovate a dover studiare un fascio di particelle di cui conoscete lrsquoenergia

ndash 2 GeV ndash ma non la massa a quanto ne sapete potrebbero essere composti daelettroni (di massa 511 keVc2) o protoni (938 MeVc2) Avete a disposizionedue rivelatori identici in grado di registrare con precisione il tempo in cui unaparticella li attraversa Come potete utilizzarli per determinare se il vostrofascio contiene elettroni o protoni

Soluzione dellrsquoesercizio 20Disponendo i due rivelatori lungo la direzione del fascio a distanza ∆L

lrsquouno dallrsquoaltro si puograve misurare il tempo impiegato dalle particelle per passaredallrsquouno allrsquoaltro e quindi la loro massa infatti se ∆t egrave la distanza temporalefra i segnali dei due rivelatori

∆L = v∆t = βc∆t =pc2

E∆t

Questo principio egrave usato nei cosiddetti rivelatori di time of flight che sono usatiper discriminare diversi tipi di particelle una particella di tipo 1 e una di tipo2 infatti percorreranno la distanza ∆L in tempi diversi legati alle rispettivemasse dalla relazione

∆L1 = ∆L2

p1c2

E1∆t1 =

radicE2 minusm2

1c2c

E∆t1 =

radicE2 minusm2

2c2c

E∆t2

13

Esercizio 21 Energia di sogliaDue fisici delle particelle vogliono produrre il bosone Z una particella di

carica neutra e di massamZ = 91 GeVc2 e discutono come fare Alice proponedi far scontrare fasci di elettroni e positroni di energia identica che viaggianodunque con impulso spaziale uguale in modulo e direzione ma di verso oppostoproducendo Z tramite il processo

e+ + eminus rarr Z

mentre Bob preferisce scontrare un fascio di protoni su un bersaglio fisso diidrogeno tramite il processo

p+ prarr Z + p+ p

Chi dei due avragrave bisogno di fasci di particelle di energia piugrave alta La massa delprotone egrave di 9383 MeVc2 quella dellrsquoelettrone di 511 keVc2

Esercizio 22 Leggi di conservazioneUn fotone (particella di massa nulla) puograve decadere in un elettrone e in un

positrone (entrambi di massa 511 keVc2) tramite il processo

γ rarr e+ + eminus

Esercizio 23 Decadimento βminus

Quali sono lrsquoenergia minima e massima dellrsquoelettrone nel decadimento

nrarr p+ eminus + νe

se il neutrone decade da fermo La massa del neutrone egrave di 9396 MeVc2 quelladel protone di 9383 MeVc2 e quella dellrsquoelettrone di 511 keVc2 si assuma chelrsquoantineutrino elettronico νe abbia massa nulla

Esercizio 24 Leggi di conservazioneIl decadimento

prarr n+ e+ + νe

egrave permesso

4 Lezione 4 (3 aprile 2020)

Esercizio 25Usando il fatto che hc = 1973 MeVfm si dimostri che in un sistema di unitagrave dimisura in cui h = c = 1 vale

14

1 1 GeVminus2 = 0389 mb

2 1 m = 5068times 1015 GeVminus1

3 1 s = 15times 1024 GeVminus1

Ricordiamo che 1 b = 1times 10minus28 m2 e che

[hc] = [Jsms] = [E][L]

Soluzione dellrsquoesercizio 25Lrsquoidea egrave di capire per quale potenza di hc e c va moltiplicato il termine a

sinistra di ciascuna equazione per ottenere il termine di destraPer cui

bull [1 GeVminus2][hc]α = [E]minus2[E]α[L]α = [0389 mb] = [L]2 da cui segue α = 2

e 1 GeVminus2(hc)2 = 1973 MeVfm1 GeV =

(01973times 10minus15 GeVm

1 GeV

)2

= 0389 mb

bull [1 m][hc]α = [L][E]α[L]α = [5068times 1015 GeVminus1] = [E]minus1 da cui segueα = minus1 e 1 m(hc)minus1 = 1 m

1973 MeVfm = 1 m01973times 10minus15 GeVm

= 5068times 1015 GeVminus1

bull [1 s][hc]α[c]β = [T ][E]α[L]α[L]β [T ]minusβ = [T ][E]α[L]α+β [T ]minusβ = [15times 1024 GeVminus1] =[E]minus1 da cui segue α = minus1 β = 1 e 1 s(hc)minus1c = 1 s

1973 MeVfm299 792 458 ms =299 792 458 m

01973times 10minus15 GeVm= 15times 1024 GeVminus1

Esercizio 26 Massa invarianteTre protoni (mp = 938 MeVc2) hanno impulsi uguali in modulo (p =

3 GeVc) e che formano angoli di 120 lrsquouno con lrsquoaltro Qual egrave la massainvariante del sistema

Esercizio 27 Energia di sogliaSi consideri il processo

γ + prarr p+ π0

dove il fotone ha massa nulla il protone ha massa di 938 MeVc2 e il π0 hamassa di 135 MeVc2

1 Se il protone egrave a riposo qual egrave lrsquoenergia minima che deve avere il fotoneincidente percheacute la reazione abbia luogo

2 La stessa reazione puograve avvenire nellrsquouniverso in cui un protone dei raggicosmici di alta energia puograve collidere con uno dei fotoni della radiazionecosmica di fondo di energia dellrsquoordine di 1 meV Qual egrave in questo casolrsquoenergia minima che deve avere il protone percheacute la reazione abbia luogo

Esercizio 28 Conseguenze della relativitagraveUn misterioso supereroe pattuglia a velocitagrave molto elevata la periferia roma-

na Allrsquoincrocio con via di Tor Bella Monaca incontra un semaforo e ndash vedendolo

15

verde ndash attraversa senza rallentare Una pattuglia della polizia municipale loferma e lo multa per esser passato col rosso Assumendo sia il supereroe che ivigili siano nel giusto a che velocitagrave viaggiava il supereroe

Soluzione dellrsquoesercizio 28Per la polizia municipale il semaforo emette fotoni di energia

E0 = hν0 =hc

λrossoasymp hc

630 nm

mentre il supereoe vede fotoni di energia

E = hν =hc

λverdeasymp hc

490 nm

e dalle trasformazioni di Lorentz indicando con γ e β le variabili calcolateusando la velocitagrave del supereroe misurata dalla municipale possiamo scrivere

E = γ(E0minusβp0) = γ(E0minusβE0) =1minus βradic1minus β2

E0 =1minus βradic

(1 + β)(1minus β)E0 =

radic1minus βradic1 + β

E0

dove abbiamo usato il fatto che i fotoni hanno massa nulla Perciograve

E

E0=

hcλverdehc

λrosso

=λrosso

λverde=

radic1minus βradic1 + β

rarr v = βc = 025c

Esercizio 29 Conseguenze della relativitagraveI neutrini sono particelle di massa molto piccola e al momento ignota Uno

dei modi con cui egrave stato possibile dedurre un limite superiore al suo valore egravestata lrsquoosservazione nel 1987 di neutrini prodotti dalla supernova 1987A1 chesi trova a 168000 anni luce dalla Terra Sono stati osservati due segnali dineutrini che possono essere schematizzati come segue si egrave osservato prima unneutrino di 35 MeV di energia seguito a 9 s di distanza da un secondo segnaledi 13 MeV Si assuma che questo ritardo sia dovuto al fatto che la massa delneutrino non egrave nulla e si calcoli questrsquoultima

Esercizio 30 DecadimentoAnimali e piante assumono dallrsquoatmosfera diversi composti contenenti car-

bonio Il carbonio presente nellrsquoatmosfera egrave predominantemente 126 C ma sono

presenti piccole concentrazioni del suo isotopo 146 C (un atomo ogni 1012) che

decade con emissione di elettroni attraverso il processo2

146 C rarr14

7 N + eminus + νe

con un tempo di dimezzamento di 5700 anni1httpsenwikipediaorgwikiSN_1987ANeutrino_emissions2Di altro non si tratta che del decadimento βminus nrarr p+ e+ νe

16

1 Qual egrave la concentrazione di 146 C dopo 11400 anni

2 Animali e piante assumono in vita proporzioni fisse di 146 C e 12

6 C mentrealla loro morte la quantitagrave di 14

6 C inizia a diminuire Avete a portata dimano un relitto di legno per cui misurate una emissione di elettroni daldecadimento di 14

6 C pari al 61 di quella di un pezzo di legno vivodella stessa massa quanti anni ha il manufatto

Esercizio 31 Energia cinetica e trasformazioni di LorentzDue particelle identiche di massa m ed energia cinetica T collidono frontal-

mente Qual egrave la loro energia cinetica relativa (ossia lrsquoenergia cinetica di unaparticella misurata nel sistema di riferimento dellrsquoaltra particella)

Esercizio 32 Energia nel centro di massaLa reazione

πminusprarr K0Λ0

avviene con unrsquoenergia nel centro di massa diradics = 3 GeV La massa del πminus egrave

di 1396 MeVc2 la massa del protone egrave di 938 MeVc2 la massa del Kminus egrave di498 MeVc2 e quella della Λ0 di 11 GeVc2

1 Calcolare lrsquoimpulso di πminus e Λ0 nel sistema di riferimento del centro dimassa

2 Se il protone egrave a riposo il K puograve essere emesso allrsquoindietro nel sistemadi riferimento del laboratorio

Esercizio 33 DecadimentoIl mesone φ0 egrave una particella neutra3 di circa 1 GeVc2 di massa che puograve

decadere in una coppia di particelle

φ0 rarr K+ +Kminus

di massa identicamK = 494 MeVc2 Si assuma di produrre φ0 di impulso notoegrave possibile che uno dei due K sia prodotto a riposo nel sistema di riferimentodel laboratorio

Esercizio 34 DecadimentoUn fascio di anti-neutrini muonici νmicro si puograve generare selezionando pioni

o kaoni π+ e K+ e facendoli passare in un lungo tubo in cui egrave stato fattoil vuoto4 in modo che dopo un certo tragitto L una buona parte di loro saragrave

3Il collisore DAFNE ai Laboratori Nazionali di Frascati produce specificatamente parti-celle di questo tipo tramite il processo e+ + eminus rarr φ0 httpswwwyoutubecomwatchv=L5yB9gDGKms

4Una tecnica di questo tipo egrave stata usata per inviare ai Laboratori Nazionali del Gran Sassodei fasci di neutrini prodotti al CERN di Ginevra httpsvideoscernchrecord985892

17

decaduta tramite i processi

π+ rarr microminus + νmicro

K+ rarr microminus + νmicro

Se lrsquoimpulso di pioni e kaoni egrave di 200 GeVc e la loro vita media di 26 ns e12 ns rispettivamente

1 Quanto a lungo viaggiano nel laboratorio i due tipi di particelle2 Se L = 1000 m quale saragrave la frazione di pioni e kaoni che saragrave decaduta

alla fine del tubo3 Qual egrave lrsquoenergia massima dei neutrini che egrave possibile misurare nel sistema

di riferimento del laboratorio nei due casi

5 Lezione 5 (16 aprile 2020)

Esercizio 35 Energia cineticaTra le eccellenze con sede in territorio elvetico primeggiano indiscutibilmente lacioccolata al latte e il Large Hadron Collider (LHC) Questrsquoultimo egrave un collisoredi particelle situato al CERN di Ginevra che fa scontrare due fasci identici diprotoni di impulso uguale in modulo e direzione ma verso opposto I fasci sonocomposti da circa 2800 gruppi (pacchetti) di 1011 particelle ciascuno Sapendoche lrsquoenergia nel centro di massa della collisione fra due protoni egrave

radics = 13 TeV

quanta cioccolata al latte dovete mangiare per assumere un numero di caloriepari allrsquoenergia cinetica di ciascun fascio di protoni di LHC

Soluzione dellrsquoesercizio 35Poicheacute si tratta di un collider e le energie in gioco sono molto maggiori della

massa del protone lrsquoenergia di ciascun protone egrave diradics2 = 65 TeV e quella

totale di un fascio egrave

2800times 1011 times 65 middot 1012 times 16 middot 10minus19J asymp 300 MJ

Secondo Google 100 g di cioccolata al latte apportano circa 500 cal = 500 times4184 J asymp 2 MJ Lrsquoenergia di un fascio di LHC corrisponde quindi a quella dicirca 15 kg di cioccolata

Esercizio 36 Scattering RutherfordUn fascio di particelle α di 100 MeV di energia e 032 nA di corrente5 collide

contro un bersaglio fisso di alluminio spesso 1 cm Una sperimentatrice prendeun rivelatore di 1 cm times 1 cm di superficie e lo posiziona ad un angolo di 30

5Per una spiegazione breve su come (e percheacute) si misura la corrente di un fascio di particel-le vedi httpswwwlhc-closerestaking_a_closer_look_at_lhc0beam_current Unatrattazione piugrave completa egrave data ad esempio da httpscdscernchrecord1213275filesp141pdf

18

rispetto al fascio di particelle a 1 m di distanza dal bersaglio Quante particelleα incideranno sul rivelatore ogni secondo

Soluzione dellrsquoesercizio 36Lrsquoalluminio ha una densitagrave di 27 gcm3 numero atomico 13 e massa atomica

27 uPoicheacute le particelle α sono nuclei di elio hanno carica 2e e la corrente di

032 nA corrisponde a un miliardo di particelle incidenti al secondo

dNidt

=032 nCs

2times 16times 10minus19 C= 1times 109 sminus1

Il rivelatore vede un angolo solido di

∆Ω equiv superficieraggio

2

=1 cm2

(1 m)2= 1times 10minus4 sr

Si tratta di uno scattering alla Rutherford per cui la sezione drsquourto per unitagravedi angolo solido rilevata ad un certo angolo θ vale

dΩ=

(zαzAle

2

4πε04E

1

sin2(θ2)

)2

pari a

dΩasymp(

2times 13times 4times etimes 16times 10minus19 C

4π times 89times 10minus12 Fmtimes 4times 100times 106 eV

1

sin2(π180 times 302)

)2

asymp 2times 10minus30 m2sr = 20 mbsr

(1)

e il numero di particelle visto dal rivelatore vale se indichiamo con nAl = ρAlNAAAl

la densitagrave numero di atomi di alluminio e con d lo spessore del rivelatore

dNrivelate

dt= ∆Ω

dΩnAld

dNi

dt

asymp 1times 10minus4 srtimes 2times 10minus30 m2srtimes 1times 104 cm2m2 times 27 gcm3 6times 1023 molminus1

27 gmol

= 120 Hz

Esercizio 37 Sezione drsquourtoUn bersaglio di idrogeno liquido di densitagrave ρ = 0071 gcm3 e volume V =

125 cm3 egrave bombardato da un fascio monoenergetico di pioni negativi con unflusso φ = 2times 107 mminus2sminus1 e si osserva la reazione

πminus + prarr π0 + n

rivelando i fotoni del decadimento π0 rarr γγ che avviene nel 988 dei casi Sela sezione drsquourto di quellrsquointerazione egrave σ = 40 mb quanti fotoni sono emessi ognisecondo

19

Esercizio 38 Sezione drsquourtoLa sezione drsquourto dellrsquoeffetto fotoelettrico per raggi X di 10 keV in carbonio

egrave 40 b per atomo Data una lastra di carbonio di 4 mm di spessore si calcoli1 il numero di bersagli per unitagrave di volume2 il coefficiente di assorbimento per effetto fotoelettrico dei raggi X di tale

energia3 la probabilitagrave che un raggio X incidente sulla lastra produca un elettrone

per effetto fotoelettrico

Esercizio 39 Sezioni drsquourtoUn bersaglio drsquooro di densitagrave superficiale ρS = 097 mgcm2 e superficie

SB = 1 cm2 viene colpito da un fascio di particelle α la cui sezione trasversaegrave contenuta completamente nellrsquoarea del bersaglio Sul bersaglio impattano37times 104 αs La sezione drsquourto di diffusione elastica ad un certo angolo θ valedσ

ddΩ = 1 bsr Calcolare1 la densitagrave di atomi del bersaglio per unitagrave di superficie2 il numero di particelle α rivelate in unrsquoora da un rivelatore di superficie

SR = 2 cm2 posto allrsquoangolo θ e a distanza DR = 01 m dal bersaglio3 Se il fascio di particelle viene sostituito da una sorgente radioattiva di

particelle α con distribuzione isotropa su tutto lrsquoangolo solido che vieneposta lungo la stessa linea del fascio a distanza DB = 20 cm dal bersaglioquanto tempo egrave necessario per rivelare con lo stesso rivelatore lo stessonumero di particelle calcolato sopra a paritagrave di sezione drsquourto

Esercizio 40 CinematicaUn fascio di positroni di 100 MeV di energia annichila su una targhetta fissa

di elettroni producendo due fotoni tramite il processo

e+ + eminus rarr γ + γ

Se uno dei due fotoni egrave emesso a 30 nel sistema del centro di massa1 quanto vale lrsquoenergia dei due fotoni nel sistema del centro di massa2 che energia e direzione avranno nel sistema del laboratorio

Esercizio 41 Sezione drsquourtoGli antineutrini νe prodotti da un reattore nucleare con potenza P = 16 GW

attraversano un bersaglio da 2000 l drsquoacqua posto a 50 m di distanza dal reat-tore

1 Supponendo che per ogni fissione sia prodotta unrsquoenergia termica di200 MeV e vengano emessi 6 antineutrini calcolare il numero medio direazioni

νe + prarr e+ + n

prodotte al giorno se la sezione drsquourto egrave σ = 1times 10minus43 cm2

20

2 Qual egrave la minima energia del neutrino per cui la reazione egrave permessa

6 Lezione 7 (8 maggio 2020)

Esercizio 42Un protone interagisce con un bersaglio producendo un pione di energia E =300 MeV Con un tracciatore posto a distanza d = 6 cm dal bersaglio egrave possi-bile rivelare la traiettoria del pione e risalire al punto di produzione del pionesul bersaglio Determinare lrsquoerrore sulla misura della posizione di tale puntocausato dalla presenza di un piano di alluminio di spessore L = 05 mm postoimmediatamente davanti al tracciatore (quindi a distanza d dal bersaglio) nellrsquoi-potesi che gli angoli delle tracce rispetto alla normale alle superfici del bersaglioe del piano di alluminio siano piccoli [mπ = 1396 MeV Al (Z = 13 A = 27ρ = 27 gcm3)]

Soluzione dellrsquoesercizio 42La deviazione standard dellrsquoangolo di diffusione coulombiana multipla vale

radic〈θ2〉 = 21 MeV

z

βc|p|

radicx

X0= 21 MeV

z

βc|p|

radicLρ

X0

dove p =radicE2 minusm2

π = 266 MeV β = pE = 0885 z = 1 Per lrsquoalluminioabbiamo

X0 = 24 gcm2 rarr X0ρ = 89 cm

per cui radic〈θ2〉 asymp 00067

e lo spostamento medio corrispondente rispetto alla posizione nominale egrave

δs = tan(00067)times 6 cm = 04 mm

Esercizio 43Un fascio misto di elettroni e antiprotoni passa attraverso una regione con

B = 2 T e dopo aver curvato per R = 3 m raggiunge una regione senza campomagnetico

1 Qual egrave lrsquoimpulso delle particelle selezionate2 Come discriminereste fra i due tipi di particelle3 Se usaste due scintillatori posti a 15 m di distanza quale risoluzione

temporale sarebbe necessaria per discriminare i due tipi di particelle4 Se i due scintillatori fossero spessi 2 cm e avessero una lunghezza di ra-

diazione X0 = 40 cm quanta energia perderebbero elettroni e protoninellrsquoattraversarli Si assuma una perdita di energia per ionizzazione di2 MeVcm e 25 MeVcm per protoni ed elettroni rispettivamente

21

5 Che indice di rifrazione dovrebbe avere un rivelatore a luce Cherenkovper discriminare elettroni e protoni

Soluzione dellrsquoesercizio 43Dalla definizione di forza

F =dpdt

= qvtimesB

|F| = mv2

R

per cui se il campo magnetico egrave ortogonale alla traiettoria

pc = qcBR

che poicheacute c = 03 mns possiamo esprimere in maniera piugrave conveniente come

pc[GeV] = 03B[T]R[m]

Ne segue che nel nostro esercizio (e tornando a c = 1)

p = 18 GeV

Trascurando le perdite di energia e ricordando che me mp i tempi divolo di elettroni e protoni valgono rispettivamente

te =L

βecasymp L

casymp 50 ns

etp =

L

βpc=

Lpradic

p2+m2p

casymp 56 ns

per cui serve una risoluzione dellrsquoordine del ns per discriminare elettroni e muonidal tempo di volo

Se gli scintillatori sono spessi 2 cm i protoni perderanno in ciascuno di essi2 cm times 2 MeVcm = 4 MeV per ionizzazione Gli elettroni perderanno invecenel primo rivelatore 5 MeV per ionizzazione e (me asymp 0)

18 GeV times(

1minus exp

(minus 2 cm

40 cm

))asymp 88 MeV

cioegrave dopo il primo rivelatore avranno 18 GeVminus93 MeV di energia e nel secondone perderanno altri

5 MeV + (18 GeV minus 93 MeV)

(1minus exp

(minus 2 cm

40 cm

))asymp 88 MeV

Lrsquoangolo di emissione di luce Cherenkov egrave dato da

cos θc =1

βnle 1

22

e poicheacute βe asymp 1 βp asymp 0887 per avere luce Cherenkov solo al passaggio deglielettroni (e non dei protoni) egrave necessario che

1 lt n lt 113

Esercizio 44Come misurereste la massa di una particella carica E quella di una parti-

cella neutra

Esercizio 45Un muone di energia E = 400 GeV penetra verticalmente nel mare Attra-

verso quale processo puograve essere rivelato A quale profonditagrave arriva prima didecadere

Esercizio 46Avete a disposizione dei tubi fotomoltiplicatori sensibili a lunghezze drsquoonda

fra 300 nm e 500 nm e volete rivelare la luce Cherenkov prodotta dal passaggio diun elettrone di 1 MeV di energia in un metro drsquoacqua Quanti fotoni vi aspettateche vengano prodotti Confrontate con il numero di elettroni di ionizzazioneche produrrebbe una particella α di 5 keV nello stesso rivelatore

Esercizio 47Determinare quali sono i processi piugrave probabili (cioegrave quelli di sezione drsquourto

piugrave alta) nellrsquointerazione fra1 fotoni di 1 MeV e atomi di alluminio2 fotoni di 100 keV e H23 fotoni di 100 keV e atomi di ferro4 fotoni di 10 MeV e atomi di carbonio5 fotoni di 10 MeV e atomi di piombo

Esercizio 48Volete misurare lrsquoimpulso di una particella carica che attraversa un campo

magnetico B ortogonale alla sua traiettoria Avete a disposizione tre rivelatori diposizione della stessa risoluzione spaziale δx come li disponete Che risoluzionein impulso vi aspettate di ottenere

23

  • Lezione 1 (13 marzo 2020)
  • Lezione 2 (20 marzo 2020)
  • Lezione 3 (27 marzo 2020)
  • Lezione 4 (3 aprile 2020)
  • Lezione 5 (16 aprile 2020)
  • Lezione 7 (8 maggio 2020)
Page 14: Esercizi per casa (risolti)ippolitv/pdf/fns1_2020_solutions.pdfEsercizi per casa (risolti) Valerio Ippolito 13 maggio 2020 1 Lezione 1 (13 marzo 2020) Esercizio1 Dilatazionedeitempi

Esercizio 21 Energia di sogliaDue fisici delle particelle vogliono produrre il bosone Z una particella di

carica neutra e di massamZ = 91 GeVc2 e discutono come fare Alice proponedi far scontrare fasci di elettroni e positroni di energia identica che viaggianodunque con impulso spaziale uguale in modulo e direzione ma di verso oppostoproducendo Z tramite il processo

e+ + eminus rarr Z

mentre Bob preferisce scontrare un fascio di protoni su un bersaglio fisso diidrogeno tramite il processo

p+ prarr Z + p+ p

Chi dei due avragrave bisogno di fasci di particelle di energia piugrave alta La massa delprotone egrave di 9383 MeVc2 quella dellrsquoelettrone di 511 keVc2

Esercizio 22 Leggi di conservazioneUn fotone (particella di massa nulla) puograve decadere in un elettrone e in un

positrone (entrambi di massa 511 keVc2) tramite il processo

γ rarr e+ + eminus

Esercizio 23 Decadimento βminus

Quali sono lrsquoenergia minima e massima dellrsquoelettrone nel decadimento

nrarr p+ eminus + νe

se il neutrone decade da fermo La massa del neutrone egrave di 9396 MeVc2 quelladel protone di 9383 MeVc2 e quella dellrsquoelettrone di 511 keVc2 si assuma chelrsquoantineutrino elettronico νe abbia massa nulla

Esercizio 24 Leggi di conservazioneIl decadimento

prarr n+ e+ + νe

egrave permesso

4 Lezione 4 (3 aprile 2020)

Esercizio 25Usando il fatto che hc = 1973 MeVfm si dimostri che in un sistema di unitagrave dimisura in cui h = c = 1 vale

14

1 1 GeVminus2 = 0389 mb

2 1 m = 5068times 1015 GeVminus1

3 1 s = 15times 1024 GeVminus1

Ricordiamo che 1 b = 1times 10minus28 m2 e che

[hc] = [Jsms] = [E][L]

Soluzione dellrsquoesercizio 25Lrsquoidea egrave di capire per quale potenza di hc e c va moltiplicato il termine a

sinistra di ciascuna equazione per ottenere il termine di destraPer cui

bull [1 GeVminus2][hc]α = [E]minus2[E]α[L]α = [0389 mb] = [L]2 da cui segue α = 2

e 1 GeVminus2(hc)2 = 1973 MeVfm1 GeV =

(01973times 10minus15 GeVm

1 GeV

)2

= 0389 mb

bull [1 m][hc]α = [L][E]α[L]α = [5068times 1015 GeVminus1] = [E]minus1 da cui segueα = minus1 e 1 m(hc)minus1 = 1 m

1973 MeVfm = 1 m01973times 10minus15 GeVm

= 5068times 1015 GeVminus1

bull [1 s][hc]α[c]β = [T ][E]α[L]α[L]β [T ]minusβ = [T ][E]α[L]α+β [T ]minusβ = [15times 1024 GeVminus1] =[E]minus1 da cui segue α = minus1 β = 1 e 1 s(hc)minus1c = 1 s

1973 MeVfm299 792 458 ms =299 792 458 m

01973times 10minus15 GeVm= 15times 1024 GeVminus1

Esercizio 26 Massa invarianteTre protoni (mp = 938 MeVc2) hanno impulsi uguali in modulo (p =

3 GeVc) e che formano angoli di 120 lrsquouno con lrsquoaltro Qual egrave la massainvariante del sistema

Esercizio 27 Energia di sogliaSi consideri il processo

γ + prarr p+ π0

dove il fotone ha massa nulla il protone ha massa di 938 MeVc2 e il π0 hamassa di 135 MeVc2

1 Se il protone egrave a riposo qual egrave lrsquoenergia minima che deve avere il fotoneincidente percheacute la reazione abbia luogo

2 La stessa reazione puograve avvenire nellrsquouniverso in cui un protone dei raggicosmici di alta energia puograve collidere con uno dei fotoni della radiazionecosmica di fondo di energia dellrsquoordine di 1 meV Qual egrave in questo casolrsquoenergia minima che deve avere il protone percheacute la reazione abbia luogo

Esercizio 28 Conseguenze della relativitagraveUn misterioso supereroe pattuglia a velocitagrave molto elevata la periferia roma-

na Allrsquoincrocio con via di Tor Bella Monaca incontra un semaforo e ndash vedendolo

15

verde ndash attraversa senza rallentare Una pattuglia della polizia municipale loferma e lo multa per esser passato col rosso Assumendo sia il supereroe che ivigili siano nel giusto a che velocitagrave viaggiava il supereroe

Soluzione dellrsquoesercizio 28Per la polizia municipale il semaforo emette fotoni di energia

E0 = hν0 =hc

λrossoasymp hc

630 nm

mentre il supereoe vede fotoni di energia

E = hν =hc

λverdeasymp hc

490 nm

e dalle trasformazioni di Lorentz indicando con γ e β le variabili calcolateusando la velocitagrave del supereroe misurata dalla municipale possiamo scrivere

E = γ(E0minusβp0) = γ(E0minusβE0) =1minus βradic1minus β2

E0 =1minus βradic

(1 + β)(1minus β)E0 =

radic1minus βradic1 + β

E0

dove abbiamo usato il fatto che i fotoni hanno massa nulla Perciograve

E

E0=

hcλverdehc

λrosso

=λrosso

λverde=

radic1minus βradic1 + β

rarr v = βc = 025c

Esercizio 29 Conseguenze della relativitagraveI neutrini sono particelle di massa molto piccola e al momento ignota Uno

dei modi con cui egrave stato possibile dedurre un limite superiore al suo valore egravestata lrsquoosservazione nel 1987 di neutrini prodotti dalla supernova 1987A1 chesi trova a 168000 anni luce dalla Terra Sono stati osservati due segnali dineutrini che possono essere schematizzati come segue si egrave osservato prima unneutrino di 35 MeV di energia seguito a 9 s di distanza da un secondo segnaledi 13 MeV Si assuma che questo ritardo sia dovuto al fatto che la massa delneutrino non egrave nulla e si calcoli questrsquoultima

Esercizio 30 DecadimentoAnimali e piante assumono dallrsquoatmosfera diversi composti contenenti car-

bonio Il carbonio presente nellrsquoatmosfera egrave predominantemente 126 C ma sono

presenti piccole concentrazioni del suo isotopo 146 C (un atomo ogni 1012) che

decade con emissione di elettroni attraverso il processo2

146 C rarr14

7 N + eminus + νe

con un tempo di dimezzamento di 5700 anni1httpsenwikipediaorgwikiSN_1987ANeutrino_emissions2Di altro non si tratta che del decadimento βminus nrarr p+ e+ νe

16

1 Qual egrave la concentrazione di 146 C dopo 11400 anni

2 Animali e piante assumono in vita proporzioni fisse di 146 C e 12

6 C mentrealla loro morte la quantitagrave di 14

6 C inizia a diminuire Avete a portata dimano un relitto di legno per cui misurate una emissione di elettroni daldecadimento di 14

6 C pari al 61 di quella di un pezzo di legno vivodella stessa massa quanti anni ha il manufatto

Esercizio 31 Energia cinetica e trasformazioni di LorentzDue particelle identiche di massa m ed energia cinetica T collidono frontal-

mente Qual egrave la loro energia cinetica relativa (ossia lrsquoenergia cinetica di unaparticella misurata nel sistema di riferimento dellrsquoaltra particella)

Esercizio 32 Energia nel centro di massaLa reazione

πminusprarr K0Λ0

avviene con unrsquoenergia nel centro di massa diradics = 3 GeV La massa del πminus egrave

di 1396 MeVc2 la massa del protone egrave di 938 MeVc2 la massa del Kminus egrave di498 MeVc2 e quella della Λ0 di 11 GeVc2

1 Calcolare lrsquoimpulso di πminus e Λ0 nel sistema di riferimento del centro dimassa

2 Se il protone egrave a riposo il K puograve essere emesso allrsquoindietro nel sistemadi riferimento del laboratorio

Esercizio 33 DecadimentoIl mesone φ0 egrave una particella neutra3 di circa 1 GeVc2 di massa che puograve

decadere in una coppia di particelle

φ0 rarr K+ +Kminus

di massa identicamK = 494 MeVc2 Si assuma di produrre φ0 di impulso notoegrave possibile che uno dei due K sia prodotto a riposo nel sistema di riferimentodel laboratorio

Esercizio 34 DecadimentoUn fascio di anti-neutrini muonici νmicro si puograve generare selezionando pioni

o kaoni π+ e K+ e facendoli passare in un lungo tubo in cui egrave stato fattoil vuoto4 in modo che dopo un certo tragitto L una buona parte di loro saragrave

3Il collisore DAFNE ai Laboratori Nazionali di Frascati produce specificatamente parti-celle di questo tipo tramite il processo e+ + eminus rarr φ0 httpswwwyoutubecomwatchv=L5yB9gDGKms

4Una tecnica di questo tipo egrave stata usata per inviare ai Laboratori Nazionali del Gran Sassodei fasci di neutrini prodotti al CERN di Ginevra httpsvideoscernchrecord985892

17

decaduta tramite i processi

π+ rarr microminus + νmicro

K+ rarr microminus + νmicro

Se lrsquoimpulso di pioni e kaoni egrave di 200 GeVc e la loro vita media di 26 ns e12 ns rispettivamente

1 Quanto a lungo viaggiano nel laboratorio i due tipi di particelle2 Se L = 1000 m quale saragrave la frazione di pioni e kaoni che saragrave decaduta

alla fine del tubo3 Qual egrave lrsquoenergia massima dei neutrini che egrave possibile misurare nel sistema

di riferimento del laboratorio nei due casi

5 Lezione 5 (16 aprile 2020)

Esercizio 35 Energia cineticaTra le eccellenze con sede in territorio elvetico primeggiano indiscutibilmente lacioccolata al latte e il Large Hadron Collider (LHC) Questrsquoultimo egrave un collisoredi particelle situato al CERN di Ginevra che fa scontrare due fasci identici diprotoni di impulso uguale in modulo e direzione ma verso opposto I fasci sonocomposti da circa 2800 gruppi (pacchetti) di 1011 particelle ciascuno Sapendoche lrsquoenergia nel centro di massa della collisione fra due protoni egrave

radics = 13 TeV

quanta cioccolata al latte dovete mangiare per assumere un numero di caloriepari allrsquoenergia cinetica di ciascun fascio di protoni di LHC

Soluzione dellrsquoesercizio 35Poicheacute si tratta di un collider e le energie in gioco sono molto maggiori della

massa del protone lrsquoenergia di ciascun protone egrave diradics2 = 65 TeV e quella

totale di un fascio egrave

2800times 1011 times 65 middot 1012 times 16 middot 10minus19J asymp 300 MJ

Secondo Google 100 g di cioccolata al latte apportano circa 500 cal = 500 times4184 J asymp 2 MJ Lrsquoenergia di un fascio di LHC corrisponde quindi a quella dicirca 15 kg di cioccolata

Esercizio 36 Scattering RutherfordUn fascio di particelle α di 100 MeV di energia e 032 nA di corrente5 collide

contro un bersaglio fisso di alluminio spesso 1 cm Una sperimentatrice prendeun rivelatore di 1 cm times 1 cm di superficie e lo posiziona ad un angolo di 30

5Per una spiegazione breve su come (e percheacute) si misura la corrente di un fascio di particel-le vedi httpswwwlhc-closerestaking_a_closer_look_at_lhc0beam_current Unatrattazione piugrave completa egrave data ad esempio da httpscdscernchrecord1213275filesp141pdf

18

rispetto al fascio di particelle a 1 m di distanza dal bersaglio Quante particelleα incideranno sul rivelatore ogni secondo

Soluzione dellrsquoesercizio 36Lrsquoalluminio ha una densitagrave di 27 gcm3 numero atomico 13 e massa atomica

27 uPoicheacute le particelle α sono nuclei di elio hanno carica 2e e la corrente di

032 nA corrisponde a un miliardo di particelle incidenti al secondo

dNidt

=032 nCs

2times 16times 10minus19 C= 1times 109 sminus1

Il rivelatore vede un angolo solido di

∆Ω equiv superficieraggio

2

=1 cm2

(1 m)2= 1times 10minus4 sr

Si tratta di uno scattering alla Rutherford per cui la sezione drsquourto per unitagravedi angolo solido rilevata ad un certo angolo θ vale

dΩ=

(zαzAle

2

4πε04E

1

sin2(θ2)

)2

pari a

dΩasymp(

2times 13times 4times etimes 16times 10minus19 C

4π times 89times 10minus12 Fmtimes 4times 100times 106 eV

1

sin2(π180 times 302)

)2

asymp 2times 10minus30 m2sr = 20 mbsr

(1)

e il numero di particelle visto dal rivelatore vale se indichiamo con nAl = ρAlNAAAl

la densitagrave numero di atomi di alluminio e con d lo spessore del rivelatore

dNrivelate

dt= ∆Ω

dΩnAld

dNi

dt

asymp 1times 10minus4 srtimes 2times 10minus30 m2srtimes 1times 104 cm2m2 times 27 gcm3 6times 1023 molminus1

27 gmol

= 120 Hz

Esercizio 37 Sezione drsquourtoUn bersaglio di idrogeno liquido di densitagrave ρ = 0071 gcm3 e volume V =

125 cm3 egrave bombardato da un fascio monoenergetico di pioni negativi con unflusso φ = 2times 107 mminus2sminus1 e si osserva la reazione

πminus + prarr π0 + n

rivelando i fotoni del decadimento π0 rarr γγ che avviene nel 988 dei casi Sela sezione drsquourto di quellrsquointerazione egrave σ = 40 mb quanti fotoni sono emessi ognisecondo

19

Esercizio 38 Sezione drsquourtoLa sezione drsquourto dellrsquoeffetto fotoelettrico per raggi X di 10 keV in carbonio

egrave 40 b per atomo Data una lastra di carbonio di 4 mm di spessore si calcoli1 il numero di bersagli per unitagrave di volume2 il coefficiente di assorbimento per effetto fotoelettrico dei raggi X di tale

energia3 la probabilitagrave che un raggio X incidente sulla lastra produca un elettrone

per effetto fotoelettrico

Esercizio 39 Sezioni drsquourtoUn bersaglio drsquooro di densitagrave superficiale ρS = 097 mgcm2 e superficie

SB = 1 cm2 viene colpito da un fascio di particelle α la cui sezione trasversaegrave contenuta completamente nellrsquoarea del bersaglio Sul bersaglio impattano37times 104 αs La sezione drsquourto di diffusione elastica ad un certo angolo θ valedσ

ddΩ = 1 bsr Calcolare1 la densitagrave di atomi del bersaglio per unitagrave di superficie2 il numero di particelle α rivelate in unrsquoora da un rivelatore di superficie

SR = 2 cm2 posto allrsquoangolo θ e a distanza DR = 01 m dal bersaglio3 Se il fascio di particelle viene sostituito da una sorgente radioattiva di

particelle α con distribuzione isotropa su tutto lrsquoangolo solido che vieneposta lungo la stessa linea del fascio a distanza DB = 20 cm dal bersaglioquanto tempo egrave necessario per rivelare con lo stesso rivelatore lo stessonumero di particelle calcolato sopra a paritagrave di sezione drsquourto

Esercizio 40 CinematicaUn fascio di positroni di 100 MeV di energia annichila su una targhetta fissa

di elettroni producendo due fotoni tramite il processo

e+ + eminus rarr γ + γ

Se uno dei due fotoni egrave emesso a 30 nel sistema del centro di massa1 quanto vale lrsquoenergia dei due fotoni nel sistema del centro di massa2 che energia e direzione avranno nel sistema del laboratorio

Esercizio 41 Sezione drsquourtoGli antineutrini νe prodotti da un reattore nucleare con potenza P = 16 GW

attraversano un bersaglio da 2000 l drsquoacqua posto a 50 m di distanza dal reat-tore

1 Supponendo che per ogni fissione sia prodotta unrsquoenergia termica di200 MeV e vengano emessi 6 antineutrini calcolare il numero medio direazioni

νe + prarr e+ + n

prodotte al giorno se la sezione drsquourto egrave σ = 1times 10minus43 cm2

20

2 Qual egrave la minima energia del neutrino per cui la reazione egrave permessa

6 Lezione 7 (8 maggio 2020)

Esercizio 42Un protone interagisce con un bersaglio producendo un pione di energia E =300 MeV Con un tracciatore posto a distanza d = 6 cm dal bersaglio egrave possi-bile rivelare la traiettoria del pione e risalire al punto di produzione del pionesul bersaglio Determinare lrsquoerrore sulla misura della posizione di tale puntocausato dalla presenza di un piano di alluminio di spessore L = 05 mm postoimmediatamente davanti al tracciatore (quindi a distanza d dal bersaglio) nellrsquoi-potesi che gli angoli delle tracce rispetto alla normale alle superfici del bersaglioe del piano di alluminio siano piccoli [mπ = 1396 MeV Al (Z = 13 A = 27ρ = 27 gcm3)]

Soluzione dellrsquoesercizio 42La deviazione standard dellrsquoangolo di diffusione coulombiana multipla vale

radic〈θ2〉 = 21 MeV

z

βc|p|

radicx

X0= 21 MeV

z

βc|p|

radicLρ

X0

dove p =radicE2 minusm2

π = 266 MeV β = pE = 0885 z = 1 Per lrsquoalluminioabbiamo

X0 = 24 gcm2 rarr X0ρ = 89 cm

per cui radic〈θ2〉 asymp 00067

e lo spostamento medio corrispondente rispetto alla posizione nominale egrave

δs = tan(00067)times 6 cm = 04 mm

Esercizio 43Un fascio misto di elettroni e antiprotoni passa attraverso una regione con

B = 2 T e dopo aver curvato per R = 3 m raggiunge una regione senza campomagnetico

1 Qual egrave lrsquoimpulso delle particelle selezionate2 Come discriminereste fra i due tipi di particelle3 Se usaste due scintillatori posti a 15 m di distanza quale risoluzione

temporale sarebbe necessaria per discriminare i due tipi di particelle4 Se i due scintillatori fossero spessi 2 cm e avessero una lunghezza di ra-

diazione X0 = 40 cm quanta energia perderebbero elettroni e protoninellrsquoattraversarli Si assuma una perdita di energia per ionizzazione di2 MeVcm e 25 MeVcm per protoni ed elettroni rispettivamente

21

5 Che indice di rifrazione dovrebbe avere un rivelatore a luce Cherenkovper discriminare elettroni e protoni

Soluzione dellrsquoesercizio 43Dalla definizione di forza

F =dpdt

= qvtimesB

|F| = mv2

R

per cui se il campo magnetico egrave ortogonale alla traiettoria

pc = qcBR

che poicheacute c = 03 mns possiamo esprimere in maniera piugrave conveniente come

pc[GeV] = 03B[T]R[m]

Ne segue che nel nostro esercizio (e tornando a c = 1)

p = 18 GeV

Trascurando le perdite di energia e ricordando che me mp i tempi divolo di elettroni e protoni valgono rispettivamente

te =L

βecasymp L

casymp 50 ns

etp =

L

βpc=

Lpradic

p2+m2p

casymp 56 ns

per cui serve una risoluzione dellrsquoordine del ns per discriminare elettroni e muonidal tempo di volo

Se gli scintillatori sono spessi 2 cm i protoni perderanno in ciascuno di essi2 cm times 2 MeVcm = 4 MeV per ionizzazione Gli elettroni perderanno invecenel primo rivelatore 5 MeV per ionizzazione e (me asymp 0)

18 GeV times(

1minus exp

(minus 2 cm

40 cm

))asymp 88 MeV

cioegrave dopo il primo rivelatore avranno 18 GeVminus93 MeV di energia e nel secondone perderanno altri

5 MeV + (18 GeV minus 93 MeV)

(1minus exp

(minus 2 cm

40 cm

))asymp 88 MeV

Lrsquoangolo di emissione di luce Cherenkov egrave dato da

cos θc =1

βnle 1

22

e poicheacute βe asymp 1 βp asymp 0887 per avere luce Cherenkov solo al passaggio deglielettroni (e non dei protoni) egrave necessario che

1 lt n lt 113

Esercizio 44Come misurereste la massa di una particella carica E quella di una parti-

cella neutra

Esercizio 45Un muone di energia E = 400 GeV penetra verticalmente nel mare Attra-

verso quale processo puograve essere rivelato A quale profonditagrave arriva prima didecadere

Esercizio 46Avete a disposizione dei tubi fotomoltiplicatori sensibili a lunghezze drsquoonda

fra 300 nm e 500 nm e volete rivelare la luce Cherenkov prodotta dal passaggio diun elettrone di 1 MeV di energia in un metro drsquoacqua Quanti fotoni vi aspettateche vengano prodotti Confrontate con il numero di elettroni di ionizzazioneche produrrebbe una particella α di 5 keV nello stesso rivelatore

Esercizio 47Determinare quali sono i processi piugrave probabili (cioegrave quelli di sezione drsquourto

piugrave alta) nellrsquointerazione fra1 fotoni di 1 MeV e atomi di alluminio2 fotoni di 100 keV e H23 fotoni di 100 keV e atomi di ferro4 fotoni di 10 MeV e atomi di carbonio5 fotoni di 10 MeV e atomi di piombo

Esercizio 48Volete misurare lrsquoimpulso di una particella carica che attraversa un campo

magnetico B ortogonale alla sua traiettoria Avete a disposizione tre rivelatori diposizione della stessa risoluzione spaziale δx come li disponete Che risoluzionein impulso vi aspettate di ottenere

23

  • Lezione 1 (13 marzo 2020)
  • Lezione 2 (20 marzo 2020)
  • Lezione 3 (27 marzo 2020)
  • Lezione 4 (3 aprile 2020)
  • Lezione 5 (16 aprile 2020)
  • Lezione 7 (8 maggio 2020)
Page 15: Esercizi per casa (risolti)ippolitv/pdf/fns1_2020_solutions.pdfEsercizi per casa (risolti) Valerio Ippolito 13 maggio 2020 1 Lezione 1 (13 marzo 2020) Esercizio1 Dilatazionedeitempi

1 1 GeVminus2 = 0389 mb

2 1 m = 5068times 1015 GeVminus1

3 1 s = 15times 1024 GeVminus1

Ricordiamo che 1 b = 1times 10minus28 m2 e che

[hc] = [Jsms] = [E][L]

Soluzione dellrsquoesercizio 25Lrsquoidea egrave di capire per quale potenza di hc e c va moltiplicato il termine a

sinistra di ciascuna equazione per ottenere il termine di destraPer cui

bull [1 GeVminus2][hc]α = [E]minus2[E]α[L]α = [0389 mb] = [L]2 da cui segue α = 2

e 1 GeVminus2(hc)2 = 1973 MeVfm1 GeV =

(01973times 10minus15 GeVm

1 GeV

)2

= 0389 mb

bull [1 m][hc]α = [L][E]α[L]α = [5068times 1015 GeVminus1] = [E]minus1 da cui segueα = minus1 e 1 m(hc)minus1 = 1 m

1973 MeVfm = 1 m01973times 10minus15 GeVm

= 5068times 1015 GeVminus1

bull [1 s][hc]α[c]β = [T ][E]α[L]α[L]β [T ]minusβ = [T ][E]α[L]α+β [T ]minusβ = [15times 1024 GeVminus1] =[E]minus1 da cui segue α = minus1 β = 1 e 1 s(hc)minus1c = 1 s

1973 MeVfm299 792 458 ms =299 792 458 m

01973times 10minus15 GeVm= 15times 1024 GeVminus1

Esercizio 26 Massa invarianteTre protoni (mp = 938 MeVc2) hanno impulsi uguali in modulo (p =

3 GeVc) e che formano angoli di 120 lrsquouno con lrsquoaltro Qual egrave la massainvariante del sistema

Esercizio 27 Energia di sogliaSi consideri il processo

γ + prarr p+ π0

dove il fotone ha massa nulla il protone ha massa di 938 MeVc2 e il π0 hamassa di 135 MeVc2

1 Se il protone egrave a riposo qual egrave lrsquoenergia minima che deve avere il fotoneincidente percheacute la reazione abbia luogo

2 La stessa reazione puograve avvenire nellrsquouniverso in cui un protone dei raggicosmici di alta energia puograve collidere con uno dei fotoni della radiazionecosmica di fondo di energia dellrsquoordine di 1 meV Qual egrave in questo casolrsquoenergia minima che deve avere il protone percheacute la reazione abbia luogo

Esercizio 28 Conseguenze della relativitagraveUn misterioso supereroe pattuglia a velocitagrave molto elevata la periferia roma-

na Allrsquoincrocio con via di Tor Bella Monaca incontra un semaforo e ndash vedendolo

15

verde ndash attraversa senza rallentare Una pattuglia della polizia municipale loferma e lo multa per esser passato col rosso Assumendo sia il supereroe che ivigili siano nel giusto a che velocitagrave viaggiava il supereroe

Soluzione dellrsquoesercizio 28Per la polizia municipale il semaforo emette fotoni di energia

E0 = hν0 =hc

λrossoasymp hc

630 nm

mentre il supereoe vede fotoni di energia

E = hν =hc

λverdeasymp hc

490 nm

e dalle trasformazioni di Lorentz indicando con γ e β le variabili calcolateusando la velocitagrave del supereroe misurata dalla municipale possiamo scrivere

E = γ(E0minusβp0) = γ(E0minusβE0) =1minus βradic1minus β2

E0 =1minus βradic

(1 + β)(1minus β)E0 =

radic1minus βradic1 + β

E0

dove abbiamo usato il fatto che i fotoni hanno massa nulla Perciograve

E

E0=

hcλverdehc

λrosso

=λrosso

λverde=

radic1minus βradic1 + β

rarr v = βc = 025c

Esercizio 29 Conseguenze della relativitagraveI neutrini sono particelle di massa molto piccola e al momento ignota Uno

dei modi con cui egrave stato possibile dedurre un limite superiore al suo valore egravestata lrsquoosservazione nel 1987 di neutrini prodotti dalla supernova 1987A1 chesi trova a 168000 anni luce dalla Terra Sono stati osservati due segnali dineutrini che possono essere schematizzati come segue si egrave osservato prima unneutrino di 35 MeV di energia seguito a 9 s di distanza da un secondo segnaledi 13 MeV Si assuma che questo ritardo sia dovuto al fatto che la massa delneutrino non egrave nulla e si calcoli questrsquoultima

Esercizio 30 DecadimentoAnimali e piante assumono dallrsquoatmosfera diversi composti contenenti car-

bonio Il carbonio presente nellrsquoatmosfera egrave predominantemente 126 C ma sono

presenti piccole concentrazioni del suo isotopo 146 C (un atomo ogni 1012) che

decade con emissione di elettroni attraverso il processo2

146 C rarr14

7 N + eminus + νe

con un tempo di dimezzamento di 5700 anni1httpsenwikipediaorgwikiSN_1987ANeutrino_emissions2Di altro non si tratta che del decadimento βminus nrarr p+ e+ νe

16

1 Qual egrave la concentrazione di 146 C dopo 11400 anni

2 Animali e piante assumono in vita proporzioni fisse di 146 C e 12

6 C mentrealla loro morte la quantitagrave di 14

6 C inizia a diminuire Avete a portata dimano un relitto di legno per cui misurate una emissione di elettroni daldecadimento di 14

6 C pari al 61 di quella di un pezzo di legno vivodella stessa massa quanti anni ha il manufatto

Esercizio 31 Energia cinetica e trasformazioni di LorentzDue particelle identiche di massa m ed energia cinetica T collidono frontal-

mente Qual egrave la loro energia cinetica relativa (ossia lrsquoenergia cinetica di unaparticella misurata nel sistema di riferimento dellrsquoaltra particella)

Esercizio 32 Energia nel centro di massaLa reazione

πminusprarr K0Λ0

avviene con unrsquoenergia nel centro di massa diradics = 3 GeV La massa del πminus egrave

di 1396 MeVc2 la massa del protone egrave di 938 MeVc2 la massa del Kminus egrave di498 MeVc2 e quella della Λ0 di 11 GeVc2

1 Calcolare lrsquoimpulso di πminus e Λ0 nel sistema di riferimento del centro dimassa

2 Se il protone egrave a riposo il K puograve essere emesso allrsquoindietro nel sistemadi riferimento del laboratorio

Esercizio 33 DecadimentoIl mesone φ0 egrave una particella neutra3 di circa 1 GeVc2 di massa che puograve

decadere in una coppia di particelle

φ0 rarr K+ +Kminus

di massa identicamK = 494 MeVc2 Si assuma di produrre φ0 di impulso notoegrave possibile che uno dei due K sia prodotto a riposo nel sistema di riferimentodel laboratorio

Esercizio 34 DecadimentoUn fascio di anti-neutrini muonici νmicro si puograve generare selezionando pioni

o kaoni π+ e K+ e facendoli passare in un lungo tubo in cui egrave stato fattoil vuoto4 in modo che dopo un certo tragitto L una buona parte di loro saragrave

3Il collisore DAFNE ai Laboratori Nazionali di Frascati produce specificatamente parti-celle di questo tipo tramite il processo e+ + eminus rarr φ0 httpswwwyoutubecomwatchv=L5yB9gDGKms

4Una tecnica di questo tipo egrave stata usata per inviare ai Laboratori Nazionali del Gran Sassodei fasci di neutrini prodotti al CERN di Ginevra httpsvideoscernchrecord985892

17

decaduta tramite i processi

π+ rarr microminus + νmicro

K+ rarr microminus + νmicro

Se lrsquoimpulso di pioni e kaoni egrave di 200 GeVc e la loro vita media di 26 ns e12 ns rispettivamente

1 Quanto a lungo viaggiano nel laboratorio i due tipi di particelle2 Se L = 1000 m quale saragrave la frazione di pioni e kaoni che saragrave decaduta

alla fine del tubo3 Qual egrave lrsquoenergia massima dei neutrini che egrave possibile misurare nel sistema

di riferimento del laboratorio nei due casi

5 Lezione 5 (16 aprile 2020)

Esercizio 35 Energia cineticaTra le eccellenze con sede in territorio elvetico primeggiano indiscutibilmente lacioccolata al latte e il Large Hadron Collider (LHC) Questrsquoultimo egrave un collisoredi particelle situato al CERN di Ginevra che fa scontrare due fasci identici diprotoni di impulso uguale in modulo e direzione ma verso opposto I fasci sonocomposti da circa 2800 gruppi (pacchetti) di 1011 particelle ciascuno Sapendoche lrsquoenergia nel centro di massa della collisione fra due protoni egrave

radics = 13 TeV

quanta cioccolata al latte dovete mangiare per assumere un numero di caloriepari allrsquoenergia cinetica di ciascun fascio di protoni di LHC

Soluzione dellrsquoesercizio 35Poicheacute si tratta di un collider e le energie in gioco sono molto maggiori della

massa del protone lrsquoenergia di ciascun protone egrave diradics2 = 65 TeV e quella

totale di un fascio egrave

2800times 1011 times 65 middot 1012 times 16 middot 10minus19J asymp 300 MJ

Secondo Google 100 g di cioccolata al latte apportano circa 500 cal = 500 times4184 J asymp 2 MJ Lrsquoenergia di un fascio di LHC corrisponde quindi a quella dicirca 15 kg di cioccolata

Esercizio 36 Scattering RutherfordUn fascio di particelle α di 100 MeV di energia e 032 nA di corrente5 collide

contro un bersaglio fisso di alluminio spesso 1 cm Una sperimentatrice prendeun rivelatore di 1 cm times 1 cm di superficie e lo posiziona ad un angolo di 30

5Per una spiegazione breve su come (e percheacute) si misura la corrente di un fascio di particel-le vedi httpswwwlhc-closerestaking_a_closer_look_at_lhc0beam_current Unatrattazione piugrave completa egrave data ad esempio da httpscdscernchrecord1213275filesp141pdf

18

rispetto al fascio di particelle a 1 m di distanza dal bersaglio Quante particelleα incideranno sul rivelatore ogni secondo

Soluzione dellrsquoesercizio 36Lrsquoalluminio ha una densitagrave di 27 gcm3 numero atomico 13 e massa atomica

27 uPoicheacute le particelle α sono nuclei di elio hanno carica 2e e la corrente di

032 nA corrisponde a un miliardo di particelle incidenti al secondo

dNidt

=032 nCs

2times 16times 10minus19 C= 1times 109 sminus1

Il rivelatore vede un angolo solido di

∆Ω equiv superficieraggio

2

=1 cm2

(1 m)2= 1times 10minus4 sr

Si tratta di uno scattering alla Rutherford per cui la sezione drsquourto per unitagravedi angolo solido rilevata ad un certo angolo θ vale

dΩ=

(zαzAle

2

4πε04E

1

sin2(θ2)

)2

pari a

dΩasymp(

2times 13times 4times etimes 16times 10minus19 C

4π times 89times 10minus12 Fmtimes 4times 100times 106 eV

1

sin2(π180 times 302)

)2

asymp 2times 10minus30 m2sr = 20 mbsr

(1)

e il numero di particelle visto dal rivelatore vale se indichiamo con nAl = ρAlNAAAl

la densitagrave numero di atomi di alluminio e con d lo spessore del rivelatore

dNrivelate

dt= ∆Ω

dΩnAld

dNi

dt

asymp 1times 10minus4 srtimes 2times 10minus30 m2srtimes 1times 104 cm2m2 times 27 gcm3 6times 1023 molminus1

27 gmol

= 120 Hz

Esercizio 37 Sezione drsquourtoUn bersaglio di idrogeno liquido di densitagrave ρ = 0071 gcm3 e volume V =

125 cm3 egrave bombardato da un fascio monoenergetico di pioni negativi con unflusso φ = 2times 107 mminus2sminus1 e si osserva la reazione

πminus + prarr π0 + n

rivelando i fotoni del decadimento π0 rarr γγ che avviene nel 988 dei casi Sela sezione drsquourto di quellrsquointerazione egrave σ = 40 mb quanti fotoni sono emessi ognisecondo

19

Esercizio 38 Sezione drsquourtoLa sezione drsquourto dellrsquoeffetto fotoelettrico per raggi X di 10 keV in carbonio

egrave 40 b per atomo Data una lastra di carbonio di 4 mm di spessore si calcoli1 il numero di bersagli per unitagrave di volume2 il coefficiente di assorbimento per effetto fotoelettrico dei raggi X di tale

energia3 la probabilitagrave che un raggio X incidente sulla lastra produca un elettrone

per effetto fotoelettrico

Esercizio 39 Sezioni drsquourtoUn bersaglio drsquooro di densitagrave superficiale ρS = 097 mgcm2 e superficie

SB = 1 cm2 viene colpito da un fascio di particelle α la cui sezione trasversaegrave contenuta completamente nellrsquoarea del bersaglio Sul bersaglio impattano37times 104 αs La sezione drsquourto di diffusione elastica ad un certo angolo θ valedσ

ddΩ = 1 bsr Calcolare1 la densitagrave di atomi del bersaglio per unitagrave di superficie2 il numero di particelle α rivelate in unrsquoora da un rivelatore di superficie

SR = 2 cm2 posto allrsquoangolo θ e a distanza DR = 01 m dal bersaglio3 Se il fascio di particelle viene sostituito da una sorgente radioattiva di

particelle α con distribuzione isotropa su tutto lrsquoangolo solido che vieneposta lungo la stessa linea del fascio a distanza DB = 20 cm dal bersaglioquanto tempo egrave necessario per rivelare con lo stesso rivelatore lo stessonumero di particelle calcolato sopra a paritagrave di sezione drsquourto

Esercizio 40 CinematicaUn fascio di positroni di 100 MeV di energia annichila su una targhetta fissa

di elettroni producendo due fotoni tramite il processo

e+ + eminus rarr γ + γ

Se uno dei due fotoni egrave emesso a 30 nel sistema del centro di massa1 quanto vale lrsquoenergia dei due fotoni nel sistema del centro di massa2 che energia e direzione avranno nel sistema del laboratorio

Esercizio 41 Sezione drsquourtoGli antineutrini νe prodotti da un reattore nucleare con potenza P = 16 GW

attraversano un bersaglio da 2000 l drsquoacqua posto a 50 m di distanza dal reat-tore

1 Supponendo che per ogni fissione sia prodotta unrsquoenergia termica di200 MeV e vengano emessi 6 antineutrini calcolare il numero medio direazioni

νe + prarr e+ + n

prodotte al giorno se la sezione drsquourto egrave σ = 1times 10minus43 cm2

20

2 Qual egrave la minima energia del neutrino per cui la reazione egrave permessa

6 Lezione 7 (8 maggio 2020)

Esercizio 42Un protone interagisce con un bersaglio producendo un pione di energia E =300 MeV Con un tracciatore posto a distanza d = 6 cm dal bersaglio egrave possi-bile rivelare la traiettoria del pione e risalire al punto di produzione del pionesul bersaglio Determinare lrsquoerrore sulla misura della posizione di tale puntocausato dalla presenza di un piano di alluminio di spessore L = 05 mm postoimmediatamente davanti al tracciatore (quindi a distanza d dal bersaglio) nellrsquoi-potesi che gli angoli delle tracce rispetto alla normale alle superfici del bersaglioe del piano di alluminio siano piccoli [mπ = 1396 MeV Al (Z = 13 A = 27ρ = 27 gcm3)]

Soluzione dellrsquoesercizio 42La deviazione standard dellrsquoangolo di diffusione coulombiana multipla vale

radic〈θ2〉 = 21 MeV

z

βc|p|

radicx

X0= 21 MeV

z

βc|p|

radicLρ

X0

dove p =radicE2 minusm2

π = 266 MeV β = pE = 0885 z = 1 Per lrsquoalluminioabbiamo

X0 = 24 gcm2 rarr X0ρ = 89 cm

per cui radic〈θ2〉 asymp 00067

e lo spostamento medio corrispondente rispetto alla posizione nominale egrave

δs = tan(00067)times 6 cm = 04 mm

Esercizio 43Un fascio misto di elettroni e antiprotoni passa attraverso una regione con

B = 2 T e dopo aver curvato per R = 3 m raggiunge una regione senza campomagnetico

1 Qual egrave lrsquoimpulso delle particelle selezionate2 Come discriminereste fra i due tipi di particelle3 Se usaste due scintillatori posti a 15 m di distanza quale risoluzione

temporale sarebbe necessaria per discriminare i due tipi di particelle4 Se i due scintillatori fossero spessi 2 cm e avessero una lunghezza di ra-

diazione X0 = 40 cm quanta energia perderebbero elettroni e protoninellrsquoattraversarli Si assuma una perdita di energia per ionizzazione di2 MeVcm e 25 MeVcm per protoni ed elettroni rispettivamente

21

5 Che indice di rifrazione dovrebbe avere un rivelatore a luce Cherenkovper discriminare elettroni e protoni

Soluzione dellrsquoesercizio 43Dalla definizione di forza

F =dpdt

= qvtimesB

|F| = mv2

R

per cui se il campo magnetico egrave ortogonale alla traiettoria

pc = qcBR

che poicheacute c = 03 mns possiamo esprimere in maniera piugrave conveniente come

pc[GeV] = 03B[T]R[m]

Ne segue che nel nostro esercizio (e tornando a c = 1)

p = 18 GeV

Trascurando le perdite di energia e ricordando che me mp i tempi divolo di elettroni e protoni valgono rispettivamente

te =L

βecasymp L

casymp 50 ns

etp =

L

βpc=

Lpradic

p2+m2p

casymp 56 ns

per cui serve una risoluzione dellrsquoordine del ns per discriminare elettroni e muonidal tempo di volo

Se gli scintillatori sono spessi 2 cm i protoni perderanno in ciascuno di essi2 cm times 2 MeVcm = 4 MeV per ionizzazione Gli elettroni perderanno invecenel primo rivelatore 5 MeV per ionizzazione e (me asymp 0)

18 GeV times(

1minus exp

(minus 2 cm

40 cm

))asymp 88 MeV

cioegrave dopo il primo rivelatore avranno 18 GeVminus93 MeV di energia e nel secondone perderanno altri

5 MeV + (18 GeV minus 93 MeV)

(1minus exp

(minus 2 cm

40 cm

))asymp 88 MeV

Lrsquoangolo di emissione di luce Cherenkov egrave dato da

cos θc =1

βnle 1

22

e poicheacute βe asymp 1 βp asymp 0887 per avere luce Cherenkov solo al passaggio deglielettroni (e non dei protoni) egrave necessario che

1 lt n lt 113

Esercizio 44Come misurereste la massa di una particella carica E quella di una parti-

cella neutra

Esercizio 45Un muone di energia E = 400 GeV penetra verticalmente nel mare Attra-

verso quale processo puograve essere rivelato A quale profonditagrave arriva prima didecadere

Esercizio 46Avete a disposizione dei tubi fotomoltiplicatori sensibili a lunghezze drsquoonda

fra 300 nm e 500 nm e volete rivelare la luce Cherenkov prodotta dal passaggio diun elettrone di 1 MeV di energia in un metro drsquoacqua Quanti fotoni vi aspettateche vengano prodotti Confrontate con il numero di elettroni di ionizzazioneche produrrebbe una particella α di 5 keV nello stesso rivelatore

Esercizio 47Determinare quali sono i processi piugrave probabili (cioegrave quelli di sezione drsquourto

piugrave alta) nellrsquointerazione fra1 fotoni di 1 MeV e atomi di alluminio2 fotoni di 100 keV e H23 fotoni di 100 keV e atomi di ferro4 fotoni di 10 MeV e atomi di carbonio5 fotoni di 10 MeV e atomi di piombo

Esercizio 48Volete misurare lrsquoimpulso di una particella carica che attraversa un campo

magnetico B ortogonale alla sua traiettoria Avete a disposizione tre rivelatori diposizione della stessa risoluzione spaziale δx come li disponete Che risoluzionein impulso vi aspettate di ottenere

23

  • Lezione 1 (13 marzo 2020)
  • Lezione 2 (20 marzo 2020)
  • Lezione 3 (27 marzo 2020)
  • Lezione 4 (3 aprile 2020)
  • Lezione 5 (16 aprile 2020)
  • Lezione 7 (8 maggio 2020)
Page 16: Esercizi per casa (risolti)ippolitv/pdf/fns1_2020_solutions.pdfEsercizi per casa (risolti) Valerio Ippolito 13 maggio 2020 1 Lezione 1 (13 marzo 2020) Esercizio1 Dilatazionedeitempi

verde ndash attraversa senza rallentare Una pattuglia della polizia municipale loferma e lo multa per esser passato col rosso Assumendo sia il supereroe che ivigili siano nel giusto a che velocitagrave viaggiava il supereroe

Soluzione dellrsquoesercizio 28Per la polizia municipale il semaforo emette fotoni di energia

E0 = hν0 =hc

λrossoasymp hc

630 nm

mentre il supereoe vede fotoni di energia

E = hν =hc

λverdeasymp hc

490 nm

e dalle trasformazioni di Lorentz indicando con γ e β le variabili calcolateusando la velocitagrave del supereroe misurata dalla municipale possiamo scrivere

E = γ(E0minusβp0) = γ(E0minusβE0) =1minus βradic1minus β2

E0 =1minus βradic

(1 + β)(1minus β)E0 =

radic1minus βradic1 + β

E0

dove abbiamo usato il fatto che i fotoni hanno massa nulla Perciograve

E

E0=

hcλverdehc

λrosso

=λrosso

λverde=

radic1minus βradic1 + β

rarr v = βc = 025c

Esercizio 29 Conseguenze della relativitagraveI neutrini sono particelle di massa molto piccola e al momento ignota Uno

dei modi con cui egrave stato possibile dedurre un limite superiore al suo valore egravestata lrsquoosservazione nel 1987 di neutrini prodotti dalla supernova 1987A1 chesi trova a 168000 anni luce dalla Terra Sono stati osservati due segnali dineutrini che possono essere schematizzati come segue si egrave osservato prima unneutrino di 35 MeV di energia seguito a 9 s di distanza da un secondo segnaledi 13 MeV Si assuma che questo ritardo sia dovuto al fatto che la massa delneutrino non egrave nulla e si calcoli questrsquoultima

Esercizio 30 DecadimentoAnimali e piante assumono dallrsquoatmosfera diversi composti contenenti car-

bonio Il carbonio presente nellrsquoatmosfera egrave predominantemente 126 C ma sono

presenti piccole concentrazioni del suo isotopo 146 C (un atomo ogni 1012) che

decade con emissione di elettroni attraverso il processo2

146 C rarr14

7 N + eminus + νe

con un tempo di dimezzamento di 5700 anni1httpsenwikipediaorgwikiSN_1987ANeutrino_emissions2Di altro non si tratta che del decadimento βminus nrarr p+ e+ νe

16

1 Qual egrave la concentrazione di 146 C dopo 11400 anni

2 Animali e piante assumono in vita proporzioni fisse di 146 C e 12

6 C mentrealla loro morte la quantitagrave di 14

6 C inizia a diminuire Avete a portata dimano un relitto di legno per cui misurate una emissione di elettroni daldecadimento di 14

6 C pari al 61 di quella di un pezzo di legno vivodella stessa massa quanti anni ha il manufatto

Esercizio 31 Energia cinetica e trasformazioni di LorentzDue particelle identiche di massa m ed energia cinetica T collidono frontal-

mente Qual egrave la loro energia cinetica relativa (ossia lrsquoenergia cinetica di unaparticella misurata nel sistema di riferimento dellrsquoaltra particella)

Esercizio 32 Energia nel centro di massaLa reazione

πminusprarr K0Λ0

avviene con unrsquoenergia nel centro di massa diradics = 3 GeV La massa del πminus egrave

di 1396 MeVc2 la massa del protone egrave di 938 MeVc2 la massa del Kminus egrave di498 MeVc2 e quella della Λ0 di 11 GeVc2

1 Calcolare lrsquoimpulso di πminus e Λ0 nel sistema di riferimento del centro dimassa

2 Se il protone egrave a riposo il K puograve essere emesso allrsquoindietro nel sistemadi riferimento del laboratorio

Esercizio 33 DecadimentoIl mesone φ0 egrave una particella neutra3 di circa 1 GeVc2 di massa che puograve

decadere in una coppia di particelle

φ0 rarr K+ +Kminus

di massa identicamK = 494 MeVc2 Si assuma di produrre φ0 di impulso notoegrave possibile che uno dei due K sia prodotto a riposo nel sistema di riferimentodel laboratorio

Esercizio 34 DecadimentoUn fascio di anti-neutrini muonici νmicro si puograve generare selezionando pioni

o kaoni π+ e K+ e facendoli passare in un lungo tubo in cui egrave stato fattoil vuoto4 in modo che dopo un certo tragitto L una buona parte di loro saragrave

3Il collisore DAFNE ai Laboratori Nazionali di Frascati produce specificatamente parti-celle di questo tipo tramite il processo e+ + eminus rarr φ0 httpswwwyoutubecomwatchv=L5yB9gDGKms

4Una tecnica di questo tipo egrave stata usata per inviare ai Laboratori Nazionali del Gran Sassodei fasci di neutrini prodotti al CERN di Ginevra httpsvideoscernchrecord985892

17

decaduta tramite i processi

π+ rarr microminus + νmicro

K+ rarr microminus + νmicro

Se lrsquoimpulso di pioni e kaoni egrave di 200 GeVc e la loro vita media di 26 ns e12 ns rispettivamente

1 Quanto a lungo viaggiano nel laboratorio i due tipi di particelle2 Se L = 1000 m quale saragrave la frazione di pioni e kaoni che saragrave decaduta

alla fine del tubo3 Qual egrave lrsquoenergia massima dei neutrini che egrave possibile misurare nel sistema

di riferimento del laboratorio nei due casi

5 Lezione 5 (16 aprile 2020)

Esercizio 35 Energia cineticaTra le eccellenze con sede in territorio elvetico primeggiano indiscutibilmente lacioccolata al latte e il Large Hadron Collider (LHC) Questrsquoultimo egrave un collisoredi particelle situato al CERN di Ginevra che fa scontrare due fasci identici diprotoni di impulso uguale in modulo e direzione ma verso opposto I fasci sonocomposti da circa 2800 gruppi (pacchetti) di 1011 particelle ciascuno Sapendoche lrsquoenergia nel centro di massa della collisione fra due protoni egrave

radics = 13 TeV

quanta cioccolata al latte dovete mangiare per assumere un numero di caloriepari allrsquoenergia cinetica di ciascun fascio di protoni di LHC

Soluzione dellrsquoesercizio 35Poicheacute si tratta di un collider e le energie in gioco sono molto maggiori della

massa del protone lrsquoenergia di ciascun protone egrave diradics2 = 65 TeV e quella

totale di un fascio egrave

2800times 1011 times 65 middot 1012 times 16 middot 10minus19J asymp 300 MJ

Secondo Google 100 g di cioccolata al latte apportano circa 500 cal = 500 times4184 J asymp 2 MJ Lrsquoenergia di un fascio di LHC corrisponde quindi a quella dicirca 15 kg di cioccolata

Esercizio 36 Scattering RutherfordUn fascio di particelle α di 100 MeV di energia e 032 nA di corrente5 collide

contro un bersaglio fisso di alluminio spesso 1 cm Una sperimentatrice prendeun rivelatore di 1 cm times 1 cm di superficie e lo posiziona ad un angolo di 30

5Per una spiegazione breve su come (e percheacute) si misura la corrente di un fascio di particel-le vedi httpswwwlhc-closerestaking_a_closer_look_at_lhc0beam_current Unatrattazione piugrave completa egrave data ad esempio da httpscdscernchrecord1213275filesp141pdf

18

rispetto al fascio di particelle a 1 m di distanza dal bersaglio Quante particelleα incideranno sul rivelatore ogni secondo

Soluzione dellrsquoesercizio 36Lrsquoalluminio ha una densitagrave di 27 gcm3 numero atomico 13 e massa atomica

27 uPoicheacute le particelle α sono nuclei di elio hanno carica 2e e la corrente di

032 nA corrisponde a un miliardo di particelle incidenti al secondo

dNidt

=032 nCs

2times 16times 10minus19 C= 1times 109 sminus1

Il rivelatore vede un angolo solido di

∆Ω equiv superficieraggio

2

=1 cm2

(1 m)2= 1times 10minus4 sr

Si tratta di uno scattering alla Rutherford per cui la sezione drsquourto per unitagravedi angolo solido rilevata ad un certo angolo θ vale

dΩ=

(zαzAle

2

4πε04E

1

sin2(θ2)

)2

pari a

dΩasymp(

2times 13times 4times etimes 16times 10minus19 C

4π times 89times 10minus12 Fmtimes 4times 100times 106 eV

1

sin2(π180 times 302)

)2

asymp 2times 10minus30 m2sr = 20 mbsr

(1)

e il numero di particelle visto dal rivelatore vale se indichiamo con nAl = ρAlNAAAl

la densitagrave numero di atomi di alluminio e con d lo spessore del rivelatore

dNrivelate

dt= ∆Ω

dΩnAld

dNi

dt

asymp 1times 10minus4 srtimes 2times 10minus30 m2srtimes 1times 104 cm2m2 times 27 gcm3 6times 1023 molminus1

27 gmol

= 120 Hz

Esercizio 37 Sezione drsquourtoUn bersaglio di idrogeno liquido di densitagrave ρ = 0071 gcm3 e volume V =

125 cm3 egrave bombardato da un fascio monoenergetico di pioni negativi con unflusso φ = 2times 107 mminus2sminus1 e si osserva la reazione

πminus + prarr π0 + n

rivelando i fotoni del decadimento π0 rarr γγ che avviene nel 988 dei casi Sela sezione drsquourto di quellrsquointerazione egrave σ = 40 mb quanti fotoni sono emessi ognisecondo

19

Esercizio 38 Sezione drsquourtoLa sezione drsquourto dellrsquoeffetto fotoelettrico per raggi X di 10 keV in carbonio

egrave 40 b per atomo Data una lastra di carbonio di 4 mm di spessore si calcoli1 il numero di bersagli per unitagrave di volume2 il coefficiente di assorbimento per effetto fotoelettrico dei raggi X di tale

energia3 la probabilitagrave che un raggio X incidente sulla lastra produca un elettrone

per effetto fotoelettrico

Esercizio 39 Sezioni drsquourtoUn bersaglio drsquooro di densitagrave superficiale ρS = 097 mgcm2 e superficie

SB = 1 cm2 viene colpito da un fascio di particelle α la cui sezione trasversaegrave contenuta completamente nellrsquoarea del bersaglio Sul bersaglio impattano37times 104 αs La sezione drsquourto di diffusione elastica ad un certo angolo θ valedσ

ddΩ = 1 bsr Calcolare1 la densitagrave di atomi del bersaglio per unitagrave di superficie2 il numero di particelle α rivelate in unrsquoora da un rivelatore di superficie

SR = 2 cm2 posto allrsquoangolo θ e a distanza DR = 01 m dal bersaglio3 Se il fascio di particelle viene sostituito da una sorgente radioattiva di

particelle α con distribuzione isotropa su tutto lrsquoangolo solido che vieneposta lungo la stessa linea del fascio a distanza DB = 20 cm dal bersaglioquanto tempo egrave necessario per rivelare con lo stesso rivelatore lo stessonumero di particelle calcolato sopra a paritagrave di sezione drsquourto

Esercizio 40 CinematicaUn fascio di positroni di 100 MeV di energia annichila su una targhetta fissa

di elettroni producendo due fotoni tramite il processo

e+ + eminus rarr γ + γ

Se uno dei due fotoni egrave emesso a 30 nel sistema del centro di massa1 quanto vale lrsquoenergia dei due fotoni nel sistema del centro di massa2 che energia e direzione avranno nel sistema del laboratorio

Esercizio 41 Sezione drsquourtoGli antineutrini νe prodotti da un reattore nucleare con potenza P = 16 GW

attraversano un bersaglio da 2000 l drsquoacqua posto a 50 m di distanza dal reat-tore

1 Supponendo che per ogni fissione sia prodotta unrsquoenergia termica di200 MeV e vengano emessi 6 antineutrini calcolare il numero medio direazioni

νe + prarr e+ + n

prodotte al giorno se la sezione drsquourto egrave σ = 1times 10minus43 cm2

20

2 Qual egrave la minima energia del neutrino per cui la reazione egrave permessa

6 Lezione 7 (8 maggio 2020)

Esercizio 42Un protone interagisce con un bersaglio producendo un pione di energia E =300 MeV Con un tracciatore posto a distanza d = 6 cm dal bersaglio egrave possi-bile rivelare la traiettoria del pione e risalire al punto di produzione del pionesul bersaglio Determinare lrsquoerrore sulla misura della posizione di tale puntocausato dalla presenza di un piano di alluminio di spessore L = 05 mm postoimmediatamente davanti al tracciatore (quindi a distanza d dal bersaglio) nellrsquoi-potesi che gli angoli delle tracce rispetto alla normale alle superfici del bersaglioe del piano di alluminio siano piccoli [mπ = 1396 MeV Al (Z = 13 A = 27ρ = 27 gcm3)]

Soluzione dellrsquoesercizio 42La deviazione standard dellrsquoangolo di diffusione coulombiana multipla vale

radic〈θ2〉 = 21 MeV

z

βc|p|

radicx

X0= 21 MeV

z

βc|p|

radicLρ

X0

dove p =radicE2 minusm2

π = 266 MeV β = pE = 0885 z = 1 Per lrsquoalluminioabbiamo

X0 = 24 gcm2 rarr X0ρ = 89 cm

per cui radic〈θ2〉 asymp 00067

e lo spostamento medio corrispondente rispetto alla posizione nominale egrave

δs = tan(00067)times 6 cm = 04 mm

Esercizio 43Un fascio misto di elettroni e antiprotoni passa attraverso una regione con

B = 2 T e dopo aver curvato per R = 3 m raggiunge una regione senza campomagnetico

1 Qual egrave lrsquoimpulso delle particelle selezionate2 Come discriminereste fra i due tipi di particelle3 Se usaste due scintillatori posti a 15 m di distanza quale risoluzione

temporale sarebbe necessaria per discriminare i due tipi di particelle4 Se i due scintillatori fossero spessi 2 cm e avessero una lunghezza di ra-

diazione X0 = 40 cm quanta energia perderebbero elettroni e protoninellrsquoattraversarli Si assuma una perdita di energia per ionizzazione di2 MeVcm e 25 MeVcm per protoni ed elettroni rispettivamente

21

5 Che indice di rifrazione dovrebbe avere un rivelatore a luce Cherenkovper discriminare elettroni e protoni

Soluzione dellrsquoesercizio 43Dalla definizione di forza

F =dpdt

= qvtimesB

|F| = mv2

R

per cui se il campo magnetico egrave ortogonale alla traiettoria

pc = qcBR

che poicheacute c = 03 mns possiamo esprimere in maniera piugrave conveniente come

pc[GeV] = 03B[T]R[m]

Ne segue che nel nostro esercizio (e tornando a c = 1)

p = 18 GeV

Trascurando le perdite di energia e ricordando che me mp i tempi divolo di elettroni e protoni valgono rispettivamente

te =L

βecasymp L

casymp 50 ns

etp =

L

βpc=

Lpradic

p2+m2p

casymp 56 ns

per cui serve una risoluzione dellrsquoordine del ns per discriminare elettroni e muonidal tempo di volo

Se gli scintillatori sono spessi 2 cm i protoni perderanno in ciascuno di essi2 cm times 2 MeVcm = 4 MeV per ionizzazione Gli elettroni perderanno invecenel primo rivelatore 5 MeV per ionizzazione e (me asymp 0)

18 GeV times(

1minus exp

(minus 2 cm

40 cm

))asymp 88 MeV

cioegrave dopo il primo rivelatore avranno 18 GeVminus93 MeV di energia e nel secondone perderanno altri

5 MeV + (18 GeV minus 93 MeV)

(1minus exp

(minus 2 cm

40 cm

))asymp 88 MeV

Lrsquoangolo di emissione di luce Cherenkov egrave dato da

cos θc =1

βnle 1

22

e poicheacute βe asymp 1 βp asymp 0887 per avere luce Cherenkov solo al passaggio deglielettroni (e non dei protoni) egrave necessario che

1 lt n lt 113

Esercizio 44Come misurereste la massa di una particella carica E quella di una parti-

cella neutra

Esercizio 45Un muone di energia E = 400 GeV penetra verticalmente nel mare Attra-

verso quale processo puograve essere rivelato A quale profonditagrave arriva prima didecadere

Esercizio 46Avete a disposizione dei tubi fotomoltiplicatori sensibili a lunghezze drsquoonda

fra 300 nm e 500 nm e volete rivelare la luce Cherenkov prodotta dal passaggio diun elettrone di 1 MeV di energia in un metro drsquoacqua Quanti fotoni vi aspettateche vengano prodotti Confrontate con il numero di elettroni di ionizzazioneche produrrebbe una particella α di 5 keV nello stesso rivelatore

Esercizio 47Determinare quali sono i processi piugrave probabili (cioegrave quelli di sezione drsquourto

piugrave alta) nellrsquointerazione fra1 fotoni di 1 MeV e atomi di alluminio2 fotoni di 100 keV e H23 fotoni di 100 keV e atomi di ferro4 fotoni di 10 MeV e atomi di carbonio5 fotoni di 10 MeV e atomi di piombo

Esercizio 48Volete misurare lrsquoimpulso di una particella carica che attraversa un campo

magnetico B ortogonale alla sua traiettoria Avete a disposizione tre rivelatori diposizione della stessa risoluzione spaziale δx come li disponete Che risoluzionein impulso vi aspettate di ottenere

23

  • Lezione 1 (13 marzo 2020)
  • Lezione 2 (20 marzo 2020)
  • Lezione 3 (27 marzo 2020)
  • Lezione 4 (3 aprile 2020)
  • Lezione 5 (16 aprile 2020)
  • Lezione 7 (8 maggio 2020)
Page 17: Esercizi per casa (risolti)ippolitv/pdf/fns1_2020_solutions.pdfEsercizi per casa (risolti) Valerio Ippolito 13 maggio 2020 1 Lezione 1 (13 marzo 2020) Esercizio1 Dilatazionedeitempi

1 Qual egrave la concentrazione di 146 C dopo 11400 anni

2 Animali e piante assumono in vita proporzioni fisse di 146 C e 12

6 C mentrealla loro morte la quantitagrave di 14

6 C inizia a diminuire Avete a portata dimano un relitto di legno per cui misurate una emissione di elettroni daldecadimento di 14

6 C pari al 61 di quella di un pezzo di legno vivodella stessa massa quanti anni ha il manufatto

Esercizio 31 Energia cinetica e trasformazioni di LorentzDue particelle identiche di massa m ed energia cinetica T collidono frontal-

mente Qual egrave la loro energia cinetica relativa (ossia lrsquoenergia cinetica di unaparticella misurata nel sistema di riferimento dellrsquoaltra particella)

Esercizio 32 Energia nel centro di massaLa reazione

πminusprarr K0Λ0

avviene con unrsquoenergia nel centro di massa diradics = 3 GeV La massa del πminus egrave

di 1396 MeVc2 la massa del protone egrave di 938 MeVc2 la massa del Kminus egrave di498 MeVc2 e quella della Λ0 di 11 GeVc2

1 Calcolare lrsquoimpulso di πminus e Λ0 nel sistema di riferimento del centro dimassa

2 Se il protone egrave a riposo il K puograve essere emesso allrsquoindietro nel sistemadi riferimento del laboratorio

Esercizio 33 DecadimentoIl mesone φ0 egrave una particella neutra3 di circa 1 GeVc2 di massa che puograve

decadere in una coppia di particelle

φ0 rarr K+ +Kminus

di massa identicamK = 494 MeVc2 Si assuma di produrre φ0 di impulso notoegrave possibile che uno dei due K sia prodotto a riposo nel sistema di riferimentodel laboratorio

Esercizio 34 DecadimentoUn fascio di anti-neutrini muonici νmicro si puograve generare selezionando pioni

o kaoni π+ e K+ e facendoli passare in un lungo tubo in cui egrave stato fattoil vuoto4 in modo che dopo un certo tragitto L una buona parte di loro saragrave

3Il collisore DAFNE ai Laboratori Nazionali di Frascati produce specificatamente parti-celle di questo tipo tramite il processo e+ + eminus rarr φ0 httpswwwyoutubecomwatchv=L5yB9gDGKms

4Una tecnica di questo tipo egrave stata usata per inviare ai Laboratori Nazionali del Gran Sassodei fasci di neutrini prodotti al CERN di Ginevra httpsvideoscernchrecord985892

17

decaduta tramite i processi

π+ rarr microminus + νmicro

K+ rarr microminus + νmicro

Se lrsquoimpulso di pioni e kaoni egrave di 200 GeVc e la loro vita media di 26 ns e12 ns rispettivamente

1 Quanto a lungo viaggiano nel laboratorio i due tipi di particelle2 Se L = 1000 m quale saragrave la frazione di pioni e kaoni che saragrave decaduta

alla fine del tubo3 Qual egrave lrsquoenergia massima dei neutrini che egrave possibile misurare nel sistema

di riferimento del laboratorio nei due casi

5 Lezione 5 (16 aprile 2020)

Esercizio 35 Energia cineticaTra le eccellenze con sede in territorio elvetico primeggiano indiscutibilmente lacioccolata al latte e il Large Hadron Collider (LHC) Questrsquoultimo egrave un collisoredi particelle situato al CERN di Ginevra che fa scontrare due fasci identici diprotoni di impulso uguale in modulo e direzione ma verso opposto I fasci sonocomposti da circa 2800 gruppi (pacchetti) di 1011 particelle ciascuno Sapendoche lrsquoenergia nel centro di massa della collisione fra due protoni egrave

radics = 13 TeV

quanta cioccolata al latte dovete mangiare per assumere un numero di caloriepari allrsquoenergia cinetica di ciascun fascio di protoni di LHC

Soluzione dellrsquoesercizio 35Poicheacute si tratta di un collider e le energie in gioco sono molto maggiori della

massa del protone lrsquoenergia di ciascun protone egrave diradics2 = 65 TeV e quella

totale di un fascio egrave

2800times 1011 times 65 middot 1012 times 16 middot 10minus19J asymp 300 MJ

Secondo Google 100 g di cioccolata al latte apportano circa 500 cal = 500 times4184 J asymp 2 MJ Lrsquoenergia di un fascio di LHC corrisponde quindi a quella dicirca 15 kg di cioccolata

Esercizio 36 Scattering RutherfordUn fascio di particelle α di 100 MeV di energia e 032 nA di corrente5 collide

contro un bersaglio fisso di alluminio spesso 1 cm Una sperimentatrice prendeun rivelatore di 1 cm times 1 cm di superficie e lo posiziona ad un angolo di 30

5Per una spiegazione breve su come (e percheacute) si misura la corrente di un fascio di particel-le vedi httpswwwlhc-closerestaking_a_closer_look_at_lhc0beam_current Unatrattazione piugrave completa egrave data ad esempio da httpscdscernchrecord1213275filesp141pdf

18

rispetto al fascio di particelle a 1 m di distanza dal bersaglio Quante particelleα incideranno sul rivelatore ogni secondo

Soluzione dellrsquoesercizio 36Lrsquoalluminio ha una densitagrave di 27 gcm3 numero atomico 13 e massa atomica

27 uPoicheacute le particelle α sono nuclei di elio hanno carica 2e e la corrente di

032 nA corrisponde a un miliardo di particelle incidenti al secondo

dNidt

=032 nCs

2times 16times 10minus19 C= 1times 109 sminus1

Il rivelatore vede un angolo solido di

∆Ω equiv superficieraggio

2

=1 cm2

(1 m)2= 1times 10minus4 sr

Si tratta di uno scattering alla Rutherford per cui la sezione drsquourto per unitagravedi angolo solido rilevata ad un certo angolo θ vale

dΩ=

(zαzAle

2

4πε04E

1

sin2(θ2)

)2

pari a

dΩasymp(

2times 13times 4times etimes 16times 10minus19 C

4π times 89times 10minus12 Fmtimes 4times 100times 106 eV

1

sin2(π180 times 302)

)2

asymp 2times 10minus30 m2sr = 20 mbsr

(1)

e il numero di particelle visto dal rivelatore vale se indichiamo con nAl = ρAlNAAAl

la densitagrave numero di atomi di alluminio e con d lo spessore del rivelatore

dNrivelate

dt= ∆Ω

dΩnAld

dNi

dt

asymp 1times 10minus4 srtimes 2times 10minus30 m2srtimes 1times 104 cm2m2 times 27 gcm3 6times 1023 molminus1

27 gmol

= 120 Hz

Esercizio 37 Sezione drsquourtoUn bersaglio di idrogeno liquido di densitagrave ρ = 0071 gcm3 e volume V =

125 cm3 egrave bombardato da un fascio monoenergetico di pioni negativi con unflusso φ = 2times 107 mminus2sminus1 e si osserva la reazione

πminus + prarr π0 + n

rivelando i fotoni del decadimento π0 rarr γγ che avviene nel 988 dei casi Sela sezione drsquourto di quellrsquointerazione egrave σ = 40 mb quanti fotoni sono emessi ognisecondo

19

Esercizio 38 Sezione drsquourtoLa sezione drsquourto dellrsquoeffetto fotoelettrico per raggi X di 10 keV in carbonio

egrave 40 b per atomo Data una lastra di carbonio di 4 mm di spessore si calcoli1 il numero di bersagli per unitagrave di volume2 il coefficiente di assorbimento per effetto fotoelettrico dei raggi X di tale

energia3 la probabilitagrave che un raggio X incidente sulla lastra produca un elettrone

per effetto fotoelettrico

Esercizio 39 Sezioni drsquourtoUn bersaglio drsquooro di densitagrave superficiale ρS = 097 mgcm2 e superficie

SB = 1 cm2 viene colpito da un fascio di particelle α la cui sezione trasversaegrave contenuta completamente nellrsquoarea del bersaglio Sul bersaglio impattano37times 104 αs La sezione drsquourto di diffusione elastica ad un certo angolo θ valedσ

ddΩ = 1 bsr Calcolare1 la densitagrave di atomi del bersaglio per unitagrave di superficie2 il numero di particelle α rivelate in unrsquoora da un rivelatore di superficie

SR = 2 cm2 posto allrsquoangolo θ e a distanza DR = 01 m dal bersaglio3 Se il fascio di particelle viene sostituito da una sorgente radioattiva di

particelle α con distribuzione isotropa su tutto lrsquoangolo solido che vieneposta lungo la stessa linea del fascio a distanza DB = 20 cm dal bersaglioquanto tempo egrave necessario per rivelare con lo stesso rivelatore lo stessonumero di particelle calcolato sopra a paritagrave di sezione drsquourto

Esercizio 40 CinematicaUn fascio di positroni di 100 MeV di energia annichila su una targhetta fissa

di elettroni producendo due fotoni tramite il processo

e+ + eminus rarr γ + γ

Se uno dei due fotoni egrave emesso a 30 nel sistema del centro di massa1 quanto vale lrsquoenergia dei due fotoni nel sistema del centro di massa2 che energia e direzione avranno nel sistema del laboratorio

Esercizio 41 Sezione drsquourtoGli antineutrini νe prodotti da un reattore nucleare con potenza P = 16 GW

attraversano un bersaglio da 2000 l drsquoacqua posto a 50 m di distanza dal reat-tore

1 Supponendo che per ogni fissione sia prodotta unrsquoenergia termica di200 MeV e vengano emessi 6 antineutrini calcolare il numero medio direazioni

νe + prarr e+ + n

prodotte al giorno se la sezione drsquourto egrave σ = 1times 10minus43 cm2

20

2 Qual egrave la minima energia del neutrino per cui la reazione egrave permessa

6 Lezione 7 (8 maggio 2020)

Esercizio 42Un protone interagisce con un bersaglio producendo un pione di energia E =300 MeV Con un tracciatore posto a distanza d = 6 cm dal bersaglio egrave possi-bile rivelare la traiettoria del pione e risalire al punto di produzione del pionesul bersaglio Determinare lrsquoerrore sulla misura della posizione di tale puntocausato dalla presenza di un piano di alluminio di spessore L = 05 mm postoimmediatamente davanti al tracciatore (quindi a distanza d dal bersaglio) nellrsquoi-potesi che gli angoli delle tracce rispetto alla normale alle superfici del bersaglioe del piano di alluminio siano piccoli [mπ = 1396 MeV Al (Z = 13 A = 27ρ = 27 gcm3)]

Soluzione dellrsquoesercizio 42La deviazione standard dellrsquoangolo di diffusione coulombiana multipla vale

radic〈θ2〉 = 21 MeV

z

βc|p|

radicx

X0= 21 MeV

z

βc|p|

radicLρ

X0

dove p =radicE2 minusm2

π = 266 MeV β = pE = 0885 z = 1 Per lrsquoalluminioabbiamo

X0 = 24 gcm2 rarr X0ρ = 89 cm

per cui radic〈θ2〉 asymp 00067

e lo spostamento medio corrispondente rispetto alla posizione nominale egrave

δs = tan(00067)times 6 cm = 04 mm

Esercizio 43Un fascio misto di elettroni e antiprotoni passa attraverso una regione con

B = 2 T e dopo aver curvato per R = 3 m raggiunge una regione senza campomagnetico

1 Qual egrave lrsquoimpulso delle particelle selezionate2 Come discriminereste fra i due tipi di particelle3 Se usaste due scintillatori posti a 15 m di distanza quale risoluzione

temporale sarebbe necessaria per discriminare i due tipi di particelle4 Se i due scintillatori fossero spessi 2 cm e avessero una lunghezza di ra-

diazione X0 = 40 cm quanta energia perderebbero elettroni e protoninellrsquoattraversarli Si assuma una perdita di energia per ionizzazione di2 MeVcm e 25 MeVcm per protoni ed elettroni rispettivamente

21

5 Che indice di rifrazione dovrebbe avere un rivelatore a luce Cherenkovper discriminare elettroni e protoni

Soluzione dellrsquoesercizio 43Dalla definizione di forza

F =dpdt

= qvtimesB

|F| = mv2

R

per cui se il campo magnetico egrave ortogonale alla traiettoria

pc = qcBR

che poicheacute c = 03 mns possiamo esprimere in maniera piugrave conveniente come

pc[GeV] = 03B[T]R[m]

Ne segue che nel nostro esercizio (e tornando a c = 1)

p = 18 GeV

Trascurando le perdite di energia e ricordando che me mp i tempi divolo di elettroni e protoni valgono rispettivamente

te =L

βecasymp L

casymp 50 ns

etp =

L

βpc=

Lpradic

p2+m2p

casymp 56 ns

per cui serve una risoluzione dellrsquoordine del ns per discriminare elettroni e muonidal tempo di volo

Se gli scintillatori sono spessi 2 cm i protoni perderanno in ciascuno di essi2 cm times 2 MeVcm = 4 MeV per ionizzazione Gli elettroni perderanno invecenel primo rivelatore 5 MeV per ionizzazione e (me asymp 0)

18 GeV times(

1minus exp

(minus 2 cm

40 cm

))asymp 88 MeV

cioegrave dopo il primo rivelatore avranno 18 GeVminus93 MeV di energia e nel secondone perderanno altri

5 MeV + (18 GeV minus 93 MeV)

(1minus exp

(minus 2 cm

40 cm

))asymp 88 MeV

Lrsquoangolo di emissione di luce Cherenkov egrave dato da

cos θc =1

βnle 1

22

e poicheacute βe asymp 1 βp asymp 0887 per avere luce Cherenkov solo al passaggio deglielettroni (e non dei protoni) egrave necessario che

1 lt n lt 113

Esercizio 44Come misurereste la massa di una particella carica E quella di una parti-

cella neutra

Esercizio 45Un muone di energia E = 400 GeV penetra verticalmente nel mare Attra-

verso quale processo puograve essere rivelato A quale profonditagrave arriva prima didecadere

Esercizio 46Avete a disposizione dei tubi fotomoltiplicatori sensibili a lunghezze drsquoonda

fra 300 nm e 500 nm e volete rivelare la luce Cherenkov prodotta dal passaggio diun elettrone di 1 MeV di energia in un metro drsquoacqua Quanti fotoni vi aspettateche vengano prodotti Confrontate con il numero di elettroni di ionizzazioneche produrrebbe una particella α di 5 keV nello stesso rivelatore

Esercizio 47Determinare quali sono i processi piugrave probabili (cioegrave quelli di sezione drsquourto

piugrave alta) nellrsquointerazione fra1 fotoni di 1 MeV e atomi di alluminio2 fotoni di 100 keV e H23 fotoni di 100 keV e atomi di ferro4 fotoni di 10 MeV e atomi di carbonio5 fotoni di 10 MeV e atomi di piombo

Esercizio 48Volete misurare lrsquoimpulso di una particella carica che attraversa un campo

magnetico B ortogonale alla sua traiettoria Avete a disposizione tre rivelatori diposizione della stessa risoluzione spaziale δx come li disponete Che risoluzionein impulso vi aspettate di ottenere

23

  • Lezione 1 (13 marzo 2020)
  • Lezione 2 (20 marzo 2020)
  • Lezione 3 (27 marzo 2020)
  • Lezione 4 (3 aprile 2020)
  • Lezione 5 (16 aprile 2020)
  • Lezione 7 (8 maggio 2020)
Page 18: Esercizi per casa (risolti)ippolitv/pdf/fns1_2020_solutions.pdfEsercizi per casa (risolti) Valerio Ippolito 13 maggio 2020 1 Lezione 1 (13 marzo 2020) Esercizio1 Dilatazionedeitempi

decaduta tramite i processi

π+ rarr microminus + νmicro

K+ rarr microminus + νmicro

Se lrsquoimpulso di pioni e kaoni egrave di 200 GeVc e la loro vita media di 26 ns e12 ns rispettivamente

1 Quanto a lungo viaggiano nel laboratorio i due tipi di particelle2 Se L = 1000 m quale saragrave la frazione di pioni e kaoni che saragrave decaduta

alla fine del tubo3 Qual egrave lrsquoenergia massima dei neutrini che egrave possibile misurare nel sistema

di riferimento del laboratorio nei due casi

5 Lezione 5 (16 aprile 2020)

Esercizio 35 Energia cineticaTra le eccellenze con sede in territorio elvetico primeggiano indiscutibilmente lacioccolata al latte e il Large Hadron Collider (LHC) Questrsquoultimo egrave un collisoredi particelle situato al CERN di Ginevra che fa scontrare due fasci identici diprotoni di impulso uguale in modulo e direzione ma verso opposto I fasci sonocomposti da circa 2800 gruppi (pacchetti) di 1011 particelle ciascuno Sapendoche lrsquoenergia nel centro di massa della collisione fra due protoni egrave

radics = 13 TeV

quanta cioccolata al latte dovete mangiare per assumere un numero di caloriepari allrsquoenergia cinetica di ciascun fascio di protoni di LHC

Soluzione dellrsquoesercizio 35Poicheacute si tratta di un collider e le energie in gioco sono molto maggiori della

massa del protone lrsquoenergia di ciascun protone egrave diradics2 = 65 TeV e quella

totale di un fascio egrave

2800times 1011 times 65 middot 1012 times 16 middot 10minus19J asymp 300 MJ

Secondo Google 100 g di cioccolata al latte apportano circa 500 cal = 500 times4184 J asymp 2 MJ Lrsquoenergia di un fascio di LHC corrisponde quindi a quella dicirca 15 kg di cioccolata

Esercizio 36 Scattering RutherfordUn fascio di particelle α di 100 MeV di energia e 032 nA di corrente5 collide

contro un bersaglio fisso di alluminio spesso 1 cm Una sperimentatrice prendeun rivelatore di 1 cm times 1 cm di superficie e lo posiziona ad un angolo di 30

5Per una spiegazione breve su come (e percheacute) si misura la corrente di un fascio di particel-le vedi httpswwwlhc-closerestaking_a_closer_look_at_lhc0beam_current Unatrattazione piugrave completa egrave data ad esempio da httpscdscernchrecord1213275filesp141pdf

18

rispetto al fascio di particelle a 1 m di distanza dal bersaglio Quante particelleα incideranno sul rivelatore ogni secondo

Soluzione dellrsquoesercizio 36Lrsquoalluminio ha una densitagrave di 27 gcm3 numero atomico 13 e massa atomica

27 uPoicheacute le particelle α sono nuclei di elio hanno carica 2e e la corrente di

032 nA corrisponde a un miliardo di particelle incidenti al secondo

dNidt

=032 nCs

2times 16times 10minus19 C= 1times 109 sminus1

Il rivelatore vede un angolo solido di

∆Ω equiv superficieraggio

2

=1 cm2

(1 m)2= 1times 10minus4 sr

Si tratta di uno scattering alla Rutherford per cui la sezione drsquourto per unitagravedi angolo solido rilevata ad un certo angolo θ vale

dΩ=

(zαzAle

2

4πε04E

1

sin2(θ2)

)2

pari a

dΩasymp(

2times 13times 4times etimes 16times 10minus19 C

4π times 89times 10minus12 Fmtimes 4times 100times 106 eV

1

sin2(π180 times 302)

)2

asymp 2times 10minus30 m2sr = 20 mbsr

(1)

e il numero di particelle visto dal rivelatore vale se indichiamo con nAl = ρAlNAAAl

la densitagrave numero di atomi di alluminio e con d lo spessore del rivelatore

dNrivelate

dt= ∆Ω

dΩnAld

dNi

dt

asymp 1times 10minus4 srtimes 2times 10minus30 m2srtimes 1times 104 cm2m2 times 27 gcm3 6times 1023 molminus1

27 gmol

= 120 Hz

Esercizio 37 Sezione drsquourtoUn bersaglio di idrogeno liquido di densitagrave ρ = 0071 gcm3 e volume V =

125 cm3 egrave bombardato da un fascio monoenergetico di pioni negativi con unflusso φ = 2times 107 mminus2sminus1 e si osserva la reazione

πminus + prarr π0 + n

rivelando i fotoni del decadimento π0 rarr γγ che avviene nel 988 dei casi Sela sezione drsquourto di quellrsquointerazione egrave σ = 40 mb quanti fotoni sono emessi ognisecondo

19

Esercizio 38 Sezione drsquourtoLa sezione drsquourto dellrsquoeffetto fotoelettrico per raggi X di 10 keV in carbonio

egrave 40 b per atomo Data una lastra di carbonio di 4 mm di spessore si calcoli1 il numero di bersagli per unitagrave di volume2 il coefficiente di assorbimento per effetto fotoelettrico dei raggi X di tale

energia3 la probabilitagrave che un raggio X incidente sulla lastra produca un elettrone

per effetto fotoelettrico

Esercizio 39 Sezioni drsquourtoUn bersaglio drsquooro di densitagrave superficiale ρS = 097 mgcm2 e superficie

SB = 1 cm2 viene colpito da un fascio di particelle α la cui sezione trasversaegrave contenuta completamente nellrsquoarea del bersaglio Sul bersaglio impattano37times 104 αs La sezione drsquourto di diffusione elastica ad un certo angolo θ valedσ

ddΩ = 1 bsr Calcolare1 la densitagrave di atomi del bersaglio per unitagrave di superficie2 il numero di particelle α rivelate in unrsquoora da un rivelatore di superficie

SR = 2 cm2 posto allrsquoangolo θ e a distanza DR = 01 m dal bersaglio3 Se il fascio di particelle viene sostituito da una sorgente radioattiva di

particelle α con distribuzione isotropa su tutto lrsquoangolo solido che vieneposta lungo la stessa linea del fascio a distanza DB = 20 cm dal bersaglioquanto tempo egrave necessario per rivelare con lo stesso rivelatore lo stessonumero di particelle calcolato sopra a paritagrave di sezione drsquourto

Esercizio 40 CinematicaUn fascio di positroni di 100 MeV di energia annichila su una targhetta fissa

di elettroni producendo due fotoni tramite il processo

e+ + eminus rarr γ + γ

Se uno dei due fotoni egrave emesso a 30 nel sistema del centro di massa1 quanto vale lrsquoenergia dei due fotoni nel sistema del centro di massa2 che energia e direzione avranno nel sistema del laboratorio

Esercizio 41 Sezione drsquourtoGli antineutrini νe prodotti da un reattore nucleare con potenza P = 16 GW

attraversano un bersaglio da 2000 l drsquoacqua posto a 50 m di distanza dal reat-tore

1 Supponendo che per ogni fissione sia prodotta unrsquoenergia termica di200 MeV e vengano emessi 6 antineutrini calcolare il numero medio direazioni

νe + prarr e+ + n

prodotte al giorno se la sezione drsquourto egrave σ = 1times 10minus43 cm2

20

2 Qual egrave la minima energia del neutrino per cui la reazione egrave permessa

6 Lezione 7 (8 maggio 2020)

Esercizio 42Un protone interagisce con un bersaglio producendo un pione di energia E =300 MeV Con un tracciatore posto a distanza d = 6 cm dal bersaglio egrave possi-bile rivelare la traiettoria del pione e risalire al punto di produzione del pionesul bersaglio Determinare lrsquoerrore sulla misura della posizione di tale puntocausato dalla presenza di un piano di alluminio di spessore L = 05 mm postoimmediatamente davanti al tracciatore (quindi a distanza d dal bersaglio) nellrsquoi-potesi che gli angoli delle tracce rispetto alla normale alle superfici del bersaglioe del piano di alluminio siano piccoli [mπ = 1396 MeV Al (Z = 13 A = 27ρ = 27 gcm3)]

Soluzione dellrsquoesercizio 42La deviazione standard dellrsquoangolo di diffusione coulombiana multipla vale

radic〈θ2〉 = 21 MeV

z

βc|p|

radicx

X0= 21 MeV

z

βc|p|

radicLρ

X0

dove p =radicE2 minusm2

π = 266 MeV β = pE = 0885 z = 1 Per lrsquoalluminioabbiamo

X0 = 24 gcm2 rarr X0ρ = 89 cm

per cui radic〈θ2〉 asymp 00067

e lo spostamento medio corrispondente rispetto alla posizione nominale egrave

δs = tan(00067)times 6 cm = 04 mm

Esercizio 43Un fascio misto di elettroni e antiprotoni passa attraverso una regione con

B = 2 T e dopo aver curvato per R = 3 m raggiunge una regione senza campomagnetico

1 Qual egrave lrsquoimpulso delle particelle selezionate2 Come discriminereste fra i due tipi di particelle3 Se usaste due scintillatori posti a 15 m di distanza quale risoluzione

temporale sarebbe necessaria per discriminare i due tipi di particelle4 Se i due scintillatori fossero spessi 2 cm e avessero una lunghezza di ra-

diazione X0 = 40 cm quanta energia perderebbero elettroni e protoninellrsquoattraversarli Si assuma una perdita di energia per ionizzazione di2 MeVcm e 25 MeVcm per protoni ed elettroni rispettivamente

21

5 Che indice di rifrazione dovrebbe avere un rivelatore a luce Cherenkovper discriminare elettroni e protoni

Soluzione dellrsquoesercizio 43Dalla definizione di forza

F =dpdt

= qvtimesB

|F| = mv2

R

per cui se il campo magnetico egrave ortogonale alla traiettoria

pc = qcBR

che poicheacute c = 03 mns possiamo esprimere in maniera piugrave conveniente come

pc[GeV] = 03B[T]R[m]

Ne segue che nel nostro esercizio (e tornando a c = 1)

p = 18 GeV

Trascurando le perdite di energia e ricordando che me mp i tempi divolo di elettroni e protoni valgono rispettivamente

te =L

βecasymp L

casymp 50 ns

etp =

L

βpc=

Lpradic

p2+m2p

casymp 56 ns

per cui serve una risoluzione dellrsquoordine del ns per discriminare elettroni e muonidal tempo di volo

Se gli scintillatori sono spessi 2 cm i protoni perderanno in ciascuno di essi2 cm times 2 MeVcm = 4 MeV per ionizzazione Gli elettroni perderanno invecenel primo rivelatore 5 MeV per ionizzazione e (me asymp 0)

18 GeV times(

1minus exp

(minus 2 cm

40 cm

))asymp 88 MeV

cioegrave dopo il primo rivelatore avranno 18 GeVminus93 MeV di energia e nel secondone perderanno altri

5 MeV + (18 GeV minus 93 MeV)

(1minus exp

(minus 2 cm

40 cm

))asymp 88 MeV

Lrsquoangolo di emissione di luce Cherenkov egrave dato da

cos θc =1

βnle 1

22

e poicheacute βe asymp 1 βp asymp 0887 per avere luce Cherenkov solo al passaggio deglielettroni (e non dei protoni) egrave necessario che

1 lt n lt 113

Esercizio 44Come misurereste la massa di una particella carica E quella di una parti-

cella neutra

Esercizio 45Un muone di energia E = 400 GeV penetra verticalmente nel mare Attra-

verso quale processo puograve essere rivelato A quale profonditagrave arriva prima didecadere

Esercizio 46Avete a disposizione dei tubi fotomoltiplicatori sensibili a lunghezze drsquoonda

fra 300 nm e 500 nm e volete rivelare la luce Cherenkov prodotta dal passaggio diun elettrone di 1 MeV di energia in un metro drsquoacqua Quanti fotoni vi aspettateche vengano prodotti Confrontate con il numero di elettroni di ionizzazioneche produrrebbe una particella α di 5 keV nello stesso rivelatore

Esercizio 47Determinare quali sono i processi piugrave probabili (cioegrave quelli di sezione drsquourto

piugrave alta) nellrsquointerazione fra1 fotoni di 1 MeV e atomi di alluminio2 fotoni di 100 keV e H23 fotoni di 100 keV e atomi di ferro4 fotoni di 10 MeV e atomi di carbonio5 fotoni di 10 MeV e atomi di piombo

Esercizio 48Volete misurare lrsquoimpulso di una particella carica che attraversa un campo

magnetico B ortogonale alla sua traiettoria Avete a disposizione tre rivelatori diposizione della stessa risoluzione spaziale δx come li disponete Che risoluzionein impulso vi aspettate di ottenere

23

  • Lezione 1 (13 marzo 2020)
  • Lezione 2 (20 marzo 2020)
  • Lezione 3 (27 marzo 2020)
  • Lezione 4 (3 aprile 2020)
  • Lezione 5 (16 aprile 2020)
  • Lezione 7 (8 maggio 2020)
Page 19: Esercizi per casa (risolti)ippolitv/pdf/fns1_2020_solutions.pdfEsercizi per casa (risolti) Valerio Ippolito 13 maggio 2020 1 Lezione 1 (13 marzo 2020) Esercizio1 Dilatazionedeitempi

rispetto al fascio di particelle a 1 m di distanza dal bersaglio Quante particelleα incideranno sul rivelatore ogni secondo

Soluzione dellrsquoesercizio 36Lrsquoalluminio ha una densitagrave di 27 gcm3 numero atomico 13 e massa atomica

27 uPoicheacute le particelle α sono nuclei di elio hanno carica 2e e la corrente di

032 nA corrisponde a un miliardo di particelle incidenti al secondo

dNidt

=032 nCs

2times 16times 10minus19 C= 1times 109 sminus1

Il rivelatore vede un angolo solido di

∆Ω equiv superficieraggio

2

=1 cm2

(1 m)2= 1times 10minus4 sr

Si tratta di uno scattering alla Rutherford per cui la sezione drsquourto per unitagravedi angolo solido rilevata ad un certo angolo θ vale

dΩ=

(zαzAle

2

4πε04E

1

sin2(θ2)

)2

pari a

dΩasymp(

2times 13times 4times etimes 16times 10minus19 C

4π times 89times 10minus12 Fmtimes 4times 100times 106 eV

1

sin2(π180 times 302)

)2

asymp 2times 10minus30 m2sr = 20 mbsr

(1)

e il numero di particelle visto dal rivelatore vale se indichiamo con nAl = ρAlNAAAl

la densitagrave numero di atomi di alluminio e con d lo spessore del rivelatore

dNrivelate

dt= ∆Ω

dΩnAld

dNi

dt

asymp 1times 10minus4 srtimes 2times 10minus30 m2srtimes 1times 104 cm2m2 times 27 gcm3 6times 1023 molminus1

27 gmol

= 120 Hz

Esercizio 37 Sezione drsquourtoUn bersaglio di idrogeno liquido di densitagrave ρ = 0071 gcm3 e volume V =

125 cm3 egrave bombardato da un fascio monoenergetico di pioni negativi con unflusso φ = 2times 107 mminus2sminus1 e si osserva la reazione

πminus + prarr π0 + n

rivelando i fotoni del decadimento π0 rarr γγ che avviene nel 988 dei casi Sela sezione drsquourto di quellrsquointerazione egrave σ = 40 mb quanti fotoni sono emessi ognisecondo

19

Esercizio 38 Sezione drsquourtoLa sezione drsquourto dellrsquoeffetto fotoelettrico per raggi X di 10 keV in carbonio

egrave 40 b per atomo Data una lastra di carbonio di 4 mm di spessore si calcoli1 il numero di bersagli per unitagrave di volume2 il coefficiente di assorbimento per effetto fotoelettrico dei raggi X di tale

energia3 la probabilitagrave che un raggio X incidente sulla lastra produca un elettrone

per effetto fotoelettrico

Esercizio 39 Sezioni drsquourtoUn bersaglio drsquooro di densitagrave superficiale ρS = 097 mgcm2 e superficie

SB = 1 cm2 viene colpito da un fascio di particelle α la cui sezione trasversaegrave contenuta completamente nellrsquoarea del bersaglio Sul bersaglio impattano37times 104 αs La sezione drsquourto di diffusione elastica ad un certo angolo θ valedσ

ddΩ = 1 bsr Calcolare1 la densitagrave di atomi del bersaglio per unitagrave di superficie2 il numero di particelle α rivelate in unrsquoora da un rivelatore di superficie

SR = 2 cm2 posto allrsquoangolo θ e a distanza DR = 01 m dal bersaglio3 Se il fascio di particelle viene sostituito da una sorgente radioattiva di

particelle α con distribuzione isotropa su tutto lrsquoangolo solido che vieneposta lungo la stessa linea del fascio a distanza DB = 20 cm dal bersaglioquanto tempo egrave necessario per rivelare con lo stesso rivelatore lo stessonumero di particelle calcolato sopra a paritagrave di sezione drsquourto

Esercizio 40 CinematicaUn fascio di positroni di 100 MeV di energia annichila su una targhetta fissa

di elettroni producendo due fotoni tramite il processo

e+ + eminus rarr γ + γ

Se uno dei due fotoni egrave emesso a 30 nel sistema del centro di massa1 quanto vale lrsquoenergia dei due fotoni nel sistema del centro di massa2 che energia e direzione avranno nel sistema del laboratorio

Esercizio 41 Sezione drsquourtoGli antineutrini νe prodotti da un reattore nucleare con potenza P = 16 GW

attraversano un bersaglio da 2000 l drsquoacqua posto a 50 m di distanza dal reat-tore

1 Supponendo che per ogni fissione sia prodotta unrsquoenergia termica di200 MeV e vengano emessi 6 antineutrini calcolare il numero medio direazioni

νe + prarr e+ + n

prodotte al giorno se la sezione drsquourto egrave σ = 1times 10minus43 cm2

20

2 Qual egrave la minima energia del neutrino per cui la reazione egrave permessa

6 Lezione 7 (8 maggio 2020)

Esercizio 42Un protone interagisce con un bersaglio producendo un pione di energia E =300 MeV Con un tracciatore posto a distanza d = 6 cm dal bersaglio egrave possi-bile rivelare la traiettoria del pione e risalire al punto di produzione del pionesul bersaglio Determinare lrsquoerrore sulla misura della posizione di tale puntocausato dalla presenza di un piano di alluminio di spessore L = 05 mm postoimmediatamente davanti al tracciatore (quindi a distanza d dal bersaglio) nellrsquoi-potesi che gli angoli delle tracce rispetto alla normale alle superfici del bersaglioe del piano di alluminio siano piccoli [mπ = 1396 MeV Al (Z = 13 A = 27ρ = 27 gcm3)]

Soluzione dellrsquoesercizio 42La deviazione standard dellrsquoangolo di diffusione coulombiana multipla vale

radic〈θ2〉 = 21 MeV

z

βc|p|

radicx

X0= 21 MeV

z

βc|p|

radicLρ

X0

dove p =radicE2 minusm2

π = 266 MeV β = pE = 0885 z = 1 Per lrsquoalluminioabbiamo

X0 = 24 gcm2 rarr X0ρ = 89 cm

per cui radic〈θ2〉 asymp 00067

e lo spostamento medio corrispondente rispetto alla posizione nominale egrave

δs = tan(00067)times 6 cm = 04 mm

Esercizio 43Un fascio misto di elettroni e antiprotoni passa attraverso una regione con

B = 2 T e dopo aver curvato per R = 3 m raggiunge una regione senza campomagnetico

1 Qual egrave lrsquoimpulso delle particelle selezionate2 Come discriminereste fra i due tipi di particelle3 Se usaste due scintillatori posti a 15 m di distanza quale risoluzione

temporale sarebbe necessaria per discriminare i due tipi di particelle4 Se i due scintillatori fossero spessi 2 cm e avessero una lunghezza di ra-

diazione X0 = 40 cm quanta energia perderebbero elettroni e protoninellrsquoattraversarli Si assuma una perdita di energia per ionizzazione di2 MeVcm e 25 MeVcm per protoni ed elettroni rispettivamente

21

5 Che indice di rifrazione dovrebbe avere un rivelatore a luce Cherenkovper discriminare elettroni e protoni

Soluzione dellrsquoesercizio 43Dalla definizione di forza

F =dpdt

= qvtimesB

|F| = mv2

R

per cui se il campo magnetico egrave ortogonale alla traiettoria

pc = qcBR

che poicheacute c = 03 mns possiamo esprimere in maniera piugrave conveniente come

pc[GeV] = 03B[T]R[m]

Ne segue che nel nostro esercizio (e tornando a c = 1)

p = 18 GeV

Trascurando le perdite di energia e ricordando che me mp i tempi divolo di elettroni e protoni valgono rispettivamente

te =L

βecasymp L

casymp 50 ns

etp =

L

βpc=

Lpradic

p2+m2p

casymp 56 ns

per cui serve una risoluzione dellrsquoordine del ns per discriminare elettroni e muonidal tempo di volo

Se gli scintillatori sono spessi 2 cm i protoni perderanno in ciascuno di essi2 cm times 2 MeVcm = 4 MeV per ionizzazione Gli elettroni perderanno invecenel primo rivelatore 5 MeV per ionizzazione e (me asymp 0)

18 GeV times(

1minus exp

(minus 2 cm

40 cm

))asymp 88 MeV

cioegrave dopo il primo rivelatore avranno 18 GeVminus93 MeV di energia e nel secondone perderanno altri

5 MeV + (18 GeV minus 93 MeV)

(1minus exp

(minus 2 cm

40 cm

))asymp 88 MeV

Lrsquoangolo di emissione di luce Cherenkov egrave dato da

cos θc =1

βnle 1

22

e poicheacute βe asymp 1 βp asymp 0887 per avere luce Cherenkov solo al passaggio deglielettroni (e non dei protoni) egrave necessario che

1 lt n lt 113

Esercizio 44Come misurereste la massa di una particella carica E quella di una parti-

cella neutra

Esercizio 45Un muone di energia E = 400 GeV penetra verticalmente nel mare Attra-

verso quale processo puograve essere rivelato A quale profonditagrave arriva prima didecadere

Esercizio 46Avete a disposizione dei tubi fotomoltiplicatori sensibili a lunghezze drsquoonda

fra 300 nm e 500 nm e volete rivelare la luce Cherenkov prodotta dal passaggio diun elettrone di 1 MeV di energia in un metro drsquoacqua Quanti fotoni vi aspettateche vengano prodotti Confrontate con il numero di elettroni di ionizzazioneche produrrebbe una particella α di 5 keV nello stesso rivelatore

Esercizio 47Determinare quali sono i processi piugrave probabili (cioegrave quelli di sezione drsquourto

piugrave alta) nellrsquointerazione fra1 fotoni di 1 MeV e atomi di alluminio2 fotoni di 100 keV e H23 fotoni di 100 keV e atomi di ferro4 fotoni di 10 MeV e atomi di carbonio5 fotoni di 10 MeV e atomi di piombo

Esercizio 48Volete misurare lrsquoimpulso di una particella carica che attraversa un campo

magnetico B ortogonale alla sua traiettoria Avete a disposizione tre rivelatori diposizione della stessa risoluzione spaziale δx come li disponete Che risoluzionein impulso vi aspettate di ottenere

23

  • Lezione 1 (13 marzo 2020)
  • Lezione 2 (20 marzo 2020)
  • Lezione 3 (27 marzo 2020)
  • Lezione 4 (3 aprile 2020)
  • Lezione 5 (16 aprile 2020)
  • Lezione 7 (8 maggio 2020)
Page 20: Esercizi per casa (risolti)ippolitv/pdf/fns1_2020_solutions.pdfEsercizi per casa (risolti) Valerio Ippolito 13 maggio 2020 1 Lezione 1 (13 marzo 2020) Esercizio1 Dilatazionedeitempi

Esercizio 38 Sezione drsquourtoLa sezione drsquourto dellrsquoeffetto fotoelettrico per raggi X di 10 keV in carbonio

egrave 40 b per atomo Data una lastra di carbonio di 4 mm di spessore si calcoli1 il numero di bersagli per unitagrave di volume2 il coefficiente di assorbimento per effetto fotoelettrico dei raggi X di tale

energia3 la probabilitagrave che un raggio X incidente sulla lastra produca un elettrone

per effetto fotoelettrico

Esercizio 39 Sezioni drsquourtoUn bersaglio drsquooro di densitagrave superficiale ρS = 097 mgcm2 e superficie

SB = 1 cm2 viene colpito da un fascio di particelle α la cui sezione trasversaegrave contenuta completamente nellrsquoarea del bersaglio Sul bersaglio impattano37times 104 αs La sezione drsquourto di diffusione elastica ad un certo angolo θ valedσ

ddΩ = 1 bsr Calcolare1 la densitagrave di atomi del bersaglio per unitagrave di superficie2 il numero di particelle α rivelate in unrsquoora da un rivelatore di superficie

SR = 2 cm2 posto allrsquoangolo θ e a distanza DR = 01 m dal bersaglio3 Se il fascio di particelle viene sostituito da una sorgente radioattiva di

particelle α con distribuzione isotropa su tutto lrsquoangolo solido che vieneposta lungo la stessa linea del fascio a distanza DB = 20 cm dal bersaglioquanto tempo egrave necessario per rivelare con lo stesso rivelatore lo stessonumero di particelle calcolato sopra a paritagrave di sezione drsquourto

Esercizio 40 CinematicaUn fascio di positroni di 100 MeV di energia annichila su una targhetta fissa

di elettroni producendo due fotoni tramite il processo

e+ + eminus rarr γ + γ

Se uno dei due fotoni egrave emesso a 30 nel sistema del centro di massa1 quanto vale lrsquoenergia dei due fotoni nel sistema del centro di massa2 che energia e direzione avranno nel sistema del laboratorio

Esercizio 41 Sezione drsquourtoGli antineutrini νe prodotti da un reattore nucleare con potenza P = 16 GW

attraversano un bersaglio da 2000 l drsquoacqua posto a 50 m di distanza dal reat-tore

1 Supponendo che per ogni fissione sia prodotta unrsquoenergia termica di200 MeV e vengano emessi 6 antineutrini calcolare il numero medio direazioni

νe + prarr e+ + n

prodotte al giorno se la sezione drsquourto egrave σ = 1times 10minus43 cm2

20

2 Qual egrave la minima energia del neutrino per cui la reazione egrave permessa

6 Lezione 7 (8 maggio 2020)

Esercizio 42Un protone interagisce con un bersaglio producendo un pione di energia E =300 MeV Con un tracciatore posto a distanza d = 6 cm dal bersaglio egrave possi-bile rivelare la traiettoria del pione e risalire al punto di produzione del pionesul bersaglio Determinare lrsquoerrore sulla misura della posizione di tale puntocausato dalla presenza di un piano di alluminio di spessore L = 05 mm postoimmediatamente davanti al tracciatore (quindi a distanza d dal bersaglio) nellrsquoi-potesi che gli angoli delle tracce rispetto alla normale alle superfici del bersaglioe del piano di alluminio siano piccoli [mπ = 1396 MeV Al (Z = 13 A = 27ρ = 27 gcm3)]

Soluzione dellrsquoesercizio 42La deviazione standard dellrsquoangolo di diffusione coulombiana multipla vale

radic〈θ2〉 = 21 MeV

z

βc|p|

radicx

X0= 21 MeV

z

βc|p|

radicLρ

X0

dove p =radicE2 minusm2

π = 266 MeV β = pE = 0885 z = 1 Per lrsquoalluminioabbiamo

X0 = 24 gcm2 rarr X0ρ = 89 cm

per cui radic〈θ2〉 asymp 00067

e lo spostamento medio corrispondente rispetto alla posizione nominale egrave

δs = tan(00067)times 6 cm = 04 mm

Esercizio 43Un fascio misto di elettroni e antiprotoni passa attraverso una regione con

B = 2 T e dopo aver curvato per R = 3 m raggiunge una regione senza campomagnetico

1 Qual egrave lrsquoimpulso delle particelle selezionate2 Come discriminereste fra i due tipi di particelle3 Se usaste due scintillatori posti a 15 m di distanza quale risoluzione

temporale sarebbe necessaria per discriminare i due tipi di particelle4 Se i due scintillatori fossero spessi 2 cm e avessero una lunghezza di ra-

diazione X0 = 40 cm quanta energia perderebbero elettroni e protoninellrsquoattraversarli Si assuma una perdita di energia per ionizzazione di2 MeVcm e 25 MeVcm per protoni ed elettroni rispettivamente

21

5 Che indice di rifrazione dovrebbe avere un rivelatore a luce Cherenkovper discriminare elettroni e protoni

Soluzione dellrsquoesercizio 43Dalla definizione di forza

F =dpdt

= qvtimesB

|F| = mv2

R

per cui se il campo magnetico egrave ortogonale alla traiettoria

pc = qcBR

che poicheacute c = 03 mns possiamo esprimere in maniera piugrave conveniente come

pc[GeV] = 03B[T]R[m]

Ne segue che nel nostro esercizio (e tornando a c = 1)

p = 18 GeV

Trascurando le perdite di energia e ricordando che me mp i tempi divolo di elettroni e protoni valgono rispettivamente

te =L

βecasymp L

casymp 50 ns

etp =

L

βpc=

Lpradic

p2+m2p

casymp 56 ns

per cui serve una risoluzione dellrsquoordine del ns per discriminare elettroni e muonidal tempo di volo

Se gli scintillatori sono spessi 2 cm i protoni perderanno in ciascuno di essi2 cm times 2 MeVcm = 4 MeV per ionizzazione Gli elettroni perderanno invecenel primo rivelatore 5 MeV per ionizzazione e (me asymp 0)

18 GeV times(

1minus exp

(minus 2 cm

40 cm

))asymp 88 MeV

cioegrave dopo il primo rivelatore avranno 18 GeVminus93 MeV di energia e nel secondone perderanno altri

5 MeV + (18 GeV minus 93 MeV)

(1minus exp

(minus 2 cm

40 cm

))asymp 88 MeV

Lrsquoangolo di emissione di luce Cherenkov egrave dato da

cos θc =1

βnle 1

22

e poicheacute βe asymp 1 βp asymp 0887 per avere luce Cherenkov solo al passaggio deglielettroni (e non dei protoni) egrave necessario che

1 lt n lt 113

Esercizio 44Come misurereste la massa di una particella carica E quella di una parti-

cella neutra

Esercizio 45Un muone di energia E = 400 GeV penetra verticalmente nel mare Attra-

verso quale processo puograve essere rivelato A quale profonditagrave arriva prima didecadere

Esercizio 46Avete a disposizione dei tubi fotomoltiplicatori sensibili a lunghezze drsquoonda

fra 300 nm e 500 nm e volete rivelare la luce Cherenkov prodotta dal passaggio diun elettrone di 1 MeV di energia in un metro drsquoacqua Quanti fotoni vi aspettateche vengano prodotti Confrontate con il numero di elettroni di ionizzazioneche produrrebbe una particella α di 5 keV nello stesso rivelatore

Esercizio 47Determinare quali sono i processi piugrave probabili (cioegrave quelli di sezione drsquourto

piugrave alta) nellrsquointerazione fra1 fotoni di 1 MeV e atomi di alluminio2 fotoni di 100 keV e H23 fotoni di 100 keV e atomi di ferro4 fotoni di 10 MeV e atomi di carbonio5 fotoni di 10 MeV e atomi di piombo

Esercizio 48Volete misurare lrsquoimpulso di una particella carica che attraversa un campo

magnetico B ortogonale alla sua traiettoria Avete a disposizione tre rivelatori diposizione della stessa risoluzione spaziale δx come li disponete Che risoluzionein impulso vi aspettate di ottenere

23

  • Lezione 1 (13 marzo 2020)
  • Lezione 2 (20 marzo 2020)
  • Lezione 3 (27 marzo 2020)
  • Lezione 4 (3 aprile 2020)
  • Lezione 5 (16 aprile 2020)
  • Lezione 7 (8 maggio 2020)
Page 21: Esercizi per casa (risolti)ippolitv/pdf/fns1_2020_solutions.pdfEsercizi per casa (risolti) Valerio Ippolito 13 maggio 2020 1 Lezione 1 (13 marzo 2020) Esercizio1 Dilatazionedeitempi

2 Qual egrave la minima energia del neutrino per cui la reazione egrave permessa

6 Lezione 7 (8 maggio 2020)

Esercizio 42Un protone interagisce con un bersaglio producendo un pione di energia E =300 MeV Con un tracciatore posto a distanza d = 6 cm dal bersaglio egrave possi-bile rivelare la traiettoria del pione e risalire al punto di produzione del pionesul bersaglio Determinare lrsquoerrore sulla misura della posizione di tale puntocausato dalla presenza di un piano di alluminio di spessore L = 05 mm postoimmediatamente davanti al tracciatore (quindi a distanza d dal bersaglio) nellrsquoi-potesi che gli angoli delle tracce rispetto alla normale alle superfici del bersaglioe del piano di alluminio siano piccoli [mπ = 1396 MeV Al (Z = 13 A = 27ρ = 27 gcm3)]

Soluzione dellrsquoesercizio 42La deviazione standard dellrsquoangolo di diffusione coulombiana multipla vale

radic〈θ2〉 = 21 MeV

z

βc|p|

radicx

X0= 21 MeV

z

βc|p|

radicLρ

X0

dove p =radicE2 minusm2

π = 266 MeV β = pE = 0885 z = 1 Per lrsquoalluminioabbiamo

X0 = 24 gcm2 rarr X0ρ = 89 cm

per cui radic〈θ2〉 asymp 00067

e lo spostamento medio corrispondente rispetto alla posizione nominale egrave

δs = tan(00067)times 6 cm = 04 mm

Esercizio 43Un fascio misto di elettroni e antiprotoni passa attraverso una regione con

B = 2 T e dopo aver curvato per R = 3 m raggiunge una regione senza campomagnetico

1 Qual egrave lrsquoimpulso delle particelle selezionate2 Come discriminereste fra i due tipi di particelle3 Se usaste due scintillatori posti a 15 m di distanza quale risoluzione

temporale sarebbe necessaria per discriminare i due tipi di particelle4 Se i due scintillatori fossero spessi 2 cm e avessero una lunghezza di ra-

diazione X0 = 40 cm quanta energia perderebbero elettroni e protoninellrsquoattraversarli Si assuma una perdita di energia per ionizzazione di2 MeVcm e 25 MeVcm per protoni ed elettroni rispettivamente

21

5 Che indice di rifrazione dovrebbe avere un rivelatore a luce Cherenkovper discriminare elettroni e protoni

Soluzione dellrsquoesercizio 43Dalla definizione di forza

F =dpdt

= qvtimesB

|F| = mv2

R

per cui se il campo magnetico egrave ortogonale alla traiettoria

pc = qcBR

che poicheacute c = 03 mns possiamo esprimere in maniera piugrave conveniente come

pc[GeV] = 03B[T]R[m]

Ne segue che nel nostro esercizio (e tornando a c = 1)

p = 18 GeV

Trascurando le perdite di energia e ricordando che me mp i tempi divolo di elettroni e protoni valgono rispettivamente

te =L

βecasymp L

casymp 50 ns

etp =

L

βpc=

Lpradic

p2+m2p

casymp 56 ns

per cui serve una risoluzione dellrsquoordine del ns per discriminare elettroni e muonidal tempo di volo

Se gli scintillatori sono spessi 2 cm i protoni perderanno in ciascuno di essi2 cm times 2 MeVcm = 4 MeV per ionizzazione Gli elettroni perderanno invecenel primo rivelatore 5 MeV per ionizzazione e (me asymp 0)

18 GeV times(

1minus exp

(minus 2 cm

40 cm

))asymp 88 MeV

cioegrave dopo il primo rivelatore avranno 18 GeVminus93 MeV di energia e nel secondone perderanno altri

5 MeV + (18 GeV minus 93 MeV)

(1minus exp

(minus 2 cm

40 cm

))asymp 88 MeV

Lrsquoangolo di emissione di luce Cherenkov egrave dato da

cos θc =1

βnle 1

22

e poicheacute βe asymp 1 βp asymp 0887 per avere luce Cherenkov solo al passaggio deglielettroni (e non dei protoni) egrave necessario che

1 lt n lt 113

Esercizio 44Come misurereste la massa di una particella carica E quella di una parti-

cella neutra

Esercizio 45Un muone di energia E = 400 GeV penetra verticalmente nel mare Attra-

verso quale processo puograve essere rivelato A quale profonditagrave arriva prima didecadere

Esercizio 46Avete a disposizione dei tubi fotomoltiplicatori sensibili a lunghezze drsquoonda

fra 300 nm e 500 nm e volete rivelare la luce Cherenkov prodotta dal passaggio diun elettrone di 1 MeV di energia in un metro drsquoacqua Quanti fotoni vi aspettateche vengano prodotti Confrontate con il numero di elettroni di ionizzazioneche produrrebbe una particella α di 5 keV nello stesso rivelatore

Esercizio 47Determinare quali sono i processi piugrave probabili (cioegrave quelli di sezione drsquourto

piugrave alta) nellrsquointerazione fra1 fotoni di 1 MeV e atomi di alluminio2 fotoni di 100 keV e H23 fotoni di 100 keV e atomi di ferro4 fotoni di 10 MeV e atomi di carbonio5 fotoni di 10 MeV e atomi di piombo

Esercizio 48Volete misurare lrsquoimpulso di una particella carica che attraversa un campo

magnetico B ortogonale alla sua traiettoria Avete a disposizione tre rivelatori diposizione della stessa risoluzione spaziale δx come li disponete Che risoluzionein impulso vi aspettate di ottenere

23

  • Lezione 1 (13 marzo 2020)
  • Lezione 2 (20 marzo 2020)
  • Lezione 3 (27 marzo 2020)
  • Lezione 4 (3 aprile 2020)
  • Lezione 5 (16 aprile 2020)
  • Lezione 7 (8 maggio 2020)
Page 22: Esercizi per casa (risolti)ippolitv/pdf/fns1_2020_solutions.pdfEsercizi per casa (risolti) Valerio Ippolito 13 maggio 2020 1 Lezione 1 (13 marzo 2020) Esercizio1 Dilatazionedeitempi

5 Che indice di rifrazione dovrebbe avere un rivelatore a luce Cherenkovper discriminare elettroni e protoni

Soluzione dellrsquoesercizio 43Dalla definizione di forza

F =dpdt

= qvtimesB

|F| = mv2

R

per cui se il campo magnetico egrave ortogonale alla traiettoria

pc = qcBR

che poicheacute c = 03 mns possiamo esprimere in maniera piugrave conveniente come

pc[GeV] = 03B[T]R[m]

Ne segue che nel nostro esercizio (e tornando a c = 1)

p = 18 GeV

Trascurando le perdite di energia e ricordando che me mp i tempi divolo di elettroni e protoni valgono rispettivamente

te =L

βecasymp L

casymp 50 ns

etp =

L

βpc=

Lpradic

p2+m2p

casymp 56 ns

per cui serve una risoluzione dellrsquoordine del ns per discriminare elettroni e muonidal tempo di volo

Se gli scintillatori sono spessi 2 cm i protoni perderanno in ciascuno di essi2 cm times 2 MeVcm = 4 MeV per ionizzazione Gli elettroni perderanno invecenel primo rivelatore 5 MeV per ionizzazione e (me asymp 0)

18 GeV times(

1minus exp

(minus 2 cm

40 cm

))asymp 88 MeV

cioegrave dopo il primo rivelatore avranno 18 GeVminus93 MeV di energia e nel secondone perderanno altri

5 MeV + (18 GeV minus 93 MeV)

(1minus exp

(minus 2 cm

40 cm

))asymp 88 MeV

Lrsquoangolo di emissione di luce Cherenkov egrave dato da

cos θc =1

βnle 1

22

e poicheacute βe asymp 1 βp asymp 0887 per avere luce Cherenkov solo al passaggio deglielettroni (e non dei protoni) egrave necessario che

1 lt n lt 113

Esercizio 44Come misurereste la massa di una particella carica E quella di una parti-

cella neutra

Esercizio 45Un muone di energia E = 400 GeV penetra verticalmente nel mare Attra-

verso quale processo puograve essere rivelato A quale profonditagrave arriva prima didecadere

Esercizio 46Avete a disposizione dei tubi fotomoltiplicatori sensibili a lunghezze drsquoonda

fra 300 nm e 500 nm e volete rivelare la luce Cherenkov prodotta dal passaggio diun elettrone di 1 MeV di energia in un metro drsquoacqua Quanti fotoni vi aspettateche vengano prodotti Confrontate con il numero di elettroni di ionizzazioneche produrrebbe una particella α di 5 keV nello stesso rivelatore

Esercizio 47Determinare quali sono i processi piugrave probabili (cioegrave quelli di sezione drsquourto

piugrave alta) nellrsquointerazione fra1 fotoni di 1 MeV e atomi di alluminio2 fotoni di 100 keV e H23 fotoni di 100 keV e atomi di ferro4 fotoni di 10 MeV e atomi di carbonio5 fotoni di 10 MeV e atomi di piombo

Esercizio 48Volete misurare lrsquoimpulso di una particella carica che attraversa un campo

magnetico B ortogonale alla sua traiettoria Avete a disposizione tre rivelatori diposizione della stessa risoluzione spaziale δx come li disponete Che risoluzionein impulso vi aspettate di ottenere

23

  • Lezione 1 (13 marzo 2020)
  • Lezione 2 (20 marzo 2020)
  • Lezione 3 (27 marzo 2020)
  • Lezione 4 (3 aprile 2020)
  • Lezione 5 (16 aprile 2020)
  • Lezione 7 (8 maggio 2020)
Page 23: Esercizi per casa (risolti)ippolitv/pdf/fns1_2020_solutions.pdfEsercizi per casa (risolti) Valerio Ippolito 13 maggio 2020 1 Lezione 1 (13 marzo 2020) Esercizio1 Dilatazionedeitempi

e poicheacute βe asymp 1 βp asymp 0887 per avere luce Cherenkov solo al passaggio deglielettroni (e non dei protoni) egrave necessario che

1 lt n lt 113

Esercizio 44Come misurereste la massa di una particella carica E quella di una parti-

cella neutra

Esercizio 45Un muone di energia E = 400 GeV penetra verticalmente nel mare Attra-

verso quale processo puograve essere rivelato A quale profonditagrave arriva prima didecadere

Esercizio 46Avete a disposizione dei tubi fotomoltiplicatori sensibili a lunghezze drsquoonda

fra 300 nm e 500 nm e volete rivelare la luce Cherenkov prodotta dal passaggio diun elettrone di 1 MeV di energia in un metro drsquoacqua Quanti fotoni vi aspettateche vengano prodotti Confrontate con il numero di elettroni di ionizzazioneche produrrebbe una particella α di 5 keV nello stesso rivelatore

Esercizio 47Determinare quali sono i processi piugrave probabili (cioegrave quelli di sezione drsquourto

piugrave alta) nellrsquointerazione fra1 fotoni di 1 MeV e atomi di alluminio2 fotoni di 100 keV e H23 fotoni di 100 keV e atomi di ferro4 fotoni di 10 MeV e atomi di carbonio5 fotoni di 10 MeV e atomi di piombo

Esercizio 48Volete misurare lrsquoimpulso di una particella carica che attraversa un campo

magnetico B ortogonale alla sua traiettoria Avete a disposizione tre rivelatori diposizione della stessa risoluzione spaziale δx come li disponete Che risoluzionein impulso vi aspettate di ottenere

23

  • Lezione 1 (13 marzo 2020)
  • Lezione 2 (20 marzo 2020)
  • Lezione 3 (27 marzo 2020)
  • Lezione 4 (3 aprile 2020)
  • Lezione 5 (16 aprile 2020)
  • Lezione 7 (8 maggio 2020)